PEDI #4 FINAL

Ace your homework & exams now with Quizwiz!

The nurse is caring for a school-age child with tinea captitis. The child has open lesions from the disease and has lost hair in the areas affected. Which nursing diagnoses would be a part of this patient's care plan? Select all that apply. A) Impaired skin integrity B) Risk for infection C) Disturbed body image D) Bathing, self-care deficit E) Altered nutrition

A) Impaired skin integrity B) Risk for infection C) Disturbed body image Feedback: Tinea is a fungal disease of the skin occurring on any part of the body, in this case the head (scalp, eyebrows, or eyelashes). Since this child has open lesions and hair loss from affected areas, there is impairment of skin integrity (which makes the areas at risk for infection. Body image is disturbed since the hair loss is visible. There is no indication of bathing deficit or altered nutrition.

A group of students is reviewing information about glucose-6-phosphate dehydrogenase (G6PD) deficiency. The students demonstrate understanding of the material what as the cause of the disorder? A)X-linked recessive inheritance B)Deficiency in clotting factors C)An excess supply of iron D)Autosomal recessive inheritance

A. X-linked recessive inheritance G6PD deficiency is an X-linked recessive disorder that affects the functioning of the red blood cells. A deficiency in clotting factors is associated with disorders such as idiopathic thrombocytopenia purpura, DIC, or hemophilia. An excess supply of iron refers to hemosiderosis, a complication of thalassemia, an autosomal recessive disorder.

The nurse is preparing to administer intravenous fluids to manage a child with dehydration. The medical record indicates the child weighs 60 pounds (27.2 kg). How many milliliters will initially be administered? Record your answer using two decimal places.

Ans: 545.45 Nursing goals for the infant or child with dehydration are aimed at restoring fluid volume and preventing progression to hypovolemia. Provide oral rehydration to children for mild to moderate states of dehydration. Children with severe dehydration should receive intravenous fluids. Initially, administer 20 mL/kg of normal saline or lactated Ringer, and then reassess the hydration status.

A nurse is preparing a presentation for an expectant parent group about neural tube defects and prevention. Which would the nurse emphasize? A) Smoking cessation B) Aerobic exercise C) Increased calcium intake D) Folic acid supplementation

Folic acid supplementation The cause of neural tube defects is unknown, but there is strong evidence to support the use of folic acid supplementation for prevention. Smoking cessation and aerobic exercise are general health recommendations unrelated to neural tube defects. Increased calcium intake is important for fetal growth and development, but it is not linked to preventing neural tube defects.

When providing care to a newborn infant who was born at 29 weeks' gestation, the nurse integrates knowledge of potential complications, being alert for signs and symptoms of what condition? A) Neonatal conjunctivitis B) Facial deformities C) Intracranial hemorrhage D) Incomplete myelinization

Intracranial hemorrhage Premature infants have more fragile capillaries in the periventricular area than term infants, which puts them at greater risk for intracranial hemorrhage. Neonatal conjunctivitis can occur in any newborn during birth and is caused by viruses, bacteria, or chemicals. Facial deformities are typical of babies of alcoholic mothers. Incomplete myelinization is present in all newborns.

30. An infant is diagnosed with infantile glaucoma. When developing the plan of care for the infant, the nurse would expect to prepare the infant and family for which of the following? A) Goniotomy B) Antibiotic therapy C) Contact lenses D) Patching of affected eye

A) Goniotomy Therapeutic management of infantile glaucoma is focused on surgical intervention via a goniotomy. Antibiotic therapy would be used to treat an infection. Contact lenses would be indicated for refractive errors and following removal of congenital cataracts. Patching of the affected eye is used for treating amblyopia and after surgery for congenital cataract.

The nurse is assessing heart rate for children on the pediatric ward. What is a normal finding based on developmental age? A)An infant's rate is 90 bpm. B)A toddler's rate is 150 bpm. C)A preschooler's rate is 130 bpm. D)A school-age child's rate is 50 bpm.

A)An infant's rate is 90 bpm. The normal heart rate for an infant is 80 to 150 bpm, for a toddler is 70 to 120 bpm, for a preschooler is 65 to 110 bpm, and for a school-age child is 60 to 100 bpm.

The nurse is inspecting the genitals of a prepubescent girl. Which is a normal sign of the onset of puberty? A)Appearance of pubic hair around 11 to 13 years old B)Swelling or redness of the labia minora C)Presence of labial adhesions D)Lesions on the external genitalia

A)Appearance of pubic hair around 11 to 13 years old Infants and young girls (particularly those of dark-skinned races) may have a small amount of downy pubic hair. Otherwise, the appearance of pubic hair indicates the onset of pubertal changes, sometimes prior to breast changes. Pubic hair generally begins to appear by age 11 years, with age 13 being the latest. Redness or swelling of the labia may occur with infection, sexual abuse, or masturbation. Lesions on the external genitalia may indicate sexually transmitted infection.

6. The nurse is preparing a teaching plan for the family and their 6-year-old son who has just been diagnosed with diabetes mellitus. What would the nurse identify as the initial goal for the teaching plan? A) Developing management and decision-making skills B) Educating the parents about diabetes mellitus type 1 C) Developing a nutritionally sound, 30-day meal plan D) Promoting independence with self-administration of insulin

Ans: A Feedback: Developing basic management and decision-making skills related to the diabetes is the initial goal of the teaching plan for this child and family. The nurse would have provided a basic description of the disorder after it was diagnosed. Development of a detailed monthly meal plan would come later, perhaps after consulting with a nutritionist. It is too soon to expect the boy to administer his own insulin.

Origin: Chapter 6, 24 24. The nurse is caring for a 7-year-old girl who is scheduled for a hernia repair and is very scared. Which fear would she also most likely have at this age? A) Fear of being kidnapped B) Fear of cutting her finger C) Fear of sudden loud noises D) Fear of the neighbor's dog

Ans: A Feedback: At this age, the child will be fearful of being kidnapped. She should have outgrown her fears of harm to her body, noises, and dogs, all of which are typical preschooler fears.

4. The nurse is caring for families with vulnerable child syndrome. Which situation would be most likely to predispose the family to this condition? A) Having a postterm infant B) Having an infant who is reluctant to feed properly C) Having a child diagnosed with impetigo at age 10 D) Having a child with juvenile diabetes

Ans: B Feedback: "Vulnerable child syndrome" is a clinical state in which the parents' reactions to a serious illness or event in the child's past continue to have long-term psychologically harmful effects on the child and parents for many years. Risk factors for the development of vulnerable child syndrome include preterm birth, congenital anomaly, newborn jaundice, handicapping condition, an accident or illness that the child was not expected to recover from, or crying or feeding problems in the first 5 years of life.

Origin: Chapter 6, 16 16. What finding would the nurse most likely discover in a 10-year-old child in the period of concrete operational thought? A) Participation in abstract thinking B) Ability to classify similar objects C) Problem solving via the scientific method D) Ability to make independent decisions

Ans: B Feedback: During the period of concrete operational thought, children are able to classify or group objects based on their common elements. Abstract thinking, problem solving via the scientific method, and independent decision making are higher-level functions, typically seen in adolescents.

Origin: Chapter 6, 19 19. The nurse has determined that an 8-year-old girl is at risk for being overweight. Which intervention would be a priority prior to developing the care plan? A) Determining the need for additional caloric intake B) Asking the parents who they want to work with the child C) Interviewing the parents about their eating habits D) Discussing the influence of peers on the child's diet

Ans: C Feedback: The nurse would need to find out what the parents' eating habits are like. It would not be necessary to determine the need for additional caloric intake. Developing a multidisciplinary plan is an intervention for a child with growth and development problems. Discussing the influence of peers is an intervention used for preventing injury.

Origin: Chapter 5, 10 10. The mother of a 4-year-old boy tells the nurse that her son occasionally wets his pants during the day. How should the nurse respond? A) "Is there a family history of diabetes?" B) 'Suddenly having accidents can be a sign of diabetes." C) "That's normal; don't worry about it." D) "Tell me about the circumstances when this occurs."

Ans: D Feedback: Bladder control is present in 4- and 5-year-olds, but an occasional accident may occur, particularly in stressful situations or when the child is absorbed in an interesting activity. The nurse needs to ask an open-ended question to determine the circumstances when the child has had accidents. Simply telling the mother that it is normal does not address the mother's concerns. The nurse does need to gather more information, because accidents in a previously potty-trained child can be a sign of diabetes.

A group of students are reviewing information about major and minor congenital disorders. The students demonstrate understanding of the information when they identify what condition as a minor disorder? A) Webbed neck B) Omphalocele C) Cutaneous hemangioma D) Facial asymmetry

Ans: A A minor congenital anomaly is webbed neck. Omphalocele, cutaneous hemangioma, and facial asymmetry are considered major congenital anomalies.

The nurse is performing developmental surveillance for children at a medical home. Which infants are most at risk for developmental delays? Select all that apply. A)A child whose birthweight was 1,600 g B)A child whose parent has a mental illness C)A child raised by a single parent D)A child with a lead level above 10 mg/dL E)A child with hypertonia or hypotonia F)A child with gestational age more than 33 weeks

B)A child whose parent has a mental illness C)A child raised by a single parent D)A child with a lead level above 10 mg/dL E)A child with hypertonia or hypotonia Risk factors for developmental delays include having a single parent, a parent with developmental disability or mental illness, hypertonia or hypotonia, birthweight less than 1,500 g, lead level above 5 mg/dL, and gestational age less than 33 weeks.

A large portion of the nurse's efforts is dedicated to health supervision for children who use the facility as their primary medical contact. At which facility does the nurse work? A)An urgent care center B)A pediatric practice C)A mobile outreach immunization program D)A dermatology practice

B)A pediatric practice A pediatric practice is most likely to fulfill the characteristics for primary care, also known as a medical home. An urgent care center does not provide preventative care activities. Mobile outreach would not provide for any care requiring hospitalization. A dermatology practice is unlikely to provide service outside its area of specialization.

The nurse is measuring the blood pressure of a 12-year-old boy with an oscillometric device. The boy's reading is greater than the 90th percentile for gender and height. What is the appropriate nursing action? A)Repeat the reading with the oscillometric device. B)Repeat the blood pressure reading using auscultation. C)Measure the blood pressure in all four extremities. D)Measure the blood pressure with a Doppler.

B)Repeat the blood pressure reading using auscultation. The nurse should repeat the reading using auscultation. The nurse should not use the Doppler ultrasound method in this circumstance. The nurse would only measure the blood pressure in all four extremities with a child presenting with cardiac complaints.

A 10-year-old girl is brought to the emergency department by her father after tripping over a rock while running in the yard. She tells the nurse, "I think I twisted my ankle." When assessing the child, what would the nurse most likely assess? A)Bruising B)Edema C)Limited range of motion D)Absent pulse

B. Edema The girl is describing a sprain, which is frequently accompanied by edema. Bruising may or may not be present. The nurse should not attempt to perform passive range of motion on the affected body part. A pulse should be present; if one is not, neurovascular compromise is present.

The nurse is reviewing the white blood cell differential of a 4-year-old girl. Which value would lead the nurse to be concerned? A)Bands: 8% B)Segs: 28% C)Eosinophils: 10% D)Basophils: 0%

C. Eosinophils: 10% For a 4-year-old, normally eosinophils range from 0% to 3%; therefore, a result of 10% would be abnormal and a cause for concern. Bands of 8%, segs of 28%, and basophils of 0% are normal values for this age.

The nurse is providing anticipatory guidance to the parents of an 18-month-old girl. Which guidance will be most helpful for toilet teaching? A)Telling them either one may demonstrate toilet use B)Assuring them that bladder control occurs first C)Telling them that curiosity is a sure sign of readiness D)Advising them to use praise, not scolding

D)Advising them to use praise, not scolding The most helpful guidance for toilet teaching is to urge the parents to use only praise, but never to scold, throughout the process. It is best for the same-sex parent to demonstrate toilet use. Bowel control will occur first. It may take additional months for nighttime bladder control to be achieved. Curiosity is a sign of readiness for toilet teaching, but by no means a sure sign.

The nurse is performing a cognitive assessment of a 2-year-old. Which behavior would alert the nurse to a developmental delay in this area? A)The child cannot say name, age, and gender. B)The child cannot follow a series of two independent commands. C)The child has a vocabulary of 40 to 50 words. D)The child does not point to named body parts.

D)The child does not point to named body parts. The 2-year-old can point to named body parts and has a vocabulary of 40 to 50 words. At 30 months old a child can follow a series of two independent commands and at 3-years old a child can say name, age, and gender.

A nurse is assessing the fontanels of a crying newborn and notes that the posterior fontanel pulsates and briefly bulges. What do these findings indicate? A)Increased intracranial pressure B)Overhydration C)Dehydration D)These are normal findings.

D)These are normal findings. It is common to see the fontanel pulsate or briefly bulge if a baby cries. Overhydration or increased intracranial pressure would cause a persistent bulging. Dehydration would cause the fontanel to be sunken.

The nurse is performing a physical examination on a sleeping newborn. Which body system should the nurse examine last? A)Heart B)Abdomen C)Lungs D)Throat

D)Throat If the infant is asleep, the nurse should auscultate the heart, lungs, and abdomen first while the baby is quiet. The nurse performs the assessment in a head-to-toe manner, leaving the most traumatic procedures, such as examination of the ears, nose, mouth, and throat, until last.

The nurse is taking a health history of a child with suspected acute poststreptococcal glomerulonephritis. Which response would alert the nurse to a confirmed risk factor for this condition? A)"She has been very healthy up to now." B)"He just got over a head cold with laryngitis." C)"My child is just 18 months old." D)"My child has not been sick at all."

B. "He just got over a head cold with laryngitis." Known risk factors include a recent episode of pharyngitis or other streptococcal infection, age older than 2 years, and male sex.

The nurse is preparing a class for a group of adolescents about promoting safety. What would the nurse plan to include as the leading cause of adolescent injuries? A) Motor vehicles B) Firearms C) Water D) Fires

A

5. A 7-year-old boy has reentered the hospital for the second time in a month. Which intervention is particularly important at this time? A) Assessing his parents' coping abilities B) Seeking his parents' input about their child's needs C) Educating his family about the procedure D) Notifying the care team about his hospitalization

Ans: A Feedback: Transition times, such as when the child reenters the hospital, create additional stress on the parents and child. Assessing the parents' coping abilities is particularly important at this time. Seeking parental input, educating about a procedure, and notifying the care team are basic activities of family-centered care and care coordination.

15. The nurse is caring for a 10-year-old girl who is in an isolation room. Which intervention would be a priority intervention for this child? A) Reduce noise as much as possible. B) Provide age-appropriate toys and games. C) Discourage visits from family members. D) Put on mask prior to entering the room.

Ans: B Feedback: Children in this setting may experience sensory deprivation due to the limited contact with others and the use of personal protective equipment such as gloves, masks, and gowns. The nurse should stimulate the child by playing with her with age-appropriate toys/games. Reducing noise would be appropriate for sensory overload. The nurse should encourage the family to visit often, introduce himself or herself before entering the room, and allow the child to view his or her face before applying a mask.

Origin: Chapter 14, 28 28. The nurse is explaining the effects of heat application for pain relief. Which effect would the nurse be likely to include? A) Decreased blood flow to the area B) Increased pressure on nociceptive fibers C) Possible release of endogenous opioids D) Altered capillary permeability

Ans: B Feedback: Heat causes an increase in blood flow. This alters capillary permeability, leading to a reduction in swelling and pressure on nociceptive fibers. Heat also may trigger the release of endogenous opioids, which mediate the pain response.

22. When assessing the adolescent with anorexia, what would the nurse expect to find? A) Tachycardia B) Hypertension C) Fever D) Sparse body hair

Ans: D Feedback: An adolescent with anorexia often exhibits a low body temperature; bradycardia; and hypotension; as well as soft, sparse body hair and thinning scalp hair.

10. What finding would the nurse expect to assess in a child with hypothyroidism? A) Nervousness B) Heat intolerance C) Smooth velvety skin D) Weight gain

Ans: D Feedback: Hypothyroidism is manifested by weight gain, fatigue, cold intolerance, and dry skin. Nervousness, heat intolerance, and smooth velvety skin are associated with hyperthyroidism.

The nurse is teaching the parents of a 12-year-old boy about appropriate approaches when raising an adolescent. Which comment should be included in the discussion? A) "Find out if his friends are worthy of him." B) "Try to be open to his views." C) "Maintain a firm set of rules." D) "Remind him that he is still your little boy."

B

The nurse knows that barriers to the adolescent's health and successful achievement of the tasks of adolescence exist. What is the major barrier to health for this population? A) Cultural B) Socioeconomic C) Marital status D) Racial

B

The nurse is caring for a 12-year-old boy with idiopathic thrombocytopenia. The nurse is providing discharge instructions about home care and safety recommendations to the boy and his parents. Which response indicates a need for further teaching? A)"We should avoid aspirin and drugs like ibuprofen." B)"He can resume participation in football in 2 weeks." C)"Swimming would be a great activity." D)"Our son cannot take any antihistamines."

B. "He can resume participation in football in 2 weeks." The nurse must emphasize to the parents that they need to prevent trauma to their son by avoiding activities that may cause injury. Participation in contact sports like football is not recommended. Aspirin, nonsteroidal anti-inflammatory drugs, and antihistamines should be avoided because they could precipitate anemia. Swimming, a noncontact sport, is an appropriate choice.

The nurse is preparing a presentation for a parent group about childhood cancers, focusing on brain tumors in children. What would the nurse describe as the most common type of brain tumor? A)Brain stem glioma B)Medulloblastoma C)Ependymoma D)Astrocytoma

B. Medulloblastoma Of all the types of brain tumors listed, a medulloblastoma is the most common type. It is invasive, is highly malignant, and grows rapidly.

The nurse is caring for a 19-month-old boy who has been admitted to the emergency department with a skull fracture. The parents state that the child fell down when running through the house and hit his head on the floor. Based on normal characteristics of skull fractures, what should be the initial focus of the assessment? A) Possible physical abuse B) Possible bone cancer C) Possible chronic neurological disease D) Possible developmental delay

Possible physical abuse Physical abuse must be investigated first because it takes a great deal of force to produce a skull fracture in infants and children younger than 2 years old. Due to the flexibility of the immature skull, it is able to withstand a great degree of deformation before a fracture will occur.

A nurse is providing teaching to the parents of a child who has had a shunt inserted as treatment for hydrocephalus. The parents demonstrate understanding of the teaching when they make what statement? A) "Having the shunt put in decreases his risk for developmental problems." B) "If he doesn't get an infection in the first week, the risk is greatly reduced." C) "He will need more surgeries to replace the shunt as he grows." D) "The shunt will help to prevent any further complications from his disease."

"He will need more surgeries to replace the shunt as he grows." Parents need to know that hydrocephalus is a chronic illness that requires lifelong follow-up and regular evaluations, including future surgeries as the child grows. The risk for infection is ever present, but is most common 1 to 2 months after shunt placement. The child with a shunt and hydrocephalus is at risk for potential growth and developmental disabilities as well as complications such as infection and malfunction of the shunt.

2. The nurse is discussing discharge instructions with the parents of a 6-year-old who had a tonsillectomy. What is the most important thing to stress? A. Administer analgesics. B. Encourage the child to drink liquids. C. Inspect the throat for bleeding. D. Apply an ice collar.

Answer: C Rationale: Inspecting the throat for bleeding is the most important discharge information to give the parents. Hemorrhage is unusual postoperatively but may occur any time from the immediate postoperative period to as late as 10 days after surgery. The nurse should inspect the throat for bleeding. Mucus tinged with blood may be expected, but fresh blood in the secretions indicates bleeding. Administering analgesics, encouraging fluids and applying an ice color are important but not as important as assessing for bleeding.

21. A child is brought to the emergency department by his parents because he suddenly developed a barking cough. Further assessment leads the nurse to suspect that the child is experiencing croup. What would the nurse have most likely assessed? A. High fever B. Dysphagia C. Toxic appearance D. Inspiratory stridor

Answer: D Rationale: A child with croup typically develops a bark-like cough often at night. This may be accompanied by inspiratory stridor and suprasternal retractions. Temperature may be normal or slightly elevated. A high fever, dysphagia, and toxic appearance are associated with epiglottitis.

17. Which of the following would the nurse include when teaching parents how to prevent otitis externa? A) Daily ear cleaning with cotton swabs B) Wearing ear plugs when swimming C) Using a hair dryer on high to dry the ear canals D) Using hydrogen peroxide to dry the canal skin

B) Wearing ear plugs when swimming To prevent otitis externa, the nurse would teach parents and children to wear earplugs when swimming and to avoid use of cotton swabs, headphones, and earphones. A hair dryer on a low setting can be used to dry the ear canals. A mixture of half rubbing alcohol and half vinegar can be used to dry the canal and alter the pH to discourage organism growth.

The nurse is inspecting the fingernails of an 18-month-old girl. What finding indicates chronic hypoxemia? A)Nails that curve inward B)Clubbing of the nails C)Nails that curve outward D)Dry, brittle nails

B)Clubbing of the nails Clubbing of the nails indicates chronic hypoxemia related to either respiratory or cardiac disease. Nails that curve inward or outward may be hereditary or linked with injury, infection, or iron-deficiency anemia. Dry, brittle nails may indicate a nutritional deficiency.

While auscultating the heart of a 5-year-old child, the nurse notes a murmur that is soft and quiet and heard each time the heart is auscultated. The nurse documents this finding as what grade? A)Grade 1 B)Grade 2 C)Grade 3 D)Grade 4

B)Grade 2 A grade 2 murmur is soft and quiet and is heard each time the chest is auscultated. A grade 1 murmur is barely audible and is heard at some times and not at other times. A grade 3 murmur is audible with intermediate intensity. A grade 4 murmur is audible and accompanied by a palpable thrill.

A group of nursing students are reviewing information about inflammatory bowel disease in preparation for a class discussion on the topic. The students demonstrate understanding of the material when they identify which characteristics of Crohn disease? Select all that apply. A)Distributed in a continuous fashion B)Most common between the ages of 10 to 20 years C)Elevated erythrocyte sedimentation rate D)Low serum iron levels E)Tenesmus F)Loss of haustra within bowel

B, C, D Crohn disease is most common between the ages of 10 and 20 years. Erythrocyte sedimentation rate is elevated and serum iron levels are low. Ulcerative colitis is distributed continuously distal to proximal, with tenesmus and loss of haustra within the bowel. Crohn disease is segmental, with disease-free skip areas common, and the bowel wall has a cobblestone appearance.

After teaching a group of students about acute rheumatic fever, the instructor determines that the teaching was successful when the students identify which assessment finding? A)Janeway lesions B)Jerky movements of the face and upper extremities C)Black lines D)Osler nodes

B. Jerky movements of the face and upper extremities Sydenham chorea is a movement disorder of the face and upper extremities associated with acute rheumatic fever. Janeway lesions, black lines, and Osler nodes are associated with infective endocarditis.

13. A school-age child diagnosed with depression is receiving antidepressant therapy. What behavior would the nurse instruct the parents to watch for and to notify the physician immediately if the child demonstrates it? A) Loss of interest B) Gastric upset C) Sedation D) Urinary retention

Ans: A Feedback: Children taking antidepressants are at risk for the development of presuicidal behavior, which may be indicated by a loss of interest or pleasure. Gastric upset, sedation, and urinary retention may or may not occur, but none of these would be as important to report as the potential for self-harm.

The nurse is developing a plan of care for an infant with heart failure who is receiving digoxin. The nurse would hold the dose of digoxin and notify the physician if the infant's apical pulse rate was: A)140 beats per minute. B)120 beats per minute. C)100 beats per minute. D)80 beats per minute.

D. 80 bpm In an infant, if the apical pulse rate is less than 90 beats per minute, the dose is held and the physician should be notified.

The mother of a 14-year-old girl complains to the nurse that her daughter is moody, shuts herself in her room, and fights with her younger sister. Which comment is most valuable to the mother? A) "Calmly talk to her about your concerns." B) "This is normal for her age." C) "She may be hanging with a bad crowd." D) "Set some rules for family etiquette."

A

The nurse is describing the maturation of various organ systems during toddlerhood to the parents. What would the nurse correctly include in this description? A)Myelinization of the brain and spinal cord is complete at about 24 months. B)Alveoli reach adult numbers by 3 years of age. C)Urine output in a toddler typically averages approximately 30 mL/hour. D)Toddlers typically have strong abdominal muscles by the age of 2.

A)Myelinization of the brain and spinal cord is complete at about 24 months.

The nurse is reviewing the laboratory test results of a child who is suspected of having systemic lupus erythematosus (SLE). What would the nurse identify as supporting this diagnosis? Select all that apply. A)Positive antinuclear antibody (ANA) B)Increased C3 levels C)Thrombocytopenia D)Leukopenia E)Increased hematocrit

A, C, D Laboratory findings may include decreased hemoglobin and hematocrit, decreased platelet count, and low white blood cell count. Complement levels, C3 and C4, will also be decreased. Though not specific to SLE, the ANA is usually positive in children with SLE.

Origin: Chapter 14, 2 2. The nurse is managing children who have chronic diseases in a neighborhood clinic. What are some examples of chronic conditions? Select all that apply. A) Diabetes mellitus B) Myocardial infarction C) Rheumatoid arthritis D) Compound fracture E) Acute asthma F) Bronchopneumonia

Ans: A, C, E Feedback: Chronic pain is defined as pain that continues past the expected point of healing for injured tissue. Diabetes, arthritis, and asthma are examples of chronic pain. Acute pain is defined as pain that is associated with a rapid onset of varying intensity. It usually indicates tissue damage and resolves with healing of the injury. Examples include heart attack, fractures, and bronchopneumonia.

Origin: Chapter 5, 20 20. The nurse is counseling parents of a picky eater on how to promote healthy eating habits in their child. Which intervention would be appropriate advice? A) Allow the child to pick out his or her own foods for meals. B) Present the food matter-of-factly and allow the child to choose what to eat. C) Offer high-fat snacks if the child does not eat to get them to eat something. D) Offer the child a special treat if he or she eats all the food on the plate.

Ans: B Feedback: The parents should maintain a matter-of-fact approach, offer the meal or snack, and then allow the child to decide how much of the food, if any, he or she is going to eat. High-fat, nutrient-poor snacks should not be substituted for healthy foods just to coax the child to "eat something." If the preschooler is growing well, then the pickiness is not a cause for concern. A larger concern may be the negative relationship that can develop between the parent and child relating to mealtime. The more the parent coaxes, cajoles, bribes, and threatens, the less likely the child is to try new foods or even eat the ones he or she likes that are served. The child should be offered a healthy diet, with foods from all groups over the course of the day as recommended by the U.S. Department of Agriculture.

18. The nurse is reviewing the medical record of a child who has dyspraxia. This child will experience difficulty with: A) reading and writing. B) mathematics and computation. C) manual dexterity and coordination. D) composition and spelling.

Ans: C Feedback: Dyspraxia refers to problems with manual dexterity and coordination. Dyslexia involves difficulty with reading, writing, and spelling. Dyscalculia involves problems with mathematics and computation. Dysgraphia involves difficulty producing the written word in composition, spelling, and writing.

What finding would lead the nurse to suspect that a child has Turner syndrome? A) Webbed neck B) Microcephaly C) Gynecomastia D) Cognitive delay

Ans: A Manifestations of Turner syndrome include webbed neck, low posterior hairline, wide-spaced nipples, edema of the hands and feet, amenorrhea, and absence of secondary sex characteristics, along with short stature and slow growth. Microcephaly is commonly associated with trisomy 13. Gynecomastia and cognitive delay are associated with Klinefelter syndrome.

10.The nurse is implementing care for a hospitalized toddler. What communication technique would the nurse use with the child to reflect the child's developmental level? A) Allow the child extra time to complete thoughts. B) Communicate solely through play. C) Provide simple but honest and straightforward responses. D) Remain nonjudgmental to avoid alienation.

Ans: A Feedback: When working with toddlers and preschoolers, the nurse should allow them time to complete their thoughts. Though language acquisition at this age is exponential, it often takes longer for the young child to find the right words, particularly in response to a query. Infants communicate nonverbally and often through play. School-age children need simple but honest and straightforward responses, and nurses should be nonjudgmental with adolescents to avoid alienating them and to keep lines of communication open.

12.A 2-week-old child responds to a bell during an initial health supervision examination. The child's records do not show that a newborn hearing screening was done. Which is the best action for the nurse to take? A) Do nothing because responding to the bell proves he does not have a hearing deficit. B) Immediately schedule the infant for a newborn hearing screening. C) Ask the mother to observe for signs that the infant is not hearing well. D) Screen again with the bell at the 2-month-old health supervision visit.

Ans: B Feedback: Guidelines for infant hearing screening recommend universal screening with an auditory brainstem response (ABR) or evoked otoacoustic emissions (EOAE) test by 1 month of age. All the other answers rely on behavioral observation. Studies have shown that behavioral observations are not a reliable method of screening for hearing loss.

A child is diagnosed with cri-du-chat syndrome. What findings would the nurse expect to assess? Select all answers that apply. A) Hypertonia B) Short stature C) Simian crease D) Wide and flat nasal bridge E) Hydrocephaly

Ans: B, C, D: Manifestations of cri-du-chat syndrome include hypotonia, short stature, microcephaly, moon-like round face, bilateral epicanthal folds, wide and flat nasal bridge, and simian crease.

The nurse is preparing to administer an intramuscular injection to an 8-month-old infant. Which site would the nurse select? A)Rectus femoris B)Vastus lateralis C)Dorsogluteal muscle D)Deltoid

Ans:B The preferred injection site in infants is the vastus lateralis muscle. An alternative site is the rectus femoris. The dorsogluteal site is not used in children until the child has been walking for at least 1 year. The deltoid muscle is used as a site in children after the age of 4 or 5 years.

7. The nurse is examining a 7-year-old boy with blepharitis. Which of the following would the nurse least likely expect to assess? A) Redness B) Scaling C) Pain D) Edema

C) Pain Blepharitis has symptoms of redness, scaling, and edema, but not pain. Pain is typically associated with hordeolum.

A child has undergone surgery using steel bar placement to correct pectus excavatum. What position would the nurse instruct the parents to avoid? A)Semi-Fowler B)Supine C)High Fowler D)Side-lying

D. Side-lying After surgery to correct pectus excavatum, the nurse would instruct the parents to avoid positioning the child on either side because this could disrupt the bar's position. Semi- or high Fowler's position and the supine position would be appropriate.

The nurse is helping the parents and their underweight adolescent collaborate on planning a healthy menu. Of which nutritional requirement of adolescents should the nurse be aware? A) Teenagers have a need for increased calories, zinc, calcium, and iron for growth. B) Teenage girls who are active require about 1,800 calories per day. C) Teenage boys who are active require between 2,000 and 2,500 calories per day. D) Adolescents require about 1,000 to 1,200 mg of calcium each day.

A

When providing care to a child with aplastic anemia, which nursing diagnosis would be the priority? A)Risk for injury B)Imbalanced nutrition, less than body requirements C)Ineffective tissue perfusion D)Impaired gas exchange

A. Risk for injury For the child with aplastic anemia, safety is of the utmost concern, with injury prevention essential to prevent hemorrhage. Nutrition, tissue perfusion, and gas exchange may or may not be associated with the child's condition.

22. When preparing to apply a restraint to a child, what would be most important for the nurse to do? A) Expect to keep the restraint on for at least 8 hours. B) Explain that safety, not punishment, is the reason for the restraint. C) Plan to use a square knot to secure the restraint to the side rails. D) Use a limb restraint rather than a jacket restraint for most issues.

Ans: B Feedback: Before applying a restraint, the nurse needs to explain the reason for the restraint to the child, emphasizing that the restraint is for safety, not to punish the child. The least restrictive type of restraint should be used, and it should be applied for the shortest time necessary. A clove-hitch knot is used to secure the restraint with ties to the bed or crib frame, not the side rails.

Origin: Chapter 6, 15 15. The nurse is performing a physical examination of an 11-year-old girl. What observations would be expected? A) The child has not gained weight since last year. B) The child has grown 2.5 inches since last year. C) The child breathes abdominally. D) The child's third molars are about to erupt.

Ans: B Feedback: From 6 to 12 years of age, children grow an average of 2.5 inches (6 to 7 cm) per year, increasing their height by at least 1 foot. An increase of 7 pounds (3 to 3.5 kg) per year in weight is expected. Abdominal breathing is typical of a preschooler and would have disappeared several years earlier. The third molars do not erupt until late adolescence.

13. A child with growth hormone deficiency is receiving growth hormone. What result would the nurse interpret as indicating effectiveness of this therapy? A) Rapid weight gain B) Complaints of headaches C) Height increase of 4 inches D) Growth plate closure

Ans: C Feedback: Effectiveness of growth hormone therapy is indicated by at least a 3- to 5-inch increase in linear growth in the first year of treatment. Rapid weight gain and headaches are adverse reactions of this therapy. The drug is stopped when the epiphyseal growth plates close.

The nurse is assessing a child with a possible fracture. What would the nurse identify as the most reliable indicator? A)Lack of spontaneous movement B)Point tenderness C)Bruising D)Inability to bear weight

B. point tenderness Point tenderness is one of the most reliable indicators of a fracture in a child. Neglect of an extremity, inability to bear weight, bruising, erythema, and pain may be present, but these findings can also suggest other conditions.

A child with systemic lupus erythematosus is receiving high-dose corticosteroid therapy over the long term. The nurse would instruct the parents and child to report: A)difficulty urinating. B)visual changes. C)joint pain. D)rash.

C. Joint pain Avascular necrosis (lack of blood supply to a joint, resulting in tissue damage) may occur as an adverse effect of long-term or high-dose corticosteroid use. Teach families to report new onset of joint pain, particularly with weight bearing, or limited range of motion. Complications of systemic lupus erythematosus include nephritis manifested by urinary changes and visual changes. Rash may develop secondary to photosensitivity. These are unrelated to the long-term or high-dose corticosteroid use.

Origin: Chapter 5, 21 21. Which food suggestion would be most appropriate for the mother of a preschooler to ensure an adequate intake of calcium? A) Spinach B) White beans C) Enriched bread D) Fortified cereal

Ans: B Feedback: To ensure an adequate intake of calcium, the nurse should suggest white beans, because 1 ounce of dried white beans when cooked provides 160 mg of calcium. Spinach, enriched bread, and fortified cereal are good sources of iron.

During a health check-up without his parents, a 17-year-old tells the nurse he is gay. Which approach should the nurse take? A) "Tell me what makes you think you are gay." B) "This puts you in an at-risk category." C) "We need to talk about safe sex." D) "You're not gay; you're confused."

A

23. An infant is diagnosed with a congenital cataract. Which of the following would the nurse expect to assess? A) Absent red reflex B) Rapid irregular eye movement C) Misalignment of the eyes D) Enlarged eye appearance

A) Absent red reflex Assessment findings associated with congenital cataract include a history of lack of visual awareness; clouding of the cornea, which may or may not be visible; and no red reflex. Rapid irregular eye movement would suggest nystagmus. Misalignment of the eyes would suggest strabismus. Enlarged appearance of the eye is associated with infantile glaucoma.

Auscultation of a child's heart reveals a loud murmur with a precordial thrill. The nurse documents this as which grade? A)Grade II B)Grade III C)Grade IV D)Grade V

C. Grade IV A grade IV murmur is loud with a precordial thrill. A grade II murmur is soft and easily heard. A grade III murmur is characterized as loud without a thrill. A grade V murmur is characterized as loud, audible without a stethoscope.

12. A parent asks the nurse about immunizing her 7-month-old daughter against the flu. Which response by the nurse would be most appropriate? A. "She really doesn't need the vaccine until she reaches 1 year of age." B. "She will probably receive it the next time she is to get her routine shots." C. "Since your daughter is older than 6 months, she should get the vaccine every year." D. "The vaccine has many side effects, so she wouldn't get it until she's ready to go to school."

Answer: C Rationale: The current recommendations are for all children older than 6 months of age to be immunized yearly against influenza.

After assessing a 10-year-old girl, the nurse documents the appearance of breast buds, identifying this as what body change? A) Menarche B) Thelarche C) Puberty D) Tanner stage 5

B

The nurse is performing risk assessments on adolescents in the school setting. Which teen should the nurse screen for hypertension? A) An Asian female B) A white male C) An African American male D) A Jewish male

C

A group of nursing students are reviewing information about humoral and cellular immunity. The students demonstrate understanding of this material when they identify what as being involved in cellular immunity? A)B cells B)Antibodies C)Antigens D)T cells

D. T Cells Cellular immunity involves T cells, which do not recognize antigens. B cells, antibodies, and antigens are involved in humoral immunity.

A 4-year-old boy has a history of seizures and has been started on a ketogenic diet. Which food selection would be most appropriate for his lunch? A) Fried eggs, bacon, and iced tea B) A hamburger on a bun, French fries, and milk C) Spaghetti with meatballs, garlic bread, and a cola drink D) A grilled cheese sandwich, potato chips, and a milkshake

Fried eggs, bacon, and iced tea The ketogenic diet involves a high intake of fats, adequate protein intake, and a very low intake of carbohydrates, resulting in a state of ketosis. The child is kept in a mild state of dehydration. Eggs and bacon are high in fat; the tea does not contain any carbohydrates. Therefore, this is the best choice. The hamburger is fat and protein, the bun is a carbohydrate, and the French fries and the milk both contain fat and protein, but both contain a lot of carbohydrates. The pasta and the sauce for the spaghetti are carbohydrates, the meatballs are protein, and the garlic bread is a carbohydrate, as is the cola drink. The grilled cheese sandwich has the fat and protein from the cheese, but the bread and chips are primarily carbohydrates, and the milkshake has fat, protein, and carbohydrates. Only the selection in A contains a ketogenic meal.

The nurse knows that children have larger heads in relation to the body and a higher center of gravity. When developing a teaching plan for parents, the nurse includes information about an increased risk for which problem? A) Febrile seizures B) Head trauma C) Caput succedaneum D) Posterior plagiocephaly

Head trauma The larger head size in relation to the body, coupled with a higher center of gravity, causes children to hit their head more readily when involved in motor vehicle accidents, bicycle accidents, and falls. Febrile seizures are not related to anatomy or physiology. Caput succedaneum is an edematous area on the scalp caused by pressure of the uterus or vagina during head-first delivery. Posterior plagiocephaly is caused by early closure of the lamboid suture.

A group of students are reviewing information about head injuries in children. The students demonstrate understanding of this information when they identify what as the most common type of skull fracture in children? A) Linear B) Depressed C) Diastatic D) Basilar

Linear The most common type of skull fracture in children is a linear skull fracture, which can result from minor head injuries. Other, less common types of skull fractures in children include depressed, diastatic, and basilar.

Hydrocephalus is suspected in a 4-month-old infant. Which would the nurse expect to assess? A) Sunken fontanels B) Diminished reflexes C) Lower extremity spasticity D) Skull symmetry

Lower extremity spasticity Hydrocephalus is manifested by spasticity of lower extremities, bulging fontanels, brisk reflexes, and skull asymmetry.

A child with increased intracranial pressure is being treated with hyperventilation. The nurse understands the need for this treatment is based on what? A) PaCO2 levels decrease, causing vasoconstriction. B) Drainage of cerebrospinal fluid occurs. C) Activity is controlled via a stimulator. D) Hyperexcitability of the nerves is reduced.

PaCO2 levels decrease, causing vasoconstriction. Hyperventilation decreases PaCO2, which results in vasoconstriction and therefore decreases intracranial pressure. A shunt would allow for drainage of cerebrospinal fluid. A vagal nerve stimulator is used to provide an appropriate dose of stimulation to manage seizure activity. Anticonvulsants decrease the hyperexcitability of nerves.

What finding would lead the nurse to suspect that a child is beginning to develop increased intracranial pressure? A) Bradycardia B) Cheyne-Stokes respirations C) Fixed, dilated pupils D) Projectile vomiting

Projectile vomiting Projectile vomiting is an early sign of increased intracranial pressure. Bradycardia, Cheyne-Stokes respirations, and fixed dilated pupils are late signs of increased intracranial pressure.

A 4-year-old boy has a febrile seizure during a well-child visit. What action would be a priority? A) Hyperextending the child's head while placing him on his side B) Using a tongue blade to pry open the child's jaw C) Loosening the child's clothing to ensure a patent airway D) Protecting the child from harm during the seizure

Protecting the child from harm during the seizure During a seizure, the child should not be held down in a specific position. Protecting the child's head and body during the seizure is the priority. Ensuring a patent airway is an important intervention but is not accomplished by loosening the child's clothing or hyperextending his head. The child should be placed on his side and nothing should be inserted into his mouth to forcibly open the jaw.

A 16-year-old boy reports to the school nurse of headaches and a stiff neck. Which sign or symptom would alert the nurse that the child may have bacterial meningitis? A) Fixed and dilated pupils B) Frequent urination C) Sunset eyes D) Sunlight is "too bright"

Sunlight is "too bright" Photophobia, or intolerance of light, is another symptom of bacterial meningitis. Fixed and dilated pupils are a symptom of head trauma and warrant prompt intervention. Frequent urination is a symptom of a type I Arnold-Chiari malformation. Sunset eyes indicate increased intracranial pressure typical of hydrocephalus.

The nurse teaches parents of adolescents that adolescents need the support of parents and nurses to facilitate healthy lifestyles. What should be a priority focus of this guidance? A) Reducing risk-taking behavior B) Promoting adequate physical growth C) Maximizing learning potential D) Teaching personal hygiene routines

A

The school nurse is conducting a seminar for parents of adolescents on how to communicate with teenagers. Which guidelines might the nurse recommend? Select all that apply. A) Talk face to face and be aware of body language. B) Ask questions to see why he or she feels that way. C) Do not give praise unless the adolescent deserves it. D) Speak to your child as an authority figure, not an equal. E) Don't admit that you make mistakes. F) Don't pretend you know all the answers.

A, B, F

A 14-year-old boy is diagnosed with Hodgkin disease. When palpating for enlarged lymph nodes, the nurse would expect to find which nodes as most commonly enlarged? Select all that apply. A)Cervical B)Axillary C)Supraclavicular D)Occipital E)Inguinal

A, C Enlarged lymph nodes may feel rubbery and tend to occur in clusters. Although any lymph nodes may be involved, the lymph nodes most commonly affected are in the cervical and supraclavicular areas.

The nurse is providing care to a child with pancreatitis. When reviewing the child's laboratory test results, what would the nurse expect to find? Select all that apply. A)Leukocytosis B)Decreased C-reactive protein C)Elevated serum amylase levels D)Positive stool culture E)Decreased serum lipase levels

A, C With pancreatitis, serum amylase and lipase levels are elevated and levels three times the normal values are extremely indicative of pancreatitis. Leukocytosis is common with acute pancreatitis. C-reactive protein levels may be elevated. Stool cultures are not used to evaluate this disorder. Positive stool cultures would indicate a bacterial cause of diarrhea.

A child is diagnosed with Kawasaki disease and is in the acute phase of the disorder. What would the nurse expect the physician to prescribe? Select all that apply. A)Intravenous immunoglobulin B)Ibuprofen C)Acetaminophen D)Aspirin E)Alprostadil

A, C, D In the acute phase, high-dose aspirin in four divided doses daily and a single infusion of intravenous immunoglobulin are used. Acetaminophen is used to reduce fever. Nonsteroidal anti-inflammatory agents such as ibuprofen are avoided while the child is receiving aspirin therapy. Alprostadil is used to temporarily keep the ductus arteriosus patent in infants with ductal-dependent congenital heart defects.

The parents bring their 4-year-old son to the emergency department. The child is receiving chemotherapy for acute lymphoblastic leukemia. The parents report that the child has become lethargic and has had significant episodes of vomiting and diarrhea. Why findings would lead the nurse to suspect the child may be experiencing tumor lysis syndrome? Select all answers that apply. A)Hyperkalemia B)Hypophosphatemia C)Polyuria D)Hypocalcemia E)Hyperuricemia

A, D, E Tumor lysis syndrome is characterized by hyperuricemia, hyperkalemia, hyperphosphatemia, decreased or absent urine output, and hypocalcemia

The nurse is administering intravenous immune globulin (IVIG). The nurse assesses vital signs and for adverse reactions every 15 minutes for the first hour of administration. After the first hour, the nurse most likely would continue to assess the child at which frequency? A)Every 30 minutes B)Every 45 minutes C)Every 60 minutes D)Every 2 hours

A. Every 30 mins The nurse needs to continue assessments according to institutional protocol. Every 15 minutes for the first hour and every 30 minutes through the remainder of the infusion is the standard assessment.

After assessing a child's blood pressure, the nurse determines the pulse pressure and finds that it is narrowed. What would the nurse identify as associated with this finding? A)Aortic stenosis B)Patent ductus arteriosus C)Aortic insufficiency D)Complete heart block

A. Aortic stenosis A narrowed pulse pressure is associated with aortic stenosis. A widened pulse pressure is associated with patent ductus arteriosus, aortic insufficiency, fever, anemia, or complete heart block.

27. A child has been prescribed growth hormone. When collecting data from this patient, which report is of the greatest concern? A) "I sometimes have headaches." B) "I feel tired." C) "My hips often hurt." D) "I take this medication with food."

Ans: C Feedback: Limping or complaints of hip pain are of concern. This may signal issues with the epiphysis and warrants further evaluation. Headaches and fatigue are not associated with medication. Taking this medication with food is not contraindicated.

23. The nurse is administering 10 units of NPH insulin to a child at 8 a.m. The nurse would expect this insulin to begin acting at which time? A) By 8:15 a.m. B) Between 8:30 and 9 a.m. C) Between 9 and 11 a.m. D) Around 12 noon

Ans: C Feedback: NPH insulin has an onset of action of 1 to 3 hours, so the drug would begin to act between 9 and 11 a.m. A rapid-acting insulin would begin to act by 8:15 a.m.; regular insulin would begin to act between 8:30 and 9 a.m. No type of insulin would begin acting around 12 noon.

Origin: Chapter 6, 9 9. The nurse is assessing the gross motor skills of an 8-year-old boy. Which of interview question would facilitate this assessment? A) 'Do you like to do puzzles?' B) 'Do play any instruments?' C) 'Do you participate in any sports?' D) 'Do you like to construct models?'

Ans: C Feedback: To assess the gross motor skills of school-age children, the nurse should ask questions about participation in sports and after-school activities. For fine motor skills, the nurse could ask questions about band membership, constructing models, and writing skills.

1. The nurse is obtaining a health history from parents whose 4-month-old boy has congenital hypothyroidism. What would the nurse most likely assess? A) The child has above-normal growth for his age. B) The child is active and playful. C) The skin is pink and healthy looking. D) It is difficult to keep the child awake.

Ans: D Feedback: The parents may state, during the health history, that it is difficult to keep the child awake. Physical examination would reveal that the child is below weight and height, that his skin is pale and mottled, and that he is lethargic and irritable.

26. A child is prescribed trazodone. What would the nurse be least likely to include in the plan of care related to this drug? A) Monitoring blood pressure for orthostatic hypotension B) Assessing the child for sedation and drowsiness C) Administering the drug with a snack D) Monitoring for tardive dyskinesia

Ans: D Feedback: Trazodone does not cause tardive dyskinesia; antipsychotics do. It can cause orthostatic hypotension, sedation, and drowsiness. It should be given after meals or with snacks to minimize gastrointestinal upset.

20. The nurse is transporting a 6-month-old with a suspected blood disorder to the nursery. What is the most appropriate method of transporting the child by the nurse? A) A wagon with rails B) Cradle hold C) Football hold D) Over-the-shoulder

Ans: D Feedback: A 6-month-old should be carried in the "over-the-shoulder" method. A wagon with rails is for an older child. A cradle hold is for infants until 3 months of age. A football hold is for infants until 2 months of age.

Origin: Chapter 5, 25 25. The nurse is providing teaching about car safety to the parents of a 5-year-old girl who weighs 45 pounds. What should the nurse instruct the parents to do? A) "Place her in a booster seat with lap and shoulder belts in the front seat." B) "Place her in the back seat with the lap and shoulder belts in place." C) "Place her in a forward-facing car seat with a harness and top tether." D) "Place her in a booster seat with lap and shoulder belts in the back seat."

Ans: D Feedback: A child who weighs between 40 and 80 pounds should ride in a booster seat that utilizes both the lap and shoulder belts in the back seat. When a child is large enough to sit up straight with the knees bent at the front edge of the seat, then he or she may sit directly on the seat of the car with lap/shoulder belt securely and appropriately attached. The back seat of the car is the safest place for a child to ride. A forward-facing car seat with harness and top tether is for a preschooler who weighs less than 40 pounds.

The nurse is providing care to a child with an intussusception. The child has a bowel movement and the nurse inspects the stool. The nurse would most likely document the stool's appearance as having what quality? A)Greasy B)Clay-colored C)Currant jelly-like D)Bloody

C. Currant jelly-like The child with intussusception often exhibits currant jelly-like stools that may or may not be positive for blood. Greasy stools are associated with celiac disease. Clay-colored stools are observed with biliary atresia. Bloody stools can be seen with several gastrointestinal disorders, such as inflammatory bowel disease.

The nurse is reviewing the laboratory test results of a child with nephrotic syndrome. What would the nurse least likely expect to find? A)Hyperlipidemia B)Hypoalbuminemia C)Decreased blood urea nitrogen (BUN) D)Hypoproteinemia

C. Decreased blood urea nitrogen (BUN) With nephrotic syndrome, proteinuria, hyperlipidemia, decreased serum protein levels (hypoproteinemia), and decreased serum albumin levels (hypoalbuminemia) are present. BUN typically becomes elevated.

A group of nursing students are reviewing information about the type of skin and skeletal traction. The students demonstrate understanding of this information when they identify which of these as a type of skeletal traction? A)Russell traction B)Bryant traction C)Buck traction D)Side arm 90-90 traction

D. Side arm 90-90 traction Side arm 90-90 traction is a type of skeletal traction with force applied through a pin in the distal femur. Russell traction, Bryant traction, and Buck traction are types of skin traction.

The nurse is examining the posture of a male toddler and notes lordosis. What would be the appropriate reaction of the nurse to this finding? A)Explain that the child will need a back brace. B)Refer the toddler to a physical therapist. C)Do nothing; this is a normal condition for toddlers. D)Notify the primary care physician about the condition.

C)Do nothing; this is a normal condition for toddlers. The toddler demonstrates lordosis (swayback) and bowlegs, with a relatively large head and protuberant belly. This is a normal condition and requires no further attention.

The nurse is screening a 6-year-old child for mental ability. Which test would the nurse use to assess intelligence? A)Denver Articulation Screening B)Denver PRQ C)Goodenough-Harris Drawing Test D)Parents' Evaluation of Developmental Status (PEDS)

C)Goodenough-Harris Drawing Test The Goodenough-Harris Drawing Test is a nonverbal screen for mental ability (intelligence). The Denver Articulation Screening screens for articulation disorders. The Denver PRQ assesses personal-social, fine motor-adaptive, language, and gross motor skills. The PEDS screens for a wide range of developmental, behavioral, and family issues.

The nurse is assessing a 3-year-old boy's development during a well-child visit. Which response by the child indicates the need for further assessment? A)He says a swear word when he hurts himself playing. B)He says "pew" when his sister has soiled her diaper. C)He laughs when his brother cries getting vaccinated. D)He constantly asks "why?" whenever he is told a fact.

C)He laughs when his brother cries getting vaccinated. Laughing when his brother cries when being vaccinated indicates that the child hasn't yet developed a sense of empathy or that there may be psychosocial issues, such as sibling rivalry, that should be assessed. The child may repeat a word even if it is out of context. This is called echolalia. Older toddlers have a well-developed sense of smell and will comment if they don't like a smell. The incessant "why" is very common to toddlers' speech.

The nurse is assessing the neck of an 8-year-old child with Down syndrome. Which finding would the nurse expect during the examination? A)Webbing B)Excessive neck skin C)Lax neck skin D)Shortened neck

C)Lax neck skin Lax neck skin may occur with Down syndrome. Webbing or excessive neck skin folds may be associated with Turner syndrome. A shortened neck is expected in a child younger than age 4.

The nurse is caring for children in a physician's office where health supervision is practiced. Which are some points of focus of health supervision? Select all that apply. A)Making referrals for all health care needs B)Monitoring disease incidence C)Optimizing the child's level of functioning D)Monitoring quality of care provided E)Teaching parents to prevent injury F)Providing care developed from national guidelines

C)Optimizing the child's level of functioning E)Teaching parents to prevent injury F)Providing care developed from national guidelines Health supervision involves providing services proactively, with the goal of optimizing the child's level of functioning. It ensures the child is growing and developing appropriately and it promotes the best possible health of the child by teaching parents and children about preventing injury and illness (e.g., proper immunizations and anticipatory guidance). The framework for the health supervision visit is developed from national guidelines available through the U.S. Department of Health and Human Services (DHHS), the American Medical Association (AMA), and the American Academy of Pediatrics (AAP). Making referrals and monitoring disease incidence and quality of care provided may occur with this model, but they are not key focal points.

A mother brings her 3-year-old daughter to the emergency department because the child has been vomiting and having diarrhea for the past 36 hours. When assessing this child's temperature, which method would be least appropriate? A)Oral B)Tympanic C)Rectal D)Axillary

C)Rectal Obtaining the child's temperature via the rectal route would be least appropriate because the child has diarrhea, and insertion of the thermometer might traumatize the rectal mucosa. Additionally, the rectal route is highly invasive and a child of this age fears body invasion. Using the oral route might be problematic due to the child's age and inability to cooperate, especially in light of the child's vomiting. However, it would not be as dangerous as obtaining a rectal temperature. The tympanic or axillary method would be the most appropriate method.

The pediatric nurse is planning quiet activities for hospitalized 18-month-old. What would be an appropriate activity for this age group? A)Painting by number B)Putting shapes into appropriate holes C)Stacking blocks D)Using crayons to color in a coloring book

C)Stacking blocks At 18 months the child can stack four blocks. The 24-month-old can paint (but not by number), scribble, and color, and put round pegs into holes.

The nurse is providing guidance after observing a mother interact with her negative 2-year-old boy. For which interaction will the nurse advise the mother that she is handling the negativism properly? A)Telling the child to stop tearing pages from magazines B)Asking the child if he would please quit throwing toys C)Telling the child firmly that we don't scream in the office D)Saying, "Please come over here and sit in this chair. OK?"

C)Telling the child firmly that we don't scream in the office Telling the child firmly that we don't scream in the office gets the point across to the child that his behavior is unacceptable while role modeling appropriate communication. Telling the child to stop tearing up magazines does not give him direction for appropriate behavior. Asking the child if he would quit throwing toys gives him an opportunity to say "no," and is the same as asking "OK?" at the end of a direction.

A child is brought to the emergency department after sustaining a concussion. The child is to be discharged home with his parents. What would the nurse include in the child's discharge instructions? A) "Expect his headache to get worse initially and then disappear." B) "Wake him every 2 hours to check his movement and responses." C) "Call your medical provider if he vomits more than five times." D) "Any watery fluid draining from his ears is normal."

"Wake him every 2 hours to check his movement and responses." The nurse should instruct the parents to wake the child every 2 hours to ensure that he moves normally and wakes enough to recognize and respond appropriately to them. The parents should be instructed to call the physician or nurse practitioner or bring the child back to the emergency department if he experiences a constant headache that gets worse, vomits more than two times, or has oozing of blood or watery fluid from his ears or nose.

The nurse is caring for a child with widespread itching and has recommended bathing as a relief measure. After teaching the mother about this, which statement from the mother indicates a need for further instruction? A) "After bathing, I need to rub his skin everywhere to make sure he is completely dry." B) "I must make sure I use lukewarm water instead of hot water." C) "Oatmeal baths are helpful; we can add Aveeno skin relief bath treatment." D) "We should leave his skin moist before applying medication or moisturizer."

A) "After bathing, I need to rub his skin everywhere to make sure he is completely dry." Feedback: The nurse needs to emphasize to the mother that she must only pat the child dry and not rub his skin. Rubbing can cause further itching. Additionally, the skin should be left moist prior to applying medication or moisturizer. Lukewarm water and oatmeal baths are appropriate.

16. The mother of a school-age child brings the child to the clinic for evaluation because he is having difficulty reading. His last visual screening was normal. He also complains of headaches and dizziness. Which of the following would the nurse suspect? A) Astigmatism B) Myopia C) Hyperopia D) Nystagmus

A) Astigmatism Children with astigmatism often have blurry vision and difficulty seeing letters as a whole, affecting their reading ability. They may have headaches and dizziness and often learn to tilt their heads slightly so that they can focus more effectively (which leads to normal vision screenings). Children with myopia can see well at close range but have difficulty focusing well on the blackboard or other objects at a distance. Hyperopia is characterized by blurriness at close range, with the ability to see at a distance. Nystagmus is manifested by a very rapid irregular eye movement.

3. The nurse is caring for a 10-year-old with allergic conjunctivitis. The nurse would be alert to the child's increased risk for which of the following? A) Atopic dermatitis B) Insect bite sensitivity C) Acute otitis media D) Frequent sore throats

A) Atopic dermatitis Atopic dermatitis is a risk factor specifically for allergic conjunctivitis because of repeated exposure to the particular allergens. Acute otitis media, insect bite sensitivity, and frequent sore throats can occur but are not related to the allergic conjunctivitis.

21. A nurse is developing a plan of care for a child who is admitted to the hospital for surgery. The child is visually impaired. Which of the following would be most appropriate for the nurse to include in the child's plan of care? Select all answers that apply. A) Explaining instructions using simple and specific terms the child understands B) Allowing the child to explore the postoperative equipment with his hands C) Touching the child on his shoulder before letting the child know someone is there D) Using the child's body parts to refer to the area where he may have postoperative pain E) Speaking to the child in a voice that is slightly louder than the usual tone of voice

A) Explaining instructions using simple and specific terms the child understands B) Allowing the child to explore the postoperative equipment with his hands D) Using the child's body parts to refer to the area where he may have postoperative pain When interacting with a visually impaired child, the nurse would make directions and instructions simple and specific, encourage exploration of objects such as postoperative equipment through touch, and use the parts of the child's body as reference points for the location of items or for this child, his postoperative pain. The nurse should identify him- or herself first before touching the child and speak in a tone of voice that is appropriate to the situation.

A group of students are preparing for a class exam on skin disorders. As part of their preparation, they are reviewing information about acne vulgaris and its association with increased sebum production. The students demonstrate understanding of the information when they identify which areas as having the highest sebaceous gland activity? Select all that apply. A) Face B) Upper chest C) Neck D) Back E) Shoulders

A) Face B) Upper chest D) Back Feedback: The face, upper chest, and back are the areas of highest sebaceous activity and thus the most common areas for acne lesions to occur. The neck and shoulders are not typical areas involved with acne.

1. The nurse is developing a plan of care for a 5-year-old child with a severe hearing impairment focusing on psychosocial interventions based on assessment findings. Which behavior would the nurse have most likely assessed? A) Immature emotional behavior B) Self-stimulatory actions C) Inattention and vacant stare D) Head tilt or forward thrust

A) Immature emotional behavior Immature emotional behavior would be seen most frequently. The inability to hear impacts the socialization process and causes social problems for the child because the hearing impairment has inhibited normal development. Self-stimulatory actions, inattention, vacant stare, head tilt, or forward thrust may also cause problems with socialization, but they are typical of visually impaired children.

The nurse is providing care to a child with folliculitis. What would the nurse expect to administer? A) Topical mupirocin B) Oral cephalosporin C) Intravenous oxacillin D) Topical Eucerin cream

A) Topical mupirocin Feedback: For folliculitis, topical mupirocin is indicated in conjunction with aggressive hygiene and warm compresses. Oral cephalosporins are used for nonbullous impetigo if there are numerous lesions. Intravenous oxacillin is used for severe cases of staphylococcal scalded skin syndrome. Topical Eucerin cream is used for atopic dermatitis.

The nurse is conducting a routine health assessment of a 3-month-old boy and notices a flat occiput. The nurse provides teaching and emphasizes the importance of tummy time. Which response by the mother indicates a need for further teaching? A)"I should have him sleep on his tummy." B)"I need to watch him during his tummy time." C)"I need to change his head position while he is in an upright chair." D)"His head has flattened due to the pressure of his head position."

A)"I should have him sleep on his tummy." The nurse needs to emphasize that the boy must be observed and awake during the recommended "tummy time" and to remind the mother that the baby should still sleep on his back. The other statements are correct.

After teaching a group of parents about language development in toddlers, what if stated by a member of the group indicates successful teaching? A)"When my 3-year-old asks 'why?' all the time, this is completely normal." B)"A 15-month-old should be able to point to his eyes when asked to do so." C)"At age 2 years, my son should be able to understand things like under or on." D)"An 18-month-old would most likely use words and gestures to communicate."

A)"When my 3-year-old asks 'why?' all the time, this is completely normal." Language development occurs rapidly in a toddler. By age 3 years, the child asks "why?" Pointing to named body parts is characteristic of a 2-year-old. Understanding concepts such as on, under, or in is typical of a 3-year-old. A 1-year-old would communicate with words and gestures.

The nurse is interviewing a 3-year-old girl who tells the nurse: 'Want go potty.' The parents tell the nurse that their daughter often speaks in this type of broken speech. What would be the nurse's appropriate response to this concern? A)'This is a normal, common speech pattern in the 3-year-old and is called telegraphic speech.' B)'This is considered a developmental delay in the 3-year-old and we should consult a speech therapist.' C)'This is a condition known as echolalia and can be corrected if you work with your daughter on language skills.' D)'This is a condition known as stuttering and it is a normal pattern of speech development in the toddler.'

A)'This is a normal, common speech pattern in the 3-year-old and is called telegraphic speech.' Telegraphic speech is common in the 3-year-old. Telegraphic speech refers to speech that contains only the essential words to get the point across, much like a telegram. In telegraphic speech the nouns and verbs are present and are verbalized in the appropriate order. Echolalia (repetition of words and phrases without understanding) normally occurs in toddlers younger than 30 months of age. "Why" and "what" questions dominate the older toddler's language. Stuttering usually has its onset at between 2 and 4 years of age. It occurs more often in boys than in girls. About 75% of all cases of stuttering resolve within 1 to 2 years after they start.

The nurse is aware that the community affects the health of its members. Which statements accurately reflect a community influence of health care? Select all that apply. A)A community can be a contributor to a child's health or be the cause of his or her illnesses. B)The child's health should be separated from the health of the surrounding community. C)Community support and resources are necessary for children with significant problems. D)Poverty has not been linked to an increase in health problems in communities. E)The breakdown of community and family support systems can lead to depression and violence. F)Ideally, the child's medical home is located outside the community.

A)A community can be a contributor to a child's health or be the cause of his or her illnesses. C)Community support and resources are necessary for children with significant problems. E)The breakdown of community and family support systems can lead to depression and violence. A community can be a contributor to a child's health or be the cause of his or her illnesses. Community support and resources are necessary for children with significant problems since a close working relationship between the child's physician and community agencies is an enormous benefit to the child. Children from communities suffering the large-scale breakdown of family relationships and loss of support systems will be at increased risk for depression, violence and abuse, substance abuse, and HIV infection. The child's health cannot be totally separated from the health of the surrounding community. Poverty has been linked to low birthweight and premature birth, among other health problems. Ideally the child's medical home is within the family's community to reduce barriers such as lack of transportation, expense of travel, and time away from the parents' workplace.

When assessing the vision of a 2-month-old, what would the nurse use? A)Black-and-white checkerboard B)Red and blue circles C)Gray and blue animal drawings D)Green and yellow letters

A)Black-and-white checkerboard For infants younger than 6 months of age, objects such as a black-and-white checkerboard or concentric circles are best because an infant's vision is more attuned to these high-contrast patterns than to colors. High-contrast animal figures such as pandas or Dalmatians also work well.

During a physical assessment of a 5-month-old child, the nurse observes the first tooth has just erupted and uses the opportunity to advise the mother to schedule a dental examination for her baby. When is the correct time for the dentist visit? A)By the first birthday B)By the second birthday C)By entry into kindergarten D)By entry into first grade

A)By the first birthday The American Academy of Pediatric Dentistry recommends that a dentist examine the infant by his or her first birthday. Besides assessing routine oral health care, establishing a dental contact by the first birthday provides a resource for emergency dental care if it is needed.

The nurse performing a health history on a child asks the parents if their child has experienced increased appetite or thirst. What body system is the nurse assessing with this question? A)Endocrine B)Genitourinary C)Hematologic D)Neurologic

A)Endocrine Indicators of problems with the endocrine system include increased thirst, excessive appetite, delayed or early pubertal changes, and problems with growth. For the genitourinary system the nurse would assess urinary patterns and genitals. For the hematologic system the nurse would assess lymph nodes, skin color, and bruising. Signs of neurologic problems include numbness, tingling, difficulty learning, altered mood or ability to stay alert, tremors, tics, and seizures.

The nurse is conducting a psychosocial assessment of a child with asthma brought to the physician's office for a check-up. Which psychosocial issues may be assessed? Select all that apply. A)Health insurance coverage B)Transportation to health care facilities C)School's response to the chronic illness D)Past medical history E)Future treatment plans F)Health maintenance needs

A)Health insurance coverage B)Transportation to health care facilities C)School's response to the chronic illness Comprehensive health supervision includes frequent psychosocial assessments. Issues to be covered include health insurance coverage, transportation to health care facilities, financial stressors, family coping, and the school's response to the chronic illness. These are often stressful and emotionally charged issues. Past medical history, future treatment plans, and health maintenance needs would also be assessed; however, these are not psychosocial issues.

The nurse is teaching the student nurse the sequence for performing the assessment techniques during a physical examination. What is the appropriate order? A)Inspection, palpation, percussion, auscultation B)Inspection, percussion, palpation, auscultation C)Palpation, percussion, inspection, auscultation D)Inspection, auscultation, palpation, percussion

A)Inspection, palpation, percussion, auscultation The physical examination of children, just as for adults, begins with a systematic inspection: checking color, warmth, characteristics, and texture visually and smelling for any odor. Palpation follows inspection to validate observations. Next percussion is used to determine the location, size, and density of organs or masses. The stethoscope is used last to auscultate the heart, lungs, and abdomen.

The school nurse is teaching parents risk factors for suicide in adolescents. What would the nurse discuss? Select all that apply. A) Mental health changes B) History of previous suicide attempt C) Higher socioeconomic status D) Greatly improved school performance E) Family disorganization F) Substance abuse

A, B, E, F

The nurse working in a community clinic attempts to establish a free vaccination program to refer low-income families. What is the key strategy for success when implementing a health promotion activity? A)Partnership development B)Funding for projects C)Finding an audience D)Adequate staffing

A)Partnership development Partnership development is the key strategy for success when implementing a health promotion activity. Identifying key stakeholders from the community allows problems to be solved and provides additional venues for disseminating information. Funding, finding an audience, and staffing a project are elements of a public health promotion activity, but developing a partnership helps empower children and families at the individual and community levels to develop resources to optimize their health.

The nurse is preparing to assess the pulse of an 18-month-old child. Which pulse would be most difficult for the nurse to palpate? A)Radial B)Brachial C)Pedal D)Femoral

A)Radial In a child younger than 2 years of age, the radial pulse is very difficult to palpate, whereas the pedal, brachial, and femoral pulses are usually easily palpated.

The nurse is teaching the parents of an overweight 18-month-old girl about diet. Which intervention will be most effective for promoting proportionate growth? A)Remove high-calorie, low-nutrient foods from the diet. B)Ensure 30 minutes of unstructured activity per day. C)Avoid sharing your snacks and candy with the child. D)Reduce the amount of high-fat food the child eats.

A)Remove high-calorie, low-nutrient foods from the diet. The most effective intervention will be to remove high-calorie, low-nutrient foods from the diet in order to reduce the number of calories and increase the nutritional value. Exercise is also important, but a child this age should have 30 minutes of structured physical activity plus several hours of unstructured physical activity per day. The parents should set an example for good eating habits. Dietary fat should not be restricted for an 18-month-old child because it is necessary for nervous system development.

The nurse is examining a 15-month-old child who was able to walk at the last visit and now can no longer walk. What would be the nurse's best intervention in this case? A)Schedule a full evaluation since this may indicate a neurologic disorder. B)Note the regression in the child's chart and recheck in another month. C)Document the findings as a developmental delay since this is a normal occurrence. D)Ask the parents if they have changed the child's schedule to a less active one.

A)Schedule a full evaluation since this may indicate a neurologic disorder. Any child who "loses" a developmental milestone—for example, the child able to sit without support who now cannot—needs an immediate full evaluation, since this indicates a significant neurologic problem.

Which would be least effective in gaining the cooperation of a toddler during a physical examination? A)Tell the child that another child the same age wasn't afraid. B)Allow the child to touch and hold the equipment when possible. C)Permit the child to sit on the parent's lap during the examination. D)Offer immediate praise for holding still or doing what was asked.

A)Tell the child that another child the same age wasn't afraid. Toddlers are egocentric, and telling the toddler how well another child behaved or cooperated probably will not help gain this child's cooperation. Allowing the child to touch and hold the equipment, permitting the child to sit on the parent's lap during the exam, and offering praise immediately for cooperating would foster cooperation.

The parents of a 1-year-old girl, both of whom have perfect teeth, are concerned about their child getting dental caries. Which is the best advice the nurse can provide? A)Tell the parents to limit the child's eating to meal and snack times. B)Urge the parents to take the child to a dentist for a check-up. C)Advise the parents to reduce carbohydrates in the child's diet. D)Advise the parents to use fluoride toothpaste.

A)Tell the parents to limit the child's eating to meal and snack times. Telling the parents to limit eating to meal and snack times is the best advice for preventing dental caries. This reduces the amount of exposure the child's teeth have to food. Urging them to take the child to see a dentist is sound advice but doesn't suggest actions they can take now to prevent caries. Carbohydrates react with oral bacteria to cause caries, but they should not be reduced from the diet. Avoiding fluoridated toothpaste may help prevent fluorosis.

The nurse is questioning the parents of a 2-year-old child to obtain a functional history. Which topics might the nurse include? Select all answers that apply. A)The child's toileting habits B)Use of car seats and other safety measures C)Problems with growth and development D)Prenatal and perinatal history E)The child's race and ethnicity F)Use of supplements and vitamins

A)The child's toileting habits B)Use of car seats and other safety measures F)Use of supplements and vitamins The functional history should contain information about the child's daily routine, such as toileting habits, safety measures, and nutrition. Problems with growth and development would be covered in the developmental history. Prenatal and perinatal history is assessed in the past health history and the child's race and ethnicity is part of the demographics.

A mother of three brings her children in for their vaccinations. The mother tells the nurse that her mother recently died and her husband just lost his job due to his company downsizing. Which parenting behaviors is the nurse likely to observe? Select all that apply. A)The mother rarely looks at her infant when the nurse is assessing the child. B)The mother voices pride in the academic accomplishments of her 7-year-old child. C)The mother becomes very frustrated and tells the nurse she can't handle her toddler's temper tantrum. D)The mother asks if the nurse has suggestions on ways to potty train her toddler. E)The mother utilizes the correct size of infant car seat for her 3-month-old child.

A)The mother rarely looks at her infant when the nurse is assessing the child. C)The mother becomes very frustrated and tells the nurse she can't handle her toddler's temper tantrum. When the family is faced with excessive stressors, the nurse may be able to ascertain the stress by observing the parent-child interaction during the health supervision visit. The nurse can learn much about the family dynamic by observing the family for behavioral clues. Lack of eye contact and care of the infant is a clue to family stress, as well as effective parenting techniques for behaviors such as temper tantrums.

When assessing adolescents for health risks, the nurse must keep in mind the factors related to the prevalence of adolescent injuries. What accurately describes these factors? Select all that apply. A) Increased physical growth B) Insufficient psychomotor coordination C) Tiredness, lack of energy D) Lack of impulsivity E) Peer pressure F) Inexperience

A, B, E, F

The nurse observing toddlers in a day care center notes that they may be happy and pleasant one moment and overreact to limit setting the next minute by throwing a tantrum. What is the focus of the toddler's developmental task that is driving this behavior? A)The need for separation and control B)The need for love and belonging C)The need for safety and security D)The need for peer approval

A)The need for separation and control Emotional development in the toddler years is focused on separation and individuation. The focus in infancy is on love and belonging, and the need for peer approval occurs in the adolescent. Safety and security are concerns in all levels of development, but not the primary focus.

The nurse is watching toddlers at play. Which normal behavior would the nurse observe? A)Toddlers engage in parallel play. B)Toddlers engage in solitary play. C)Toddlers engage in cooperative play. D)Toddlers do not engage in play outside the home.

A)Toddlers engage in parallel play. Toddlers typically play alongside another child (parallel play) rather than cooperatively. Infants engage in solitary play.

The nurse is developing a plan of care for a child with thalassemia. What information would the nurse expect to include? Select all that apply. A)Packed RBC transfusions B)Deferoxamine therapy C)Heparin therapy D)Opioid analgesics E)Platelet transfusions F)Intravenous immunoglobulin

A, B RBC transfusions and deferoxamine for chelation are used to treat thalassemia. Heparin therapy is used for treating DIC. Opioid analgesics would be used to treat severe pain associated with sickle cell crisis. Platelet transfusions and intravenous immunoglobulin would be used to treat idiopathic thrombocytopenia purpura.

The nurse is reviewing the laboratory test results of a child who is receiving chemotherapy. To calculate the child's absolute neutrophil count, in addition to the total number of white blood cells, which results would the nurse use? Select all that apply. A)Bands B)Segs C)Eosinophils D)Basophils

A, B To calculate the absolute neutrophil count, the nurse would add together the percentage of banded and segmented neutrophils and then multiply the total number of white blood cells reported on the complete blood count by the sum.

The nurse is performing a cognitive assessment on a 16 year-old client. Which behaviors demonstrated will the nurse identify as middle formal operational, according to Piaget's theory? Select all that apply. A) Reporting that he smokes marijuana occasionally. B) Wanting to make decisions about health care independently C) Being very concerned with implications of the Affordable Care Act regarding health care benefits D) Wanting their friends to visit them in the hospital more than their parents E) Difficulty understanding the implications their diagnosis might present

A, B, C

While performing an assessment of a patient who is immunocompromised, the nurse notes the child to have thrush in the mouth, tenderness over the spleen upon palpation, and a white blood cell count of 3,000. Which nursing diagnoses will the nurse include in the care plan of this child based on these findings? Select all that apply. A)Ineffective protection B)Risk for imbalanced nutrition, less than body requirements C)Pain D)Impaired skin integrity E)Delayed growth and development

A, B, C Based on these symptoms the diagnosis of Ineffective protection is related to the decreased white blood cell count; Risk for imbalanced nutrition, less than body requirements, is related to the thrush; and Pain is related to the tenderness over the spleen and the thrush. There is no evidence to support the diagnoses of Impaired skin integrity or Delayed growth and development.

The nurse is assessing a child with aplastic anemia. What would the nurse expect to assess? Select all that apply. A)Ecchymoses B)Tachycardia C)Guaiac-positive stool D)Epistaxis E)Severe pain F)Warm tender joints

A, B, C, D Assessment findings associated with aplastic anemia include ecchymoses, epistaxis, guaiac-positive stools, and tachycardia. Severe pain and warm tender joints are most often associated with sickle cell crisis.

A nurse is assessing a child who may have a latex allergy. The nurse asks the child about allergic reactions to certain foods. Which foods if identified by the child as experiencing an allergic reaction would help support the suspected latex allergy? Select all answers that apply. A)Peaches B)Plums C)Carrots D)Tomatoes E)Apples F)Lettuce

A, B, C, D Foods with a known cross-sensitivity to latex include pear, peach, passion fruit, plum, pineapple, kiwi, fig, grape, cherry, melon, nectarine, papaya, apple, apricot, banana, chestnut, carrot, celery, avocado, tomato, or potato. Apples and lettuce are not associated with a cross-sensitivity.

The nurse is teaching the parents of a child diagnosed with iron-deficiency anemia about ways to increase their child's intake of iron. The parents demonstrate understanding of the teaching when they identify which foods as good choices for the child? Select all that apply. A)Tuna B)Salmon C)Tofu D)Cow's milk E)Dried fruits

A, B, C, E Foods high in iron include red meats, tuna, salmon, eggs, tofu, enriched grains, dried beans and peas, dried fruits, leafy green vegetables, and iron-fortified breakfast cereals.

A nursing student is reviewing information about primary immunodeficiencies. The student demonstrates understanding of the material by identifying which immunodeficiencies as affecting only males? Select all that apply. A)X-linked agammaglobulinemia B)Wiskott-Aldrich syndrome C)Selective IgA deficiency D)X-linked hyper-IgM syndrome E)IgG subclass deficiency F)Severe combined immune deficiency

A, B, D X-linked agammaglobulinemia, Wiskott-Aldrich syndrome, and X-linked hyper-IgM syndrome affect males only. Selective IgA deficiency, IgG subclass deficiency, and severe combined immune deficiency affect boys and girls.

A 15-year-old adolescent is brought to the emergency department by his parents. The adolescent is febrile with chills that started suddenly. He states, "I had a sinus infection and sore throat a couple of days ago." The nurse suspects bacterial meningitis based on which findings? Select all that apply. A) Complaints of stiff neck B) Photophobia C) Absent headache D) Negative Brudzinski sign E) Vomiting

A, B, E In addition to the adolescent's complaints and history, other findings suggesting bacterial meningitis include complaints of a stiff neck, photophobia, headache, positive Brudzinski sign, and vomiting.

A child is diagnosed with a food allergy to milk. When teaching the parents about this allergy, what would the nurse suggest as possible substitutions for milk? Select all that apply. A)Fruit juice B)Rice milk C)Yogurt D)Nondairy creamers E)Soy milk

A, B, E Milk can be replaced with water, fruit juice, rice milk, or soy milk. Yogurt contains milk and some nondairy products such as creamers may contain milk and should be avoided.

An 8-month-old infant is brought to the clinic for evaluation. The mother tells the nurse that she has noticed some white patches on the infant's tongue that look like curdled milk after breastfeeding. The nurse suspects oral candidiasis (thrush). Which question would the nurse use to help confirm this suspicion? A)"Are you having breast pain when you nurse the baby?" B)"Has he had any dairy problems recently?" C)"Is he experiencing any vomiting lately?" D)"How have his stools been this past week?"

A. "Are you having breast pain when you nurse the baby?" The infant may develop thrush from the mother if the mother has a fungal infection of the breast. Asking the mother about breast pain would be important because this type of infection can cause the mother a great deal of pain with nursing. Dairy products are not associated with oral candidiasis but are associated with the development of infectious diarrhea in infants. Vomiting is unrelated to thrush. The infant also may have candidal diaper rash, but this would be manifested on the skin as a beefy-red rash with satellite lesions, not in his stools.

The nurse is educating the parents of a 4-year-old boy with a Wilms tumor who is about to have chemotherapy prior to surgery. Which statement by the parents indicates that the nurse should review the instructions about preventing infection? A)"He takes his antibiotic twice a day." B)"We check his temperature orally." C)"We keep him away from crowds." D)"He must be clean and his teeth brushed."

A. "He takes his antibiotic twice a day." The parents have heard the instructions for the antibiotic administration incorrectly. The trimethoprim-sulfamethoxazole should be administered twice daily for 3 consecutive days each week to prevent Pneumocystis pneumoniae. The parents understand to avoid rectal temperatures and crowds, and to maintain his hygiene meticulously.

The nurse is taking a health history of an 11-year-old girl with recurrent abdominal pain. Which response would lead the nurse to suspect irritable bowel syndrome? A)"I always feel better after I have a bowel movement." B)"I don't take any medicine right now." C)"The pain comes and goes." D)"The pain doesn't wake me up in the middle of the night."

A. "I always feel better after I have a bowel movement." In cases of irritable bowel syndrome, the pain may be relieved by defecation. Use of medications and pain that comes and goes or wakes the person up in the middle of the night are all relevant findings pertinent to recurrent abdominal pain.

The nurse is caring for a child with thalassemia who is receiving chelation therapy at home using a battery-operated pump. After teaching the parents about this treatment, which statement by the mother indicates a need for additional teaching? A)"I can have the nurse administer the chelation therapy if I am uncomfortable." B)"I must be very careful to strictly adhere to the chelation regimen." C)"The deferoxamine binds to the iron so it can be removed from the body." D)"The medication can be administered while my child is sleeping."

A. "I can have the nurse administer the chelation therapy if I am uncomfortable." The nurse needs to emphasize to the mother that therapy must be maintained at home to continuously decrease the iron levels in the child's body. Family members need to be taught to administer deferoxamine subcutaneously with a small battery-powered infusion pump over a several-hour period each night (usually while the child is sleeping).

The nurse is providing home care instructions for a 13-year-old girl recently diagnosed with systemic lupus erythematosus. Which response by the girl indicates a need for further teaching? A)"I need to wear sunscreen in the summer to prevent rashes." B)"I need to eat a healthy diet, exercise, and get plenty of sleep." C)"I need an eye examination every year." D)"I need to be careful when it is cold; I should always wear gloves."

A. "I need to wear sunscreen in the summer to prevent rashes." The nurse needs to emphasize that the girl should apply sunscreen every day, not just in the summer, to prevent rashes resulting from photosensitivity. A healthy diet, sleep, yearly eye examinations, and protection from cold weather are appropriate measures.

The nurse is teaching the mother of a 5-year-old boy with a history of impaction how to administer enemas at home. Which response from the mother indicates a need for further teaching? A)"I should position him on his abdomen with knees bent." B)"He will require 250 to 500 mL of enema solution." C)"I should wash my hands and then wear gloves." D)"He should retain the solution for 5 to 10 minutes."

A. "I should position him on his abdomen with knees bent." A 5-year-old child should lie on his left side with his right leg flexed toward the chest. An infant or toddler is positioned on his abdomen. Using 250 to 500 mL of solution, washing hands and wearing gloves, and retaining the solution for 5 to 10 minutes are appropriate responses.

The nurse is caring for a 3-year-old girl with short bowel syndrome as a result of trauma to the small intestine. The girl's mother is extremely anxious and tells the nurse she is afraid she will never learn how to care for her daughter at home. How should the nurse respond? A)"I will help you become an expert on your daughter's care." B)"You must learn how to care for your daughter at home." C)"You really need the support of your husband." D)"There is a lot to learn and you need a positive attitude."

A. "I will help you become an expert on your daughter's care." The nurse needs to empower families to become the experts on their children's needs and conditions via education and participation in care. The most positive approach in this case is to let the mother know the nurse will support her and help her become an expert on her daughter's care. Telling the mother that she must learn how to care for her daughter or that she must have a positive attitude is not helpful. Telling her that she needs the support of her husband is irrelevant and unhelpful.

The nurse is providing postoperative care for a 14-month-old girl who has undergone a myelomeningocele repair. The girl's mother is extremely anxious and tells the nurse she is afraid she will never learn how to care for her daughter at home. Which response by the nurse would be most appropriate? A)"I will help you become comfortable in caring for your daughter." B)"You must learn how to care for your daughter at home." C)"You will need to learn to collaborate with all the caregivers." D)"There is a lot to learn, and you need a positive attitude."

A. "I will help you become comfortable in caring for your daughter." The nurse needs to empower families to become the experts on their child's needs and conditions via education and participation in care. The most positive approach is to let the mother know the nurse will support her and help her become an expert on her daughter's care. Telling the mother that she must learn how to care for her daughter or that she must have a positive attitude is not helpful. Telling her that she needs to collaborate with the caregivers is true, but does not address her fears.

The nurse is caring for a 12-year-old girl with nephrotic syndrome. The girl confides that she feels like a "freak" compared to her peers because of her weight, edema, and moon face. Which response by the nurse would be most appropriate? A)"Let's put you in touch with some other girls who are also having the same body changes." B)"Luckily, this is just a temporary, unfortunate part of your condition; you need to accept it." C)"Your real friends do not care about your appearance and just want you to get well." D)"You are beautiful in your own way; what matters is what is on the inside."

A. "Let's put you in touch with some other girls who are also having the same body changes." It is important to introduce the girl to other youngsters with chronic renal conditions so she does not feel so isolated. Adolescents need interaction with peers. Telling the girl that this is a temporary condition, her real friends don't care about her appearance, and she is beautiful in her own way dismisses the girl's concerns and does not offer solutions. Nephrotic syndrome is a chronic condition, so telling her the condition is temporary also is inaccurate.

A nurse is caring for a 12-year-old girl with a severe peanut allergy. The girl's parents are upset because the school does not permit her to carry her EpiPen with her. It must remain in the school's office per school regulations. Which response by the nurse would be most appropriate? A)"She is allowed by law to carry her EpiPen with her; I will talk to school authorities." B)"Let's file an action plan and keep it in the school office in the event of anaphylaxis." C)"Make sure she wears a medical alert bracelet so that school staff know she has allergies." D)"I will be happy to train school authorities and staff to recognize anaphylaxis."

A. "She is allowed by law to carry her EpiPen with her; I will talk to school authorities." Public Law No. 108-377, the Asthmatic Schoolchildren's Treatment and Health Management Act of 2004, was passed by the U.S. Congress. This law is intended to ensure that students with severe allergies can carry prescribed medications such as an EpiPen with them at all times. The nurse must contact the school and inform them of this law so that the girl is allowed to carry her EpiPen on her person at all times. The school staff should be trained to recognize anaphylaxis, there should be an action plan on file, and the girl should wear a medical alert bracelet as well. However, the most important action is to notify school authorities of the law.

The nurse is caring for a 4-year-old girl with vulvovaginitis. After explaining to the girl's mother how to help prevent subsequent episodes, which statement by the mother indicates a need for additional teaching? A)"She needs to wipe from front to back." B)"I will make sure she changes her underwear every day." C)"She should probably avoid bubble baths." D)"I will help supervise her wiping after bowel movements."

A. "She needs to wipe from front to back." At the age of 4, the mother should not assume that the girl will wipe properly. The mother will need to supervise her wiping in order to train her properly. Making sure the child changes her underwear daily, avoiding bubble baths, and supervising her wiping after bowel movements indicate that the mother has understood the instructions.

When evaluating the hemogram of an 8-month-old infant, the nurse would identify which type of hemoglobin as being the predominant type? A)Hemoglobin A B)Hemoglobin F C)Hemoglobin A2 D)Hemoglobin S

A. Hemoglobin A. Three types of normal hemoglobin are present at any given time in the blood: A, F, and A2. By 6 months of age, hemoglobin A is the predominant type. Hemoglobin S is associated with sickle cell disease.

The nurse is caring for a 14-month-old boy with rickets who was recently adopted from overseas. His condition was likely a result of a diet very low in milk products. The nurse is providing teaching regarding treatment. Which response by the parents indicates a need for further teaching? A)"We must give him calcium and phosphorus with food every morning." B)"He must take vitamin D as prescribed and spend some time in the sunlight." C)"He must take calcium at breakfast and phosphorus at bedtime." D)"We should encourage him to have fish, dairy, and liver if he will eat it."

A. "We must give him calcium and phosphorus with food every morning." The nurse should emphasize that the calcium and phosphorus supplements should be administered at alternate times to promote proper absorption of both of these supplements. Taking vitamin D, spending time in the sun, and encouraging intake of fish, dairy, and liver are appropriate responses.

A child with Duchenne muscular dystrophy is to receive prednisone as part of his treatment plan. After teaching the child's parents about this drug, which statement by the parents indicates the need for additional teaching? A)"We should give this drug before he eats anything." B)"We need to watch carefully for possible infection." C)"The drug should not be stopped suddenly." D)"He might gain some weight with this drug."

A. "We should give this drug before he eats anything." Corticosteroids such as prednisone can cause gastric upset, so the medication should be given with food to reduce this risk. The drug may mask the signs of infection, so the parents need to monitor the child closely for any changes. Treatment with this drug should not be stopped abruptly due to the risk for acute adrenal insufficiency. Common side effects of this drug include weight gain, osteoporosis, and mood changes.

A child with heart failure is receiving supplemental oxygen. The nurse understands that in addition to improving oxygen saturation, this intervention also has what effect? A)Cause vasodilation B)Increase pulmonary vascular resistance C)Promote diuresis D)Mobilize secretions

A. Cause vasodilation Oxygen improves oxygen saturation and also functions as a vasodilator and decreases pulmonary vascular resistance. Diuretics promote dieresis. Chest physiotherapy helps to mobilize secretions.

A nurse is caring for a 7-year-old girl scheduled for an intravenous pyelogram (IVP). Which action would be the priority before the test? A)Checking with the parents for any allergies B)Ensuring adequate hydration C)Giving the girl an enema D)Screening her for pregnancy

A. Checking with the parents for any allergies It is important to double-check whether the girl has any allergies. The test is contraindicated in children allergic to shellfish or iodine. Adequate hydration is also important, but the check for allergies is a priority. Only females of reproductive age must be screened for pregnancy. An enema is not necessary at all institutions.

What would the nurse expect to find in a male infant with Wiskott-Aldrich syndrome? A)Eczema B)Thrombocytosis C)Lymphadenopathy D)Pneumonia

A. Eczema Wiskott-Aldrich syndrome is manifested by eczema that usually worsens with time, petechiae, bloody diarrhea, or a bleeding episode in the first 6 months of life. Thrombocytopenia is present. Lymphadenopathy is associated with hypogammaglobulinemia. Pneumonia is associated with severe combined immune deficiency.

The nurse is caring for a child who is experiencing an acute renal transplant rejection and is to receive muromonab-CD3. What would the nurse most likely expect to assess after the first dose is administered? A)Fever with chills, chest tightness B)Cough, hyperkalemia C)Photosensitivity, gastrointestinal (GI) upset D)Urinary retention, decreased appetite

A. Fever with chills, chest tightness The first dose of muromonab-CD3 can cause fever, chills, chest tightness, wheezing, nausea, and vomiting. Cough and hyperkalemia are associated with angiotensin-converting enzyme inhibitors. Photosensitivity and GI upset are often associated with diuretics. Urinary retention and decreased appetite are associated with imipramine.

The nurse is caring for a 13-year-old boy with acute myelogenous leukemia who is experiencing feelings of powerlessness due to the effects of chemotherapy. What intervention will best help the teen's sense of control? A)Involving the boy in decisions whenever possible B)Acknowledging the boy's feelings of anger with the disease C)Providing realistic expectations of treatments and outcomes D)Recognizing abilities that are unaffected by the disease

A. Involving the boy in decisions whenever possible. Involving the boy in the decision-making process will best help his sense of control. Whether he is included in important decisions about therapy or minor decisions like menus or dress, it will give him a sense of control over his situation. Acknowledging feelings of anger, recognizing his abilities, and providing realistic expectations will reduce body image disturbance and build self-esteem.

A nurse is reviewing the medical record of a child and finds that the child has a grade III murmur. After auscultating the child's heart sounds, how would the nurse document this murmur? A)Loud without a thrill B)Loud with a precordial thrill C)Soft and easily heard D)Loud, audible with a stethoscope

A. Loud w/o a thrill A grade III murmur is loud without a thrill. Grade II is soft and easily heard. Grade IV is loud with a precordial thrill. Grade V is characterized as loud, audible with a stethoscope.

The nurse is planning a discussion group for parents with children who have cancer. How would the nurse describe a difference between cancer in children and adults? A)Most childhood cancers affect the tissues rather than organs. B)Childhood cancers are usually localized when found. C)Unlike adult cancers, childhood cancers are less responsive to treatment. D)The majority of childhood cancers can be prevented.

A. Most childhood cancers affect the tissues rather than organs. Childhood cancers usually affect the tissues, not the organs, as in adults. Metastasis often is present when the childhood cancer is diagnosed. Childhood cancers, unlike adult cancers, are very responsive to treatment. Unfortunately, little is known about cancer prevention in children.

An 8-year-old girl was diagnosed with a closed fracture of the radius at approximately 2 p.m. The fracture was reduced in the emergency department and her arm placed in a cast. At 11 p.m. her mother brings her back to the emergency department due to unrelenting pain that has not been relieved by the prescribed narcotics. Which action would be the priority? A)Notifying the doctor immediately B)Applying ice C)Elevating the arm D)Giving additional pain medication as ordered

A. Notifying the dr. immediately The nurse should notify the doctor immediately because the girl's symptoms are the classic sign of compartment syndrome. Immediate treatment is required to prevent excessive swelling and to detect neurovascular compromise as quickly as possible. The ice should be removed and the arm brought below the level of the heart to facilitate whatever circulation is present. Giving additional pain medication will not help in this situation.

The nurse is caring for a child undergoing highly active antiretroviral therapy (HAART) for HIV infection. The nurse is preparing to administer the prescribed medication. In addition to the nucleoside analog reverse transcriptase inhibitors (NRTIs) and the nonnucleoside analog reverse transcriptase inhibitors (NNRTIs), the nurse is cognizant that the child will be taking which additional medication as part of the three-drug regimen? A)Protease inhibitors B)Corticosteroids C)Cytotoxic drugs D)Disease-modifying antirheumatic drugs (DMARDs)

A. Protease inhibitors The nurse understands that the child will be taking protease inhibitors as part of the three-drug regimen for HAART. Corticosteroids, cytotoxic agents, and DMARDs are typically used for the treatment of juvenile idiopathic arthritis (JIA).

The nurse is conducting a physical examination of a toddler with suspected lead poisoning. Lab results indicate blood lead level 52 mcg/dL. Which action would the nurse expect to happen next? A)Repeat testing within 2 days and prepare to begin chelation therapy as ordered. B)Repeat testing within 1 week with education to decrease lead exposure. C)Confirm with repeat testing in 1 month and referral to local health department. D)Prepare to admit child to begin chelation therapy.

A. Repeat testing within 2 days and prepare to begin chelation therapy as ordered. The recommendation for blood lead levels of 45 to 69 mcg/dL is to confirm the level with a repeat laboratory test within 2 days and educate the parents to decreased lead exposure. She should also expect to begin chelation therapy as ordered and refer the case to the local health department for investigation of home lead reduction with referrals for support services. Repeat testing in 1 week with parent education is appropriate for lead levels between 20 and 44 mcg/dL. Repeat testing in 1 month and education would be appropriate for levels between 15 and 19 mcg/dL. Preparing to admit the child to begin chelation therapy immediately would be appropriate for lead levels greater than 70 mcg/dL.

The nurse is caring for a 10-year-old in traction. While performing a skin assessment, the nurse notices that the skin over the calcaneus appears slightly red and irritated. Which action would the nurse take first? A)Reposition the child's foot on a pressure-reducing device. B)Apply lotion to his foot to maintain skin integrity. C)Make sure the skin is clean and dry. D)Gently massage his foot to promote circulation.

A. Reposition the child's foot on a pressure-reducing device. The nurse's first action is to remove continuous pressure from this area. The other actions can help decrease the potential for skin breakdown, but the pressure must be relieved first.

The nurse is providing care to a child with a long-leg hip spica cast. What is the priority nursing diagnosis? A)Risk for impaired skin integrity due to cast and location B)Deficient knowledge related to cast care C)Risk for delayed development related to immobility D)Self-care deficit related to immobility

A. Risk for impaired skin integrity due to cast and location Although deficient knowledge, risk for delayed development, and self-care deficit may be applicable, the child is at increased risk for skin breakdown due to the size of the cast and its location. In addition, the cast has an opening, which allows for elimination. Soiling of cast edges or leakage of urine or stool can lead to skin breakdown.

A nurse is caring for a 14-year-old girl scheduled for a barium swallow/upper gastrointestinal (GI) series. Before providing instructions, what would be the priority? A)Screening the girl for pregnancy B)Reminding her to drink plenty of fluids after the procedure C)Ordering a bowel preparation D)Reminding the girl about potential light-colored stools

A. Screening the girl for pregnancy. Females of reproductive age must be screened for pregnancy prior to the test because radiography is used. A bowel preparation is not necessary for a barium swallow/upper GI series. The reminders about fluids and light-colored stools are appropriate but are not the first priority.

A child with hypogammaglobulinemia is to receive intravenous immunoglobulin (IVIG). What action would not be correct to take? A)Shake the vial after reconstituting it B)Premedicate the child with acetaminophen C)Obtain preinfusion vital signs D)Check serum blood urea nitrogen and creatinine levels

A. Shake the vial after reconstituting it. Many IVIG products are packed as two vials, one the IVIG powder and one the sterile diluents. Once reconstituted, the IVIG should not be shaken because this leads to foaming and may cause the immunoglobulin protein to degrade. The child can be premedicated with acetaminophen or diphenhydramine. Baseline serum blood urea nitrogen and creatinine should be assessed because acute renal insufficiency may occur as a serious adverse reaction.

The nurse is conducting a physical examination of a child with a suspected cardiovascular disorder. Which finding would the nurse most likely expect to assess if the child had transposition of the great vessels? A)Significant cyanosis without presence of a murmur B)Abrupt cessation of chest output with an increase in heart rate/filling pressure C)Soft systolic ejection D)Holosystolic murmur

A. Significant cyanosis without presence of a murmur Significant cyanosis without presence of a murmur is highly indicative of transposition. Abrupt cessation of chest output accompanied by an increase in heart rate and filling pressure is indicative of cardiac tamponade. A soft systolic ejection or holosystolic murmur can be found with other disorders, such as hypoplastic left heart syndrome, but is not highly suspicious of transposition.

When developing the plan of care for a child with cerebral palsy, which treatment would the nurse expect as least likely? A)Skeletal traction B)Physical therapy C)Orthotics D)Occupational therapy

A. Skeletal traction Skeletal traction would be the least likely treatment for a child with cerebral palsy. Physical therapy, orthotics and braces, and occupational therapy are all common treatments used for cerebral palsy.

The nurse is assessing the neuromusculoskeletal system of a newborn. What is an abnormal assessment finding? A)Sluggish deep tendon reflexes B)Full range of motion in extremities C)Absence of hypotonia D)Lack of purposeful muscular control

A. Sluggish deep tendon reflexes Deep tendon reflexes are present at birth and are initially brisk in the newborn and progress to average over the first few months. Sluggish deep tendon reflexes indicate an abnormality. The newborn is capable of spontaneous movement but lacks purposeful control. Full range of motion is present at birth. Healthy infants and children demonstrate normal muscle tone; hypertonia or hypotonia is an abnormal finding.

The mother of a 5-year-old girl brings the child to the clinic for an evaluation. The mother tells the nurse, "She seems to be so tired and irritable lately. And she looks so pale." Further assessment reveals pale conjunctiva and oral mucous membranes. The nurse suspects iron-deficiency anemia. Which additional finding would help provide additional evidence for this suspicion? A)Spooned nails B)Negative splenomegaly C)Oxygen saturation: 99% D)Bradycardia

A. Spooned nails Spooning or concave shape of the nails suggests iron-deficiency anemia. Other findings would include decreased oxygen saturation levels, tachycardia, and possible splenomegaly.

Origin: Chapter 14, 19 19. Prior to administering morphine to a 10-year-old child, the nurse reviews the adverse effects of the drug. Which system is primarily affected by the drug, causing most of the adverse effects? A) Central nervous system B) Peripheral nervous system C) Digestive system D) Musculoskeletal system

Ans: A Feedback: Opioid agonists, such as morphine, are associated with numerous adverse effects, resulting primarily from their depressant action on the central nervous system.

The nurse is assessing a 13-year-old girl with a family history of kidney cancer who has come to the clinic complaining of abdominal pain, nausea, and vomiting. Which finding would the nurse identify as least likely indicative of cancer in a child? A)The child reports rectal bleeding and diarrhea. B)Observation reveals an asymmetric abdomen. C)The child experiences a broken bone without trauma. D)Palpation determines an abdominal mass.

A. The child reports rectal bleeding and diarrhea. Rectal bleeding and diarrhea are symptoms of rectal cancer in adults and are not typical of children with cancer. The child reporting that a bone broke without any trauma, the nurse observing asymmetric swelling in the abdomen, or palpation revealing a mass in the abdomen are findings in children with cancer.

Origin: Chapter 14, 18 18. For which child would nonopioid analgesics be recommended? A) A child with juvenile arthritis B) A child with end-stage cancer C) A child with a broken arm D) A child with severe postoperative pain

Ans: A Feedback: Nonopioid analgesics may be used to treat mild to moderate pain, often for conditions such as arthritis; joint, bone, and muscle pain; headache; dental pain; and menstrual pain. Opioid analgesics are typically used for moderate to severe pain as can occur with cancer, broken bones, and postoperative healing.

Origin: Chapter 5, 16 16. When observing a group of preschoolers at play in the clinic waiting room, which type of play would the nurse be least likely to note? A) Parallel play B) Cooperative play C) Dramatic play D) Fantasy play

Ans: A Feedback: Parallel play is associated with toddlers. Cooperative, dramatic, and fantasy play are commonly used by preschoolers.

When reviewing the history of a child with suspected primary immunodeficiency, what would the nurse be least likely to find? A)Weight appropriate for height B)Antibiotic therapy for the past 3 months without effect C)Ten episodes of otitis media in the last year D)Three bouts of sinusitis within a year's time

A. Weight appropriate for height Weight appropriate for height would not be associated with primary immunodeficiency. Rather, failure to thrive is considered a warning sign. Other warning signs of primary immunodeficiency include eight or more episodes of acute otitis media in 1 year; two or more episodes of severe sinusitis in 1 year; treatment with antibiotics for 2 months or longer with little effect; two or more episodes of pneumonia in 1 year; recurrent deep skin or organ abscesses; persistent oral thrush or skin candidiasis after age 1 year; history of infections that do not clear with antibiotics; two or more serious infections; and a family history of primary immunodeficiency.

A group of students are reviewing information about gallbladder disease in children. The students demonstrate a need for additional review when they state: A)cholesterol gallstones are more frequently found in males. B)pigment stones are found primarily in the common bile duct. C)pancreatitis is a common complication of cholecystitis in children. D)cholecystitis is due to chemical irritation from obstructed bile flow.

A. cholesterol gallstones are more frequently found in males Cholesterol gallstones are seen more often in females than males and increased risk occurs with age and onset of puberty. Pigment stones are usually found in the common bile duct. Pancreatitis is a common complication in children with gallstone disease. Cholecystitis is an inflammation of the gallbladder that is caused by chemical irritation due to the obstruction of bile flow from the gallbladder into the cystic ducts.

A 15-year-old boy comes to the emergency department accompanied by his parents. The boy reports an abrupt onset of sudden pain on the right side of his scrotum. When asked to rate his pain on a scale of 1 to 10, with 10 being the most severe, the boy states, "It's a 12." Further assessment reveals a blue-black swelling on the affected side. The nurse suspects testicular torsion and immediately notifies the physician because: A)the condition is a surgical emergency. B)the boy is at risk for sepsis. C)intravenous antibiotics need to be initiated. D)renal failure is imminent.

A. the condition is a surgical emergency Testicular torsion is a surgical emergency that necessitates immediate surgical correction to prevent testicular necrosis and possible gangrene. There is no infection with testicular torsion, intravenous antibiotics are not used to treat this condition, and renal failure is not imminent.

A child returns from surgery in which a stoma was created in the abdominal wall to the bladder. The nurse identifies this as a: A)vesicostomy. B)ureteral stent. C)continent urinary diversion. D)bladder augmentation.

A. vesicostomy A vesicostomy refers to a stoma created in the abdominal wall to the bladder. A ureteral stent is a thin catheter temporarily placed in the ureter to drain urine. A continent urinary diversion uses a piece of the intestine to create a bladder that can be catheterized. Bladder augmentation involves the use of a piece of the stomach or intestine to enlarge bladder capacity.

16. A child is receiving therapy in which he is learning to replace automatic negative thought patterns with alternative ones. The nurse interprets this as which type of therapy? A) Cognitive therapy B) Behavioral therapy C) Milieu therapy D) Individual therapy

Ans: A Feedback: Cognitive therapy teaches children to change reactions so that automatic negative thought patterns are replaced with alternative ones. Behavioral therapy uses stimulus and response conditioning to manage or alter behavior, reinforcing desired behaviors and replacing the inappropriate ones. Milieu therapy involves a specially structured setting designed to promote the child's adaptive and social skills. With individual therapy, the child and therapist work together to resolve the conflicts, emotions, or behavior problems.

A 10-month-old is brought to the emergency department by her parents after they found her face down in the bathtub. The mother said, "I just left the bathroom to answer the phone. When I came back, I found her." Which assessment would be the priority? A) Airway, breathing, and circulation B) Level of consciousness C) Vital signs D) Pupillary response

Airway, breathing, and circulation With a submersion injury, hypoxia is the primary problem. Therefore, assessment of airway, breathing, and circulation are the priority assessments for which the nurse would institute resuscitative measures. Other assessments such as level of consciousness, vital signs, and papillary response would be done once the child's airway, breathing, and circulation are assessed and emergency interventions are instituted.

17. A nurse is preparing a program for a parent group about various techniques that can be used to manage behavior. What would the nurse be least likely to include? A) Focus the child's attention on the negative behavior. B) Set limits with the child for responsible behavior. C) Ignore inappropriate behaviors. D) Provide positive feedback for self-control efforts.

Ans: A Feedback: Behavior management techniques include redirecting the child's attention when needed, setting limits for responsible behavior, ignoring inappropriate behaviors, and providing praise and positive feedback for the child's self-control efforts.

8. The nurse is caring for a 13-year-old boy with a history of inappropriate behavior. Which statement by the mother would lead the nurse to suspect oppositional defiant disorder rather than conduct disorder? A) "He has frequent temper tantrums." B) "He was pulling the neighbor's dog around by his leash." C) "He is constantly lying to me." D) "He has stolen hundreds of dollars from my purse."

Ans: A Feedback: Reports of frequent temper tantrums point to oppositional defiant disorder rather than conduct disorder. Reports of cruelty to animals, excessive lying, and stealing point to conduct disorder.

17. A child with diabetes insipidus is being treated with vasopressin. The nurse would assess the child closely for signs and symptoms of which condition? A) Syndrome of inappropriate antidiuretic hormone (SIADH) B) Thyroid storm C) Cushing syndrome D) Vitamin D toxicity

Ans: A Feedback: SIADH, although rare in children, is a potential complication of excessive administration of vasopressin. Thyroid storm may result from overadministration of levothyroxine (thyroid hormone replacement). Cushing syndrome is associated with corticosteroid use. Vitamin D toxicity may result from the use of vitamin D as treatment of hypoparathyroidism.

25. The nurse identifies a nursing diagnosis of impaired social interaction related to altered social skills as evidenced by impulsivity and intrusive behavior. The nurse plans to identify factors that aggravate the child's behavior for which reason? A) Minimize stimuli that exacerbate the child's undesired behaviors. B) Improve the child's ability to deal with external stressors. C) Promote increased ability to follow through. D) Encourage the child to adopt expectations into his routine.

Ans: A Feedback: The nurse identifies aggravating factors to help minimize stimuli that exacerbate the child's undesired behaviors. This must be accomplished first before any other interventions would be effective. Improving the child's ability to deal with external stressors is achieved by modifying the environment to decrease distracting stimuli. Actions such as speaking directly to the child and maintaining eye contact promote engagement and an increased ability to follow through. Providing positive feedback encourages the child to adopt expectations into his routine.

10. The nurse is caring for a child with bipolar disorder. The child is taking lithium as ordered. The parents inquire about the potential side effects. Which response by the nurse would be most appropriate? A) "You might see excessive urination and thirst, tremor, nausea, weight gain, and diarrhea." B) "He might experience a significant decrease in his appetite and difficulty sleeping." C) "You need to watch for dry mouth, urinary retention, and constipation." D) "This medication can cause seizures, agitation, headache, and nausea."

Ans: A Feedback: The nurse needs to explain that the potential side effects of lithium include polyuria, polydipsia, tremors, nausea, weight gain, and diarrhea. Decreased appetite and difficulty sleeping are associated with psychostimulants. Anticholinergic effects such as dry mouth, urinary retention, and constipation are often associated with tricyclic antidepressants as well as a-agonist antihypertensive agents such as clonidine. Seizures, agitation, headache, and nausea are associated with atypical antipsychotic agents.

3. The nurse is developing a plan of care for a 7-year-old boy with diabetes insipidus. What is the priority nursing diagnosis? A) Deficient fluid volume related to dehydration B) Excess fluid volume related to edema C) Deficient knowledge related to fluid intake regimen D) Imbalanced nutrition, more than body requirements related to excess weight

Ans: A Feedback: The priority nursing diagnosis most likely would be deficient fluid volume related to dehydration, due to a deficiency in the secretion of antidiuretic hormone (ADH). Excess fluid would result from a disorder that leads to water retention, such as syndrome of inappropriate antidiuretic hormone (SIADH). Deficient knowledge related to fluid intake regimen is a nursing diagnosis for this child, but a secondary one. Imbalanced nutrition, more than body requirements related to excess weight would be inappropriate for this child since he probably has lost weight secondary to the fluid loss.

Origin: Chapter 6, 17 17. After teaching the parents of a 9-year-old girl about safety, which statement indicates the need for additional teaching? A) "She can ride in the front seat of the car once she is 10 years old." B) "We need to buy her a helmet so she can ride her scooter." C) "She should ride her bike with the traffic on the side of the road." D) "We signed her up for swim lesions at the local community center."

Ans: A Feedback: Children younger than 12 years of age must sit in the back seat of the car. Laws in most states require helmets for riding bicycles and scooters. When riding a bike, the child should ride on the side of the road traveling with the traffic. Children should know how to swim. If swimming skills are limited, the child must wear a life preserver at all times.

5. The nurse is caring for a hospitalized 13-year-old girl, who is questioning everything the medical staff is doing and is resistant to treatment. How should the nurse respond? A) "Let's work together to plan your day along with your treatments." B) "The sooner you cooperate, the sooner you are going to leave." C) "If you are more cooperative, perhaps we can arrange a visit from friends." D) "Please don't make me call your parents about this."

Ans: A Feedback: Collaborating with the adolescent will provide the teen with increased control. The nurse should work with the teen to provide a mutually agreeable schedule that allows for the teen's preferences while incorporating the required nursing care. Threatening to call the parents will most likely promote further resistance. The nurse should try to immediately engage the girl, rather than making the nurse's cooperation conditional upon the girl's cooperation. Telling the girl that the sooner she cooperates, the sooner she will leave is inappropriate. The nurse is incorrectly implying that her behavior, rather than her medical needs, is going to determine when she will be discharged from the hospital.

Origin: Chapter 6, 7 7. The nurse explains to parents of school-age children that according to Kohlberg's theory of moral development, their child is at the conventional stage of moral development. What is the motivation for school-age children to follow rules? A) They follow rules out of a sense of being a 'good person.' B) They follow rules out of fear of being punished. C) They follow rules in order to receive praise from caretakers. D) They follow rules because it is in their nature to do so.

Ans: A Feedback: During the school-age years, the child's sense of morality is constantly being developed. According to Kohlberg, the school-age child is at the conventional stage of moral development. The 7- to 10-year-old usually follows rules out of a sense of being a "good person." He or she wants to be a good person to his or her parents, friends, and teachers and to himself or herself.

6. The nurse is caring for a special needs infant. Which intervention will be most important in helping the child reach her maximum developmental potential? A) Directing her parents to an early intervention program B) Monitoring her progress in elementary school C) Serving on an individualized education program committee D) Preparing a plan for her to transition to college

Ans: A Feedback: Early intervention is critical to maximizing the child's developmental potential by laying the foundation for health and development. While important, intervention in elementary or secondary school does not have the impact of early intervention. When the time arrives, it is important to have a written plan for transition to college, if this is a possibility for the grown child.

27. The parents of a child with physical and developmental special needs state, "We wish our child could get some kind of educational experience." How should the nurse respond? A) "This must be difficult for you. Let's talk with the social worker to see what programs are available for your child." B) "I am sure it must be difficult to know that your child will never be able to go to school like other children." C) "Since all children can attend school regardless of their special need, I suggest you talk with your local school about enrolling your child." D) "It would be very difficult for your child to attend school with all of their disabilities. It's unfortunate, but it is reality."

Ans: A Feedback: Education is federally mandated. Contacting the social worker gives the parents the support they need to find and choose the appropriate school. Telling them to contact their local school is not supportive of the parent's needs.

25. The nurse working in community nursing uses epidemiology as a tool. What information can be obtained using this process? A) Health needs of a population B) Cultural needs of a population C) Income levels of a population D) Mortality rates of a population

Ans: A Feedback: Epidemiology can help determine the health and health needs of a population and assist in planning health services. Community health nurses perform epidemiologic investigations in order to help analyze and develop health policy and community health initiatives. The nurse provides culturally competent care but does not use epidemiology to determine culture, income levels, or mortality rates of children.

Origin: Chapter 6, 5 5. The nurse knows that the school-age child is in Erikson's stage of industry versus inferiority. Which best examplifies a school-ager working toward accomplishing this developmental task? A) The child signs up for after-school activities. B) The child performs his bedtime preparations autonomously. C) The child becomes aware of the opposite sex. D) The child is developing a conscience.

Ans: A Feedback: Erikson (1963) describes the task of the school-age years to be a sense of industry versus inferiority. During this time, the child is developing his or her sense of self-worth by becoming involved in multiple activities at home, at school, and in the community, which develops his or her cognitive and social skills. Achieving independence is a task of the preschooler who also is developing a conscience at that age. Awareness of the opposite sex occurs in, but is not the focus of, the school-age child.

10. The mother of a hospitalized child reports that her daughter, who is having some difficulty eating, just had a 4-ounce cup of ice chips. The nurse documents this on the child's intake flow sheet as how much? A) 2 ounces B) 4 ounces C) 6 ounces D) 8 ounces

Ans: A Feedback: Ice chips are included as fluid intake, and the amount is approximately equivalent to half the same amount of water. Therefore, the nurse would document this fluid intake as 2 ounces.

Origin: Chapter 6, 21 21. The mother of a 7-year-old girl tells the school nurse that her child is deathly afraid of going to school. What would be the best intervention the nurse could suggest in this situation? A) Return the child to school and investigate the cause of the fear. B) Have the child stay home from school until any issues causing this fear are resolved. C) Investigate a new school for the child to attend that the child will not be afraid of. D) Tell the child that privileges will be taken away if she does not return to school.

Ans: A Feedback: It is important to investigate specific causes of school refusal/school phobia and take appropriate action. The parents should return the child to school, investigate the cause of the fear, support the child, collaborate with teachers, and praise success in school attendance. This is not a situation for punishment, and changing schools would not solve the child's school phobia.

Origin: Chapter 5, 19 19. The parents of a 4-year-old who is a picky eater ask the nurse what foods to include in their child's diet to provide adequate iron consumption. Which food would the nurse recommend? A) Cooked lentils B) Whole milk C) Oranges D) Sweet potatoes

Ans: A Feedback: Lentils are a good source of iron. Whole milk, oranges, and sweet potatoes are good sources of calcium.

Origin: Chapter 5, 27 27. When providing anticipatory guidance to parents about their preschool son who was caught in a lie, what would the nurse emphasize? A) "You need to determine the reason for lying before punishing the child." B) "Lying should never be tolerated and the child should be punished." C) "The misbehavior is usually more serious than the lying itself." D) "It is okay to become angry when dealing with the child's lying."

Ans: A Feedback: Lying is common in preschool children and occurs for a variety of reasons, such as fearing punishment, getting carried away by imagination, or imitating what another person has done. Regardless, the parent should ascertain the reason for the lying before punishing the child. The child also needs to learn that the lying is usually far worse than the misbehavior. Parents need to remain calm and serve as a role model of an even temper.

Origin: Chapter 14, 3 3. The nurse is caring for a child who is recovering from an appendectomy. What is the appropriate term for the pain this child is experiencing? A) Nociceptive pain B) Neuropathic pain C) Chronic pain D) Superficial somatic pain

Ans: A Feedback: Nociceptive pain reflects pain due to noxious stimuli that damages normal tissues or has the potential to do so if the pain is prolonged. Nociceptive pain ranges from sharp or burning; to dull, aching, or cramping; to deep aching or sharp stabbing. Examples of conditions that result in nociceptive pain include chemical burns, sunburn, cuts, appendicitis, and bladder distention. Neuropathic pain is pain due to malfunctioning of the peripheral or central nervous system. Chronic pain is defined as pain that continues past the expected point of healing for injured tissue. Superficial somatic pain, often called cutaneous pain, involves stimulation of nociceptors in the skin, subcutaneous tissue, or mucous membranes.

7. The nurse is caring for a 4-year-old girl with special care needs in the hospital. Which intervention would have the most positive effect on this child? A) Taking her on an adventure down the hall B) Helping her do a simple craft project C) Introducing her to children in the playroom D) Limiting the staff providing care for her

Ans: A Feedback: Preschool-age children need to develop a sense of initiative, and helping the child to explore her area of the hospital would help accomplish this developmental need. Craft projects and introducing the child to other children would help build a sense of industry and peer relationships, both of which are needs of the school-age, not preschool, child. Limiting the number of people providing care is a trust-building intervention, beginning in infancy.

Origin: Chapter 5, 23 23. The nurse is conducting a well-child assessment for a 5-year-old boy in preparation for kindergarten. The boy's grandmother is his primary caregiver because the boy's mother has suffered from depression and substance abuse issues. The nurse understands that the child is at increased risk for which developmental problem? A) Lack of social and emotional readiness for school B) Stuttering C) Speech and language delays D) Fine motor skills delay

Ans: A Feedback: Risk factors for lack of social and emotional readiness for school include insecure attachment in the early years, maternal depression, parental substance abuse, and low socioeconomic status.

Origin: Chapter 5, 26 26. The nurse is caring for a premature baby in the NICU. The mother reports that the infant's normally happy and outgoing 5-year-old sister is acting sad and withdrawn. The nurse understands that due to her developmental stage, the girl is at risk of what happening? A) Viewing her baby sister's illness as her fault B) Harming the baby C) Experiencing clinical depression D) Creating an imaginary friend to cope with the situation

Ans: A Feedback: Since the preschool child is facing the psychosocial task of initiative versus guilt, it is natural for the child to experience guilt when something goes wrong. The child may have a strong belief that if someone is ill or dying, he or she may be at fault and the illness or death is punishment. It is less likely that the girl would be at risk of harming the baby or experiencing clinical depression as a result of the baby's illness. The child may create an imaginary friend to cope with the illness, but would not withdraw or express sadness as a result of the imaginary friend.

2. The nurse is caring for a 7-year-old girl hospitalized in isolation. The nurse notices that she has begun sucking her thumb and changing her speech patterns to those of a toddler. What condition is the girl manifesting? A) Regression B) Suppression C) Repression D) Denial

Ans: A Feedback: Sucking the thumb and changing of speech pattern (such as to baby talk) are signs of regression, a defense mechanism used by children to deal with unpleasant experiences by returning to a previous stage that may be more comfortable to the child. Suppression is a conscious inhibition of an idea or desire. Repression is an unconscious inhibition of an idea or desire. Denial would be exhibited by expressions of resignation instead of true contentment, not thumb sucking or baby talk.

Origin: Chapter 14, 22 22. The nurse is preparing to administer a topical anesthetic for a 10-year-old girl with a chin laceration. The nurse would expect to apply what as ordered in preparation for sutures? A) TAC (tetracaine, epinephrine, cocaine) B) Iontophoretic lidocaine C) EMLA D) Vapocoolant spray

Ans: A Feedback: TAC (tetracaine, epinephrine, cocaine) is commonly used for lacerations that require suturing. The agent can be applied directly to the wound with a cotton ball or swab for 20 to 30 minutes until the area is numb. EMLA and iontophoretic lidocaine are applied to intact skin, not to open wounds or lacerations. A vapocoolant spray, which should not be applied over a wound, is only effective for 1 to 2 minutes.

Origin: Chapter 14, 13 13. The nurse is assessing the pain of a postoperative newborn. The nurse measures the infant's facial expression, body movement, sleep, verbal or vocal ability, consolability, and response to movements and touch. Which behavioral assessment tool is being used by the nurse? A) Riley Infant Pain Scale B) Pain Observation Scale for Young Children C) CRIES Scale for Neonatal Postoperative Pain Assessment D) FLACC Behavioral Scale for Postoperative Pain in Young Children

Ans: A Feedback: The Riley Infant Pain Scale measures six parameters: facial expression, body movement, sleep, verbal or vocal ability, consolability, and response to movements and touch. The Pain Observation Scale for Young Children (POCIS) measures seven parameters: facial expression, cry, breathing, torso, arms and fingers, legs and toes, and state of arousal. The CRIES tool assesses five parameters: cry, oxygen required for saturation levels less than 95%, increased vital signs, facial expression, and sleeplessness. The FLACC tool measures five parameters: facial expression, legs, activity, cry, and consolability.

Origin: Chapter 5, 11 11. The nurse is explaining to parents that the preschooler's developmental task is focused on the development of initiative rather than guilt. What is a priority intervention the nurse might recommend for parents of preschoolers to stimulate initiative? A) Reward the child for initiative in order to build self-esteem. B) Change the routine of the preschooler often to stimulate initiative. C) Do not set limits on the preschooler's behavior as this results in low self-esteem. D) As a parent, decide how and with whom the child will play.

Ans: A Feedback: The building of self-esteem continues throughout the preschool period. It is of particular importance during these years, as the preschooler's developmental task is focused on the development of initiative rather than guilt. A sense of guilt will contribute to low self-esteem, whereas a child who is rewarded for his or her initiative will have increased self-confidence. Routine and ritual continue to be important throughout the preschool years, as they help the child to develop a sense of time as well as provide the structure for the child to feel safe and secure. Also, consistent limits provide the preschooler with expectation and guidance. Giving children opportunities to decide how and with whom they want to play also helps them develop initiative.

Origin: Chapter 6, 2 2. The nurse is performing an annual check-up for an 8-year-old child. Compared to the previous assessment of this child, which characteristic would most likely be observed? A) Breathing is diaphragmatic. B) Pulse rate is increased. C) Secondary sex characteristics are present. D) Blood pressure has reached adult level.

Ans: A Feedback: The child's respiratory system is maturing, so abdominal breathing has been replaced by diaphragmatic breathing. Pulse rate will decrease, rather than increase, during this time. Secondary sex characteristics will not appear until the late school-age years. Blood pressure will not reach the adult level until adolescence.

29. The nurse caring for a child on a pediatric intensive care unit notices that when the parents go to work the child is very angry and cries easily. What does the nurse suspect is occurring with this patient? A) Protest phase of separation anxiety B) Regressive behavior C) Detachment from the parents D) Despair

Ans: A Feedback: The first phase of separation anxiety, protest, occurs when the child is separated from the parents or primary caretaker. This phase may last from a few hours to several days and is characterized by crying, expressing agitation, rejecting others who attempt to offer comfort, anger, and inconsolable grief.

Origin: Chapter 5, 4 4. The nurse is caring for a 5-year-old girl posttonsillectomy. The girl looks out the window and tells the nurse that it is raining and says, "The sky is crying because it is sad that my throat hurts." The nurse understands that the girl is demonstrating which mental process? A) Magical thinking B) Centration C) Transduction D) Animism

Ans: A Feedback: The nurse understands that the girl is demonstrating magical thinking. Magical thinking is a normal part of preschool development. The preschool-age child believes her thoughts to be all-powerful. Transduction is reasoning by viewing one situation as the basis for another situation whether or not they are truly causally linked. Animism is attributing life-like qualities to inanimate objects. Centration is focusing on one aspect of a situation while neglecting others.

Origin: Chapter 5, 22 22. The nurse is providing teaching about accidental poisoning to the family of a 3-year-old. The nurse understands that a child of this age is at increased risk of accidental ingestion due to which sensory alteration? A) A less discriminating sense of taste B) A lack of fully developed hearing C) Visual acuity that has not fully developed D) A less discriminating sense of touch

Ans: A Feedback: The young preschooler may have a less discriminating sense of taste than the older child, making him or her at increased risk for accidental ingestion. A less discriminating sense of touch and developing visual acuity would not increase the risk. Hearing is intact at birth and it does not increase the child's risk for accidental ingestion.

Origin: Chapter 6, 12 12. A mother brings her 6-year-old son in for a check-up because the child is reporting stomachaches. It is the beginning of the school year. What might the mother also mention? A) The child cries before going to school. B) The child made friends the first day of school. C) The child fights with siblings more often. D) The child loves the crowds in the lunchroom.

Ans: A Feedback: This child has a slow-to-warm-up temperament. The child may also be crying before going to school. Making friends the first day of school and enjoying the crowds in the lunchroom are typical of a child with an easy temperament. Irritability is typical of a child with a difficult temperament.

Origin: Chapter 14, 25 25. The nurse is providing teaching to the parents of a newborn prior to a heelstick. The nurse is describing the procedure and recommending various methods for the parents to help comfort their baby. Which statement by the parents indicates a need for further teaching? A) "It's better if we are not in the room for this." B) "We can use kangaroo care before and after." C) "We hope you are using a very tiny needle." D) "We can offer him nonnutritive sucking to calm him."

Ans: A Feedback: Unless contraindicated, the parents should be encouraged to be present before, during, and after the procedure to provide comforting support to the child. Kangaroo care, small-gauge needles, and nonnutritive sucking are other methods to provide atraumatic care.

18. When providing care to a dying child and his family, which would be most important? A) Focusing on the family as the unit of care B) Teaching the family appropriate care measures C) Offering the child support and encouragement D) Assisting the parents in decision making

Ans: A Feedback: When caring for a dying child and his family, the most important aspect of care is focusing on the family as the unit of care. Teaching, offering support, and assisting in decision making are important, but these actions must be implemented while focusing on the family as the unit of care.

27. The nurse is preparing an educational program on behavioral management techniques used in children to help alter negative behavior. What information should the nurse include? Select all that apply. A) Set limits and hold the child responsible for their behavior. B) Do not argue, bargain or negotiate about the limits once established. C) Change caregivers occasionally so the child learns to respond to different people. D) Use a high-pitched voice and remain calm when speaking with the child. E) Ignore inappropriate behaviors.

Ans: A, B Feedback: Behavior management techniques include setting limits and holding the child responsible for his or her behavior. Not arguing, bargaining or negotiating about the limits once established. Inappropriate behaviors should be ignored. Provide consistent caregivers and establish a daily routine. Use a low-pitched, not high pitched voice and remain calm when speaking with the child.

28. A 6-year-old child has been diagnosed with growth hormone deficiency. The child's mother requests more information about this condition. Which statements should be included in the nurse's response? Select all that apply. A) "The majority of children who have this condition are born of normal weight and length." B) "There are several potential causes of this condition." C) "This condition is most likely related to dwarfism in past generations of your family." D) "Most children with this condition are nutritionally deprived." E) "Your child most likely does not eat adequate amounts of protein."

Ans: A, B Feedback: Growth hormone deficiency can result from a variety of causes. These causes may include genetic mutations, tumors, infection and birth trauma. Some cases have not identifiable causes. Most children diagnosed with this condition are of normal length and weight at birth but in childhood fall behind in growth. A small proportion of children may have nutritional concerns.

29. The nurse is caring for a child who takes dextroamphetamine for treatment of ADHD. Which comments by the patient or family would concern the nurse? Select all that apply. A) "I take my sustained released capsule at night before I go to bed." B) "We have noticed that our child shows very little emotion over the last few weeks." C) "I haven't noticed any difference in my appetite." D) "Sometimes my head hurts a little for a short time after I take my medicine." E) "We notice our child gets a little irritable occasionally."

Ans: A, B Feedback: Psychostimulants, such as dextroamphetamine, should be taken in the morning in order to avoid difficulty sleeping. A flat affect is a sign of dosages that are too high. Decreased appetite, headache, and irritability are common side effects.

29. The nurse is caring for a 9-year-old patient newly diagnosed with diabetes. The patient has polyuria, polydipsia, and weight loss. Which nursing diagnoses will the nurse include in the care plan? Select all that apply. A) Imbalanced nutrition: less than body requirements B) Deficient fluid volume C) Deficient knowledge regarding disease process D) Noncompliance E) Delayed growth and development

Ans: A, B, C Feedback: Polyuria (excessive urination), polydipsia (excessive thirst), and weight loss support the diagnoses of Deficient fluid volume and Imbalanced nutrition: less than body requirements. Being newly diagnosed with the disease at the age of 9 supports the diagnosis of Deficient knowledge regarding disease process. There is no data to support Noncompliance or Delayed growth and development.

Origin: Chapter 14, 30 30. The nurse is researching behavioral-cognitive pain relief strategies to use on a 5-year-old child with unrelieved pain. Which methods might the nurse choose? Select all that apply. A) Relaxation B) Distraction C) Thought stopping D) Massage E) Sucking

Ans: A, B, C Feedback: Common behavioral-cognitive strategies include relaxation, distraction, imagery, thought stopping, and positive self-talk. Sucking and massage are examples of biophysical interventions.

Origin: Chapter 5, 3 3. The nurse is assessing the psychosocial development of a preschooler. What are normal activities characteristic of the preschooler? Select all that apply. A) Plans activities and makes up games B) Initiates activities with others C) Acts out roles of other people D) Engages in parallel play with peers E) Classifies or groups objects by their common elements F) Understands relationships among objects

Ans: A, B, C Feedback: The many activities of the preschooler include beginning to plan activities, making up games, initiating activities with others, and acting out the roles of other people (real and imaginary). Toddlers engage in parallel play; preschoolers engage in cooperative play. School-age children classify or group objects by common elements and understand relationships among objects.

20. A nurse is preparing a teaching session for a group of parents with children newly diagnosed with attention deficit/hyperactivity disorder (ADHD). When explaining this disorder to the parents, what would the nurse include as being involved? Select all that apply. A) Impulsivity B) Inattention C) Distractibility D) Hyperactivity E) Defiance F) Anxiety

Ans: A, B, C, D Feedback: ADHD is characterized by inattention, impulsivity, distractibility, and hyperactivity. Anxiety disorder and oppositional defiant disorder may be comorbidities associated with ADHD.

Origin: Chapter 5, 15 15. The nurse is caring for preschoolers in a day care center. For this age group, of what developmental milestones should the nurse be aware? Select all that apply. A) Counting 10 or more objects B) Correctly naming at least four colors C) Understanding the concept of time D) Knowing everyday objects E) Understanding the differences of others F) Forming concepts as logical as an adult's

Ans: A, B, C, D Feedback: The child in the intuitive phase can count 10 or more objects, correctly name at least four colors, and better understand the concept of time, and he or she knows about things that are used in everyday life, such as appliances, money, and food. The preschooler forms concepts that are not as complete or as logical as the adult's, and tolerates others' differences but doesn't understand them.

Origin: Chapter 6, 6 6. The school nurse providing school health screenings knows that the 7- to 11-year-old is in Piaget's stage of concrete operational thoughts. What should this age group accomplish when developing operations? Select all that apply. A) Ability to assimilate and coordinate information about the world from different dimensions B) Ability to see things from another person's point of view and think through an action C) Ability to use stored memories of past experiences to evaluate and interpret present situations D) Ability to think about a problem from all points of view, ranking the possible solutions while solving the problem E) Ability to think outside of the present and incorporate into thinking concepts that do exist as well as concepts that might exist F) Ability to understand the principle of conservation—that matter does not change when its form changes

Ans: A, B, C, F Feedback: Piaget's stage of cognitive development for the 7- to 11-year-old is the period of concrete operational thoughts. In developing concrete operations, the child is able to assimilate and coordinate information about the world from different dimensions. He or she is able to see things from another person's point of view and think through an action, anticipating its consequences and the possibility of having to rethink the action. The school-age child is able to use stored memories of past experiences to evaluate and interpret present situations. Also, during concrete operational thinking, the school-age child develops an understanding of the principle of conservation—that matter does not change when its form changes. According to Piaget, the adolescent progresses from a concrete framework of thinking to an abstract one in the formal operational period. During this period, the adolescent is able to think about a problem from all points of view, ranking the possible solutions while solving the problem. The adolescent also develops the ability to think outside of the present; that is, he or she can incorporate into thinking concepts that do exist as well as concepts that might exist. His or her thinking becomes logical, organized, and consistent.

8. The nurse is caring for infants having the condition failure to thrive (FTT). Which infants would be at risk for this condition? Select all that apply. A) A newborn baby with tetralogy of Fallot B) An infant with a cleft palate C) An infant born to a diabetic mother D) An infant born to an impoverished mother E) An infant with bronchopulmonary dysplasia F) An infant born to a teenage mother

Ans: A, B, D, E Feedback: Infants and children with cardiac or metabolic disease, chronic lung disease (bronchopulmonary dysplasia), cleft palate, or gastroesophageal reflux disease are at particular risk for FTT. Also, poverty is the single greatest contributing risk factor (Block et al., 2005). An infant born to a diabetic mother or an infant born to a teenage mother does not have increased risk for FTT.

Origin: Chapter 5, 2 2. The nurse is teaching the parents of a 4-year-old boy about the normal maturation of the child's organs during the preschool years and their effect on body functions. Which statements accurately describe these changes? Select all that apply. A) Myelination of the spinal cord allows for bowel and bladder control to be complete in most children by age 3 years. B) The respiratory structures are continuing to grow in size, and the number of alveoli continues to increase, reaching the adult number at about 7 years of age. C) Heart rate increases and blood pressure decreases slightly during the preschool years; an innocent heart murmur may be heard upon auscultation. D) The bones continue to increase in length and the muscles continue to strengthen and mature; however, the musculoskeletal system is still not fully mature. E) The small intestine is continuing to grow in length, and stool passage usually occurs once or twice per day in the average preschooler. F) The urethra remains long in both boys and girls, making them more susceptible to urinary tract infections than adults.

Ans: A, B, D, E Feedback: Most of the body systems have matured by the preschool years. Myelination of the spinal cord allows for bowel and bladder control to be complete in most children by age 3 years. The respiratory structures are continuing to grow in size, and the number of alveoli continues to increase, reaching the adult number at about 7 years of age. The bones continue to increase in length and the muscles continue to strengthen and mature. However, the musculoskeletal system is still not fully mature. The small intestine is continuing to grow in length, and stool passage usually occurs once or twice per day in the average preschooler. The 4-year-old generally has adequate bowel control. Heart rate decreases and blood pressure increases slightly during the preschool years. An innocent heart murmur may be heard upon auscultation. The urethra remains short in both boys and girls, making them more susceptible to urinary tract infections than adults.

Origin: Chapter 5, 8 8. The school nurse is helping parents choose books for their preschoolers. What literacy skills present in the preschooler would the nurse consider when making choices? Select all that apply. A) Preschoolers enjoy books with pictures that tell stories. B) Preschoolers like stories with repeated phrases as they help keep their attention. C) Preschoolers like stories that describe experiences different from their own. D) Preschoolers demonstrate early literacy skills by reciting stories or portions of books. E) Preschoolers may retell the story from the book, pretend to read books, and ask questions about the story. F) Preschoolers do not have enough focus and expanded attention to notice when a page is skipped during reading.

Ans: A, B, D, E Feedback: Preschoolers enjoy books with pictures that tell stories. Stories with repeated phrases help to keep the child's attention. Also, children like stories that describe experiences similar to their own. The preschool child demonstrates early literacy skills by reciting stories or portions of books. He or she also may retell the story from the book, pretend to read books, and ask questions about the story. The preschool child has enough focus and expanded attention to notice when a page is skipped during reading and will call it to the parent's attention.

21. A school nurse is working with the parents of an 8-year-old who has Tourette syndrome on how best to accommodate the child. What advice would be most helpful? Select all that apply. A) Allowing for breaks when tics occur B) Providing for "time-outs" during the day C) Using a tape recorder to take notes D) Ensuring a specified amount of time for test taking E) Implementing a reward system for behavior

Ans: A, C Feedback: Together the school nurse and parents should arrange for classroom accommodations such as allowing for "tic breaks," taking untimed tests or tests in another room, or using note takers or tape recorders. Time-outs and reward systems are more appropriate for the child with ADHD.

27. The nurse referring a child to home care discusses the advantages and disadvantages with the child's family. What are disadvantages of this method of health care? Select all that apply. A) The nurse is performing care of the child in the family's home. B) The home care nurse is not always equipped to perform technical care. C) The out-of-pocket cost of home care is more expensive. D) The technical procedures may be overwhelming for the family. E) The financial burden may cause more stress for the family. F) The child does not receive continuity of care provided in the hospital setting.

Ans: A, C, D, E Feedback: There are some disadvantages to home care. The presence of health care professionals in the home can be an intrusion on family privacy. Financial issues can become a large burden: families may have higher out-of-pocket costs if their insurance does not reimburse for home care. Having one parent at home full time and not earning an income can contribute to increased financial strain, not to mention social isolation of that parent. All of these can lead to increased stress on family members. Also, caring for children with complex medical needs can be overwhelming for some families. The home care nurse should arrange for continuity of care for the child.

Origin: Chapter 14, 6 6. The nurse caring for infants in the neonatal intensive care unit (NICU) relies on the use of behavioral and physiologic indicators for determining pain. Which examples are behavioral indicators? Select all that apply. A) The infant grimaces. B) The infant's heart rate is elevated. C) The infant flails his arms and legs. D) The infant's respiratory rate is elevated. E) The infant is crying uncontrollably. F) The infant's oxygen saturation is low.

Ans: A, C, E Feedback: In preterm and term newborns, behavioral and physiologic indicators are used for determining pain. Behavioral indicators include facial expression, body movements, and crying. Physiologic indicators include changes in heart rate, respiratory rate, blood pressure, oxygen saturation levels, vagal tone, palmar sweating, and plasma cortisol or catecholamine levels.

Origin: Chapter 6, 29 29. A 12-year-old girl is experiencing prepubescence, and tells the school nurse that she feels "very out of place" in her school. What would be acceptable responses by the nurse? Select all that apply. A) "It must be difficult for you. Why don't you sit down and we can talk about it." B) "I would suggest that you talk to your parents about your feelings. This isn't something that I can talk to you about." C) "All of the girls and boys will be going through the same thing as you so that should make you feel a little better." D) "Tell me how this makes you feel. Talking about your feelings may help you feel better about school." E) "I went through the same thing when I was in school. I know it doesn't feel like it now but I promise it will get easier."

Ans: A, D Feedback: Prepubescence typically occurs in the 2 years before the beginning of puberty and is characterized by the development of secondary sexual characteristics, a period of rapid growth for girls, and a period of continued growth for boys. Acknowledging the student's feelings and encouraging her to talk about her feelings will likely help her to feel better about herself. She may not be comfortable with talking about her feelings with her parents at this point, and the nurse discussing this topic with the student is acceptable. Telling her that everyone goes through it and that it will "get easier" does not address the student's feelings and is nontherapeutic communication.

12. The nurse is preparing a hospitalized 7-year-old girl for a lumbar puncture. Which actions would help reduce her stress related to the procedure? Select all that apply. A) Pretend to perform the procedure on her doll. B) Explain the procedure to her in medical terms. C) Do not allow her to see or touch the equipment. D) Teach her the steps of the procedure. E) Tell her not to pay attention to any sounds she might hear. F) Introduce her to the health care personnel.

Ans: A, D, F Feedback: Useful techniques for reducing stress in children include the following: perform nursing care on stuffed animals or dolls and allow the child to do the same, teach the child the steps of the procedure or inform him or her exactly what will happen during the hospital stay, introduce the child to the health care personnel with whom he or she will come in contact, avoid the use of medical terms, allow the child to handle some equipment, show the child the room where he or she will be staying, explain the sounds the child may hear, and let the child sample the food that will be served.

2. The nurse is caring for an adolescent girl with anorexia nervosa. What findings would indicate to the nurse that the girl requires hospitalization? A) Weight gain of one-half pound per week B) Food refusal C) Body mass index of 18 D) Soft, sparse body hair and dry, sallow skin

Ans: B Feedback: Food refusal, severe weight loss, unstable vital signs, arrested pubertal development, and the need for enteral nutrition warrant hospitalization. Soft, sparse body hair and dry, sallow skin are signs of anorexia, but do not warrant hospitalization. A weight gain of one-half pound per week indicates progress toward therapeutic goals. A body mass index of 18 is on the low end of the normal range of body mass.

21. A 5-year-old child with type 1 diabetes is brought to the clinic by his mother for a follow-up visit after having his hemoglobin A1C level drawn. Which result would indicate to the nurse that the child is achieving long-term glucose control? A) 9.0% B) 8.2% C) 7.3% D) 6.9%

Ans: B Feedback: For a child 6 years of age and younger, the target HbA1C level should be less than 8.5% but greater than 7.5%. For children between the ages of 6 to 12 years, the target HbA1C level is less than 8%. For children and adolescents between 13 to 19 years of age, the target HbA1C level would be less than 7.5%.

16. A child is diagnosed with hyperthyroidism. Which agent would the nurse expect the physician to prescribe? A) Mineralocorticoid B) Methimazole C) Levothyroxine D) Dexamethasone

Ans: B Feedback: Methimazole is an antithyroid drug that is used to treat hyperthyroidism. Mineralocorticoid is used to treat adrenal insufficiency. Levothyroxine is used to treat hypothyroidism. Dexamethasone is used to treat congenital adrenal hyperplasia.

7. A nurse is caring for a 5-year-old girl with depression. The girl is having difficulty coping with her feelings of sadness and fear, which stem from her parents' separation and recent divorce. The girl has been prescribed antidepressant medication but the mother thinks the girl would benefit from therapy. The nurse anticipates a referral to a therapist that specializes in: A) individual therapy. B) play therapy. C) behavioral therapy. D) hypnosis.

Ans: B Feedback: Play therapy is designed to change emotional status and encourages the child to act out feelings of sadness, fear, hostility, or anger. It is particularly beneficial for the younger child. Play therapy, rather than individual therapy, is recommended for the younger child. Hypnosis promotes deep relaxation, which is not the therapeutic goal for this child. Behavioral therapy is used to encourage appropriate behavior and would not address the girl's sadness.

30. The nurse working in a pediatric mental health clinic is assessing a 4-year-old child who has suffered from physical abuse. Which type of therapy does the nurse anticipate will be most helpful in developing a trusting relationship as well as assisting in determining the patient's current emotional state? A) Behavioral therapy B) Play therapy C) Cognitive behavioral therapy D) Family therapy

Ans: B Feedback: Play therapy will be most helpful, especially in the initial phase of assessment, because it encourages the child to act out feelings of sadness, fear, hostility, or anger.

9. The nurse is caring for a 5-year-old. The child's mother reports that he is extremely sensitive to sounds that most people do not notice and that he prefers complete silence. She explains that the boy is resisting going to school due to the noise and commotion. Additionally, the mother states that he will only wear 100% cotton clothing with all of the tags cut out. The nurse interprets these findings as indicating which disorder or condition? A) Anxiety disorder B) Sensory processing disorder C) Depression D) Obsessive-compulsive disorder

Ans: B Feedback: Sensory processing disorder (sensory integration dysfunction) results in overreaction to different textures and hypersensitivity or hyposensitivity to sensory input. The reported sensitivities to sound and clothing do not point to an anxiety disorder, depression, or obsessive-compulsive disorder.

14. What would lead the nurse to suspect that an adolescent has bulimia? A) Body mass index less than 17 B) Calluses on back of knuckles C) Nail pitting D) Bradycardia

Ans: B Feedback: The adolescent with bulimia would exhibit calluses on the back of the knuckles and split fingernails and would be of normal weight or slightly overweight. A body mass index of 17, nail pitting, and bradycardia would suggest anorexia.

12. A child with diabetes reports that he is feeling a little shaky. Further assessment reveals that the child is coherent but with some slight tremors and sweating. A fingerstick blood glucose level is 70 mg/dL. What would the nurse do next? A) Administer a sliding-scale dose of insulin. B) Give 10 to 15 grams of a simple carbohydrate. C) Offer a complex carbohydrate snack. D) Administer glucagon intramuscularly.

Ans: B Feedback: The child is experiencing hypoglycemia as evidenced by the assessment findings and blood glucose level. Since the child is coherent, offering the child 10 to 15 grams of a simple carbohydrate would be appropriate. Insulin is not used because the child is hypoglycemic. A complex carbohydrate snack would be used after offering the simple carbohydrate to maintain the glucose level. Intramuscular glucagons would be used if the child was not coherent.

26. A group of nursing students are reviewing the components of the endocrine system. The students demonstrate understanding of the review when they identify what as the primary function of this system? A) Regulation of water balance B) Hormonal secretion C) Cellular metabolism D) Growth stimulation

Ans: B Feedback: The endocrine system consists of various glands, tissues, or clusters of cells that produce and release hormones. Hormones are chemical messengers that stimulate and/or regulate the actions of other tissues, organs, or endocrine glands that have specific receptors to a hormone. Along with the nervous system, the endocrine milieu influences all physiologic effects such as growth and development, metabolic processes related to fluid and electrolyte balance and energy production, sexual maturation and reproduction, and the body's response to stress. The release patterns of the hormones vary, but the level in the body is maintained within specified limits to preserve health.

1. The nurse is teaching the mother of a 12-year-old boy about the risk factors associated with drug and alcohol abuse. Which response by the mother indicates a need for further teaching? A) "A family history of alcoholism is a risk factor for substance abuse." B) "Just because his friends are experimenting does not mean that he will." C) "If my husband or I have a substance abuse problem it could increase his risk." D) "Negative life events are a potential risk factor."

Ans: B Feedback: The nurse needs to emphasize that a peer group that abuses substances is a risk factor associated with substance abuse and increases the chances of a child experimenting. Other risk factors include a family history of substance abuse, current parental substance abuse, and negative life events.

2. The nurse is caring for an 8-year-old girl with hyperpituitarism. What ordered treatment will the nurse expect to perform? A) Give desmopressin acetate intranasally B) Inject octreotide acetate C) Give 1 mg/kg/day of methimazole D) Administer glipizide orally

Ans: B Feedback: The nurse would give the child a subcutaneous injection of octreotide acetate every 12 hours as directed. Desmopressin is a synthetic antidiuretic hormone used to treat diabetes insipidus. Methimazole is an antithyroid drug used to treat hyperthyroidism. Glipizide is a hypoglycemic drug that assists insulin production in children with diabetes mellitus type 2.

7. The nurse is caring for an 8-year-old girl with an endocrine disorder involving the posterior pituitary gland. What care would the nurse expect to implement? A) Instructing the parents to report adverse reactions to the growth hormone treatment B) Teaching the parents how to administer the desmopressin acetate C) Informing the parents that treatment stops when puberty begins D) Educating the parents to report signs of acute adrenal crisis

Ans: B Feedback: The nurse would teach the parents how to administer desmopressin acetate, which treats diabetes insipidus, a disorder related to the posterior pituitary gland. Instructing parents to report adverse reactions to growth hormone is an intervention for growth hormone deficiency. Informing the parents that treatment stops at the normal time of puberty is a teaching intervention for precocious puberty. Educating the parents to report signs of an acute adrenal crisis is an intervention for congenital adrenal hyperplasia. All three of these other disorders are related to the anterior pituitary.

14. After teaching the parents of a daughter with central precocious puberty about medication therapy, which statement by the parents indicates successful teaching? A) "She needs to use the nasal spray once every day." B) "She'll start puberty again when the medication stops." C) "This medication will slow down the changes but not reverse them." D) "Once therapy is done, she'll need surgery."

Ans: B Feedback: Treatment for central precocious puberty involves administering a gonadotropin-releasing hormone (GnRH) analog. When it is stopped, puberty resumes according to the appropriate developmental stages. This analog can be given by depot injection every 3 to 4 weeks, a daily subcutaneous injection, or an intranasal spray two or three times per day. With GnRH analog treatment, secondary sexual development stabilizes or regresses. Surgery is indicated only if there is a tumor.

19. A nurse is conducting a screening program for autism in infants and children. What would the nurse identify as a warning sign? A) Lack of babbling by 6 months B) Inability to say a single word by 16 months C) Lack of gestures by 8 months D) Inability to use two words by 18 months

Ans: B Feedback: Warning signs of autism include no babbling by 12 months, no pointing or using gestures by 12 months, no single words by 16 months, no two-word utterances by 24 months, and loss of language or social skills at any age.

18. The nurse is reviewing the laboratory test results of a child with Addison disease. What would the nurse expect to find? A) Hypernatremia B) Hyperkalemia C) Hyperglycemia D) Hypercalcemia

Ans: B Feedback: With Addison disease, the child would exhibit hyperkalemia, hyponatremia, and hypoglycemia. Hypercalcemia would be associated with hyperparathyroidism.

Origin: Chapter 14, 10 10. When the nurse is assessing a child's pain, which is most important? A) Obtaining a pain rating from the child with each assessment B) Using the same tool to assess the child's pain each time C) Documenting the child's pain assessment D) Asking the parents about the child's pain tolerance

Ans: B Feedback: Although obtaining a pain rating, documenting the assessment, and asking the child's parents about the pain are important, the most important aspect of pain assessment is to use the same tool each time so that appropriate comparisons can be made and effective interventions can be planned and implemented. Consistency allows the most accurate assessment of the child's pain.

Origin: Chapter 6, 26 26. When providing anticipatory guidance to a group of parents with school-aged children, what would the nurse describe as the most important aspect of social interaction? A) School B) Peer relationships C) Family D) Temperament

Ans: B Feedback: Although school, family, and temperament are important influences on social interaction, peer relationships at this time provide the most important social interaction for school-age children.

19. The nurse is caring for a 7-year-old boy who needs his left leg immobilized. What is the priority nursing intervention? A) Enlist the assistance of a child life specialist. B) Explain to the boy that he must keep his leg very still. C) Apply a clove-hitch restraint to the boy's left leg. D) Explain that a restraint will be applied if he cannot hold still.

Ans: B Feedback: An explanation about the desired goal is necessary and appropriate for a 7-year-old child to understand what is required. In many cases, this will be all that is needed. Explaining that a restraint will be applied if the boy cannot hold still will likely be perceived as a threat or punishment. All alternative measures need to be tried before the use of restraints. Enlisting the assistance of the child life specialist is not a priority.

Origin: Chapter 14, 23 23. The nurse is caring for a child who reports chronic pain. What is the priority nursing assessment? A) How the pain impacts the child's and family's stress level B) The pain's history, onset, intensity, duration, and location C) The child's and parents' feeling of anxiety and depression D) The child's cognitive level and emotional response

Ans: B Feedback: Assessment of the child's pain is key; it is the priority assessment and is the only answer that focuses on the child's physiologic need. Assessment of how the pain impacts the child's and family's stress, feelings of anxiety, hopelessness, and depression, as well as the child's cognitive level and emotional response, are secondary after the pain is explored.

Origin: Chapter 6, 8 8. The nurse is talking with a chatty 7-year-old girl during her regular check-up. Which behaviors would the child also be expected to exhibit? A) Showing no interest in what the nurse sees in her ears B) Explaining what is right and what is wrong C) Demonstrating independence from her mother D) Showing no concern when the nurse hurts her own finger

Ans: B Feedback: At this age, behavior is seen by the child as either completely right or wrong. The child will almost surely want to know why the nurse looks in her ears. The child depends heavily on parents for support and encouragement at this age. This is a time when children gain empathy, so the child would show concern for the nurse's injury.

Origin: Chapter 14, 16 16. The student nurse is learning about the effects of heat and cold when used in a pain management plan. What accurately describes one of these effects? A) Cold results in vasodilation. B) Cold alters capillary permeability. C) Heat results in vasoconstriction. D) Heat decreases blood flow to the area.

Ans: B Feedback: Cold results in vasoconstriction and alters capillary permeability, leading to a decrease in edema at the site of the injury. Heat results in vasodilation and increases blood flow to the area.

Origin: Chapter 6, 13 13. The nurse is teaching the parents of a 9-year-old girl about the socialization that is occurring in their child through school contacts. Which information would the nurse include in her teaching plan? A) Teachers are the most influential people in the development of the school-age child's social network. B) Continuous peer relationships provide the most important social interaction for school-age children. C) Parents should establish norms and standards that signify acceptance or rejection. D) A characteristic of school-age children is their formation of groups with no rules and values involved.

Ans: B Feedback: Continuous peer relationships provide the most important social interaction for school-age children. Peer and peer-group identification are most essential to the socialization of the school-age child. Peer groups establish norms and standards that signify acceptance or rejection. Valuable lessons are learned from interactions with children their own age. A characteristic of school-age children is their formation of groups with rules and values.

Origin: Chapter 14, 14 14. The nurse tells a joke to a 12-year-old to distract him from a painful procedure. What pain management technique is the nurse using? A) Relaxation B) Distraction C) Imagery D) Thought stopping

Ans: B Feedback: Distraction involves having the child focus on another stimulus, thereby attempting to shield him from pain. Humor has been demonstrated to be an effective distracting technique for pain management.

Origin: Chapter 5, 12 12. The parents of a 5-year-old boy tell the nurse that their son is having frequent episodes of night terrors. Which of the following statements would indicate that the boy is having nightmares instead of night terrors? A) "It usually happens about an hour after he falls asleep." B) "He will tell us about what happened in his dream." C) "He is completely unaware that we are there." D) "When we try to comfort him, he screams even more."

Ans: B Feedback: During a nightmare, a child will have a memory of the occurrence and may remember the dream and talk about it later. With night terrors, the child has no memory of the event. The other statements are indicative of night terrors.

24. When describing organ donation to the family of a dying child, what would the nurse include in the discussion? A) Telling them that further harm may occur to the child through the process B) Tell them that their cultural and religious beliefs will be considered C) Including this topic in the discussion of impending death D) Informing the family that organ donation will delay the funeral

Ans: B Feedback: During organ donation, the family's cultural and religious beliefs must be considered, and the team discussing organ donation with the family must do so in a sensitive and ethical manner. The donating child will not suffer further because of organ donation. The topic of organ donation should be separated from the discussion of impending death or brain death notification. Organs are harvested in a timely fashion after the declaration of death so the family need not worry about delay of the wake or funeral.

16. The nurse is providing home care for the family of an 8-year-old boy who is dying of leukemia. Which action will be most supportive to the parents of the child? A) Encouraging organ and tissue donation B) Being patient with parental indecision C) Getting prior authorization for treatments D) Explaining how anorexia is a natural process

Ans: B Feedback: It is critical to be patient with parents who may vacillate when making decisions. Give them the information and time they need to make decisions and avoid being judgmental. Explaining about anorexia and encouraging organ donation may be discussed when the parents indicate they are concerned. Getting prior authorization facilitates care delivery and is not a supportive intervention.

25. A child is admitted to the hospital with a spinal cord injury resulting in paralysis below the level of the waist. When should the nurse begin planning with the parents for rehabilitation placement for this child after acute hospitalization? A) After hospitalization when the parents are ready B) As soon after the patient is admitted as possible C) When the child starts showing improvement in their condition D) Once the child and the parent feel it is time to seek extended care

Ans: B Feedback: It is important to begin planning for discharge to a rehabilitation facility as soon as possible so that all necessary arrangements can be made prior to discharge. Waiting until the parent or child feel the time is right or waiting until improvement is noted may not leave enough time for thorough discharge planning.

Origin: Chapter 6, 30 30. The mother of a 12-year-old boy is talking with the school nurse about her son's clumsiness. She reports that he seems to fall a lot, his writing is horrible, and as much as he practices he can't play his guitar very well. How should the nurse respond to the mother? A) "Boys tend to take a bit longer than girls to mature." B) "Have you spoken with your pediatrician about your observations?" C) "Boys tend to refine their fine motor skills by this age." D) "I will make a note of your observations and talk to his teachers."

Ans: B Feedback: Myelinization of the central nervous system is reflected by refinement of fine motor skills. The child between 10 and 12 years of age begins to exhibit manipulative skills comparable to adults. In order to determine if the child is delayed in fine motor skill development, the pediatrician should be made aware because further examination or testing may be warranted. Just stating the fact that his motor skills should be developed by this age, although true, does not address the mother's concerns. The teachers can be notified of the mother's observations, but the child should still be assessed by the pediatrician.

Origin: Chapter 5, 28 28. The nurse is providing anticipatory guidance for parents of a preschooler regarding sex education. What is a recommended guideline when dealing with this issue? A) Be prepared to thoroughly cover a topic before the child asks about it. B) Before answering questions, find out what the child thinks about the subject. C) Expand upon the topic when answering questions to prevent further confusion. D) Provide a less than honest response to shelter the child from knowledge that is too advanced.

Ans: B Feedback: Preschoolers are very inquisitive and want to learn about everything around them; therefore, they are very likely to ask questions about sex and where babies come from. Before attempting to answer questions, parents should try to find out first what the child is really asking and what the child already thinks about that subject. Then they should provide a simple, direct, and honest answer. The child needs only the information that he or she is requesting.

Origin: Chapter 6, 28 28. The nurse is providing anticipatory guidance for parents of a school-age child on teaching the dangers of drugs and alcohol. What advice might be helpful for these parents? A) School-age children are not ready to absorb information that deals with drugs and alcohol. B) School-age children can think critically to interpret messages seen in advertising, media, and sports. C) Parents must prevent their child from being exposed to messages that are in conflict with their values. D) Discussions with children need to be based on facts and focused on the past and future.

Ans: B Feedback: School-age children can be taught how to think critically to interpret messages seen in advertising, media, sports, and entertainment personalities. School-age children are ready to absorb information that deals with drugs and alcohol and may be exposed to messages that are in conflict with their parents' values regarding smoking and alcohol. This may occur at school and cannot be prevented. Discussions with children need to be based on facts and focused on the present.

Origin: Chapter 6, 10 10. The school nurse is conducting vision screening for a 7-year-old girl and documents the condition 'amblyopia.' What would the nurse tell the parents about this condition? A) 'Amblyopia is an uncorrected refractive error of the eye.' B) 'Amblyopia is reduced vision in an eye that has not been adequately used during early development.' C) 'Amblyopia is a malalignment of the eye, which occurs at birth.' D) 'Amblyopia is a clouding of the lens of the eye caused by trauma to the eye.'

Ans: B Feedback: Some problems frequently identified in school-age children include amblyopia (lazy eye), uncorrected refractive errors or other eye defects, and malalignment of the eyes (called strabismus). Amblyopia is reduced vision in an eye that has not been adequately used during early development. Inadequate use can result from conditions such as strabismus, being cross-eyed, or one eye being more nearsighted, farsighted, or astigmatic than the other eye. Amblyopia is the leading cause of visual impairment in children (National Eye Institute, 2008) and if untreated can result in vision loss.

16. An adolescent is scheduled for outpatient arthroscopic surgery on his knee next week. As part of preparing him for the procedure, which action would be most appropriate? A) Discussing the events with the adolescent and his mother upon arrival the morning of the procedure B) Providing detailed explanations of the procedure at least a week in advance of the procedure C) Encouraging the parent to stay with the adolescent as much as possible before the procedure D) Answering the adolescent's questions with simple answers, encouraging him to ask the surgeon

Ans: B Feedback: The adolescent needs a detailed explanation about the procedure at least 7 to 10 days beforehand. Waiting until the morning of the procedure would be inappropriate. However, information could be clarified and additional questions could be answered at this time. Having the parent stay with the adolescent is something that the adolescent would need to decide; he may or may not want a parent present. Referring the adolescent to the surgeon for his questions is inappropriate and ignores the adolescent's desire for control and information.

Origin: Chapter 5, 1 1. The nurse is conducting a well-child examination of a 5-year-old girl, who was 40 inches tall at her last examination at age 4. Which height measurement would be within the normal range of growth expected for a preschooler? A) 41 inches B) 43 inches C) 45 inches D) 47 inches

Ans: B Feedback: The average preschool-age child will grow 2.5 to 3 inches (6.5 to 7.8 cm) per year. The average 3-year-old is 37 inches tall (96.2 cm), the average 4-year-old is 40.5 inches tall (103.7 cm), and the average 5-year-old is 43 inches tall (118.5 cm).

19. The parents of an 11-year-old boy who is dying from cancer are concerned that he is not eating. Which intervention would serve both the parents' and child's needs? A) Urging the child to eat one good meal per day B) Serving small meals of things the child likes C) Straightening up around the child before meals D) Administering antiemetics as ordered for nausea

Ans: B Feedback: The child is more likely to eat small amounts of foods of his choosing. This accommodates the child's reduced appetite, reassures his parents that he is not starving, and gives the child a sense of control. Straightening up the child's area before meals provides a more pleasant eating environment. The use of antiemetics controls nausea but may not increase appetite. Urging the child to eat a substantial meal is unnecessary and creates stress.

Origin: Chapter 5, 13 13. The mother of a 5-year-old boy calls the nurse and seeks advice on how to assist the child with the recent death of his paternal grandfather. The boy keeps asking when his grandpa is coming back. How should the nurse respond? A) "It is best to just ignore this and to not respond to his questions." B) "This is normal; children his age do not understand the permanence of death." C) "You have to keep repeating that his grandfather is never coming back." D) "He will eventually figure this out on his own."

Ans: B Feedback: The nurse needs to remind the mother that preschoolers do not completely understand the concept of death or its permanence. Telling the mother that it is best to ignore the boy's questions or that the boy will eventually figure this out on his own does not teach. Repeating that the grandfather is not coming back does not consider the developmental stage of the child and is inappropriate.

14. The nurse is caring for an 11-year-old girl preparing to undergo a magnetic resonance imaging (MRI) scan. Which statement would best help prepare the girl for the test and decrease anxiety? A) "You won't hear a sound if you wear your headphones." B) "The machine makes a very loud rattle; however, headphones will help." C) "There are a variety of loud sounds you will hear." D) "The MRI scanner sounds like a machine gun."

Ans: B Feedback: The nurse should acknowledge that an MRI is loud and briefly describe the noises the machine makes. Then, the nurse should immediately offer a solution: headphones. Telling the girl she won't hear a sound is untrue. Telling her that there are loud sounds isn't enough and could increase anxiety. Comparing the MRI scanner to the sound of a machine gun is not appropriate imagery for a child.

21. The nurse is caring for an immunosuppressed 3-year-old girl and is providing teaching to the mother about proper oral hygiene. Which response from the mother indicates a need for further teaching? A) "I really need to carefully check for skin breakdown." B) "I must really scrub her teeth and gums well." C) "I must use a soft toothbrush." D) "I can use a soft gauze sponge to care for her gums."

Ans: B Feedback: The nurse should caution the mother that overly vigorous brushing should be avoided as it can injure or irritate the gums. The other statements are recommended guidelines for care.

6. The nurse is caring for a 10-year-old boy who is in traction. The boy has a nursing diagnosis of deficient diversional activity related to confinement in bed that is evidenced by verbalization of boredom and lack of participation in play, reading, and schoolwork. What would be the best intervention? A) Offer the child reading materials. B) Enlist the aid of a child life specialist. C) Encourage the child to complete his homework. D) Ask for the parents' assistance.

Ans: B Feedback: The nurse should enlist the aid of a child life specialist to provide suggestions for appropriate activities. Offering the child reading materials or encouraging him to complete his homework would most likely be met with resistance as he has already verbalized his boredom and disinterest in play, reading, and schoolwork. The parents could offer the child life specialist ideas about the boy's likes and dislikes; however, the child life specialist could offer expertise in assisting hospitalized children.

Origin: Chapter 5, 5 5. The parents of a 4-year-old ask the nurse when their child will be able to differentiate right from wrong and develop morals. What would be the best response of the nurse? A) 'The preschooler has no sense of right and wrong.' B) 'The preschooler is developing a conscience.' C) 'The preschooler sees morality as internal to self.' D) 'The preschooler's morals are their own, right or wrong.'

Ans: B Feedback: The preschool child can understand the concepts of right and wrong and is developing a conscience. Preschool children see morality as external to themselves; they defer to power (that of the adult). The child's moral standards are those of their parents or other adults who influence them, not necessarily their own.

9. After teaching a group of students about therapeutic play, the instructor determines that additional teaching is needed when the students identify what as a characteristic of therapeutic play? A) Focus on coping B) Use of a highly structured format C) Dramatization of emotions D) Expression of feelings

Ans: B Feedback: Therapeutic play is nondirected play, focused on helping the child cope with feelings and fears. Real-life stressors and emotions can be acted out or dramatized, allowing the child to express his or her feelings.

14. The nurse is helping a 20-year-old woman transition to adult care. Which would be the most important role of the nurse following a successful transition? A) Teacher B) Consultant C) Care provider D) Advocate

Ans: B Feedback: Transition planning involves multidisciplinary care coordination; acknowledgement of the changing roles among the youth, family, and health care professionals; and fostering of the youth's self-determination skills. Prior to transition, educating the client is the most important role of the nurse. After the transition, the nurse serves as a consultant to the adult office in relation to the teen's needs. The nurse consults with a transition services coordinator or other service agency as available in the local community.

20. The nurse is caring for a child involved in an automobile accident whose family has been informed that the child is brain dead. What teaching might the nurse provide the family regarding organ donation? A) The nurse should ask about organ donation when the family is informed of their child's condition. B) The nurse should explain that written consent is necessary for the organ donation. C) The nurse should make sure the parents know that procurement of organs may mar their child's appearance. D) The nurse should make sure the parents know that they will be responsible for expenses related to organ procurement.

Ans: B Feedback: Written consent is necessary for organ donation, so the family must be appropriately informed and educated. The discussion of organ donation should be separate from the discussion of impending death or brain death notification. Families need to know that procurement of the organs does not mar the child's appearance, so that an open casket at the child's funeral is still possible if the family desires. All expenses for organ procurement are borne by the recipient's family, not the donor's.

The nurse is teaching the student nurse the physiology involved in pain transmission. Which statements accurately describes a physiologic event in the nervous system related to pain transmission? Select all that apply. A) Thermal stimulation may involve the release of mediators, such as histamine, prostaglandins, leukotrienes, or bradykinin. B) When nociceptors are activated by noxious stimuli, the stimuli are converted to electrical impulses that are relayed to the spinal cord and brain. C) Myelinated A-delta fibers are large fibers that conduct the impulse at very rapid rates; unmyelinated small C fibers transmit the impulse slowly. D) Once in the dorsal horn of the spinal cord, the nerve fibers divide and then cross to the opposite side and rise upward to the thalamus. E) The point at which the person first feels the highest intensity of the painful stimulus is termed the pain threshold. F) Peripheral sensitization allows the nerve fibers to react to a stimulus that is of lower intensity than would be needed to cause pain.

Ans: B, C, D, F Feedback: When nociceptors are activated by noxious stimuli, the stimuli are converted to electrical impulses that are relayed along the peripheral nerves to the spinal cord and brain. Myelinated A-delta fibers are large fibers that conduct the impulse at very rapid rates; unmyelinated small C fibers transmit the impulse slowly. Once in the dorsal horn of the spinal cord, the nerve fibers divide and then cross to the opposite side and rise upward to the thalamus. Peripheral sensitization allows the nerve fibers to react to a stimulus that is of lower intensity than would be needed to cause pain. Chemical stimulation may involve the release of mediators, such as histamine, prostaglandins, leukotrienes, or bradykinin. The point at which the person first feels the lowest intensity of the painful stimulus is termed the pain threshold.

15. A child with depression is prescribed fluoxetine. The nurse identifies this as belonging to which class of drugs? A) Atypical antidepressant B) Tricyclic antidepressant C) Selective serotonin reuptake inhibitor D) Psychostimulant

Ans: C Feedback: Fluoxetine (Prozac) is a selective serotonin reuptake inhibitor. Trazodone is an atypical antidepressant; amitriptyline, desipramine, imipramine, and nortriptyline are tricyclic antidepressants. Methylphenidate and the amphetamines are psychostimulants.

4. The nurse is assessing a 9-year-old girl with a history of tuberculosis at age 6 years. She has been losing weight and has no appetite. The nurse suspects Addison disease based on which assessment findings? A) Arrested height and increased weight B) Thin, fragile skin and multiple bruises C) Hyperpigmentation and hypotension D) Blurred vision and enuresis

Ans: C Feedback: Hyperpigmentation and hypotension would point to Addison disease. Arrested height and increased weight are typical of acquired hypothyroidism; this girl has lost weight. Thin, fragile skin and multiple bruises are indicative of Cushing syndrome. Blurred vision, headaches, and enuresis would be complaints of a child with diabetes mellitus.

Origin: Chapter 6, 3 3. The pediatric nurse is aware of the maturation of organ systems in the school-age child. What accurately describes these changes? Select all that apply. A) The brain grows very slowly during the school-age years and growth is complete by the time the child is 12 years of age. B) Respiratory rates decrease, abdominal breathing disappears, and respirations become diaphragmatic in nature. C) The school-age child's blood pressure increases and the pulse rate decreases, and the heart grows more slowly during the middle years. D) The school-age child experiences more gastrointestinal upsets compared with earlier years since the stomach capacity increases. E) Bladder capacity increases, but varies among individual children, and girls generally have a greater bladder capacity than boys. F) Prepubescence typically occurs in the 2 years before the beginning of puberty and is characterized by the development of secondary sexual characteristics.

Ans: B, C, E, F Feedback: Respiratory rates decrease, abdominal breathing disappears, and respirations become diaphragmatic in nature. The school-age child's blood pressure increases and the pulse rate decreases. The heart grows more slowly during the middle years and is smaller in size in relation to the rest of the body than at any other development stage. Bladder capacity increases, but varies among individual children. Girls generally have a greater bladder capacity than boys. Prepubescence typically occurs in the 2 years before the beginning of puberty and is characterized by the development of secondary sexual characteristics. The brain and skull grow very slowly during the school-age years. Brain growth is complete by the time the child is 10 years of age. The school-age child experiences fewer gastrointestinal upsets compared with earlier years. Stomach capacity increases, which permits retention of food for longer periods of time.

24. A nurse is reviewing the medical record of an 11-year-old child with a conduct disorder. What would the nurse identify as characteristics of this disorder? Select all answers that apply. A) Easily annoyed B) Initiator of physical fights C) Temper tantrums D) Truancy E) Arrest for arson

Ans: B, D, E Feedback: Behaviors associated with conduct disorder include initiation of physical fights, arson, and truancy. Becoming easily annoyed and experiencing temper tantrums are associated with oppositional defiant disorder.

13. The nurse is performing an admission of a 10-year-old boy. Which actions will help the nurse establish a trusting and caring relationship with the child and his family? Select all that apply. A) The nurse should not minimize the child's fears by smiling. B) The nurse should initiate introductions. C) The nurse should not use formal titles at the introduction. D) The nurse should maintain eye contact at the appropriate level. E) The nurse should start communication with the child first and then move on to the family. F) The nurse should use age-appropriate communication with the child.

Ans: B, D, F Feedback: Regardless of the site of care, nursing care must begin by establishing a trusting, caring relationship with the child and family. The nurse should smile, start introductions, give his or her title, and let the child and family know what will happen and what is expected of them. The nurse should also maintain eye contact at the appropriate level, communicate with children at age-appropriate levels, and, with a younger child, start with the family first so the child can see that the family trusts you.

Origin: Chapter 5, 9 9. The parents of a preschooler ask the nurse to help them choose a preschool for their child. What are recommended guidelines and goals for choosing a preschool? Select all that apply. A) The main goal of preschool is to improve reading and writing skills and readiness for entering into grade school. B) When selecting a preschool the parent may want to consider the accreditation of the school and the teachers' qualifications. C) The teachers should decide how focused on curriculum the school should be for each individual student. D) The parent should observe the classroom, evaluating the environment, noise level, and sanitary practices. E) The type of discipline used in the school is also an important factor. Parents should choose a preschool that uses corporal punishment. F) The parent should observe the classroom to determine how the children interact with each other and how the teachers interact with the children.

Ans: B, D, F Feedback: When selecting a preschool, the parent may want to consider the accreditation of the school, the teachers' qualifications, and recommendations of other parents. The parent should observe the classroom, evaluating the environment, noise level, and sanitary practices, as well as how the children interact with each other and how the teachers interact with the children. The main goal of preschool is to foster the child's social skills and accustom him or her to the group environment. The parents must decide how focused on curriculum they want the school to be. The type of discipline used in the school is also an important factor. Parents should not choose a preschool that uses corporal punishment.

4. The nurse is caring for a 7-year-old with Tourette syndrome. The nurse would be alert for which comorbid condition? A) Depression B) Anxiety disorder C) Attention deficit/hyperactivity disorder D) Asperger syndrome

Ans: C Feedback: Attention deficit/hyperactivity disorder and obsessive-compulsive disorders occur in 50% of children with Tourette syndrome. Depression, anxiety disorder, and Asperger syndrome are not typical comorbid conditions associated with Tourette syndrome.

28. The nurse is speaking with a parent regarding their child's recent diagnosis of oppositional defiant disorder. Which statement by the parent would cause the nurse to question the diagnosis? A) "I am so tired of arguing with my daughter all the time." B) "My son purposely does exactly the opposite of what his father tells him to do." C) "I feel so bad that my daughter intentionally hurt the neighbor's cat." D) "My daughter gets so annoyed at me when she doesn't get her way."

Ans: C Feedback: Common behaviors in oppositional defiant disorder include excessive arguing with adults, active defiance, noncompliance with adult requests or limits and easily annoyed. Physical cruelty to animals or people is associated with conduct disorder, not oppositional defiant disorder.

9. The nurse is preparing a teaching plan for a 10-year-old girl with hyperthyroidism. What information would the nurse include in the plan? A) Describing surgery to remove an anterior pituitary tumor B) Teaching her parents to give injections of growth hormone C) Explaining about the radioactive iodine procedure D) Showing her parents how to give DDAVP intranasally

Ans: C Feedback: Explaining about the radioactive iodine procedure would be part of the teaching plan for a child with hyperthyroidism because this is a less invasive type of therapy for the disorder. Describing surgery to remove an anterior pituitary tumor would be included for a child with hyperpituitarism. Teaching a parent to give injections of growth hormone would be appropriate for a child with a growth hormone deficiency. Showing parents how to give DDAVP intranasally is appropriate for a child with diabetes insipidus.

23. After teaching the parents of a child with attention deficit/hyperactivity disorder about ways to control the child's behavior, the nurse determines a need for additional teaching when the parents state: A) "If he starts to act out, we'll have him do a time-out to help him refocus." B) "We can use a reward system when he behaves appropriately." C) "If he misbehaves, we need to punish him instead of reward him." D) "We need to help him set realistic goals that he can achieve."

Ans: C Feedback: Punishment for misbehaving would be inappropriate because it would lead to negative feelings and further decrease self-esteem. Appropriate behavior management strategies include time-outs, positive reinforcement, reward or privilege withdrawal, or a token system. Setting realistic goals also is helpful to foster self-esteem and independence.

6. The nurse is caring for a 3-year-old boy. The parents are concerned that he is exhibiting signs of cognitive delays. Which statement by the parents would lead the nurse to suspect autism spectrum disorder rather than possible learning disability? A) "He is not speaking in complete sentences." B) "We can understand a lot of what he says, but no one else can." C) "He seems to be speaking words less and less frequently." D) "He is unable to sit still for a short story."

Ans: C Feedback: Reports of regression or the loss of previously acquired skills points to autism rather than intellectual disability. Not speaking in complete sentences, others not being able to understand what the child is saying, and an inability to sit still for a short story suggest a learning disability.

5. A nurse is caring for a 10-year-old boy with a nursing diagnosis of ineffective coping related to an inability to deal with stressors secondary to anxiety. What action should the nurse to take first? A) Set clear limits on the child's behavior B) Teach the child problem-solving skills C) Encourage a discussion of the child's thoughts and feelings D) Role model appropriate social and conversation skills

Ans: C Feedback: The priority action is to encourage the child to discuss his thoughts and feelings. This is the initial step toward learning to deal with them appropriately. Setting clear limits, teaching problem-solving skills, and role modeling skills would be appropriate as the child begins to learn how to acknowledge and deal with his feelings.

30. A teenage girl diagnosed with polycystic ovary syndrome tells the nurse, "I refuse to take oral contraceptives since I am not sexually active." What is the best response to the girl? A) "It's important for you to take the pills even if you're not sexually active in order to prevent unwanted symptoms of the disease." B) "The doctor has prescribed these for you because it is an effective treatment method for the disease." C) "I know it's hard remembering to take those pills every day. Tell me more about what is making you not want to take the oral contraceptives." D) "Do your parents know that you are not taking the treatment medication your doctor prescribed?"

Ans: C Feedback: This response shows empathy to the client and encourages her to further discuss the reasons they are non-compliant with the prescribed treatment regimen. "It's important for you to take the pills even if you're not sexually active...," and "The doctor has prescribed these for you because it is an effective treatment..." are accurate statements, but they are not methods of therapeutic communication and do not lead to further discussion about the noncompliance. Asking if the parents know she isn't taking the medications leads to mistrust of the nurse.

15. A group of nursing students are reviewing information about the endocrine system in infants and children. The students demonstrate understanding of the information when they state: A) Endocrine glands begin developing in the third trimester of gestation. B) At birth, the endocrine glands are completely functional. C) Infants have difficulty balancing glucose and electrolytes. D) A child's endocrine system has little effect on growth and development.

Ans: C Feedback: Typically, most endocrine glands begin to develop during the first trimester of gestation, but their development is incomplete at birth. Thus, complete hormonal control is lacking during the early years of life, and the infant cannot appropriately balance fluid concentration, electrolytes, amino acids, glucose, and trace substances.

8. The nurse is assessing a 13-year-old boy with type 2 diabetes mellitus. What would the nurse correlate with disorder? A) The parents report that their child had "a cold or flu" recently. B) Blood pressure is decreased when checking vital signs. C) The parents report that their son "can't drink enough water." D) Auscultation reveals Kussmaul breathing.

Ans: C Feedback: Unquenchable thirst (polydipsia) is a common finding associated with diabetes mellitus, type 1 and 2. However, reports of flu-like illness and Kussmaul breathing are more commonly associated with type 1 diabetes. Blood pressure is normal with type 1 diabetes and elevated with type 2 diabetes.

1. The nurse working in the emergency room monitors the admission of children. Statistically, for which disorder would children younger than 5 years most commonly be admitted? A) Mental health problems B) Injuries C) Respiratory disorders D) Gastrointestinal disorders

Ans: C Feedback: According to Child Health USA 2010, diseases of the respiratory system, such as asthma and pneumonia, account for the majority of hospitalizations in children younger than 5 years of age, while diseases of the respiratory system, mental health problems, injuries, and gastrointestinal disorders lead to more hospitalizations in older children.

12. The nurse is reviewing the Adolescent Health Transition Project's recommended schedule for transition planning. According to the schedule, at what age should the nurse explore health care financing for young adults? A) 12 years old B) 14 years old C) 17 years old D) 19 years old

Ans: C Feedback: By age 14, the nurse should ensure that a transition plan is initiated and that the individualized education plan (IEP) reflects post-high school plans. By age 17, the nurse should explore health care financing for young adults. The nurse should check the teen's eligibility for Supplemental Security Income (SSI) the month the child turns 18. By age 21, the nurse should ensure that the young adult has registered with the Division of Developmental Disabilities for adult services if applicable.

Origin: Chapter 5, 7 7. The nurse is assessing the motor skills of a 5-year-old girl. Which finding would cause the nurse to be concerned? A) Can copy a square on another piece of paper B) Can dress and undress herself without help C) Draws a person with three body parts D) Is beginning to tie her own shoelaces

Ans: C Feedback: By the age of 5 years, the child should be able to draw a person with a body and at least six body parts. She should also be able to copy triangles and other geometric patterns and dress and undress herself and should be learning to tie her shoelaces.

29. The mother of a 7-year-old boy with autism tearfully reports feeling as if she is not qualified to care for her child. Which initial action by the nurse is most appropriate? A) Tell the child's mother that this is a common feeling when caring for a special needs child. B) Encourage the child's mother to keep a journal to best identify areas needing improvement in the home routine. C) Recognize the mother's positive accomplishments in caring for her child. D) Recommend the child's mother seek counseling.

Ans: C Feedback: Caring for a special needs child can be overwhelming for the parents. Feeling overwhelmed is not uncommon. Recognition of positive outcomes and activities should be performed. The child's mother may indeed benefit from counseling or participation in a support group but it is not of the highest priority. Explaining to the child's mother that others feel the same way does not address her personal concerns. Keeping a journal may be effective but suggestions that this will help improve her performance are not meeting her immediate needs.

24. The nurse is working as a community health care nurse. What would be the nurse's focus when providing care of the child? A) Providing care to the individual and family in acute care settings B) Providing care to the indigent in family care settings C) Providing care in geographically and culturally diverse settings D) Providing care for particular age groups or particular diagnoses

Ans: C Feedback: Community health nurses work in geographically and culturally diverse settings. They address current and potential health needs of the population or community. Community-based nursing focuses more on providing care to the individual or family (which, of course, impacts the community) in settings outside of acute care. They promote and preserve the health of a population and are not limited to particular age groups, income levels, or diagnoses.

Origin: Chapter 14, 27 27. The nurse is preparing a child for a lumbar puncture. How far ahead of the procedure should the nurse apply the EMLA cream? A) 30 minutes B) 1 hour C) 3 hours D) 4 hours

Ans: C Feedback: For a deeper procedure such as a lumbar puncture, the nurse needs to apply the cream 2 to 3 hours before the procedure. For a superficial procedure, the EMLA cream should be applied at least 1 hour before the procedure.

Origin: Chapter 6, 1 1. The nurse is performing a physical assessment of a 10-year-old boy. The nurse notes that during last year's check-up the child weighed 80 pounds. According to average growth for this age group, what would be his expected current weight? A) 81 pounds B) 85 pounds C) 87 pounds D) 89 pounds

Ans: C Feedback: From 6 to 12 years of age, an increase of 7 pounds (3 to 3.5 kg) per year in weight is expected.

1. The nurse is teaching a group of parents with premature infants about the various medical and developmental problems that may occur. The nurse determines that additional teaching is needed when the group identifies what as a problem? A) Sudden infant death syndrome B) Hydrocephalus C) Peptic ulcer D) Bronchopulmonary dysplasia

Ans: C Feedback: Gastroesophageal reflux disease, not peptic ulcer, is a medical problem that commonly affects premature infants. Myriad problems may occur, including sudden infant death syndrome, hydrocephalus, bronchopulmonary dysplasia, cardiac changes, growth retardation, nutrient deficiencies, bradycardia, rickets, inguinal or umbilical hernias, visual problems, hearing deficits, delayed dentition, and growth delays.

17. The nurse is providing palliative care for a 9-year-old boy in hospice. Which is unique to hospice care for children? A) Encouraging visits from friends and family B) Educating parents about terminal dehydration C) Prolonging treatment that might possibly help D) Treating constipation to relieve abdominal pain

Ans: C Feedback: Hospice for children allows for continuation of hopeful treatment so long as certain criteria are met. This is different from adult hospice. Encouraging visits from friends and family, educating parents about terminal dehydration, and treating constipation are common to family-centered care.

2. The nurse is caring for a toddler with special needs. Which developmental tasks related to toddlerhood might be delayed in the child with special needs? A) Developing body image B) Developing peer relationships C) Developing language and motor skills D) Learning through sensorimotor exploration

Ans: C Feedback: In special needs children, developmental delays may occur in all stages. In particular, motor and language skill development may be delayed if the toddler is not given adequate opportunities to test his or her limits and abilities. Development of body image may be hindered in the preschooler due to painful exposures and anxiety. Development of peer relationships may be delayed in the school-age and adolescent child. The infant's ability to learn through sensorimotor exploration may be impaired due to lack of appropriate stimulation, confinement to a crib, or increased contact with painful experiences.

Origin: Chapter 5, 29 29. The parents of a preschooler express concern to the nurse about their son's new habit of masturbating. What is an appropriate response to this concern? A) Tell the child in a firm manner that this behavior is not acceptable. B) When the child displays this behavior, place him in a 'time-out.' C) Treat the action in a matter-of-fact manner emphasizing safety. D) Consult a psychotherapist to determine the reason for this behavior.

Ans: C Feedback: Masturbation is a healthy and natural part of normal preschool development if it occurs in moderation. If the parent overreacts to this behavior, then it may occur more frequently. Masturbation should be treated in a matter-of-fact way by the parent. The child needs to learn certain rules about this activity: nudity and masturbation are not acceptable in public. The child should also be taught safety: no other person can touch the private parts unless it is the parent, doctor, or nurse checking to see when something is wrong.

Origin: Chapter 14, 5 5. The nurse is caring for a child who is experiencing pain related to chemotherapy treatment. What is a behavioral factor that might affect the child's pain experience? A) Knowledge of the therapy B) Fear about the outcome of therapy C) Participation in normal routine activities D) Ability to identify pain triggers

Ans: C Feedback: Participation in normal routine activities is a behavior factor. Knowledge of the therapy and ability to identify pain triggers are cognitive factors. Fear about the outcome of therapy is an emotional factor. Situational factors involve factors or elements that interact with the child and his or her current situation involving the experience of pain.

A group of nursing students are reviewing information about neurocutaneous syndromes. What is an example of these disorders? A) Sturge-Weber syndrome B) Marfan syndrome C) Apert syndrome D) Achondroplasia

Ans: A Sturge-Weber syndrome is an example of a neurocutaneous syndrome. Marfan syndrome, Apert syndrome, and achondroplasia are autosomal dominantly inherited genetic disorders.

4. The nurse is caring for a 4-year-old girl who has been hospitalized for over a week with severe burns. Which would be a priority intervention to help satisfy this preschool child's basic needs? A) Encourage friends to visit as often as possible. B) Suggest that a family member be present with her 24 hours a day. C) Explain necessary procedures in simple language that she will understand. D) Allow her to make choices about her meals and activities as much as permitted.

Ans: C Feedback: Preschoolers fear mutilation and are afraid of intrusive procedures since they do not understand the body's integrity. They interpret words literally and have an active imagination; therefore, procedures should be demonstrated and/or explained in simple terms. Adolescents typically do not experience separation anxiety from being away from their parents; instead, their anxiety comes from being separated from friends, and therefore encouraging friends to visit is a priority intervention. Toddlers are especially susceptible to separation anxiety and would benefit from a family member being present as much as possible. School-age children are accustomed to controlling self-care and typically are highly social; they would benefit from being involved in choices about meals and activities.

Origin: Chapter 5, 24 24. A nurse is caring for a 4-year-old girl. The mother says that the girl is afraid of cats and dogs and does not like to go to the playground anymore because she wants to avoid the dogs that are often being walked at the park. What should the nurse tell the mother? A) "It is best to avoid the playground until she outgrows the fear." B) "She needs to face her fears head-on; take her to the park as much as possible." C) "Acknowledge her fear and help her develop a strategy for dealing with it." D) "Try to minimize her fears and insist that she go to the park."

Ans: C Feedback: Preschoolers have vivid imaginations and experience a variety of fears. It is best to acknowledge the fear, rather than minimize it, and then collaborate with the child on strategies for dealing with the fear. Avoiding the playground will not address the child's fears. Forcing the child to face her fear without enlisting her input to help deal with the fear does not teach. It is also important for the mother to find out if an incident involving cats and dogs occurred without her knowledge.

26. When speaking to a group of parents at a local elementary school, the nurse describes school nursing as a specialized practice of nursing based on the fact that a healthy child has a better chance to succeed in school. What best describes the strategy school nurses use to achieve student success? A) They coordinate all school health programs. B) They link community health services. C) They work to minimize health-related barriers to learning. D) They promote student health and safety.

Ans: C Feedback: School nurses work to remove or minimize health barriers to learning to give students the best opportunity for academic success. Coordinating school health programs, linking community health programs, and promoting health and safety are individual components within the ultimate goal of removing or minimizing health barriers.

26. The parents of a child with a developmental disability tell the nurse that they feel guilty because they sometimes find themselves feeling sad and wondering how their child would be without the disability. Which response by the nurse best shows empathy and encourages the parents to vent their feelings? A) "I'm sure it must be difficult to have a child developmentally delayed." B) "There are lots of parents that are experiencing the difficulty and feelings of hopelessness and grief you're having. Maybe if you talk to someone it might help you both." C) "I can only imagine how hard it is for you. You should know that it is common for parents to have these feelings when having a child with special needs." D) "It's important to focus on the positives that can come from the experience of being the parents of a child that has these issues."

Ans: C Feedback: Showing empathy by stating, "I can only imagine how hard it is for you" is important when developing rapport and supporting the parents, and letting them know that they are not alone in the feelings they are experiencing allows them to feel less guilty. Just stating, "I'm sure it must be difficult to have a child developmentally delayed" may convey empathy but it does not allow for open conversation. "There are lots of parents that are experiencing the difficulty and feelings of hopelessness and grief you're having. Maybe if you talk to someone it might help you both" doesn't convey empathy. "It's important to focus on the positives that can come from the experience of being the parents of a child that has these issues" does not address the parents' feelings.

21. The nurse is caring for a preschool child who is receiving palliative care for end-stage cancer. What would be the focus of age-appropriate interventions for this child? A) Providing unconditional love and trust B) Providing a familiar and consistent routine C) Teaching the child that death is not punishment D) Providing specific, honest details of death

Ans: C Feedback: Spirituality in the preschool years focuses on the concept of right versus wrong. The 3- to 5-year-old may see death as punishment for wrongdoing, and the nurse must correct this misunderstanding. For the infant, unconditional love and trust are of utmost importance. The toddler, 1 to 3 years old, thrives on familiarity and routine; the nurse should maximize the toddler's time with parents, be consistent, provide favorite toys, and ensure physical comfort. The school-age child has a concrete understanding of death. Children who are 5 to 10 years old need specific, honest details (as desired).

Origin: Chapter 6, 18 18. The nurse is teaching parents to plan nutritional meals for their 7-year-old son who is overweight. Which guideline might the nurse include in the teaching plan? A) School-age children with an average body weight of 20 to 35 kg need approximately 90 calories per kilogram daily. B) The average water requirement for a school-age child per 24 hours ranges from 2,000 to 2,500 mL per day. C) The school-age child needs 28 g of protein and 800 mg of calcium for maintenance of growth and good nutrition. D) In the school-age child, calories needed to sustain weight increase, while the appetite decreases.

Ans: C Feedback: The 4- to 8-year-old child needs 28 g of protein and 800 mg of calcium for maintenance of growth and good nutrition. School-age children with an average body weight of 20 to 35 kg need approximately 70 calories per kilogram daily (1,400 to 2,100 calories per day). The average water requirement per 24 hours ranges from 1,800 to 2,200 mL per day. Growth, body composition, and body shape remain constant during the late school-age years. Needed calories decrease while the appetite increases.

18.When assessing the vision of a 2-month-old, what would the nurse use? A) Black-and-white checkerboard B) Red and blue circles C) Gray and blue animal drawings D) Green and yellow letters

Ans: A Feedback: For infants younger than 6 months of age, objects such as a black-and-white checkerboard or concentric circles are best because an infant's vision is more attuned to these high-contrast patterns than to colors. High-contrast animal figures such as pandas or Dalmatians also work well.

Origin: Chapter 5, 30 30. The parents of a 5-year-old are concerned that their son is too short for his age. The nurse measures the child's height at 40 inches (101.6 cm). How should the nurse respond? A) "Some children are short for their age during the preschool years but usually catch up during early childhood." B) "Are most of the adults in your family short? It may be hereditary that your child will be shorter than average." C) "The average height for a 5-year-old is 43 inches tall (118.5cm), so your son is within the normal range for height." D) "I am sure his height is a concern, but if you start choosing nutrient-dense foods he will likely catch up to normal in height."

Ans: C Feedback: The average preschool-age child will grow 2.5 to 3 inches (6.5 to 7.8 cm) per year. The average 3-year-old is 37 inches tall (96.2 cm), the average 4-year-old is 40.5 inches tall (103.7 cm), and the average 5-year-old is 43 inches tall (118.5 cm).

Origin: Chapter 6, 4 4. The nurse is using the formula for bladder capacity to measure the bladder capacity of a 9-year-old girl. What number would the nurse document for this measurement? A) 9 ounces B) 10 ounces C) 11 ounces D) 12 ounces

Ans: C Feedback: The formula for bladder capacity is age in years plus 2 ounces. Therefore, the bladder capacity of the 9-year-old would be 11 ounces.

22. The nurse is caring for a 5-year-old boy who is terminally ill. Which intervention would best meet the needs of this dying child? A) Offer the child decision-making opportunities. B) Provide the child with specific details. C) Assure the child that he did nothing wrong. D) Act as a confidant for the child's concerns.

Ans: C Feedback: The magical thinking of preschool-age children may cause him to think that dying is punishment for doing something wrong. Assuring him that he did nothing wrong is very important. School-age children would benefit from receiving specific details and being given decision-making opportunities. They may also use nurses as their confidants.

Origin: Chapter 14, 7 7. A nurse is providing teaching to the mother of an adolescent girl about how to manage menstrual pain nonpharmacologically. Which statements by the mother indicates a need for further teaching? A) "I need to help her learn techniques to distract her; card games, for example." B) "I need to be able to identify the subtle ways she shows pain." C) "I need to follow these instructions exactly for them to work properly." D) "I need to encourage her to practice and utilize these techniques."

Ans: C Feedback: The mother does not need to follow the instructions exactly; she needs to review the methods and modify them in a way that works best for her daughter. The other statements are correct.

7. The nurse is caring for a 13-year-old girl hospitalized for complications from type 1 diabetes. The girl has a nursing diagnosis of powerlessness related to lack of control of multiple demands associated with hospitalization, procedures, treatments, and changes in usual routine. How can the nurse help promote control? A) Ask the child to identify her areas of concern. B) Encourage participation of parents in care activities. C) Offer the girl as many choices as possible. D) Enlist the family's assistance in creating a time schedule.

Ans: C Feedback: The nurse needs to offer the girl as many choices as possible, such as options for food and drink (as her diet allows), hygiene, activities, or clothing options to promote feelings of individuality and control. Two of the other options engage the parents in the process. A 13-year-old girl is capable of making her own choices regarding activities, schedules, and routine, but she may not be able to identify her areas of concern.

Origin: Chapter 14, 21 21. The nurse has applied EMLA cream as ordered. How does the nurse assess that the cream has achieved its purpose? A) Assess the skin for redness. B) Note any blanching of skin. C) Lightly tap the area where the cream is. D) Gently poke the child with a needle.

Ans: C Feedback: The nurse should verify that sensation is absent by lightly tapping or scratching the area. Blanching or redness indicates that the medication has penetrated the skin adequately but does not indicate that sensation is absent. Using a needle to poke the skin would likely frighten the child.

Origin: Chapter 6, 25 25. The nurse is counseling the parents of a 10-year-old child who was caught stealing at school. Which topic should the nurse cover? A) Having the child return the property in front of his or her class B) Discussing ways for the child to save face C) Finding out what is currently going on at home D) Reminding the child daily that stealing is wrong

Ans: C Feedback: The parents need to understand the child's behavior. The reason for stealing at age 10 may be that the child wants the item or is trying to impress peers, or it may be a sign of anxiety. More information is needed before the nurse can effectively work with the family. The parents should work together with the child to decide how the item will be returned. The child will lose face but gain integrity by returning the stolen item. Reminding the child about stealing on a daily basis may ruin the child's self-esteem.

Origin: Chapter 14, 17 17. The nurse is teaching an 8-year-old child and his family how to manage cancer pain using nonpharmacologic methods. Which parent statement signifies successful child teaching? A) 'I will avoid using descriptive words like pinching, pulling, or heat.' B) 'I will not use positive reinforcement until the technique is perfected.' C) 'I will begin using the technique before he experiences pain.' D) 'I will be honest and tell him that the procedure will hurt a lot.'

Ans: C Feedback: The parents should begin using the technique chosen before the child experiences pain or when the child first indicates he is anxious about, or beginning to experience, pain. The parents should use descriptive terms like pushing, pulling, pinching, or heat and avoid overly descriptive or judgmental statements such as, "This will really hurt a lot" or "This will be terrible." They should offer praise, positive reinforcement, hugs, and support for using the technique even when it was not effective.

Origin: Chapter 5, 17 17. The nurse is supervising lunch time for children on a pediatric ward. Which observation is considered abnormal for this age group? A) The child has a full set of primary teeth. B) The child has no difficulty chewing and swallowing meat. C) The child uses his fingers and refuses to use a fork. D) The child is a picky eater.

Ans: C Feedback: The preschool child has learned to use utensils fairly effectively to feed himself or herself, has a full set of primary teeth, and is able to chew and swallow competently. Preschool children may be picky eaters. They may eat only a limited variety of foods or foods prepared in certain ways and may not be very willing to try new things.

30. The nurse is reviewing the therapist's documentation in the medical record of an assigned client who has cerebral palsy. The therapist has noted the parents may be experiencing vulnerable child syndrome. Which observation of the family unit best supports this potential diagnosis? A) The parents regularly attend a support group for parents of special needs children. B) The child has been diagnosed with pneumonia twice in the past year. C) The parents report they feel their child requires more therapy than the care team has indicated will be needed. D) The child is schooled at home with a private tutor.

Ans: C Feedback: Vulnerable child syndrome is a clinical state in which the parents' reactions to a serious illness or event in the child's past continue to have long-term psychologically harmful effects on the child and members of the family unit. The parents view the child as being at higher risk for medical, developmental, or behavioral problems. Parents exhibit excessive unwarranted concerns and seek health care for their child very frequently. Requests for additional therapy would be consistent with this syndrome. Attendance at a support group is important and does not indicate any underlying pathology. Pneumonia in a child with this diagnosis is not extremely unusual and does not have an increase in likelihood that the vulnerable child syndrome may be involved. Home schooling does not indicate the presence of vulnerable child syndrome.

Origin: Chapter 14, 20 20. The nurse is administering pain medication for a child with continuous pain from internal injuries. Which method would be ordered to dispense the medication? A) Administer the medication PRN (as needed). B) Administer the mediation when pain has peaked. C) Administer the medication around the clock at timed intervals. D) Administer the medication when the child complains of pain.

Ans: C Feedback: With any medication administered for pain management, the timing of administration is vital. Timing depends on the type of pain. For continuous pain, the current recommendation is to administer analgesia around the clock at scheduled intervals to achieve the necessary effect. As-needed or PRN dosing is not recommended for continuous pain. This method can lead to inadequate pain relief because of the delay before the drug reaches its peak effectiveness. For pain that can be predicted or considered temporary, such as with a procedure, analgesia is administered so that the peak action of the drug matches the time of the painful event. It is not recommended to wait until the child complains of pain because therapeutic levels will be difficult to reach at this point.

25. A nurse is preparing a presentation for a group of parents with children diagnosed with diabetes type 1. The children are all adolescents. What issues would the nurse need to address? Select all that apply. A) Self-monitoring of blood glucose levels B) Feelings of being different C) Deficient decision-making skills D) Body image conflicts E) Struggle for independence

Ans: C, D, E Feedback: Adolescents are undergoing rapid physical, emotional, and cognitive growth. Working toward a separate identity from parents and the demands of diabetic care can hinder this. This struggle for independence can lead to nonadherence of the diabetic care regimen. Conflicts develop with self-management, body image, and peer group acceptance. Teens may acquire the skills to perform tasks related to diabetic care but may lack decision-making skills needed to adjust treatment plan. Teens do not always foresee the consequences of their activities. Self-monitoring of blood glucose levels and feelings of being different are issues common to school-age children.

28. The nurse is meeting with the parents of a 7-year-old boy with Down syndrome. The child's mother reports an interest in hippotherapy. The child's father reports that this seems to be a waste of money. The parents then ask the nurse for additional information. What information may be included in the nurse's response? Select all answers that apply. A) Hippotherapy has limited research demonstrating its actual effectiveness. B) This type of therapy is most helpful for teens. C) A variety of conditions including Down syndrome have used hippotherapy with success. D) Self-esteem may be improved with hippotherapy. E) The benefits of hippotherapy are both physical and psychological.

Ans: C, D, E Feedback: Hippotherapy refers to the use of horseback riding for the handicapped, therapeutic horseback riding, or equine-facilitated psychotherapy. Individuals with almost any cognitive, physical, or emotional disability may benefit from therapeutic riding or other supervised interaction with horses. The unique movement of the horse under the child helps the child with physical disabilities to achieve increased flexibility, balance, and muscle strength. Children with mental or emotional disabilities may experience increased self-esteem, confidence, and patience as a result of the unique relationship with the horse.

24. The nurse suspects that a 4-year-old with type 1 diabetes is experiencing hypoglycemia based on what findings? Select all that apply. A) Blurred vision B) Dry, flushed skin C) Diaphoresis D) Slurred speech E) Fruity breath odor F) Tachycardia

Ans: C, D, F Feedback: Manifestations of hypoglycemia include behavioral changes, confusion, slurred speech, belligerence, diaphoresis, tremors, palpitation, and tachycardia. Blurred vision; dry, flushed skin; and fruity breath odor suggest hyperglycemia.

11. The nurse is looking into the Individuals with Disabilities Education Improvement Act of 2004 to help provide resources for a client with multiple chronic diseases. What are mandates of this legislation? Select all that apply. A) The law mandates government-funded care coordination and special education for children up to 8 years of age. B) This early intervention program is a state-funded program run at the federal level. C) This federal law allows each state to define "developmental disability" differently. D) An evaluation of the child's physical, language, emotional, and social capabilities is performed to determine eligibility. E) The primary care nurse manages the developmental services and special education that the child requires. F) The goal is to maintain a natural environment, so most services occur in the home or day care center.

Ans: C, D, F Feedback: The Individuals with Disabilities Education Improvement Act of 2004 mandates government-funded care coordination and special education for children up to 3 years of age. Federal law allows each state to define "developmental disability" differently, but in general an evaluation of the child's physical, language, emotional, and social capabilities is performed by qualified personnel to determine eligibility. The goal of the program is that the child receives services in a "natural environment," so most services occur in the home or day care center. This early intervention program is administered through each state. Children who qualify for services receive care coordination, and the service coordinator manages the developmental services and special education that the child requires.

17. The nurse is providing developmentally appropriate care for a toddler hospitalized for observation following a fall down the steps. Which measures might the nurse consider when caring for this child? Select all that apply. A) Use the en face position when holding the toddler. B) Use a bed for toddlers who have an adult present. C) Avoid leaving small objects that can be swallowed in the bed. D) Explain activities in concrete, simple terms. E) Allow the child to select meals and activities. F) Encourage parents to stay to prevent separation anxiety.

Ans: C, F Feedback: For a toddler, the nurse would avoid leaving small objects that can be swallowed in the bed and encourage parents to stay to prevent separation anxiety. The nurse would use the en face position when holding an infant and use a bed only for the older toddler who has an adult present in the room at all times. The nurse would explain activities in concrete, simple terms for a preschooler and allow a school-age child to select meals and activities.

22. The parents of a 7-year-old girl with type 1 diabetes has been recording her blood glucose measurements before meals and at bedtime for the past 4 days; they are as follows: Monday B: 120 mg/dL L: 110 mg/dL D: 90 mg/dL Bed: 110 mg/dL Tuesday 135 mg/dL 120 mg/dL 140 mg/dL 110 mg/dL Wednesday B: 124 mg/dL L: 140 mg/dL D: 130 mg/dL Bed: 160 mg/dL Thursday 200 mg/dL 220 mg/dL 200 mg/dL 240 mg/dL The parents bring the child in for a follow-up visit and show the nurse the results. Based on the results, the nurse would need to obtain additional information from the parents and child about which day? A) Monday B) Tuesday C) Wednesday D) Thursday

Ans: D Feedback: Blood glucose levels for a child who is 7 years of age should range from 90 to 180 mg/dL before meals and from 100 to 180 mg/dL before bedtime. On Thursday, the results for each testing were above normal. Therefore, the nurse needs to gather additional information about this day.

5. The nurse is caring for a 13-year-old girl with delayed puberty. Based on the nurse's knowledge of this condition, the nurse would include which nursing diagnosis in the child's plan of care? A) Disabled family coping related to the child's disorder B) Imbalanced nutrition, less than body requirements related to the child's short stature C) Noncompliance related to the need for lifelong hormone therapy D) Deficient knowledge related to the administration of estradiol

Ans: D Feedback: Deficient knowledge related to the administration of estradiol is an appropriate nursing diagnosis for this child. There are oral, transdermal, topical, injectable, and vaginal preparations available. Disabled family coping due to the child's disorder and noncompliance due to long-term therapy are not likely diagnoses because of the simplicity and brevity of the treatment for this disorder. Imbalanced nutrition evidenced by short stature would be appropriate for a child with growth hormone deficiency.

12. When reviewing the medical record of a child, what would the nurse interpret as the most sensitive indicator of intellectual disability? A) History of seizures B) Preterm birth C) Vision deficit D) Language delay

Ans: D Feedback: Due to the extent of cognition required to understand and produce speech, the most sensitive early indicator of intellectual disability is delayed language development. A history of seizures, preterm birth, and vision deficit may be associated with intellectual disability but are not the most sensitive indicators.

11. A child with attention deficit/hyperactivity disorder is prescribed long-acting methylphenidate. What information would the nurse include when teaching the child and his parents about this drug? A) "Give the drug three times a day: morning, midday, and after school." B) "This drug may cause drowsiness, so be careful when doing things." C) "Some increase in appetite may occur, so watch how much you eat." D) "Take this drug every day in the morning when you wake up."

Ans: D Feedback: Long-acting methylphenidate is administered once daily in the morning, whereas the other forms are given three times a day. The drug typically causes difficulty sleeping and decreased appetite.

3. The nurse is caring for an adolescent girl with a suspected anxiety disorder. The girl states that she is constantly double-checking that she has unplugged her curling iron and must make sure that everything is in perfect order in her room before she leaves the house. The nurse interprets these findings as indicating which disorder? A) Generalized anxiety disorder B) Posttraumatic stress disorder C) Social phobia D) Obsessive-compulsive disorder

Ans: D Feedback: Obsessive-compulsive disorder is characterized by compulsions (repetitive behaviors such as cleaning, washing, or checking something) to reduce anxiety about obsessions (unwanted and intrusive thoughts). Posttraumatic stress disorder is an anxiety disorder that occurs after a child is subjected to a traumatic event, later experiencing physiologic arousal when a stimulus triggers memories of the event. Generalized anxiety disorder is characterized by unrealistic concerns over past behavior, future events, and personal competency. Social phobia is characterized by a persistent fear of formal speaking, using public restrooms, or eating in front of others.

20. A group of students are reviewing information about the various types of insulin used to treat type 1 diabetes. The students demonstrate understanding of the information when they identify which of these insulins as having the longest duration? A) Lispro B) Regular C) NPH D) Glargine

Ans: D Feedback: Of the insulins listed, glargine (Lantus) has the longest duration of action, that is, 12 to 24 hours. Lispro lasts approximately 3 to 5 hours; regular lasts 5 to 8 hours; and NPH lasts approximately 10 to 16 hours.

19. A nurse is teaching the parents of an infant with congenital adrenal hyperplasia about the signs and symptoms of adrenal crisis. The nurse determines that the teaching was successful when the parents correctly identify what sign of adrenal crisis? A) Bradycardia B) Constipation C) Fluid overload D) Persistent vomiting

Ans: D Feedback: Signs and symptoms of acute adrenal crisis include persistent vomiting, dehydration, hyponatremia, hyperkalemia, hypotension, tachycardia, and shock.

11. The parents of a child with congenital adrenal hyperplasia bring the child to the emergency department for evaluation because the child has had persistent vomiting. What finding would lead the nurse to suspect that the child is experiencing an acute adrenal crisis? A) Hypernatremia B) Bradycardia C) Hypertension D) Hyperkalemia

Ans: D Feedback: Signs and symptoms of an acute adrenal crisis include hyperkalemia, hyponatremia, tachycardia, hypotension, persistent vomiting, dehydration, and shock.

Origin: Chapter 14, 8 8. The nurse is counseling the parents of a 9-year-old boy who is receiving morphine for postoperative pain. Which statement from the nurse accurately reflects the pain experience in children? A) 'You can expect that your child will tell you when he is experiencing pain.' B) 'Your child will learn to adapt to the pain he is experiencing.' C) 'Your child will experience more adverse effects to narcotics than adults.' D) 'It is very rare that children become addicted to narcotics.'

Ans: D Feedback: Addiction to narcotics when used in children is very rare. Often children deny pain to avoid a painful situation or procedure, embarrassment, or loss of control. Repeated exposure to pain or painful procedures can result in an increase in behavioral manifestations. The risk of adverse effects of narcotic analgesics is the same for children as for adults.

10. Which would be least appropriate to include in the discharge plan for a medically fragile child? A) Assisting with referrals for financial support B) Arranging for necessary care equipment and supplies C) Assessing the family's home environment D) Encouraging passive caregiving

Ans: D Feedback: As part of the discharge plan for a medically fragile child, the nurse would encourage active caregiving by the parents to help them increase their self-confidence in the child's care. Assisting with referrals, arranging for equipment and supplies, and determining the adequacy of the home environment are important aspects of the discharge plan.

Origin: Chapter 6, 27 27. The school nurse is teaching parents about the effects of bullying on school children. What accurately describes this developmental concern? A) Children who bully are those who report themselves as being lonely and having difficulty in forming friendships. B) Children with health issues, such as, disabilities, obesity and food allergies, are at a decreased risk of being bullied. C) In general, about 20% of all children attending school are frightened and afraid most of the day. D) Both boys and girls are bullied; boys usually bully boys and use force more often.

Ans: D Feedback: Both boys and girls are bullied and can bully others. Boys usually bully boys and use force more often, and boys are twice as likely to be victims of bullying. Bullied children are those who report themselves as being lonely and having difficulty in forming friendships. Children with health issues, such as disabilities, obesity, and food allergies, are at an increased risk of being bullied. In general, about 10% of all children attending school are frightened and afraid most of the day.

3. The nurse is providing home care for a 1-year-old girl who is technologically dependent. Which intervention will best support the family process? A) Finding an integrated health program for the family B) Teaching modifications of the medical regimen for vacation C) Assessing family expectations for the special needs child D) Creating schedules for therapies and interventions

Ans: D Feedback: Coordinating care with the schedules and capabilities of the parents provides the greatest support for the family. It gives them a sense of order and control. Integrated health care programs may not be available in the family's area. Teaching therapy modifications for travel and assessing family expectations are not supportive interventions.

Origin: Chapter 14, 4 4. The nurse is conducting an assessment of a high school track athlete. The client tells the nurse he is experiencing pain along his outer thigh. He describes it as tight, achy, and tender, particularly after he runs. The nurse understands that he is most likely experiencing what kind of pain? A) Cutaneous B) Neuropathic C) Visceral D) Deep somatic

Ans: D Feedback: Deep somatic pain typically involves the muscles, tendons, joints, fasciae, and bones. It can be localized or diffuse and is usually described as dull, aching, or cramping with tenderness. It can also be due to overuse injuries commonly experienced by athletes. Cutaneous pain usually involves the skin and is described as sharp or burning. Neuropathic pain is due to a malfunctioning of the peripheral nervous system and is described as burning or tingling. Visceral pain is pain that develops within organs.

15. The nurse caring for young children in a hospice setting is aware of the following statistics related to the occurrence of death in children. Which statement accurately reflects one of these statistics? A) Each year, about 50,000 children die in the United States; of those, about 15,000 are infants. B) It is unusual for a child's chronic illness to progress to the point of becoming a terminal illness. C) Despite strides made, diabetes remains the leading cause of death from disease in all children older than the age of 1 year. D) Congenital defects and traumatic injuries are the more common causes of diseases leading to death.

Ans: D Feedback: Diseases can lead to terminal illness in children, with congenital defects and traumatic injuries being the more common causes. Each year, about 45,000 children die in the United States; of those, about 24,500 are infants (Heron et al., 2013). In many cases, a child's chronic illness may progress to the point of becoming a terminal illness. Cancer remains the leading cause of death from disease in all children older than the age of 1 year (Heron et al., 2013).

Origin: Chapter 6, 22 22. Two working parents are discussing with the school nurse the possibility of their 12-year-old girl going home alone after school. What suggestion should the nurse make? A) Provide entertainment until the parents come home. B) Allow the child to go to a friend's house. C) Teach her how to take a message if someone calls. D) Purchase caller ID for the phone.

Ans: D Feedback: Having caller ID allows the child to answer the phone if Mom or Dad calls while ignoring all other calls. Rather than entertaining the child, this would be a better time for homework, age-appropriate chores, and limited entertainment. If the child goes to a friend's house, it should be prearranged between the parents, not spur of the moment. It is safer if the child does not answer the phone instead of taking a message.

Origin: Chapter 5, 6 6. Which activity would the nurse least likely include as exemplifying the preconceptual phase of Piaget's preoperational stage? A) Displays of animism B) Use of active imaginations C) Understanding of opposites D) Beginning questioning of parents' values

Ans: D Feedback: In the intuitive phase of Piaget's preoperational stage, the child begins to question parents' values. Animism, active imaginations, and an understanding of opposites would characterize the preconceptual phase of Piaget's preoperational stage.

Origin: Chapter 14, 26 26. The nurse is conducting a pain assessment of a 10-year-old boy who has been taking acetaminophen for chronic knee pain. The assessment indicates that the recommended dose is no longer providing adequate relief. What is the appropriate nursing action? A) Increase the dosage of the acetaminophen. B) Tell the child he is experiencing the ceiling effect. C) Use guided imagery to help his pain. D) Obtain an order for a different medication.

Ans: D Feedback: Increasing the dose of the acetaminophen will not help his pain because he has reached as high a dose of that medication that will work. This is known as the ceiling effect, but explaining that to him will not help his pain. Guided imagery is not the best therapy for his pain, so the physician needs to order a different medication to manage his pain.

9. The nurse is weighing an underweight infant diagnosed with failure to thrive (FTT) and notes that the baby does not make eye contact and is less active than the other infants. What would be a probable cause for the FTT related to the infant's body language? A) Congenital heart defect B) Cleft palate C) Gastroesophageal reflux disease D) Maternal abuse

Ans: D Feedback: Infants with FTT related to maternal neglect may avoid eye contact and be less interactive than other infants. Inorganic causes of FTT include neglect, abuse, behavioral problems, lack of appropriate maternal interaction, poor feeding techniques, lack of parental knowledge, or parental mental illness.

Origin: Chapter 6, 11 11. The school nurse knows that school-age children are developing metalinguistic awareness. Which is an example of this skill? A) The child enjoys reading books. B) The child enjoys conversations with peers. C) The child enjoys speaking on the phone. D) The child enjoys telling jokes.

Ans: D Feedback: Language skills continue to accelerate during the school-age years. School-age children develop metalinguistic awareness—an ability to think about language and comment on its properties. This enables them to enjoy jokes and riddles due to their understanding of double meanings and play on words and sounds.

28. The nurse working with children in a hospital setting notes that they are being discharged earlier and earlier. Which is a primary reason for this trend? A) Nursing shortages B) Increased funding for home care C) National health care initiatives D) Cost containment

Ans: D Feedback: Over the past century changes in health care, such as strained health care funding, shorter hospital stays, and cost containment, have led to a shift in responsibilities of care for children from the hospital to homes and communities. Nursing shortages influence the delivery of health care. National health care initiatives may or may not affect earlier discharge to home health care.

23. What would the nurse include in the plan of care for a dying child with pain? A) Administering analgesics as needed B) Using measures the nurse finds comforting C) Playing the television or radio so the child can hear it D) Changing the child's position frequently but gently

Ans: D Feedback: Pain management includes changing the child's position frequently but gently to minimize discomfort. Analgesics are given around the clock rather than as needed. The nurse would use measures that the child finds comforting to provide additional relief. A calm environment with minimal noise and light is helpful.

23. The nurse is providing discharge planning for a 12-year-old boy with multiple medical conditions. What would be the best teaching method for this child and his family? A) Demonstrate the care and ask for a return demonstration. B) Provide and review educational booklets and materials. C) Provide a written schedule for the child's care. D) Provide a trial period of home care.

Ans: D Feedback: Parents of children with multiple medical needs may benefit from a trial period of home care. This occurs while the child is still in the hospital, but the parents or caregivers provide all of the care that the child requires. The other options are also important teaching methods, but a trial period is the best solution for a child with multiple medical conditions.

Origin: Chapter 6, 20 20. The school nurse is preparing a talk on the influence of the media on school-age children to present at the next PTO meeting. Which fact might the nurse include in the introduction? A) Children in the United States spend about 6 hours a day either watching TV or playing video games. B) A child will see 2,000 murders by the end of grade school and 20,000 commercials a year. C) A school-age child cannot determine what is real from what is fantasy; therefore, TV and video games can lead to aggressive behavior. D) Parents should limit television watching and video-game playing to 2 hours per day.

Ans: D Feedback: Parents should limit television watching and video-game playing to 2 hours per day. Children in the United States spend about 4 hours a day either watching TV or playing video games. A child will see 8,000 murders by the end of grade school and 40,000 commercials a year. Although school-age children can determine what is real from what is fantasy, research has shown that this amount of time in front of the TV—watching it or playing video games—can lead to aggressive behavior, less physical activity, and altered body image.

Origin: Chapter 6, 14 14. During a well-child check-up, the parents of a 9-year-old boy tell the nurse that their son's friends told him that soccer is a stupid game, and now he wants to play baseball. Which comment by the nurse best explains the effects of peer groups? A) "The child's best friends will continue playing soccer." B) "The children will cheer for each other regardless of the sport being played." C) "Your child will rarely talk to you about his friends." D) "Acceptance by friends, especially of the same sex, is very important at this age."

Ans: D Feedback: Peer relationships, especially of the same sex, are very important and can influence the child's relationship with his parents. They can provide enough support that he can risk parental conflict and stand his ground about playing soccer. At this age, peer groups are made up of the child's best friends, and they happen to be playing baseball. Peer groups have rules and take up sides against the soccer player. Peers are an authority, so the child will let his parents know their opinions.

Origin: Chapter 14, 29 29. Pentazocine is prescribed for a child with moderate pain. The nurse identifies this drug as an example of which type? A) Nonsteroidal anti-inflammatory drug (NSAID) B) Prostaglandin inhibitor C) Opioid D) Mixed opioid agonist-antagonist

Ans: D Feedback: Pentazocine is classified as a mixed opioid agonist-antagonist. Ibuprofen, ketorolac, and naproxen are examples of NSAIDs that inhibit prostaglandin synthesis. Morphine, codeine, and fentanyl are examples of opioids.

3. The nurse is caring for an 8-year-old boy hospitalized for a bone marrow transplant. His parents are in and out of his room throughout the day. Which behaviors of the child would alert the nurse that he is in the second stage of separation anxiety? A) He ignores his parents when they return to his room. B) He cries uncontrollably whenever they leave. C) He forms superficial relationships with his caregivers. D) He sits quietly and is uninterested in playing and eating.

Ans: D Feedback: Separation anxiety consists of three stages—protest, despair, and detachment. In the protest stage, the child reacts aggressively to separation and exhibits great distress by crying, expressing agitation, and rejecting others who attempt to offer comfort. In the despair phase the child displays hopelessness by withdrawing from others, becoming quiet without crying, and exhibiting apathy, depression, lack of interest in play and food, and overall feelings of sadness. In the detachment stage the child shows interest in the environment, starts to play again, and forms superficial relationships with the nurses and other children. If the parents return, the child ignores them. A child in this phase of separation anxiety exhibits resignation, not contentment.

Origin: Chapter 6, 23 23. The parents of an 8-year-old boy are interested in promoting learning through reading to their son. Which suggestion by the nurse would best promote this goal? A) Have the parents choose what he should read initially. B) Tell the child to read instead of watching TV with his parents. C) Tell the parents that reading is for the child to do by himself. D) Take the child to the library to check out some books.

Ans: D Feedback: Taking the child to the library can be a positive start to the reading experience. It is best to let the librarian recommend books that will be appropriate for the child, but let the child choose from recommended materials. Set an example by reading instead of watching TV while the child is not in bed. Reading to the child is a valuable parent-child activity that can expose the child to classic works that are beyond the child's present reading ability.

Origin: Chapter 14, 9 9. The nurse is using the acronym QUESTT to assess the pain of a child. Which is an accurate descriptor of this process? A) Question the child's parents. B) Understand the child's pain level. C) Establish a caring relationship with the child. D) Take the cause of pain into account when intervening.

Ans: D Feedback: The acronym QUESTT stands for the following: Question the child. Use a reliable and valid pain scale. Evaluate the child's behavior and physiologic changes to establish a baseline and determine the effectiveness of the intervention. The child's behavior and motor activity may include irritability and protection as well as withdrawal of the affected painful area. Secure the parent's involvement. Take the cause of pain into account when intervening. Take action.

Origin: Chapter 14, 11 11. Which tool would be the least appropriate scale for the nurse to use when assessing a 4-year-old child's pain? A) FACES pain rating scale B) Oucher pain rating scale C) Poker chip tool D) Numeric pain intensity scale

Ans: D Feedback: The numeric pain intensity scale can be used with children as young as 5 years of age, but the preferred minimum age for using this tool is 7 years. The FACES and Oucher pain rating scales and the poker chip tool are appropriate pain assessment tools for a 4-year-old.

Origin: Chapter 14, 24 24. The nurse is monitoring a child who has received epidural analgesia with morphine. The nurse is careful to monitor for which adverse effect of the medication? A) Epidural hematoma B) Arachnoiditis C) Spinal headache D) Respiratory depression

Ans: D Feedback: The nurse needs to monitor for signs of respiratory depression, a potential adverse effect of the opioid medication. Epidural hematoma, arachnoiditis, and spinal headache are potential adverse effects of the insertion of the epidural catheter.

8. The nurse is caring for an 8-year-old girl who requires numerous venipunctures and injections daily. The nurse understands that the child is exhibiting signs of sensory overload and enlists the assistance of the child life specialist. What should the therapeutic play involve to best deal with the child's stressors? A) Puppets and dolls B) Drawing paper and crayons C) Wooden hammer and pegs D) Sewing puppets with needles

Ans: D Feedback: The nurse understands that the child may benefit from supervised needle play to assist the child undergoing frequent blood work, injections, or intravenous procedures. The child life specialist can determine what form of therapeutic play is best, but the nurse can recommend interventions based on his or her knowledge of the specific child.

Origin: Chapter 5, 14 14. The nurse is developing a nursing care plan for a hospitalized 6-year-old. Which behavior would warrant nursing intervention? A) The child pretends he is talking to an imaginary friend when the nurse addresses the child. B) The child states that her fairy godmother is going to come and take her home. C) The child starts talking about his grandmother and then quickly changes the subject to a new toy he received. D) The child does not want to play games with other children on the hospital ward.

Ans: D Feedback: The preschooler begins to plan activities, make up games, and initiate activities with others. Not wanting to play games with other children is a sign of a developmental delay and nursing intervention is recommended. The preschooler often has an imaginary friend who serves as a creative way for the preschooler to sample different activities and behaviors and practice conversational skills. Through make-believe and magical thinking, preschool children satisfy their curiosity about differences in the world around them. The preschooler uses transduction when reasoning: he or she extrapolates from a particular situation to another, even though the events may be unrelated.

13. The nurse is caring for a 14-year-old girl with special health needs. What is the priority intervention for this child? A) Encouraging the parents to promote the child's self-care B) Assessing the child for signs of depression C) Discussing how her care will change as she grows D) Monitoring for compliance with treatment

Ans: D Feedback: The priority intervention is monitoring for compliance with treatment. The girl is struggling to fit in with her peers and may try to hide or ignore her illness. Monitoring for depression and encouraging self-care have a lesser impact on the child's physical health. A transition plan to adulthood may be initiated some time in midadolescence.

10. The nurse is assessing a 4-month-old boy during a scheduled visit. Which findings might suggest a developmental problem? A) The child does not babble. B) The child does not vocally respond to voices. C) The child never squeals or yells. D) The child does not say dada or mama.

Ans: B Feedback: The fact that the child does not vocally respond to voices might suggest a developmental problem. At 4 to 5 months of age, most children are making simple vowel sounds, laughing aloud, doing raspberries, and vocalizing in response to voices. The child is too young to babble, squeal, yell, or say dada or mama.

Origin: Chapter 5, 18 18. The nurse of a preschool child is helping parents develop a healthy meal plan for their child. What nutritional requirements for this age group should the nurse consider? A) The 3- to 5-year-old requires 300 to 500 mg calcium and 10 mg iron daily. B) The 3-year-old should consume 10 mg dietary fiber daily. C) The 4- to 8-year-old requires 15 mg dietary fiber per day. D) The typical preschooler requires about 85 kcal/kg of body weight.

Ans: D Feedback: The typical preschooler requires about 85 kcal/kg of body weight. The 3- to 5-year-old requires 500 to 800 mg calcium and 10 mg iron daily. The 3-year-old should consume 19 mg dietary fiber daily, while the 4- to 8-year-old requires 25 mg dietary fiber per day.

Origin: Chapter 14, 15 15. The nurse is providing instructions to a mother on how to use thought stopping to help her child deal with anxiety and fear associated with frequent painful injections. Which statement indicates the mother understands the technique? A) "We will imagine that we are on the beach in Florida." B) "We can talk about our favorite funny movie and laugh." C) "She can let her body parts go limp, working from head to toe." D) "We'll repeat 'quick stick, feel better, go home soon' several times."

Ans: D Feedback: Thought stopping is a technique that involves the use of short, concise phrases of positive ideas. Doing so helps to promote the child's sense of control. Imagining a favorite beach in Florida is using imagery. Talking about a favorite funny movie involves humor. Letting body parts go limp is a relaxation technique.

Origin: Chapter 14, 12 12. The nurse uses the FLACC behavioral scale to assess a 6-year-old's level of postoperative pain and obtains a score of 9. The nurse interprets this to indicate that the child is experiencing: A) little to no pain. B) mild pain. C) moderate pain. D) severe pain.

Ans: D Feedback: With the FLACC behavioral scale, five parameters are measured and scored as 0, 1, or 2. They are then totaled to achieve a maximum score of 10. The higher the score, the greater the pain. A score of 9 indicates severe pain.

18. The nurse is ordered to apply restraints to a toddler who keeps pulling at the tubes in his arm. Which criteria must occur to ensure proper use of these restraints? Select all answers that apply. A) The nurse must check the restraints every 15 minutes while they are in place. B) Secure the restraints with ties to the side rails, not the bed or crib frame. C) Assess the temperature of the affected extremities, pulses, and capillary refill every 15 minutes after placement. D) Use a clove-hitch type of knot to secure the restraints with ties. E) Remove the restraint every 2 hours to allow for range of motion and repositioning. F) Encourage parent participation, providing continuous explanations about the reasons and time frame for restraints.

Ans: D, E, F Feedback: The nurse should use a clove-hitch type of knot to secure the restraints with ties, remove the restraint every 2 hours to allow for range of motion and repositioning, and encourage parent participation, providing continuous explanations about the reasons and time frame for the restraints. The nurse must check restraints 15 minutes following initial placement and then every hour for proper placement and secure the restraints with ties to the bed or crib frame, not the side rails. The nurse should also assess the temperature of the affected extremities, pulses, and capillary refill, initially after 15 minutes and then every hour after placement.

31. The nurse is assessing the developmental milestones of an infant. The infant was born 8 weeks ago and was 4 weeks premature. The nurse anticipates that the infant will be meeting milestones for what age of child? Record your answer in weeks.

Ans: 4 Feedback: To determine adjusted age, subtract the number of weeks that the infant was premature (4 weeks) from the infant's chronologic age (8 weeks).

A nursing student is preparing an oral presentation about autosomal recessive inheritance. What must occur for an offspring to demonstrate signs and symptoms of the disorder with this type of inheritance? A) Both parents must be heterozygous carriers. B) One parent must have the disease. C) The mother must be a carrier. D) The father must be affected by the disease.

Ans: A Autosomal recessive inheritance occurs when two copies of the mutant or abnormal gene in the homozygous state are necessary to produce the phenotype. In other words, two abnormal genes are needed for the individual to demonstrate signs and symptoms of the disorder. Both parents of the affected person must be heterozygous carriers of the gene (clinically normal, but carriers of the gene).

The nurse is obtaining the health history for a 15-month-old boy from the parents. The child is not yet speaking. Which finding would be eliminated as a risk factor for a possible genetic disorder? A) The child is male and Caucasian. B) The grandmother and father have hearing impairments. C) The child was a breech delivery 3 weeks early. D) The mother was 37 when she became pregnant

Ans: A Being male and Caucasian are risk factors for acute lymphoblastic leukemia, not genetic disorders. The fact that the child's grandmother and father have hearing impairments suggests a genetic disorder. The facts that the mother was 37 when she became pregnant and had a breech delivery 3 weeks early are also risk factors for genetic disorders.

The nurse is caring for 3-day-old girl with Down syndrome whose mother had no prenatal care. What is the priority nursing diagnosis? A) Imbalanced nutrition, less than body requirements related to the effects of hypotonia B) Deficient knowledge related to the presence of a genetic disorder C) Delayed growth and development related to a cognitive impairment D) Impaired physical mobility related to poor muscle tone

Ans: A Children with Down syndrome may have difficulty sucking and feeding due to lack of muscle tone and the structure of their mouths and tongues. This can lead to poor nutritional intake and makes this the priority diagnosis. This also uses the strategy that physiologic needs have priority using Maslow's hierarchy of needs. Deficient knowledge due to lack of information about the disorder is a close second in priority, as the mother did not know of her daughter's condition before birth and has much to learn now. This child is at risk for a number of complications such as infection, heart disease, and leukemia and will require frequent assessment. Most children with Down syndrome experience some degree of intellectual disability, but early intervention will allow the child maximum development within the limits of the disease. Mobility is delayed but should not be a problem at this time.

A nurse is reviewing an article about genetic disorders and patterns of inheritance. The nurse demonstrates understanding of the information by identifying which as an example of an autosomal dominant genetic disorder? A) Neurofibromatosis B) Cystic fibrosis C) Tay-Sachs disease D) Sickle cell disease

Ans: A Neurofibromatosis is an example of an autosomal dominant genetic disorder. Cystic fibrosis, Tay-Sachs disease, and sickle cell disease are examples of autosomal recessive genetic disorders.

When teaching a class about trisomy 21, the instructor would identify the cause of this disorder as: A) nondisjunction. B) X-linked recessive inheritance. C) genomic imprinting. D) autosomal dominant inheritance.

Ans: A Trisomy 21 is an example of a genetic disorder involving an abnormality in chromosomal number due to nondisjunction. X-linked recessive inheritance disorders, such as hemophilia and Duchenne muscular dystrophy, involve altered genes on the X chromosome. Genomic imprinting disorders, such as Prader-Willi syndrome, involve expression of only the maternal or paternal allele, with the other being inactive. Autosomal dominant inheritance disorders, such as neurofibromatosis and achondroplasia, involve a single gene in the heterozygous state that is capable of producing the phenotype, thus overshadowing the normal gene.

7.The nurse uses family-centered care to provide care for children in a pediatric office. Upon what concept is family-centered care based? A) The family is the constant in the child's life and the primary source of strength. B) The care provider is the constant in the child's life and the primary source of strength. C) The child must be prepared to be his or her own source of strength during times of crisis. D) The wishes of the family should direct the nursing care plan for the child.

Ans: A Feedback: Family-centered care involves a partnership between the child, family, and healthcare providers in planning, providing, and evaluating care. Family-centered care enhances parents' and caregivers' confidence in their own skills and also prepares children and young adults for assuming responsibility for their own healthcare needs. It is based on the concept that the family is the constant in the child's life and the primary source of strength and support for the child.

12.The nurse is caring for a 14-year-old boy with an osteosarcoma. Which communication technique would be least effective for him? A) Letting him choose juice or soda to take pills B) Seeking the teenager's input on all decisions C) Discussing the benefits of chemotherapy with him D) Avoiding undue criticism of noncompliance

Ans: A Feedback: Letting the child choose juice or soda to take pills is the least effective communication technique for an adolescent. It may provide some sense of control, but is not as effective as seeking his input on all care decisions, including him during discussions of the benefits of chemotherapy, and avoiding undue criticism of noncompliance.

4.The nurse contacts a child life specialist (CLS) to work with children on a pediatric ward. What is the primary goal of the CLS? A) Decrease anxiety and fear during hospitalization and painful procedure. B) Keep children who are hospitalized distracted from pain. C) Perform medical procedures using atraumatic principles. D) Act as a liaison between the nurse and the child.

Ans: A Feedback: The CLS is a specially trained individual who provides programs that prepare children for hospitalization, surgery, and other procedures that could be painful (Child Life Council, 2010a, 2010b). The goal of the CLS is to decrease the anxiety and fear while improving and encouraging understanding and cooperation of the child. The CLS may use distraction techniques and act as a liaison, but that is not the primary goal of the CLS role. The CLS does not perform medical procedures.

16.The nurse is enlisting the parents' assistance for therapeutic hugging prior to an otoscopic examination. What should the nurse emphasize to the parents? A) "You will need to keep his hands down and his head still." B) "If this does not work, we will have to apply restraints." C) "If you are not capable of this, let me know so I can get some assistance." D) "I may need you to leave the room if your son will not remain still."

Ans: A Feedback: The nurse needs to provide a specific explanation of the parents' role and what body parts to hold still in a safe manner. Implying that the parents may not be capable or may have to leave the room is inappropriate. Telling the parents that restraints may be required is not helpful, does not teach, and may be perceived as a threat.

2. The nurse is teaching a new mother about the drastic growth and developmental changes her infant will experience in the first year of life. Which statement describes a developmental milestone occurring in infancy? A) By 6 months of age, the infant's brain weighs half that of the adult brain; at age 12 months, the brain weighs 2.5 times what it did at birth. B) Most infants triple their birth weight by 4 to 6 months of age and quadruple their birth weight by the time they are 1 year old. C) The head circumference increases rapidly during the first 6 months: the average increase is about 1 in per month. D) The heart triples in size over the first year of life; the average pulse rate decreases from 120 to 140 in the newborn to about 100 in the 1-year-old.

Ans: A Feedback: By 6 months of age, the infant's brain weighs half that of the adult brain; at age 12 months, the brain weighs 2.5 times what it did at birth. Most infants double their birth weight by 4 to 6 months of age and triple their birth weight by the time they are 1 year old. The head circumference increases rapidly during the first 6 months: the average increase is about 0.6 in (1.5 cm) per month. The heart doubles in size over the first year of life. As the cardiovascular system matures, the average pulse rate decreases from 120 to 140 in the newborn to about 100 in the 1-year-old.

14.The nurse is examining a 15-month-old child who was able to walk at the last visit and now can no longer walk. What would be the nurse's best intervention in this case? A) Schedule a full evaluation since this may indicate a neurologic disorder. B) Note the regression in the child's chart and recheck in another month. C) Document the findings as a developmental delay since this is a normal occurrence. D) Ask the parents if they have changed the child's schedule to a less active one.

Ans: A Feedback: Any child who "loses" a developmental milestone—for example, the child able to sit without support who now cannot—needs an immediate full evaluation, since this indicates a significant neurologic problem.

6. A new mother expresses concern to the nurse that her baby is crying and grunting when passing stool. What is the nurse's best response to this observation? A) "This is normal behavior for infants unless the stool passed is hard and dry." B) "This is normal behavior for infants due to the immaturity of the gastrointestinal system." C) "This indicates a blockage in the intestine and must be reported to the health care provider." D) "This is normal behavior for infants unless the stool passed is black or green."

Ans: A Feedback: Due to the immaturity of the gastrointestinal system, newborns and young infants often grunt, strain, or cry while attempting to have a bowel movement. This is not of concern unless the stool is hard and dry. Stool color and texture may change depending on the foods that the infant is ingesting. Iron supplements may cause the stool to appear black or very dark green.

19. The nurse in a community clinic is caring for a 6-month-old boy and his mother. Which intervention is priority to promote adequate growth? A) Monitoring the child's weight and height B) Encouraging a more frequent feeding schedule C) Assessing the child's current feeding pattern D) Recommending higher-calorie solid foods

Ans: A Feedback: Monitoring the child's weight and height is the priority intervention to promote adequate growth. Encouraging a more frequent feeding schedule, assessing the child's current feeding pattern, and recommending higher-calorie solid foods are interventions when the nursing diagnosis is that nutrition level does not meet body requirements.

27.The nurse working in a community clinic attempts to establish a free vaccination program to refer low-income families. What is the key strategy for success when implementing a health promotion activity? A) Partnership development B) Funding for projects C) Finding an audience D) Adequate staffing

Ans: A Feedback: Partnership development is the key strategy for success when implementing a health promotion activity. Identifying key stakeholders from the community allows problems to be solved and provides additional venues for disseminating information. Funding, finding an audience, and staffing a project are elements of a public health promotion activity, but developing a partnership helps empower children and families at the individual and community levels to develop resources to optimize their health.

15.During a physical assessment of a 5-month-old child, the nurse observes the first tooth has just erupted and uses the opportunity to advise the mother to schedule a dental examination for her baby. When is the correct time for the dentist visit? A) By the first birthday B) By the second birthday C) By entry into kindergarten D) By entry into first grade

Ans: A Feedback: The American Academy of Pediatric Dentistry recommends that a dentist examines the infant by his or her first birthday. Besides assessing routine oral health care, establishing a dental contact by the first birthday provides a resource for emergency dental care if it is needed.

20. The nurse is caring for a 7-month-old girl during a well-child visit. Which intervention is most appropriate for this child? A) Discussing the type of sippy cup to use B) Advising about increased caloric needs C) Explaining how to prepare table meats D) Describing the tongue extrusion reflex

Ans: A Feedback: The cup may be introduced at 6 to 8 months of age. Old-fashioned sippy cups are preferred compared to the new style. The nurse would not advise about increased caloric needs as caloric needs drop at this age. Transition to table meat will not take place until age 10 to 12 months. Tongue extrusion reflex has disappeared at age 4 to 6 months.

18. The nurse is providing anticipatory guidance to the mother of a 9-month-old girl during a well-baby visit. Which topic would be most appropriate? A) Advising how to create a toddler-safe home B) Warning about small objects left on the floor C) Cautioning about putting the baby in a walker D) Telling about safety procedures during baths

Ans: A Feedback: The most appropriate topic for this mother would be advising her on how to create a toddler-safe home. The child will very soon be pulling herself up to standing and cruising the house. This will give her access to areas yet unexplored. Warning about small objects left on the floor, telling about safety procedures during baths, and cautioning about using baby walkers would no longer be anticipatory guidance as the child has passed these stages.

27. The parent of a 6-month-old infant asks the nurse for advice about his son's thumb sucking. What would be the nurse's best response to this parent? A) "Thumb sucking is a healthy self-comforting activity." B) "Thumb sucking leads to the need for orthodontic braces." C) "Caregivers should pay special attention to the thumb sucking to stop it." D) "Thumb sucking should be replaced with the use of a pacifier."

Ans: A Feedback: Thumb sucking is a healthy self-comforting activity. Infants who suck their thumbs or pacifiers often are better able to soothe themselves than those who do not. Studies have not shown that sucking either thumbs or pacifiers leads to the need for orthodontic braces unless the sucking continues well beyond the early school-age period. The infant who has become attached to thumb sucking should not have additional attention drawn to the issue, as that may prolong thumb sucking. Pacifiers should not be used to replace thumb sucking as this habit will also need to be discouraged as the child grows.

22. A nurse is providing care for a child hospitalized with a diagnosis of aplastic anemia. In planning the child's care, which intervention(s) will assist the child in adapting to being hospitalized? Select all that apply. A) Provide opportunities for the parents to participate in the child's care. B) Encourage the parents to bring personal items to make the child feel more at home. C) Make the child's room off limits to invasive procedures. D) Discuss the plan of care out of earshot of the child. E) Answer any questions the child may have in generalized terms.

Ans: A, B, C Feedback: Atraumatic care is important to a child's well-being during hospitalization. Examples of this include providing opportunities for the parents and the child to participate in care, encouraging parents to bring personal items, and maintaining the child's room as a safe place, off limits to invasive procedures. It is important to be honest with the child and include the child in all plan of care discussions.

23. A nurse is assisting the health care provider with suturing a laceration on a preschooler's leg. What distraction methods can the nurse perform to promote atraumatic care? Select all that apply. A) Ask the child to squeeze the nurse's hand. B) Sing a song and have the child sing along. C) Have the child blow bubbles. D) Allow the child to play with surgical instruments. E) Let the child suture a doll.

Ans: A, B, C Feedback: Distraction methods for preschoolers include asking the child to squeeze the nurse's hand, encouraging the child to count aloud, singing a song and having the child sing along, pointing out any pictures on the ceiling, having the child blow bubbles, and playing music appealing to the child. Suturing a doll or playing with surgical instruments would be activities better suited for school-age children.

The nurse is caring for a couple who have just learned that their infant has a genetic disorder. What would be least appropriate for the nurse to do at this time? A) Actively listening to the parents' concerns B) Teaching the parents about the child's medical needs C) Providing time for the parents to ask questions D) Offering suggestions for support services

Ans: B The parents are most likely overwhelmed with learning the diagnosis and are dealing with a wide range of emotions and reactions. Therefore, it would be inappropriate at this time to attempt teaching them. Rather, the nurse would provide emotional support, actively listening to the parents, allowing time for questions, and offering suggestions for support to assist them in dealing with this new challenge. Teaching can be done at a later time.

7.The nurse is conducting a psychosocial assessment of a child with asthma brought to the healthcare provider's office for a check-up. Which psychosocial issues may be assessed? Select all that apply. A) Health insurance coverage B) Transportation to healthcare facilities C) School's response to the chronic illness D) Past medical history E) Future treatment plans F) Health maintenance needs

Ans: A, B, C Feedback: Comprehensive health supervision includes frequent psychosocial assessments. Issues to be covered include health insurance coverage, transportation to healthcare facilities, financial stressors, family coping, and the school's response to the chronic illness. These are often stressful and emotionally charged issues. Past medical history, future treatment plans, and health maintenance needs would also be assessed; however, these are not psychosocial issues.

30. A new mother tells the nurse that she is having difficulty breastfeeding her baby. When observing the mother, which actions prompt the nurse to provide teaching about proper breastfeeding techniques? Select all that apply. A) The mother carefully washes her breasts prior to feeding the infant. B) The mother feeds the infant every hour. C) The mother supplements feedings with water. D) The mother holds her breast in the "C" position. E) The mother strokes the nipple against the infant's face.

Ans: A, B, C Feedback: The mother should wash her hands prior to breastfeeding the infant. There is no need to wash the breasts in most circumstances. The best time to feed the infant is on demand rather than hourly, and there is no need to supplement breastfeeding with water. The "C" position and stroking the nipple against the infant's face promote effective breastfeeding.

29. The nurse is assessing the infants in the nursery for the six stages of consciousness. The nurse becomes concerned when assessing which infants? Select all that apply. A) An infant rapidly moves from deep sleep to crying. B) An infant moves from active alert state to drowsiness. C) An infant progresses slowly from deep sleep to light sleep. D) An infant frequently skips the quiet alert state during the six stages of consciousness. E) An infant ends the stages of consciousness with crying.

Ans: A, B, D Feedback: The nurse becomes concerned if the infant does not move slowly through six states of consciousness, which begin with deep sleep. The infant should then progress as follows: light sleep, drowsiness, quiet alert state, active alert state, and finally crying. States are not normally skipped.

1.The nurse is providing atraumatic care to children in a hospital setting. What are principles of this philosophy of care? Select all that apply. A) Avoid or reduce painful procedures. B) Avoid or reduce physical distress. C) Minimize parent-child interactions. D) Provide child-centered care. E) Minimize child control. F) Use core primary nursing.

Ans: A, B, F Feedback: When using atraumatic care, the nurse would avoid or reduce painful procedures, avoid or reduce physical distress, use core primary nursing, maximize parent-child interactions, provide family-centered care, and provide opportunities for control, such as participating in care, attempting to normalize daily schedule, and providing direct suggestions.

17.The nurse is preparing to perform a dressing change on a 13-year-old client who is being treated for burns he received 2 weeks ago. The client prefers not to take pain medication before the dressing change because it causes drowsiness. What nursing interventions would provide atraumatic care? Select all that apply. A) The nurse asks the client if he would like the television on during the dressing change. B) The nurse asks the client if a small group of nursing students can observe the dressing change. C) The nurse encourages the client to wear headphones to listen to music during the dressing change. D) The nurse encourages the parent to talk to the child about taking pain medication prior to the procedure. E) The nurse tells the client that the dressing change will not be performed unless pain medication is taken.

Ans: A, C Feedback: Minimizing stress prior to and during a procedure helps provide atraumatic care. Since the child chooses to not take pain medication, watching television or using headphones during the procedure provides distraction to the discomfort of the procedure. Students observing does not provide distraction. The child has chosen for the last 2 weeks to not receive pain medication, so having the parent talk to the child again does not provide atraumatic care. The nurse cannot force the child to take pain medication.

29.A mother of three brings her children in for their vaccinations. The mother tells the nurse that her mother recently died and her husband just lost his job due to his company downsizing. Which parenting behaviors is the nurse likely to observe? Select all that apply. A) The mother rarely looks at her infant when the nurse is assessing the child. B) The mother voices pride in the academic accomplishments of her 7-year-old child. C) The mother becomes very frustrated and tells the nurse she can't handle her toddler's temper tantrum. D) The mother asks if the nurse has suggestions on ways to potty train her toddler. E) The mother utilizes the correct size of infant car seat for her 3-month-old child.

Ans: A, C Feedback: When the family is faced with excessive stressors, the nurse may be able to ascertain the stress by observing the parent-child interaction during the health supervision visit. The nurse can learn much about the family dynamic by observing the family for behavioral clues. Lack of eye contact and care of the infant is a clue to family stress, as well as effective parenting techniques for behaviors such as temper tantrums.

When the nurse is assessing a 2-day-old newborn and suspects Down syndrome, what factors would lead to this assessment? Select all that apply. A) Flat facial profile B) Downward slant to the eyes C) Large tongue compared to mouth D) Simian crease E) Epicanthal folds F) Rigid joints

Ans: A, C, D, E Common clinical manifestations of Down syndrome include flat facial profile, upward slant to the eyes (oblique palpebral fissures), tongue that is large in comparison to the mouth size, simian, crease, epicanthal folds, and loose joints.

When performing a physical examination on a small child, the nurse observes approximately 8 to 10 light-brown spots concentrated primarily on the trunk and extremities, two small lumps on the posterior trunk, and axillary freckling. What condition do these findings suggest? A) Klinefelter syndrome B) Neurofibromatosis C) Fragile X syndrome D) Sturge-Weber syndrome

Ans: B The hallmark of neurofibromatosis is café-au-lait spots appearing all over the body, particularly the trunk and extremities. Additional findings include benign tumors, axillary freckling, and pigmented nevi. Klinefelter syndrome is associated with a lack of secondary sex characteristics, decreased facial hair, gynecomastia, decreased pubic hair, and hypogonadism. Fragile X syndrome is manifested by minor dysmorphic features and developmental delay. Sturge-Weber syndrome is associated with facial nevus, seizures, hemiparesis, and intracranial calcifications.

6.The nurse is aware that the community affects the health of its members. Which statements accurately reflect a community influence of health care? Select all that apply. A) A community can be a contributor to a child's health or be the cause of his or her illnesses. B) The child's health should be separated from the health of the surrounding community. C) Community support and resources are necessary for children with significant problems. D) Poverty has not been linked to an increase in health problems in communities. E) The breakdown of community and family support systems can lead to depression and violence. F) Ideally, the child's medical home is located outside the community.

Ans: A, C, E Feedback: A community can be a contributor to a child's health or be the cause of his or her illnesses. Community support and resources are necessary for children with significant problems since a close working relationship between the child's healthcare provider and community agencies is an enormous benefit to the child. Children from communities suffering the large-scale breakdown of family relationships and loss of support systems will be at increased risk for depression, violence and abuse, substance abuse, and HIV infection. The child's health cannot be totally separated from the health of the surrounding community. Poverty has been linked to low birthweight and premature birth, among other health problems. Ideally the child's medical home is within the family's community to reduce barriers such as lack of transportation, expense of travel, and time away from the parents' workplace.

8. The nurse caring for newborns knows that infants exhibit phenomenal increases in their gross motor skills over the first 12 months of life. Which statements accurately describe the typical infant's achievement of these milestones? Select all that apply. A) At 1 month, the infant lifts and turns the head to the side in the prone position. B) At 2 months, the infant rolls from supine to prone to back again. C) At 6 months, the infant pulls to stand up. D) At 7 months, the infant sits alone with some use of hands for support. E) At 9 months, the infant crawls with the abdomen off the floor. F) At 12 months, the infant walks independently.

Ans: A, D, E, F Feedback: At 1 month, the infant lifts and turns the head to the side in the prone position. At 7 months, the infant sits alone with some use of hands for support. At 9 months, the infant crawls with the abdomen off the floor. At 12 months, the infant walks independently. At 4 months, the infant lifts the head and looks around. At 10 months, the infant pulls to stand up.

3. The nurse is assessing the respiratory system of a newborn. Which anatomic differences place the infant at risk for respiratory compromise? Select all that apply. A) The nasal passages are narrower. B) The trachea and chest wall are less compliant. C) The bronchi and bronchioles are shorter and wider. D) The larynx is more funnel shaped. E) The tongue is smaller. F) There are significantly fewer alveoli.

Ans: A, D, F Feedback: In comparison with the adult, in the infant, the nasal passages are narrower, the trachea and chest wall are more compliant, the bronchi and bronchioles are shorter and narrower, the larynx is more funnel shaped, the tongue is larger, and there are significantly fewer alveoli. These anatomic differences place the infant at higher risk for respiratory compromise. The respiratory system does not reach adult levels of maturity until about 7 years of age

A nursing instructor is preparing a class discussion on the benefits and drawbacks associated with genetic advances and the Human Genome Project. What would the instructor address as a potential problem? A) Early detection possibilities B) Risk profiling C) Focus on causes D) Rapid diagnosis

Ans: B Although current and potential applications of the Human Genome Project to health care are numerous, risk profiling presents a potential problem. Risk profiling based on an individual's unique genetic makeup can be used to tailor prevention, treatment, and ongoing management of health conditions, but it will raise issues associated with client privacy and confidentiality related to workplace discrimination and access to health insurance. Early detection possibilities, focus on causes, and rapid diagnosis are benefits to the information gained from the Human Genome Project.

A pregnant woman is to undergo testing to evaluate for chromosomal abnormalities. Which test would the nurse expect to be done the earliest? A) Amniocentesis B) Chorionic villi sampling C) Triple screen D) Fetal nuchal translucency

Ans: B Chorionic villi sampling is performed at 7 to 11 weeks' gestation. Amniocentesis usually is performed after 15 weeks' gestation. A triple screen is usually done between 16 and 19 weeks' gestation. Fetal nuchal translucency must be performed between 11 and 14 weeks.

When teaching the parents of a child with phenylketonuria, the nurse would instruct them to include which food in the child's diet? A) Milk B) Oranges C) Meat D) Eggs

Ans: B Foods that contain phenylalanine are to be avoided. These include milk, meat, and eggs. Foods such as oranges would be allowed

The nurse is teaching a couple about X-linked disorders. They are concerned that they might pass on hemophilia to their children. What response by the couple indicates the need for further teaching? A) "The father can't be a carrier if he doesn't have hemophilia." B) "If the father doesn't have it, then his kids won't either." C) "If the mother is a carrier, her daughter could be one too." D) "If the mother is a carrier, her sons may have hemophilia."

Ans: B Hemophilia is an X-linked recessive disorder. This means that both the father and the mother must have the gene for hemophilia to pass it on to their children. Also, their male children will have hemophilia, while their female children have only a 50% chance of having the disorder. If the father has hemophilia and the mother has hemophilia, their children will have the disease. If the father has hemophilia and the mother is a carrier, all their children have a 50% chance of getting the disease.

When providing support and education to the family of a child who is diagnosed with a serious genetic abnormality, what would be the priority? A) Assisting with scheduling follow-up visits B) Establishing a trusting relationship C) Teaching the family what to expect D) Using measures to promote growth and development

Ans: B Regardless of the genetic abnormality, learning of a genetic abnormality may be shattering to the family. Therefore, the initial priority is to establish a trusting relationship. Once this is accomplished, other aspects of care, such as assisting with scheduling follow-up visits, teaching, and implementing measures to promote growth and development, can be addressed.

3.The nurse is preparing a presentation to a local parent group about pediatric health supervision. Which would the nurse emphasize as the focus? A) Injury prevention B) Wellness C) Health maintenance D) Developmental surveillance

Ans: B Feedback: The focus of pediatric health supervision is wellness. Injury and disease prevention, health maintenance and promotion, and developmental surveillance are all critical components of wellness.

19.The nurse is caring for a child who is scheduled to begin chemotherapy. When planning education for the parents, what action by the nurse is most correct? A) Obtain a large classroom to allow the nurse to stand at the front and present information. B) Obtain a small conference room and arrange the chairs in a circle for both the nurse and family members to sit. C) Provide written information to the family and allow them to review it, with instructions to contact the nurse if there are additional questions. D) Provide a video of information to the family, with instructions to contact the nurse if there are additional questions.

Ans: B Feedback: Teaching is an important function of the nurse. When providing education, it is important to offer the information in an environment that is conducive to learning. A circular set of chairs will allow the nurse to face the parents during the exchange. A large class that has the nurse standing and the parents sitting does not provide the ability for a personal interaction needed for this session. Giving the parents information in writing should be done in conjunction with a face-to-face teaching session. Video information may be beneficial but does not replace the face-to-face teaching session.

21. A nurse is preparing to start an intravenous (IV) line in a child with severe pneumonia. The nervous child asks the nurse to wait until later to do the procedure. How should the nurse proceed? A) Inform the child that the procedure will have to happen immediately. B) Explain to the child why the IV is needed and find creative games to utilize while inserting the IV. C) Call the health care provider to see if the medication can be given in liquid form by mouth. D) Ask the parent to hold the child down so that the procedure can be completed.

Ans: B Feedback: When a procedure is necessary the nurse should use a firm, positive, and confident approach that provides the child with a sense of security. The child should be allowed to express feelings of anger, anxiety, fear or frustration but also know the procedure is necessary. In atraumatic care, the nurse should use a topical anesthetic at the IV site prior to the IV insertion to minimize pain. The parents should not be used as a restraint. This causes severe anxiety for the parent and the child. If an IV is prescribed to be placed, then most likely IV medications will be needed. Just because the child does not want the IV, the child should not be allowed to dictate care.

6.The child life specialist (CLS) is preparing a 6-year-old child for a magnetic resonance imaging (MRI) scan. Which statement reflects the use of atraumatic principles when explaining the procedure? A) "You will be taken to a magnetic resonance imaging machine for an x-ray of your liver." B) "You may hear some loud noises when you are lying in the machine, but they won't hurt you." C) "You have nothing to worry about; the MRI machine is safe and will not cause you any pain." D) "Let's just get you to the x-ray department for your test and you'll see how simple it is."

Ans: B Feedback: When using atraumatic principles, the CLS would explain any sensations, such as noises that will be experienced. The language should be simple and at the child's developmental age; using the technical term for the machine might frighten the child. Telling the child there is nothing to worry about does not allay the child's fears. Allowing the child to experience the machine without explaining the sensations does not follow atraumatic principles.

20.The nurse is caring for a teen who will be hospitalized for physical rehabilitation for an extended period of time after an auto accident. When working to promote a good working relationship with the teen, what action by the nurse will be most beneficial? A) Allow the teen to control the daily schedule. B) Keep your word with regard to promises and statements made to the teen. C) Allow the teen to make decisions about the plan of care. D) Include the teen in the weekly interdisciplinary care conferences

Ans: B Feedback: When working with teens, the establishment of trust and rapport is of the highest priority. Establishing trust can best be done by demonstrating consistency and keeping promises made to the teen. Control of the daily schedule may not be feasible. The teen can be allowed to have an impact on some elements of the plan of care but this does not have a greater importance than the establishment of trust. The teen may be able to attend care conferences, but this is not of the highest priority.

4.A large portion of the nurse's efforts is dedicated to health supervision for children who use the facility as their primary medical contact. At which facility does the nurse work? A) An urgent care center B) A pediatric practice C) A mobile outreach immunization program D) A dermatology practice

Ans: B Feedback: A pediatric practice is most likely to fulfill the characteristics for primary care, also known as a medical home. An urgent care center does not provide preventative care activities. Mobile outreach would not provide for any care requiring hospitalization. A dermatology practice is unlikely to provide service outside its area of specialization.

26.After teaching the mother about follow-up immunizations for her daughter, who received the varicella vaccine at age 14 months, the nurse determines that the teaching was successful when the mother states that a follow-up dose should be given at which time? A) When the child is 20 to 36 months of age B) When the child is 4 to 6 years of age C) When the child is 11 to 12 years of age D) When the child is 13 to 15 years of age

Ans: B Feedback: A second dose of varicella vaccine should be given when the child is 4 to 6 years of age. Hepatitis A vaccine should be given to infants at age 12 months, with a repeat dose given in 6 to 12 months. The human papillomavirus (HPV) vaccine should be given to children beginning at age 11 to 12 years, with catch-up doses to begin at 13 to 14 years of age.

5. Which reflex, if found in a 4-month-old infant, would cause the nurse to be concerned? A) Plantar grasp B) Step C) Babinski D) Neck righting

Ans: B Feedback: Appropriate appearance and disappearance of primitive reflexes, along with the development of protective reflexes, indicates a healthy neurologic system. The step reflex is a primitive reflex that appears at birth and disappears at 4 to 8 weeks of age. The plantar grasp reflex is a primitive reflex that appears at birth and disappears at about the age of 9 months. The Babinski reflex is a primitive reflex that appears at birth and disappears around the age of 12 months. The neck righting reflex is a protective reflex that appears around the age of 4 to 6 months and persists.

16. The nurse is caring for a 4-week-old girl and her mother. Which is the most appropriate subject for anticipatory guidance? A) Promoting the digestibility of breast milk B) Telling how and when to introduce rice cereal C) Describing root reflex and latching on D) Advising how to choose a good formula

Ans: B Feedback: Telling the mother how to introduce rice cereal is the most appropriate subject for anticipatory guidance. Since this mother is already breast- or bottle-feeding her baby, educating her about these subjects would not inform her about what to expect in the next phase of development.

23. The nurse is educating a first-time mother who has a 1-week-old boy. Which is the most accurate anticipatory guidance? A) Describing the effect of neonatal teeth on breastfeeding B) Explaining that the stomach holds less than 1 ounce C) Informing that fontanels will close by 6 months D) Telling that the step reflex persists until the child walks

Ans: B Feedback: Explaining that the child's stomach holds less than 1 ounce gives the mother a reason for frequent, small feedings and is the most helpful and accurate anticipatory guidance. Telling that the step reflex persists until the child walks and informing that fontanels will close by 6 months are inaccurate. The step reflex disappears at about 2 months and fontanels close between 12 and 18 months. Neonatal teeth are highly unusual and need no explanation unless they occur.

17. The nurse is providing anticipatory guidance to a mother of a 5-month-old boy about introducing solid foods. Which statement by the mother indicates that effective teaching has occurred? A) "I'll start with baby oatmeal cereal mixed with low-fat milk." B) "The cereal should be a fairly thin consistency at first." C) "I can puree the meat that we are eating to give to my baby." D) "Once he gets used to the cereal, then we'll try giving him a cup."

Ans: B Feedback: Iron-fortified rice cereal mixed with a small amount of formula or breast milk to a fairly thin consistency is typically the first solid food used. As the infant gets older, a thicker consistency is appropriate. Strained, pureed, or mashed meats may be introduced at 10 to 12 months of age. A cup is typically introduced at 6 to 8 months of age regardless of what or how much solid food is being consumed

28. At which age would the nurse expect to find the beginning of object permanence? A) 1 month B) 6 months C) 9 months D) 12 months

Ans: B Feedback: Object permanence begins to develop between 4 and 7 months of age and is solidified by approximately age 8 months. By age 12 months, the infant knows he or she is separate from the parent or caregiver.

5.The nurse strives to provide culturally competent care for children in a health clinic that follows the principles of health supervision. Which nursing action reflects this type of care? A) The nurse treats all children the same regardless of their culture. B) The nurse negotiates a care plan with the child and family. C) The nurse researches the child's culture and provides care based on the findings. D) The nurse provides future-based care for culturally diverse children.

Ans: B Feedback: Optimal wellness for the child requires the nurse and the family to negotiate a mutually acceptable plan of care. The nurse must consider the culture of children because if the goals of the healthcare plan are not consistent with the health belief system of the family, the plan has little chance for success. Researching the culture is helpful, but the nurse should not assume all children follow cultural directives and base the care plan solely on the research. Most health promotion and disease prevention strategies in the United States have a future-based orientation; however, significant numbers of children belong to cultures with a present-based orientation. For these children, health promotion activities need shorter-term goals and outcomes to be useful.

22.The nurse is explaining the difference between active and passive immunity to the student nurse. Which statement accurately describes a characteristic of the process of immunity? A) Active immunity is produced when the immunoglobulins of one person are transferred to another. B) Passive immunity can be obtained by injection of exogenous immunoglobulins. C) Active immunity can be transferred from mothers to infants via colostrum or the placenta. D) Passive immunity is acquired when a person's own immune system generates the immune response.

Ans: B Feedback: Passive immunity can be obtained by injection of exogenous immunoglobulins. Passive immunity is produced when the immunoglobulins of one person are transferred to another. Passive immunity can also be transferred from mothers to infants via colostrum or the placenta. Active immunity is acquired when a person's own immune system generates the immune response.

4. A new mother shows the nurse that her baby grasps her finger when she touches the baby's palm. How might the nurse respond to this information? A) "This is a primitive reflex known as the plantar grasp." B) "This is a primitive reflex known as the palmar grasp." C) "This is a protective reflex known as rooting." D) "This is a protective reflex known as the Moro reflex."

Ans: B Feedback: Primitive reflexes are subcortical and involve a whole-body response. Selected primitive reflexes present at birth include Moro, root, suck, asymmetric tonic neck, plantar and palmar grasp, step, and Babinski. During the palmar grasp, the infant reflexively grasps when the palm is touched. The plantar grasp occurs when the infant reflexively grasps with the bottom of the foot when pressure is applied to the plantar surface. The root reflex occurs when the infant's cheek is stroked and the infant turns to that side, searching with mouth. The Moro reflex is displayed when with sudden extension of the head, the arms abduct and move upward and the hands form a "C."

26. The nurse is counseling the mother of a newborn who is concerned about her baby's constant crying. What teaching would be appropriate for this mother? A) Carrying the baby may increase the length of crying. B) Reducing stimulation may decrease the length of crying. C) Using vibration, white noise, or swaddling may increase crying. D) Using a swing or car ride may increase the incidence of crying episodes.

Ans: B Feedback: Prolonged crying leads to increased stress among caregivers. Reducing stimulation may decrease the length of crying, and carrying the infant more may be helpful. Some infants respond to the motion of an infant swing or a car ride. Vibration, white noise, or swaddling may also help to decrease fussing in some infants. Parents should try one intervention at a time, taking care not to stimulate the infant excessively in the process of searching for solutions.

19.The nurse is performing a risk assessment of a 5-year-old and determines the child has a risk factor for cystic fibrosis. What type of screening would the nurse perform to confirm or rule out this disease? A) Universal screening B) Selective screening C) Hyperlipidemia screening D) Developmental screening

Ans: B Feedback: Selective screening is done when a risk assessment indicates the child has one or more risk factors for the disorder. In universal screening, an entire population is screened regardless of the child's individual risk. Selectively screening children at high risk for hyperlipidemia can reduce their lifelong risk of coronary artery disease; it does not screen for cystic fibrosis. Developmental screening is performed to detect developmental delays.

24.The nurse is discussing vaccination for Haemophilus influenzae type B (Hib) with the mother of a 6-month-old child. Which comment provides the most compelling reason to get the vaccination? A) "These bacteria live in every human." B) "Young children are especially susceptible to these bacteria." C) "You have a choice of two excellent vaccines." D) "Your child needs this final dose for protection."

Ans: B Feedback: The most compelling reason for vaccination is that the highest rate of illness from influenza is in children. The fact that Hib is an opportunistic bacterium that lives in humans and only causes disease when resistance is lowered may be difficult for the parent to understand. A choice of two vaccines conveys no benefits to the mother. Need for the final dose is vague.

20.The nurse is caring for an infant who had hyperbilirubinemia requiring exchange transfusion. Based on this information, this infant is at risk for what type of disorder? A) Vision loss B) Hearing loss C) Hypertension D) Hyperlipidemia

Ans: B Feedback: There are many conditions that place an infant at risk for hearing loss, including an exchange transfusion with hyperbilirubinemia. A risk factor for vision loss is history of ocular structural abnormalities. Risk factors for systemic hypertension include preterm birth, very low birthweight, renal disease, organ transplant, congenital heart disease, or other illnesses associated with hypertension. A risk factor for hyperlipidemia is family history.

9. The nurse is teaching a new mother about the development of sensory skills in her newborn. What would alert the mother to a sensory deficit in her child? A) The newborn's eyes wander and occasionally are crossed. B) The newborn does not respond to a loud noise. C) The newborn's eyes focus on near objects. D) The newborn becomes more alert with stroking when drowsy.

Ans: B Feedback: Though hearing should be fully developed at birth, the other senses continue to develop as the infant matures. The newborn should respond to noises. Sight, smell, taste, and touch all continue to develop after birth. The newborn's eyes wander and occasionally cross, and the newborn is nearsighted, preferring to view objects at a distance of 8 to 15 in. Holding, stroking, rocking, and cuddling calm infants when they are upset and make them more alert when they are drowsy.

11. The nurse is preparing a nursing care plan for a child hospitalized for cardiac surgery. Which are examples of interventions that nurses perform in the 'building a trusting relationship' stage? Select all that apply. A) Gathering information about the child using the child's own toys B) Preparing the child for a procedure by playing games C) Explaining in simple terms what will happen during surgery D) Allowing the child to devise an exercise plan following surgery E) Praising the child for how well he is doing following instructions F) Giving the child a favorite toy to cuddle following a painful procedure

Ans: B, C Feedback: The introduction phase involves the initial contact with children and their families and it establishes the foundation for a trusting relationship. A trusting relationship can be built by using appropriate language, games, and play such as singing a song during a procedure, preparing the child adequately for procedures, and providing explanations and encouragement. In the decision-making phase, the nurse gives some control over to the child by allowing him to participate in making certain decisions, s

18.The nurse is admitting a 7-year-old child to the medical-surgical unit. The child answers questions with very short answers, makes little eye contact with the nurse, and looks to the parent to answer most questions. Which interventions would be appropriate during this admission assessment? Select all that apply. A) Tell the child that you are going to be their nurse so it would be best if they answered your questions. B) When asking questions, look at the child as well as the parent. C) Sit at the child's eye level during the admission questioning process. D) Stop asking questions for the present time and return later when the child feels more comfortable. E) Ask the child if they are always nervous around new people.

Ans: B, C Feedback: The goal is to establish rapport with the client and encourage communication. It is common for young children to be shy, so it is acceptable for the nurse to ask both the child and parent questions until the child feels comfortable talking with the nurse. Sitting at eye level is less intimidating and may help in establishing a trusting relationship. Telling the child that they need to answer the questions appears as condemning the child's behavior. Admission questions are important and can't be delayed until a later time. Asking the child if they are nervous around new people is intimidating and may further block communication.

The pediatric nurse is conducting an information session for parents regarding genetic alterations in children. Which comments by the parents of a child with a cleft palate indicate learning occurred during the session? Select all that apply. A) "I can't believe that it is because of my genes that our child has this disorder." B) "Our child will need to be counseled when reaching adulthood since the risk for passing along the gene that causes cleft palate is increased." C) "While I'm not sure what they are, I know that environmental factors have likely played a role in our child's cleft palate." D) "I wish we had genetic testing before having children. We might have felt a little more prepared for the possibility of disorders in our children." E) "This makes it almost certain that if we have more children they will have cleft palate as well."

Ans: B, C, D Many of the common congenital malformations, such as cleft palate, are attributed to multifactorial inheritance (a combination of genes from both parents, along with unknown environmental factors, produces the trait or condition). These disorders tend to have familial tendencies as well.

13.The nurse is performing developmental surveillance for children at a medical home. Which infants are most at risk for developmental delays? Select all that apply. A) A child whose birthweight was 1,600 g B) A child whose parent has a mental illness C) A child raised by a single parent D) A child with a lead level above 10 mg/dL E) A child with hypertonia or hypotonia F) A child with gestational age more than 33 weeks

Ans: B, C, D, E Feedback: Risk factors for developmental delays include having a single parent, a parent with developmental disability or mental illness, hypertonia or hypotonia, birthweight less than 1,500 g, lead level above 5 mg/dL, and gestational age less than 33 weeks.

12. The nurse is performing a health assessment of a 3-month-old African-American boy. For what condition should this infant be monitored based on his race? A) Jaundice B) Iron deficiency C) Lactose intolerance D) Gastroesophageal reflux disease (GERD)

Ans: C Feedback: Many dietary practices are affected by culture, both in the types of food eaten and in the approach to progression of infant feeding. Some ethnic groups tend to be lactose intolerant (particularly blacks, Native Americans, and Asians); therefore, alternative sources of calcium must be offered. Jaundice, iron deficiency, and GERD are not seen at a significantly higher rate in African-American infants.

11. The nurse observes an infant interacting with his parents. What are normal social behavioral developments for this age group? Select all that apply. A) Around 5 months, the infant may develop stranger anxiety. B) Around 2 months, the infant exhibits a first real smile. C) Around 3 months, the infant smiles widely and gurgles when interacting with the caregiver. D) Around 3 months, the infant will mimic the parent's facial movements, such as sticking out the tongue. E) Around 3 to 6 months of age, the infant may enjoy socially interactive games such as patty-cake and peek-a-boo. F) Separation anxiety may also start in the last few months of infancy.

Ans: B, C, D, F Feedback: The infant exhibits a first real smile at age 2 months. By about 3 months of age, the infant will start an interaction with a caregiver by smiling widely and possibly gurgling. The 3- to 4-month-old will also mimic the parent's facial movements, such as widening the eyes and sticking out the tongue. Separation anxiety may also start in the last few months of infancy. Around the age of 8 months, the infant may develop stranger anxiety. At 6 to 8 months of age, the infant may enjoy socially interactive games such as patty-cake and peek-a-boo.

After teaching a class about inborn errors of metabolism, the instructor determines that additional teaching is needed when the class identifies what as an example of an inborn error of metabolism? A) Galactosemia B) Maple syrup urine disease C) Achondroplasia D) Tay-Sachs disease

Ans: C Achondroplasia is an autosomal dominant genetic disorder, not an inborn error of metabolism. Galactosemia, maple syrup urine disease, and Tay-Sachs are considered inborn errors of metabolism.

The nurse is assessing a 4-year-old boy whose mother was 40 years old when he was born. Which finding suggests this child has a genetic disorder? A) Inquiry determines the child had feeding problems. B) Observation shows nasal congestion and excess mucus. C) Inspection reveals low-set ears with lobe creases. D) Auscultation reveals the presence of wheezing.

Ans: C Low-set ears are associated with numerous genetic dysmorphisms. Additionally, the mother's age during pregnancy is a risk factor for genetic disorders. Feeding problems could have been due to low birthweight, prematurity, or a variety of other reasons. The nasal congestion may be a cold. The wheezing could be bronchiolitis or asthma.

The nurse is assessing an infant and notes that the infant's urine has a mousy or musty odor. What would the nurse suspect? A) Maple syrup urine disease B) Tyrosinemia C) Phenylketonuria D) Trimethylaminuria

Ans: C The urine of a child with phenylketonuria has a mousy or musty odor. For the child with maple syrup urine disease, excretions have a maple syrup odor. With tyrosinemia, excretions have a cabbage-like or rancid butter odor. With trimethylaminuria, excretions smell like rotting fish.

8.The nurse knows that effective communication with children and their parents is critical to providing atraumatic quality nursing care. Which statement accurately describes the communication patterns of children? A) Communication patterns are similar from one child to the next. B) Children often use more words than adults to describe their fears. C) Children rely more on nonverbal communication and silence. D) Parents more often require affective communication rather than neutral communication.

Ans: C Feedback: Children often use fewer words than adults and may rely more on nonverbal communication and silence. Communication patterns can vary greatly from one child to the next. Some children are very talkative, while others are quiet. Parents more often require neutral communication (i.e., verbal communication that is related to assessing and solving problems), whereas children more often desire affective communication (establishment of rapport and trust, giving comfort).

3.The nurse is implementing interventions to prevent physical stressors for a 9-year-old child receiving chemotherapy in the hospital. How will the nurse provide atraumatic care for this child? A) Use restraint or "holding down" of the child during the procedure to prevent injury. B) Have the parent stand near and/or rub the child's feet during the procedure. C) Insert a saline lock if the child will require multiple doses of parenteral medications. D) Avoid using numbing techniques for multiple blood draws or IV insertion.

Ans: C Feedback: The nurse should insert a saline lock if the child will require multiple doses of parenteral medications. During painful or invasive procedures, the nurse should avoid traditional restraint or "holding down" of the child and use alternative positioning such as "therapeutic hugging." If therapeutic hugging is not an option, the nurse could have the parent stand near the child's head, not his feet to provide visual and verbal comfort. The nurse should also use numbing techniques for blood draws or IV insertion.

13.The nurse is educating a 16-year-old girl who has just been diagnosed with acute myelogenous leukemia. Which statement best demonstrates therapeutic communication? A) Discussing the treatment plan in detail for the next few weeks B) Using medical terms when describing the disease C) Assessing the adolescent's emotional status in private D) Talking about clothing and the stores where she shops

Ans: C Feedback: Therapeutic communication is goal directed and purposeful. Assessing the child's emotional status in private is goal directed and purposeful. Talking about clothing and shopping is not therapeutic communication unless its purpose is to find head coverings or wigs to mask hair loss and that information was not presented. Discussing the treatment plan for the next few weeks in detail is too much information for someone who has just been diagnosed. Using medical terms when describing the disease does not promote understanding.

5.The nurse is preparing a child and his family for a lumbar puncture. Which would be a primary intervention instituted to keep the child safe? A) Distraction methods B) Stimulation methods C) Therapeutic hugging D) Therapeutic touch

Ans: C Feedback: Therapeutic hugging (a holding position that promotes close physical contact between the child and a parent or caregiver) may be used for certain procedures or treatments where the child must remain still. Alternatively, distraction or stimulation (such as with a toy) can help to gain the child's cooperation, but therapeutic hugging would be used to keep the child safe during the procedure. Therapeutic touch is an energy therapy used to promote healing and decrease anxiety and stress and is not related to safety.

When describing Prader-Willi syndrome to a group of nursing students, the instructor would describe this condition as one affecting which chromosome? A) 4 B) 5 C) 11 D) 15

Ans: D Prader-Willi syndrome involves an abnormality on chromosome 15. Cri-du-chat involves an abnormality on chromosome 5; Wolf-Hirschhorn syndrome involves an abnormality on chromosome 4; and Beckwith-Wiedemann syndrome involves an abnormality on chromosome 11.

9.The nurse is teaching the student nurse how to communicate effectively with children. Which method would the nurse recommend? A) Position self above the child's level to denote authority. B) If possible, communicate with the child apart from the parent. C) Direct questions and explanations to the child. D) Use the medical terms for body parts and medical care.

Ans: C Feedback: To communicate effectively with children, the nurse should direct questions and explanations to the child; position self at the child's level; allow the child to remain near the parent if needed, so the child can remain comfortable and relaxed; and use the child's or family's terms for body parts and medical care when possible.

14.The nurse is performing a cultural assessment of an Asian family that has a child hospitalized for leukemia. What is the best technique for providing culturally competent care for this family? A) Research the culture and base care on findings. B) Ask other Asians to explain their culture. C) Just ask the family about their culture and listen. D) Hire an interpreter to explain the family culture.

Ans: C Feedback: Understanding and respecting the family's culture helps foster good communication and improves child and family education about health care. The best way to assess the family's cultural practices is to ask and then listen. Determine the language spoken at home and observe the use of eye contact and other physical contact. Demonstrate a caring, nonjudgmental attitude and sensitivity to the child's and family's cultural diversity. An interpreter should be hired for a family who does not speak English.

9.The father of a 13-year-old boy reports his family has a strong history of depression. He questions screening for his son. What information should be provided by the nurse? A) "Are you having concerns about depression in your son?" B) "Screening in at risk teens should be completed annually after age 14." C) "Children should be screened for depression every year beginning at age 11." D) "If you notice that your son is having mood issues, we can certainly refer him for an evaluation with a therapist."

Ans: C Feedback: Academy of Pediatrics recommended screening tool [CRAFFT) and a depression screening is recommended annually beginning at age 11. It is clear that the parent is voicing concerns for his son's risk factors. The question asked does not provide the information being requested.

10.During the health history, the mother of a 4-month-old child tells the nurse she is concerned that her baby is not doing what he should be at this age. What is the nurse's best response? A) "I'll be able to tell you more after I do his physical." B) "Fill out the questionnaire and then I can let you know." C) "Tell me what concerns you." D) "All mothers worry about their babies. I'm sure he's doing well."

Ans: C Feedback: Asking about the mother's concerns is assessment and is the first thing the nurse should do. The mother has intimate knowledge of the infant and can provide invaluable information that can help structure the nurse's assessment. Relying on the physical assessment ignores the value of the mother's input. A screening questionnaire is no substitute for a developmental assessment. Minimizing the mother's concerns reduces communication between the mother and the nurse.

14. The nurse is teaching a new mother the proper techniques for breastfeeding her newborn. Which is a recommended guideline that should be implemented? A) Wash the hands and breasts thoroughly prior to breastfeeding. B) Stroke the nipple against the baby's chin to stimulate wide opening of the baby's mouth. C) Bring the baby's wide-open mouth to the breast to form a seal around all of the nipple and areola. D) When finished, the mother can break the suction by firmly pulling the baby's mouth away from the nipple.

Ans: C Feedback: Before each breastfeeding session, mothers should wash their hands, but it is not necessary to wash the breast in most cases. The mother should then stroke the nipple against the baby's cheek to stimulate opening of the mouth and bring the baby's wide-open mouth to the breast to form a seal around all of the nipple and areola. When the infant is finished feeding, the mother can break the suction by inserting her finger into the baby's mouth.

2.The nurse is providing care for children in a pediatric medical home. What is a characteristic of care in these types of facilities? A) All insurance except Medicaid is accepted. B) Ambulatory care is not provided C) A centralized database contains all child information. D) Continuity of care is provided from infancy through adulthood.

Ans: C Feedback: In a medical home, a centralized database contains all pertinent information. All insurance including Medicaid is accepted in the medical home and ambulatory care is provided. Continuity of care is also provided from infancy to adolescence.

21. The nurse is assessing a 12-month-old boy with an English-speaking father and a Spanish-speaking mother. The boy does not say mama or dada yet. What is the priority intervention? A) Performing a developmental evaluation of the child B) Encouraging the parents to speak English to the child C) Asking the mother if the child uses Spanish words D) Referring the child to a developmental specialist

Ans: C Feedback: Infants in bilingual families may use some words from each language. Therefore, the priority intervention in this situation would be to ask the mother if the child uses Spanish words. There is not enough evidence to warrant performing a developmental evaluation or referring the child to a developmental specialist. Encouraging the parents to speak English to the child is unnecessary if the child is progressing with Spanish first.

30.Three children in a family, ages 7 months, 4 years, and 9 years have been tested for lead poisoning. The two younger children's tests reflect elevated lead levels and they will be undergoing treatment. The children's mother questions why her younger children were not "spared" as their older sibling was. What response by the nurse is most correct? A) "Some children are better able to metabolize toxins such as lead after exposure." B) "Your older child has a stronger liver and kidneys, which have helped her to better rid her body of the lead." C) "Younger children are often impacted because of their play behaviors place them on the floors and they often put things into their mouths." D) "It is likely your older child may have had elevated levels earlier in life but has gotten over the condition."

Ans: C Feedback: Lead poisoning is a problem that affects children younger than age 6 the most due to the fact that they are crawling on the ground and putting things in their mouths, and their developing neurologic system is more sensitive to the effects of lead. The liver and kidney development is not an influence on the degree of lead found in children's blood specimens. Metabolism is not the greatest influence on the reason why only the younger children have been impacted by lead poisoning.

25. The nurse is providing anticipatory guidance to a mother to help promote healthy sleep for her 3-week-old baby. Which recommended guideline might be included in the teaching plan? A) Place the baby on a soft mattress with a firm, flat pillow for the head. B) Place the head of the bed near the window to provide fresh air, weather permitting. C) Place the baby on his or her back when sleeping. D) If the baby sleeps through the night, wake him or her up for the night feeding.

Ans: C Feedback: Sudden infant death syndrome (SIDS) has been associated with prone positioning of newborns and infants, so the infant should be placed to sleep on the back. The baby should sleep on a firm mattress without pillows or comforters. The baby's bed should be placed away from air conditioner vents, open windows, and open heaters. By 4 months of age, night waking may occur, but the infant should be capable of sleeping through the night and does not require a night feeding.

21.The nurse is performing a vision screening for a 4-year-old child. Which screening chart would be best for determining the child's visual acuity? A) Snellen B) Ishihara C) Allen figures D) Color Vision Testing Made Easy (CVTME)

Ans: C Feedback: The Allen figures chart is reliable for assessing visual acuity for a preschool child. The Snellen chart requires that the child has a good knowledge of the alphabet. This is not an expectation for a 4-year-old child. The Ishihara and CVTME charts are designed to assess color vision discrimination and not visual acuity.

17. The nurse is screening a 6-year-old child for mental ability. Which test would the nurse use to assess intelligence? A) Denver Articulation Screening B) Denver PRQ C) Goodenough--Harris Drawing Test D) Parents' Evaluation of Developmental Status (PEDS)

Ans: C Feedback: The Goodenough--Harris Drawing Test is a nonverbal screen for mental ability (intelligence). The Denver Articulation Screening screens for articulation disorders. The Denver PRQ assesses personal-social, fine motor-adaptive, language, and gross motor skills. The PEDS screens for a wide range of developmental, behavioral, and family issues.

1. The nurse is examining a 10-month-old boy who was born 10 weeks early. Which finding is cause for concern? A) The child has doubled his birth weight. B) The child exhibits plantar grasp reflex. C) The child's head circumference is 49.53 cm. D) No primary teeth have erupted yet.

Ans: C Feedback: The child's head size is large for his adjusted age (7.5 months), which would be cause for concern. The average head circumference of the full-term newborn is 35 cm (13.5 in). Head circumference increases about 10 cm from birth to 1 year (Levine, 2019). Birth weight doubles by about 4 months of age. Plantar grasp reflex does not disappear until 9 months adjusted age. Primary teeth may not erupt until 8 months adjusted age.

2.The nurse is consulting with a child life specialist (CLS) to help minimize the stress of hospitalization for a child. Which services would the CLS provide? Select all that apply. A) Medical preparation for tests, surgeries, and other medical procedures B) Support before and after, but not during, medical procedures C) Activities to support normal growth and development D) Grief and bereavement support E) Emergency room interventions for children and families F) Only inpatient consultations with families

Ans: C, D, E Feedback: • The CLS would provide activities to support normal growth and development, grief and bereavement support, and emergency room interventions for children and families. The CLS would also provide nonmedical preparation for tests, surgeries, and other medical procedures; support during medical procedures; and outpatient consultation with families (American Academy of Pediatrics, Committee on Hospital Care and Child Life Council, 2014, reaffirmed 2018).

1.The nurse is caring for children in a healthcare provider's office where health supervision is practiced. Which are some points of focus of health supervision? Select all that apply. A) Making referrals for all healthcare needs B) Monitoring disease incidence C) Optimizing the child's level of functioning D) Monitoring quality of care provided E) Teaching parents to prevent injury F) Providing care developed from national guidelines

Ans: C, E, F Feedback: Health supervision involves providing services proactively, with the goal of optimizing the child's level of functioning. It ensures the child is growing and developing appropriately and it promotes the best possible health of the child by teaching parents and children about preventing injury and illness (e.g., proper immunizations and anticipatory guidance). The framework for the health supervision visit is developed from national guidelines available through the U.S. Department of Health and Human Services (DHHS), the American Medical Association (AMA), and the American Academy of Pediatrics (AAP). Making referrals and monitoring disease incidence and quality of care provided may occur with this model, but they are not key focal points.

When providing guidance to the parents of a child with Down syndrome, which interaction would be most appropriate? A) Encourage the parents to home-school the child. B) Advise the parents that the child will need monthly thyroid testing. C) Instruct them on the need for yearly dental visits. D) Teach the parents about the need for a high-fiber diet.

Ans: D A high-fiber intake is important for children with Down syndrome because their lack of muscle tone may decrease peristalsis, leading to constipation. Early intervention programs with special education are important to promote growth and development. The child should be integrated into mainstream education whenever possible. Children with Down syndrome should undergo thyroid testing yearly and see the dentist every 6 months.

The nurse is caring for a 9-year-old boy with achondroplasia. What will the nurse's assessment reveal? A) Narrow passages from the nose to the throat B) Slim stature, hypotonia, and a narrow face C) Craniosynostosis and a small nasopharynx D) Trident hand and persistent otitis media

Ans: D Achondroplasia results in disordered growth with an average adult height of 4 feet for males or females. Other distinguishing symptoms are a separation between the middle and ring fingers, called trident hand, and persistent otitis media and middle ear dysfunction. Narrow passages from nose to throat are a symptom of CHARGE syndrome. Slim stature, hypotonia, and a narrow face are symptoms of Marfan syndrome. Craniosynostosis and a small nasopharynx are symptoms of Apert syndrome.

The nurse is preparing a presentation to a local community group about genetic disorders and the types of congenital anomalies that can occur. What would the nurse include as a major congenital anomaly? A) Overlapping digits B) Polydactyly C) Umbilical hernia D) Cleft palate

Ans: D Cleft palate is considered a major congenital anomaly, one that creates a significant medical problem or requires surgical or medical management. Overlapping digits, polydactyly, and umbilical hernia are considered minor congenital anomalies because they do not cause an increase in morbidity in and of themselves.

The nurse is teaching a couple about the pros and cons of genetic testing. Which statement best describes the capabilities of genetic testing? A) "Various genetic tests help the physician choose appropriate treatments." B) "Genetic testing helps couples avoid having children with fatal diseases." C) "Genetic tests identify people at high risk for preventable conditions." D) "Some genetic tests can give a probability for developing a disorder."

Ans: D The fact that some tests only provide a probability for developing a disorder raises a problem. A serious limitation of these susceptibility tests is that some people who carry a disease-associated mutation never develop the disease. The other statements affirm the value of genetic tests.

11.The nurse is caring for a 4-year-old boy with Ewing sarcoma who is scheduled for a computed axial tomography (CAT) scan tomorrow. Which is the best example of therapeutic communication? A) Telling him he will get a shot when he wakes up tomorrow morning B) Telling him how cool he looks in his baseball cap and pajamas C) Using family-familiar words and soft words when possible D) Describing what it is like to get a CAT scan using words he understands

Ans: D Feedback: Describing what it is like to get a CAT scan using age-appropriate words is the best example of therapeutic communication. It is goal-directed, focused, and purposeful communication. Using family-familiar words and soft words is a good teaching technique. Telling him how cool he looks in his baseball cap and pajamas is not goal-directed communication. Telling the child he will get a shot when he wakes up could keep him awake all night.

15.The nurse is educating the parents of a 7-year-old girl who has just been diagnosed with epilepsy. Which teaching technique would be most appropriate? A) Assessing the parents' knowledge of the anticonvulsant medications B) Demonstrating proper seizure safety procedures C) Discussing the surgical procedure for epilepsy D) Giving the parents information in small amounts at a time

Ans: D Feedback: Parents, when given a life-altering diagnosis, need time to absorb information and to ask questions. Therefore, giving the parents information in small amounts at a time is best. The child has just been diagnosed with epilepsy, and surgical intervention is not used unless seizures persist in spite of medication therapy. Therefore, discussing surgery would be inappropriate at this time. Assessing the parents' knowledge of the anticonvulsant medications identifies a knowledge gap and need to learn, but it would be unreasonable to think that they would understand the medications when the diagnosis had just been made. Demonstrating proper seizure safety procedures is an effective way to present information to an adult.

24. A 10-year-old child with sickle-cell anemia is frequently in the pediatric center of a hospital. What intervention can the nurse provide that will allow the child the sense of control that meets the goals promotes atraumatic care? A) Advocate for minimal laboratory blood draws. B) Promote family-centered care. C) Provide appropriate pain management. D) Maintain the child's home routine related to activities of daily living.

Ans: D Feedback: To promote a sense of control that meets the goals of atraumatic care, the nurse would attempt to maintain the child's home routine related to activities of daily living. In the hospital, the nurse would use primary nursing. The nurse would encourage the child to have a security item present if desired. Other measures include involving the child and family in planning care from the moment of the first encounter, empowering them by providing knowledge, allowing them choices when available, and making the environment more inviting and less intimidating. The nurse could advocate for minimum blood draws, but with the child's disease this will likely not happen. The nurse can help the child with reassurance and topical pain medication for laboratory draws to prevent the discomfort of multiple needle sticks. These actions, however, do not offer the child a sense of control.

8.The nurse is examining a 2-year-old child who was adopted from Guatemala. What would be a priority screening for this child? A) Screening for congenital defects B) Screening for abuse C) Screening for childhood illnesses D) Screening for infectious diseases

Ans: D Feedback: Although all the screenings are important, health supervision of the internationally adopted child must include comprehensive screening for infectious disease. In 2017, approximately 4,714 children were adopted from countries outside the United States, many from areas with a high prevalence of infectious diseases (Intercountry Adoption, Bureau of Consular Affairs, U.S. Department of State, 2018). Health supervision of the internationally adopted child must include comprehensive screening for infectious diseases, disorders of growth and development, along with vision and hearing and any further testing based on diseases common in their country of origin (Wilson & Simms, 2016). Proper screening is important not only to the child's health but also to the adopting family and the larger community.

22. A 6-month-old girl weighs 14.7 lb during a scheduled check-up. Her birth weight was 8 lb. What is the priority nursing intervention? A) Talking about solid food consumption B) Discouraging daily fruit juice intake C) Increasing the number of breastfeedings D) Discussing the child's feeding patterns

Ans: D Feedback: Assessing the current feeding pattern and daily intake is the priority intervention. Talking about solid food consumption may not be appropriate for this child yet. Discouraging daily fruit juice intake or increasing the number of breastfeedings may not be necessary until the situation is assessed.

28.The nurse is providing anticipatory guidance to an obese teenager. Which intervention would be most likely to promote healthy weight in teenagers? A) Make the focus of the program weight centered. B) Begin directly advising children about their weight at age 6. C) Focus physical activity on competitive sports and activities. D) Obtain nutritional histories directly from the school-age child and adolescent.

Ans: D Feedback: Before providing education to school-age and teenage children, it is important to obtain nutritional histories directly from them because increasingly they are eating meals away from the family table. The focus of healthy weight promotion should be health centered, not weight centered. Linking success to numbers on a scale increases the possibility of developing eating disorders, nutritional deficiencies, and body hatred. The nurse can begin directly advising children on healthy foods starting at age 3. The focus of physical activity should be on noncompetitive, fun activities.

The nurse if checking placement on a child's feeding tube. When the pH is checked, it is 5.3. What action by the nurse is indicated? A)Remove the tube. B)Document the findings as normal. C)Contact the health care provider. D)Re-evaluate the pH again in 2 hours.

Ans:C Gastric pH may be used to evaluate feeding tube placement. Normal gastric pH is less than 5.0. Findings greater than 5.0 indicate the need for further action. The nurse cannot remove the tube. The findings cannot be documented as normal. Evaluating the gastric pH again in 2 hours is not appropriate as the matter warrents more immediate action.

13. The nurse is promoting a healthy diet to guide a mother when feeding her 2-week-old girl. Which is the most effective anticipatory guidance? A) Substituting cow's milk if breast milk is not available B) Advocating iron supplements with bottle-feeding C) Advising fluid intake per feeding of 5 or 6 ounces D) Discouraging the addition of fruit juice to the diet

Ans: D Feedback: Discouraging the addition of fruit juice to the child's diet is the most effective anticipatory guidance. Fruit juice can displace important nutrients from breast milk or formula. Cow's milk is likely to result in an allergic reaction. If breast milk is not available, infant formula may be substituted. Advising fluid intake per feeding of 5 or 6 ounces is too much for this neonate, but is typical for an infant 4 to 6 months of age. Advocating iron supplements with bottle-feeding is unnecessary so long as the formula is fortified with iron.

11.A 3-year-old child is scheduled for a hearing screening. The nurse would prepare the child for screening by which method? A) Auditory brainstem response B) Evoked otoacoustic emissions C) Visual reinforcement audiometry D) Conditioned play audiometry

Ans: D Feedback: For children between the ages of 2 and 4 years, conditioned play audiometry would be an appropriate method for hearing screening. Auditory brainstem response and evoked otoacoustic emissions are appropriate hearing screening methods for newborns through age 6 months. Visual reinforcement audiometry is appropriate for children ages 6 months to 2 years.

24. A mother is concerned about her infant's spitting up. Which suggestion would be most appropriate? A) "Put the infant in an infant seat after eating." B) "Limit burping to once during a feeding." C) "Feed the same amount but space out the feedings." D) "Keep the baby sitting up for about 30 minutes afterward."

Ans: D Feedback: Keeping the baby upright for 30 minutes after the feeding, burping the baby at least two or three times during feedings, and feeding smaller amounts on a more frequent basis may help to decrease spitting up. Positioning the infant in an infant seat compresses the stomach and is not recommended.

7. The neonatal nurse assesses newborns for iron deficiency anemia. Which newborn is at highest risk for this disorder? A) A post-term newborn B) A term newborn with jaundice C) A newborn born to a diabetic mother D) A premature newborn

Ans: D Feedback: Maternal iron stores are transferred to the fetus throughout the last trimester of pregnancy. Infants born prematurely miss all or at least a portion of this iron store transfer, placing them at increased risk for iron deficiency anemia compared with term infants. An infant having jaundice, having been born to a mother with diabetes, or having been born postterm does not significantly place the infant at risk for iron deficiency anemia.

15. The nurse is providing discharge teaching regarding formula preparation for a new mother. Which guideline would the nurse include in the teaching plan? A) Always wash bottles and nipples in hot soapy water and rinse well; do not wash them in the dishwasher. B) Store tightly covered ready-to-feed formula can after opening in refrigerator for up to 24 hours. C) Warm bottle of formula by placing bottle in a container of hot water, or microwaving formula. D) Do not add cereal to the formula in the bottle or sweeten the formula with honey

Ans: D Feedback: Proper formula preparation includes the following: wash nipples and bottles in hot soapy water and rinse well or run nipples and bottles through the dishwasher; store tightly covered ready-to-feed formula can after opening in refrigerator for up to 48 hours; after mixing concentrate or powdered formula, store tightly covered in refrigerator for up to 48 hours; do not reheat and reuse partially used bottles; throw away the unused portion after each feeding; do not add cereal to the formula in the bottle; do not sweeten formula with honey; warm formula by placing bottle in a container of hot water; and do not microwave formula.

23.The nurse is administering a hepatitis B vaccine to a child. What is the classification of this type of vaccine? A) Killed vaccines B) Toxoid vaccines C) Conjugate vaccines D) Recombinant vaccines

Ans: D Feedback: Recombinant vaccines use genetically engineered organisms. The hepatitis B vaccine is produced by splicing a gene portion of the virus into a gene of a yeast cell. The yeast cell is then able to produce hepatitis B surface antigen to use for vaccine production. Killed vaccines contain whole dead organisms; they are incapable of reproducing but are capable of producing an immune response. Toxoid vaccines contain protein products produced by bacteria called toxins. The toxin is heat-treated to weaken its effect, but it retains its ability to produce an immune response. Conjugate vaccines are the result of chemically linking the bacterial cell wall polysaccharide (sugar-based) portions with proteins.

16.A mother and her 4-week-old infant have arrived for a health maintenance visit. Which activity will the nurse perform? A) Assess the child for an upper respiratory infection. B) Take a health history for a minor injury. C) Administer a varicella injection. D) Plot the child's head circumference on a growth chart.

Ans: D Feedback: The nurse will plot the head circumference of the child as part of developmental surveillance and screening. Assessing for an infection and taking a health history for an injury are not part of a health maintenance visit. Administering a vaccination for varicella would not occur until 12 months of age.

25.The mother of a 15-month-old child is questioning the nurse about the need for the hepatitis B vaccination. Which comment provides the most compelling reason for the vaccine? A) "The most common side effect is injection site soreness." B) "This is a recombinant or genetically engineered vaccine." C) "Immunizations are needed to protect the general population." D) "This protects your child from infection that can cause liver disease."

Ans: D Feedback: Up to 90% of neonates infected with hepatitis B develop chronic carrier status and will be predisposed to cirrhosis and hepatic cancer. The mother is not questioning side effects, safety, or disease prevention in general. Therefore, it is best to speak to her concerns.

The nurse will be administering a medication to a child that is primarily excreted by the kidney. The nurse is aware that this action is especially dangerous until the child reaches what age? Record your answer in years.

Ans:2 The immaturity of the kidneys until the age of 1 to 2 years affects renal blood flow, glomerular filtration, and active tubular secretion. This results in a longer half-life and increases the potential for toxicity of drugs primarily excreted by the kidneys.

Based on Erikson's developmental theory, what is the major developmental task of the adolescent? A) Gaining independence B) Finding an identity C) Coordinating information D) Mastering motor skills

B

The nurse is teaching the student nurse the factors that affect the pharmacodynamics of the drugs they are administering. What is a factor affecting this property of drugs? A)Immature body systems B)Weight C)Body surface D)Body composition

Ans:A Although a drug's mechanism of action is the same in any individual, the physiologic immaturity of some body systems in a child can affect a drug's pharmacodynamics (behavior of the medication at the cellular level). The child's age, weight, body surface area, and body composition also can affect the drug's pharmacokinetics (movement of drugs throughout the body via absorption, distribution, metabolism, and excretion).

The nurse is administering a crushed tablet to an 18-month-old infant. What is a recommended guideline for this intervention? A)Mix the crushed tablet with a small amount of applesauce. B)Place the crushed tablet in the infant's formula. C)Mix the crushed tablet with the infant's cereal. D)Crushed tablets should only be mixed with water.

Ans:A If a tablet or capsule is the only oral form available for children younger than 6 years, it needs to be crushed or opened and mixed with a pleasant-tasting liquid or a small amount (generally no more than a tablespoon) of a nonessential food such as applesauce. The crushed tablet or inside of a capsule may taste bitter, so it should never be mixed with formula or other essential foods. Otherwise, the child may associate the bitter taste with the food and later refuse to eat it.

The nurse is caring for a 6-year-old child who has multisystem trauma due to a motor vehicle accident. The child is receiving total parenteral nutrition (TPN). What is a recommended nursing intervention for children on TPN? A)Initially, check blood glucose levels frequently, such as every 4 to 6 hours, to evaluate for hyperglycemia. B)Be vigilant in monitoring the infusion rate, change the rate as necessary, and report any changes to the physician or nurse practitioner. C)If for any reason the TPN infusion is interrupted or stops, begin an infusion of a 10% saline at the same infusion rate as the TPN. D)Administer TPN continuously over an 8-hour period, or after initiation it may be given on a cyclic basis, such as over a 12-hour period during the night.

Ans:A Initially, the nurse should check blood glucose levels frequently, such as every 4 to 6 hours, to evaluate for hyperglycemia. Throughout TPN therapy, the nurse should be vigilant in monitoring the infusion rate, and report any changes in the infusion rate to the physician or nurse practitioner immediately. Adjustments may be made to the rate, but only as ordered by the physician or nurse practitioner. If for any reason the TPN infusion is interrupted or stops, the nurse should begin an infusion of a 10% dextrose solution at the same infusion rate as the TPN. TPN can be administered continuously over a 24-hour period, or after initiation it may be given on a cyclic basis, such as over a 12-hour period during the night.

The nurse is preparing to administer insulin to a diabetic child. Which would be the recommended route for this administration? A)Subcutaneous B)Intradermal C)Intramuscular D)Oral

Ans:A Subcutaneous (SQ) administration distributes medication into the fatty layers of the body. It is used primarily for insulin administration, heparin, and certain immunizations, such as MMR. Intradermal administration is used primarily for tuberculosis screening and allergy testing. Intramuscular administration is used to administer certain medications, such as many immunizations. Insulin is not administered orally.

A physician orders a medication dosage that is above the normal dosage. The nurse administers the medication without questioning the dosage. What error did the nurse make? A)The nurse violated one of the "rights" of medication administration. B)The nurse performed an act outside the scope of practice for nursing. C)The nurse has not made an error, but the physician did by ordering the wrong dosage of medication. D)The nurse has committed an act of maleficence by administering the medication.

Ans:A The nurse violated one of the "rights" of medication administration, the right dosage, because the nurse is responsible for being aware and questioning an incorrect dosage of medication. Medication administration is within the scope of nursing practice. Maleficence is performing a harmful act intentionally.

The nurse is preparing to administer medication to a child with a gastrostomy tube in place. What is a recommended guideline for this procedure? Select all that apply. A)Verify proper tube placement prior to instilling medication. B)Mix liquid medications with a small amount of water and add directly into the tube. C)Mix powdered medications well with cold water first. D)Crush tablets and mix with warm water to prevent tube occlusion. E)Open up capsules and mix the contents with warm water. F)Flush the tube with water after administering medications.

Ans:A, D, E, F The correct procedure includes checking proper tube placement prior to instilling medication, crushing tablets and mixing with warm water to prevent tube occlusion, opening up capsules and mixing the contents with warm water, and flushing the tube with water after administering medications. The nurse should give liquid medications directly into the tube and mix powdered medications well with warm water first.

The nurse is caring for children who are receiving IV therapy in the hospital setting. For which children would a central venous device be indicated? A)A child who is receiving an IV push B)A child who is receiving chemotherapy for leukemia C)A child who is receiving IV fluids for dehydration D)A child who is receiving a one-time dose of a medication

Ans:B Although central venous access devices can be used short term, the majority are used for moderate- to long-term therapy, such as chemotherapy. Central venous access devices are indicated when the child lacks suitable peripheral access, requires IV fluid or medication for more than 3 to 5 days, or is to receive specific treatments, such as the administration of highly concentrated solutions or irritating drugs that require rapid dilution. Peripheral IV devices are used for most other IV therapies.

The nurse is teaching the parents of a 1-month-old girl with Down syndrome how to maintain good health for the child. Which instruction would the nurse be least likely to include? A) Getting cervical radiographs between 3 and 5 years of age B) Adhering to the special dietary needs of the child C) Getting an echocardiogram before 3 months of age D) Monitoring for symptoms of respiratory infection

Ans:B Children with Down syndrome do not require a special diet unless underlying gastrointestinal disease is present. However, a balanced, high-fiber diet and regular exercise are important. Getting cervical radiographs between 3 and 5 years of age is the screening method for atlantoaxial instability, which is seen in about 14% of children with Down syndrome. Evaluation by a pediatric cardiologist before 3 months of age, including an echocardiogram, is important since children with Down syndrome are at higher risk for heart disease. The child will be more susceptible to infectious diseases.

The nurse is providing teaching on how to administer nasal drops. Which response by the parents indicates a need for further teaching? A)"We need to be careful not to stimulate a sneeze." B)"She needs to remain still for at least 10 minutes after administration." C)"Our daughter should lie on her back with her head hyperextended." D)"We must not let the dropper make contact with the nasal membranes."

Ans:B Once the drops are instilled, the child should remain in hyperextension for at least 1 minute to ensure the drops have come in contact with the nasal membranes. Ten minutes would be excessive. The other statements are correct.

The nurse is caring for an 8-year-old girl who requires medication that is only available in an enteric tablet form. The nurse is teaching the mother how to help the girl swallow the medication. Which statement indicates a need for further teaching? A)"I can encourage her to place it on the back of her tongue." B)"I can pinch her nose to make it easier to swallow." C)"We cannot crush this type of pill as it will affect the delivery of the medication." D)"We can place the tablet in a spoonful of applesauce."

Ans:B The mother should be advised to never pinch the child's nose as it increases the risk for aspiration. The other statements are correct.

The nurse is providing teaching for the mother of an infant who receives all of his nutrition through a tube. The nurse is reviewing interventions to promote growth and development. Which response from the mother indicates a need for further teaching? A)"I will give him a pacifier during feeding time." B)"We need to keep feeding time very quiet." C)"We need to make sure he doesn't lose the desire to eat by mouth." D)"Sucking produces saliva, which aids in digestion."

Ans:B The nurse needs to emphasize that it is important to talk, play music, cuddle, and rock the infant to promote a normalized feeding time. The other statements are correct.

The nurse is preparing to administer a medication to a 5-year-old who weighs 35 pounds. The prescribed single dose is 1 to 2 mg/kg/day. Which is the appropriate dose range for this child? A)8 to 16 mg B)16 to 32 mg C)35 to 70 mg D)70 to 140 mg

Ans:B The nurse should convert the child's weight in pounds to kilograms by dividing the child's weight in pounds by 2.2. (35 pounds divided by 2.2 = 16 kg). The nurse would then multiply the child's weight in kilograms by 1 mg for the low end (16 kg × 1 mg = 16 mg) and then by 2 mg for the high end (16 kg × 2 mg = 32 mg).

The parents of a child receiving total parenteral nutrition ask the nurse why their child must have their blood glucose monitored so frequently since they are not diabetic. What is the best response by the nurse? A)"We like to keep a close check on the blood glucose for all children receiving total parenteral nutrition." B)"It is important to monitor the blood glucose level because the solution has a high concentration of carbohydrates that convert to glucose." C)"This is a good time for us to monitor your child in case they start developing signs of diabetes related to receiving total parenteral nutrition." D)"I would suggest you ask the physician why blood glucose checks have been ordered so frequently."

Ans:B Total parenteral nutrition has a high concentration of carbohydrates, which convert to glucose. Informing the parents that this is the reason for frequent monitoring of the blood glucose adequately addresses their question. It is routine for any patient receiving total parenteral nutrition to have frequent monitoring of blood glucose, but this does not answer the parent's question. There is no need to monitor a child for diabetes without reason. There is no reason to suggest asking the physician when this question can be answered by the nurse.

The nurse notes that a child with swallowing difficulty is receiving a continuous tube feeding. The child is very active and the feeding frequently gets interrupted because the tube becomes disconnected. What should the nurse discuss with the physician about the tube feeding? A)The nurse should ask the physician if the patient could receive total parenteral nutrition. B)The nurse should ask the physician if the patient could receive bolus rather than continuous tube feedings. C)The nurse should ask the physician if the patient could receive the tube feedings during the night rather than continuously during all hours. D)The nurse should ask the physician if the patient could be given oral rather than tube feedings. E)The nurse should ask the physician if the patient could be given a sedative in order to prevent disruption of the tube feedings.

Ans:B, C A bolus feeding is a specified amount of feeding solution that is given at specific intervals, usually over a short period of time such as 15 to 30 minutes, and is given via a syringe, feeding bag, or infusion pump. Continuous feedings are given at a slower rate over a longer period of time. In some cases, the feeding may be given during the night so that the child can be free to move about and participate in activities during the day. Either of these methods could help in the disruption of the feedings. Total parenteral nutrition is intravenous feeding and cannot be given for extended periods of time, nor would it help the active child. The child has a swallowing difficulty so oral feedings are not possible at this time. Sedatives would be considered a chemical restraint if given for this purpose.

The student nurse is preparing to administer eye drops to a 2-year-old child. Which actions indicate the need for additional instruction? Select all that apply. A)The student nurse explains the medication regimen to the child's parents. B)The nurse holds the medication bottle 3 inches from the child's nurse during administration. C)The child is instructed to look down during the instillation of the medication in the eyes. D)The student nurse seeks assistance to hold the child during the medication administration. E)The child is turned so the medication flows toward the outer corner of the eye.

Ans:B, C, E When preparing to administer medications to a child teaching to the parents and the child (based upon the child's ability to comprehend) about the medication and the procedure that will be used. When a child is under the age of 3, assistance should be obtained from another health care provider. The bottle should be held one inch from the child's nose. The child should be instructed to look up and to the side for the administration. The medication should flow toward the nose.

A nurse has just administered medication via an orogastric tube. What is the priority nursing action following administration? A)Check tube placement. B)Retape the tube. C)Flush the tube. D)Remove the tube.

Ans:C After administration, the nurse should flush the tube to maintain patency and ensure that the entire amount of medication has been given. The tube should be checked prior to administering the medication. It is not necessary to retape the tube following administration. It is not appropriate to remove the tube unless it has been specifically ordered.

The school nurse is performing a physical examination on a 13-year-old boy who is on the soccer team. What is a physical quality that develops during these early adolescent years? A) Coordination B) Endurance C) Speed D) Accuracy

B

The nurse is caring for a 1-month-old girl with low-set ears and severe hypotonia who was diagnosed with trisomy 18. Which nursing diagnosis would the nurse identify as most likely? A) Interrupted family process related to the child's diagnosis B) Deficient knowledge deficit related to the genetic disorder C) Grieving related to the child's poor prognosis D) Ineffective coping related to stress of providing care

Ans:C Grieving related to the child's prognosis is a diagnosis specific to this child's care. The prognosis for trisomy 18 is that the child will not survive beyond the first year of life. Ineffective coping related to the stress of providing care, deficient knowledge related to the genetic disorder, and interrupted family process due to the child's diagnosis could be appropriate for any family of a child with a genetic disorder.

The nurse is administering immunizations to children in a neighborhood clinic. What is the most frequent route of administration? A)Oral B)Intradermal C)Intramuscular D)Topical

Ans:C Intramuscular (IM) administration delivers medication to the muscle. In children, this method of medication administration is used infrequently because it is painful and children often lack adequate muscle mass for medication absorption. However, IM administration is used to administer certain medications, such as many immunizations.

After administering eye drops to a child, the nurse applies gentle pressure to the inside corner of the eye at the nose for which reason? A)To promote dispersion over the cornea B)To enhance systemic absorption C)To ensure the medication stays in the eye D)To stabilize the eyelid

Ans:C Punctal occlusion, or gentle pressure to the inside corner of the eye at the nose, helps to slow systemic absorption and ensure that the medication stays in the eye. Having the head lower than the body aids in dispersing the medication over the cornea. Placing the heel of the hand on the child's forehead and then retracting the lower lid helps to stabilize it.

The nurse is explaining to the student nurse the therapeutic effects of total parenteral nutrition (TPN). What accurately describes the use of TPN? A)It is used short term to supply additional calories and nutrients as needed. B)It is delivered via the peripheral vein to allow rapid dilution of hypertonic solution. C)It is a highly concentrated solution of carbohydrates, electrolytes, vitamins, and minerals. D)It is usually used when the child's nutritional status is within acceptable parameters.

Ans:C TPN is a highly concentrated solution of carbohydrates, electrolytes, vitamins, and minerals. TPN provides all nutrients to meet a child's needs. It is delivered via central venous access to allow rapid dilution of hypertonic solution. It is usually used in a child with a nonfunctioning gastrointestinal (GI) tract, such as a congenital or acquired GI disorder; a child with severe failure to thrive or multisystem trauma or organ involvement; and preterm newborns.

The nurse is preparing to administer oral ampicillin to a child who weighs 40 kg. The safe dose for children is 50 to 100 mg/kg/day divided in doses administered every 6 hours. What would be the low single safe dose and high single safe dose per day for this child? A)50 to 100 mg per dose B)100 to 500 mg per dose C)500 to 1,000 mg per dose D)1,000 to 5,000 mg per dose

Ans:C To calculate the dosage, the nurse would set up a proportion to calculate the low dose as follows: 50 mg/1 kg = x mg/40 kg; solve for x by cross-multiplying: 1 × x = 50 × 40; x = 2,000 mg divided by 4 doses per day = 500 mg. Then calculate the high safe dose range using the following proportion: 100 mg/1 kg = x mg/40 kg; solve for x by cross-multiplying: 1 × x = 100 × 40; x = 4,000 mg divided by 4 doses per day = 1,000 mg.

When describing the differences affecting the pharmacokinetics of drugs administered to children, which would the nurse include? A)Oral drugs are absorbed more quickly in children than adults. B)Absorption of intramuscularly administered drugs is fairly constant. C)Topical drugs are absorbed more quickly in young children than adults. D)Absorption of drugs administered by subcutaneous injection is increased.

Ans:C Topical absorption of drugs is increased in infants and young children because the stratum corneum is thinner and well hydrated. The absorption of oral drugs is slowed by slower gastric emptying, increased intestinal motility, a proportionately larger small intestine surface area, high gastric pH, and decreased lipase and amylase secretion. The absorption of drugs given intramuscularly or subcutaneously is erratic and may be decreased.

The nurse is monitoring the output for a 10-year-old child. The medical record indicates the child weighs 78 pounds. How much urine can be anticipated for this child for a 12-hour period? 1. 78 pounds = 35kg 2. 1 mL X 35kg = 35 mL/hr and 2 mL X 35 = 70 mL/hr 3. 35 mL X 12 hours = 420 mL 4. 70 mL X 12 hours = 840 mL A)300 to 1200 mL B)360 to 900 mL C)420 to 840 mL D)600 to 1200 mL

Ans:C Urinary output for a child will vary. As a general rule, output anticipated will be approximately 1.0 to 2.0 mL/kg/hour for children and adolescents. In a child who weighs 78 pounds, this will calculate as follows: (the rest of this was not available)

The nurse is administering acetaminophen PRN to a 9-year-old child on the pediatric ward of the hospital. Which answers reflect nursing actions that follow the rules of the 'eight rights' of pediatric medication administration? Select all that apply. A)The nurse identifies the child by checking the name on the child's chart. B)The nurse makes sure the medication is given within the hour of the ordered time. C)The nurse checks the documented time of the last dosage administered. D)The nurse calculates the dosage according to the child's weight. E)The nurse explains the therapeutic effects of the medication to the child and parents. F)The nurse administers the medication even though the child is adamant about not taking it.

Ans:C, D, E Following the 'right patient' rule, the nurse checks the documented time of the last dosage administered. For the 'right dose,' the nurse calculates the dosage according to the child's weight. For the 'right to be educated,' the nurse explains the therapeutic effects of the medication to the child and parents. To ensure the 'right patient,' the nurse confirms the child's identity and then checks with the caregivers for further identification. To administer at the 'right time,' the nurse gives the medication within 20 to 30 minutes of the ordered time, and to protect the child's 'right to refuse,' the nurse respects the child's or parents' option to refuse.

The school nurse is preparing a program on sexuality and birth control for a class of 14- to 16-year-olds. Which behavior will have the most influence on how the information is presented? A) Teens are adjusting to new body images. B) Adolescents tend to take risks. C) Teenagers are able to think in the abstract. D) Adolescents understand that actions have consequences.

B

The nurse is administering a liquid medication to a 3-year-old using an oral syringe. Which action would be most appropriate? A)Direct the liquid toward the anterior side of the mouth. B)Keep the child's hand away from the oral syringe when squirting the medication. C)Give all of the drug in the syringe at one time with one squirt. D)Allow the child time to swallow the medication in between amounts.

Ans:D When using an oral syringe to administer liquid medications, give the drug slowly in small amounts and allow the child to swallow before placing more medication in the mouth. The syringe is directed toward the posterior side of the mouth. The toddler or young preschooler may enjoy helping by squirting the medication into his or her mouth.

The nurse is caring for a couple who is having a triple screen done. The nurse would least likely expect what level to be tested? A) a-Fetoprotein B) Human chorionic gonadotropin C) Unconjugated estriol D) Testosterone

Ans:D A triple screen tests a-fetoprotein (AFP), human chorionic gonadotropin (hCG), and unconjugated estriol (uE3). Testosterone is not included.

The nurse is counseling a couple who suspect that they could bear a child with a genetic abnormality. What would be most important for the nurse to incorporate into the plan of care when working with this family? A) Gathering information from at least three generations B) Informing the family of the need for a wide range of information C) Maintaining the confidentiality of the information D) Presenting the information in a nondirective manner

Ans:D It is essential to respect client autonomy and present information in a factual, nondirective manner. In these situations, the nurse needs to understand that the choice is the couple's to make. Gathering information for three generations obtains a broad overview of what has been seen in both sides of the family. Maintaining confidentiality of the information is as important as with any other client information gathered. Informing family of the need for information is necessary because of its personal nature.

A nurse is caring for a 14-year-old with a gastrostomy tube. The girl has skin breakdown and irritation at the insertion site. Which would be the most appropriate method to clean and secure the gastrostomy tube? A)Make sure the tube cannot be moved in and out of the child's stomach. B)Use adhesive tape to tape the tube in place and prevent movement. C)Place a transparent dressing over the site whether there is drainage or not. D)If any drainage is present, use a presplit 2 × 2 and place it loosely around the site.

Ans:D Skin around the gastrostomy or jejunostomy insertion site may become irritated from movement of the tube, moisture, leakage of stomach or intestinal contents, or the adhesive device holding the tube in place. Keeping the skin clean and dry is important and will help prevent most of these problems. If any drainage is present, a presplit 2 × 2 can be placed loosely around the site and changed when soiled. If no drainage is present, the nurse should not place a dressing as it can cause undue pressure and trap moisture, leading to skin irritation. Preventing movement of the tube helps reduce skin irritation; however, the tube should be able to move slightly in and out of the child's stomach.

The nurse caring for a 6-year-old patient enters the room to administer an oral medication in the form of a pill. The dad at the bedside looks at the pill and tells the nurse that his daughter has a hard time swallowing pills. Which of the following is the best response by the nurse? A)Ask the child to try swallowing the pill and offer a choice of drinks to take with it. B)Crush the pill and add it to applesauce. C)Request that the physician prescribe the medication in liquid form. D)Call the pharmacy and ask if the pill can be crushed.

Ans:D The father is the best source of knowledge on medication administration for the child. The pharmacy should be called to determine if the pill might be crushed. Asking the child to try swallowing the pill disregards the information the father has just given. Requesting that the physician order the medication in liquid form is not necessary at this point.

The nurse is caring for a child who is receiving total parenteral nutrition (TPN) for failure to thrive. Which nursing action might the nurse take to prevent complications from this therapy? A)Adhere to clean technique when caring for the catheter and administering TPN. B)Ensure that the system remains an open system at all times. C)Secure all connections and open the catheter during tubing and cap changes. D)Use occlusive dressings and chlorhexidine-impregnated sponge dressings.

Ans:D The nurse should use occlusive dressings and chlorhexidine-impregnated sponge dressings to help prevent infection. The nurse should always follow agency or institution policy and procedures, adhere to strict aseptic technique when caring for the catheter and administering TPN, ensure that the system remains a closed system at all times, and secure all connections and clamp the catheter or have the child perform the Valsalva maneuver during tubing and cap changes.

The nurse is determining the amount of IV fluids to administer in a 24-hour period to a child who weighs 40 kg. How many milliliters should the nurse administer? A)1,000 mL B)1,500 mL C)1,750 mL D)1,900 mL

Ans:D Typically, the amount of fluid to be administered in a day (24 hours) is determined by the child's weight (in kg) using the following formula: 100 mL per kg of body weight for the first 10 kg (1,000) 50 mL per kg of body weight for the next 10 kg (500) 20 mL per kg of body weight for the remainder of body weight in kg (400).

22. A child has been prescribed a nasal cannula for oxygen delivery. What should the nurse do before applying the cannula? A. Assess patency of the nares B. Test the oxygen saturation C. Add humidification to the delivery device D. Assess the lung sounds

Answer: A Rationale: A nasal cannula is a good delivery device for children, because it allows them to eat and talk unobstructed. Because the device is designed for flow through the nares, the patency of the nares should be assessed prior to using the cannula. If the nares are blocked from secretions, suctioning may be required. If there is a defect in the upper airway causing blockage, the nasal cannula may not be an appropriate oxygen delivery device. The oxygen saturation should have been measured and used as a guide for the prescription of oxygen therapy. Adding humidification is a way to keep the upper airways from becoming too dry, but oxygen can be started before humidity is added. Anytime a child is sick enough to require oxygen all respiratory assessments, including lung sounds, should be done. It does not matter, however, what the lung sounds are if the child is in enough distress to require oxygen. The lung sounds can be assessed after oxygen is started.

28. A hospitalized child suddenly begins reporting "my chest hurts," is tachypneic, and has tachycardia. The nurse auscultates the lung sounds and finds absent breath sounds on one side. After notifying the health care provide what action would the nurse take first? A. Prepare for chest tube insertion B. Administer oxygen C. Obtain oxygen saturation measurement D. Prepare for mechanical ventilation

Answer: A Rationale: A pneumothorax is a collection of air in the pleural space. Trapped air consumes space in the pleural cavity causing a partial or complete collapse. The priority symptom a nurse would assess is the decreased or absent lung sounds on the affected side. A pneumothorax can occur spontaneously in a healthy child or it can occur in a child with chronic lung disease, has been on a ventilator or has had thoracic surgery. Additional symptoms the child would experience would be chest pain, tachypnea, retractions, grunting, cyanosis and tachycardia. Many of these symptoms could be present with any child with an acute or chronic lung disease or respiratory distress, but the defining symptom is the absent breath sounds.The treatment for a pneumothoriax is with a chest tube so the priority action would be to gather supplies and prepare for the health care provider to insert a chest tube. Obtaining an oxygen saturation level measurement will only provide data, it will not help the child in distress. Oxygen may need to be administered, but with a pneumothorax it will be very ineffective. Mechanical ventilation would be a last resort and could actually make the situation worse if the lung was not reinflated.

9. A child requires supplemental oxygen therapy at 8 liters per minute. Which delivery device would the nurse most likely expect to be used? A. Simple mask B. Venturi mask C. Nasal cannula D. Oxygen hood

Answer: A Rationale: A simple mask would be used to deliver a flow rate of 8 liters per minute. A Venturi mask would be used to deliver a specific percentage of oxygen, from 24% to 50%. A nasal cannula would be used to deliver no more than 4 liters per minute. An oxygen hood requires a liter flow of 10 to 15 liters per minute.

20. The nurse is preparing to provide tracheostomy care to an infant. After gathering the necessary equipment, what would the nurse do next? A. Position the infant supine with a towel roll under the neck. B. Cut the new tracheostomy ties to the appropriate length. C. Cut the tracheostomy ties from around the tracheostomy tube. D. Cleanse around the site of the tracheostomy with the prescribed solution.

Answer: A Rationale: After gathering the necessary equipment, the nurse would position the infant supine with a blanket or towel roll to extend the neck. Then the nurse would open all the packaging and cut the new tracheostomy ties to the appropriate length. This would be followed by cleaning the site with the appropriate solution and then rinsing it. After placing the precut sterile gauze under the tracheostomy tube, the nurse would cut the ties and remove them from the tube while an assistant holds the tube in place.

6. The nurse is educating the parents of a 7-year-old boy with asthma about the medications that have been prescribed. Which drug would the nurse identify as an adjunct to a β2-adrenergic agonist for treatment of bronchospasm? A. Ipratropium B. Montelukast C. Cromolyn D. Theophylline

Answer: A Rationale: Ipratropium is an anticholinergic administered via inhalation to produce bronchodilation without systemic effects. It is generally used as an adjunct to a β2-adrenergic agonist. Montelukast decreases the inflammatory response by antagonizing the effects of leukotrienes. Cromolyn prevents release of histamine from sensitized mast cells. Theophylline provides for continuous airway relaxation.

7. The nurse is caring for a 3-year-old girl with a respiratory disorder. The nurse anticipates the need for providing supplemental oxygen to the child when performing which action? A. Suctioning a tracheostomy tube B. Administering drugs with a nebulizer C. Providing tracheostomy care D. Suctioning with a bulb syringe

Answer: A Rationale: Supplemental oxygenation may be necessary before, and is always performed after, suctioning a child with a tracheostomy tube. Providing tracheostomy care, administering drugs with a nebulizer, and suctioning with a bulb syringe do not require supplemental oxygen.

29. A child is hospitalized with pneumonia. The nurse assesses an increase in the work of breathing and in the respiratory rate. What intervention should the nurse do first to help this child? A. Elevate the head of the bed B. Administer oxygen C. Notify the health care provider D. Obtain oxygen saturation levels

Answer: A Rationale: The child who is experiencing increased work of breathing should be placed in a position to better open the airway and provide more room for lung expansion. Generally this is acomplished by elevating the head of the bed. If this does not improve the work of breathing, then administering oxygen should be done. The oxygen saturation should be measured because it will provide information as to the severity of the respiratory problem, but this measurement will not directly help the child. The health care provider should be notified if the child continues to deteriorate.

23. The nurse is obtaining a health history of a child suspected of tuberculosis. What question would the nurse ask first about the child's cough? A. "How long has your child had a cough?" B. "Does your child cough only at night?" C. "Does your child cough up anything when coughing?" D. "Has your child been around anyone who is coughing?"

Answer: A Rationale: Tuberculosis is a highly contagious disease. Most children contract it from an infected immediate household member. When taking the health history, the nurse should ask about symptoms such as malaise, weight loss, anorexia, chest tightness and a cough. The child's cough from tuberculosis is described as progressing slowly over several weeks and months rather than having an acute onset. Asking about the production from the cough is a way to determine if hemoptysis has occurred. Asking about being around anyone coughing is a way to determine if the child has been exposed to anyone with tuberculosis. Coughing only at night could be related to other respiratory disorders such as asthma.

27. A child is in the emergency department with an asthma exaccerbation. Upon asucultation the nurse is unable to hear air movement in the lungs. What action should the nurse take first? A. Administer a beta-2 adrenergic agonist B. Administer oxygen C. Start a peripheral IV D. Administer corticosteroids

Answer: A Rationale: When lungs sounds are unable to be heard in a child with asthma, the child is very ill. This means there is severe airway obstruction. The air movement is so severe wheezes cannot be heard. The priority treatment is to administer an inhaled short term bronchodilator (beta-2 adrenergic agonist). The child may require numerous inhalations until bronchodilation occurs and air can pass through the bronchi. Oxygen can be started but until the brochi are dilated no oxygen can get through to the lung fields. In IV would need to be started and IV steroids administered to reduce the inflammation, but the priority is bronchodilation.

25. The nurse is preparing a room for a child being transferred out of the intensive care unit. The child has a tracheostomy. What item(s) are essential for the nurse to have available at the bedside in case of emergency? Select all that apply. A. A new tracheostomy tube of the same size B. A new tracheostomy tube of a smaller size C. A bag valve mask D. A sterile tracheostomy kit E. Cleaning supplies for the tracheostomy

Answer: A, B, C Rationale: A child with a tracheostomy can have an emergent situation for any number of reasons. It is important to always have emergency equipment at the bedside to provide immediate care when these situations arrive. Two spare tracheostomy tubes should always be at the bedside, one the same size as in place and once a size smaller. These would be needed if the tube became dislodged. A bag valve mask needs to remain at the bedside at all times. Ideally it should be connected to oxygen, but that is an individual protocol for the health care organization. The bag can be used to hyperoxygenate the child prior to or following suctioning or it can be used in an emergent situation such as a respiratory arrest. Sterile tracheostomy kits and cleaning supplies can be available at the bedside, but they are used for routine cleaning and not for emergencies.

30. An infant with a high respiratory rate is NPO and is receiving IV fluids. What assessment(s) will the nurse make to assure this infant is hydrated? Select all that apply. A. Measure skin turgor B. Palpate anterior fontanel C. Determine urine output D. Review electrolyte laboratory results E. Assess the lung sounds

Answer: A, B, C Rationale: IV fluids are necessary many times for infants and children who are experiencing high respiratory rates. The high respiratory rates make the child very tired from the increased work of breathing. In an infant there are very little reserves so the infant tires very quickly, especially when the work of sucking is added to the compromised respiratory state. To determine if the infant is hydrated the nurse should assess the skin turgor, palpate for a flat anterior fontanel, observe for moist mucus membranes and measure the urine output. The urine output should be 1 to 2ml/kg/hr. The electrolyte laboratory results will tell the nurse if the infant has an electrolyte imbalance, not a fluid imbalance. Assessing the lung sounds will not tell if the child is hydrated, only if the lungs are "wet" and fluid overloaded. The infant would also exhibit additional signs of respiratory distress if the lungs are fluid overloaded.

24. The nurse is teaching the parent of a child with cystic fibrosis about nutrition requirements for the child. What should be included in this teaching? A. "Give your child high-calorie foods and snacks." B. "Feed your child foods that are high in protein." C. "Administer water soluble vitamins." D. "Give panreatic enzymes with meals." E. "Give your child foods high in fat."

Answer: A, B, D Rationale: Children with cystic fibrosis (CF) have trouble digesting and absorbing nutrients. They tend to be underweight. For optimal health, their diets should be high in calories and high in protein, with the supplementation of fat soluble vitamins and pancreatic enzymes. This diet helps with growth and the optimal nutrients. The fat soluble vitamins (vitamins A, D, E and K) are needed, because children with CF have trouble absorbing fat and need the vitamin supplementation to aid in fat absorption. Water soluble vitamins (the B vitamins and vitamin C) do not aid in fat absorption. The child should not have a high-fat diet, because the extra fat is difficult to digest and be absorbed. Pancreatic enzymes are necessary because they are missing due to the disease process. They are necessary to aid in digestion. They should be ingested with meals.

26. A parent with a child who has cystic fibrosis asks the nurse how to determine if the child is receiving an adequate amount of pancreatic enzymes. How should the nurse respond? Select all that apply. A. "The dose is adequate when your child is only having 1 to 2 stools per day." B. "The dose is adequate when your child's weight is improving." C. "The dose prescribed is based on your child's pancreatic laboratory values so it should be correct." D. "When your child starts to eat more quantity of food you will need to adjust the amount of enzyme pills." E. "You will need to give your child less enzyme pills when high-fat foods are eaten."

Answer: A, B, D Rationale: Pancreatic enzymes are required for the child with cystic fibrosis (CF) to help absorb nutrients from the diet and to aid in digestion. They are given with each meal and snack the child eats. The number of capsules required at each dose depends upon the diagnosis of how the pancreas is functioning and the amount of food needing to be digested. The pancreatic laboratory values may detemine a baseline for the number of pills to start with, but the dosage is adjusted regularly. The dosage of pancreatic enzymes is adjusted until an adequate growth pattern is established and the child is having no more than 1 to 2 stools per day. The child should be given an increased number of enzyme pills when a meal with high-fat content is consumed, not fewer.

15. A group of nursing students are reviewing the medications used to treat asthma. The students demonstrate understanding of the information when they identify which agent as appropriate for an acute episode of bronchospasm? A. Salmeterol B. Albuterol C. Ipratropium D. Cromolyn

Answer: B Rationale: Albuterol is a short-acting β2-adrenergic agonist that is used for treatment of acute bronchospasm. Salmeterol is a long-acting β2-adrenergic agonist used for long-term control or exercise-induced asthma. Ipratropium is an anticholinergic agent used as an adjunct to β2- adrenergic agonists for treatment of bronchospasm. Cromolyn is a mast cell stabilizer used prophylactically but not to relieve bronchospasm during an acute wheezing episode.

10. A group of nursing students are reviewing information about the variations in respiratory anatomy and physiology in children in comparison to adults. The students demonstrate understanding of the information when they identify which finding? A. Children's demand for oxygen is lower than that of adults. B. Children develop hypoxemia more rapidly than adults do. C. An increase in oxygen saturation leads to a much larger decrease in pO2. D. Children's bronchi are wider in diameter than those of an adult.

Answer: B Rationale: Children develop hypoxemia more rapidly than adults do because they have a significantly higher metabolic rate and faster resting respiratory rates than adults do, which leads to a higher demand for oxygen. A smaller decrease in oxygen saturation reflects a disproportionately much larger decrease in pO2. The bronchi in children are narrower than in adults, placing them at higher risk for lower airway obstruction.

The school nurse knows that dating is a milestone for adolescents. Which statement accurately describes a trend in teen dating? A) Most late adolescents spend more time in activities with mixed-sex groups, such as dances and parties, than they do dating as a couple. B) Most teens have been involved in at least one romantic relationship by middle adolescence. C) Teens that date frequently report slightly lower levels of self-esteem and decreased autonomy. D) Homosexual behavior as a teen usually indicates that the adolescent will maintain a homosexual orientation.

B

17. A nursing instructor is preparing a class on chronic lung disease. What information would the instructor include when describing this disorder? A. It is a result of cystic fibrosis. B. It is seen most commonly in premature infants. C. It typically affects females more often than males. D. It is characterized by bradypnea.

Answer: B Rationale: Chronic lung disease, formerly known as bronchopulmonary dysplasia, is often diagnosed in infants who have experienced respiratory distress syndrome, most commonly seen in premature infants. Male gender is a risk factor for development. Tachypnea and increased work of breathing are characteristic of chronic lung disease.

1. The nurse is examining an 8-year-old boy with tachycardia and tachypnea. The nurse anticipates which test as most helpful in determining the extent of the child's hypoxia? A. Pulmonary function test B. Pulse oximetry C. Peak expiratory flow D. Chest radiograph

Answer: B Rationale: Pulse oximetry is a useful tool for determining the extent of hypoxia. It can be used by the nurse for continuous or intermittent monitoring. Pulmonary function testing measures respiratory flow and lung volumes and is indicated for asthma, cystic fibrosis, and chronic lung disease. Peak expiratory flow testing is used to monitor the adequacy of asthma control. Chest radiographs can show hyperinflation, atelectasis, pneumonia, foreign bodies, pleural effusion, and abnormal heart or lung size.

5. The nurse is assessing a 5-year-old girl who is anxious, has a high fever, speaks in a whisper, and sits up with her neck thrust forward. Based on these findings, what would be least appropriate for the nurse to perform? A. Providing 100% oxygen B. Visualizing the throat C. Having the child sit forward D. Auscultating for lung sounds

Answer: B Rationale: The child is exhibiting signs and symptoms of epiglottitis, which can be lifethreatening. Under no circumstances should the nurse attempt to visualize the throat. Reflex laryngospasm may occur, precipitating immediate airway occlusion. Providing 100% oxygen in the least invasive manner that is most acceptable to the child is a sound intervention, as is allowing the child to assume a position of sitting forward with the neck extended. Auscultation would reveal breath sounds consistent with an obstructed airway.

14. A nurse is preparing a teaching plan for the family of a child with allergic rhinitis. When describing the immune reaction that occurs, the nurse would identify the role of which immunoglobulin? A. IgA B. IgE C. IgG D. IgM

Answer: B Rationale: The immunoglobulin involved in the immune response associated with allergic rhinitis is IgE. IgA, IgG, and IgM are not involved in this response.

16. The nurse is preparing to perform a physical examination of a child with asthma. Which technique would the nurse be least likely to perform? A. Inspection B. Palpation C. Percussion D. Auscultation

Answer: B Rationale: When examining the child with asthma, the nurse would inspect, auscultate, and percuss. Palpation would not be used.

3. A nurse is administering 100% oxygen to a child with a pneumothorax based on the understanding that this treatment is used primarily for which reason? A. Improve gas exchange B. Bypass the obstruction C. Hasten air reabsorption D. Prevent hypoxemia

Answer: C Rationale: Administration of 100% oxygen is used to treat pneumothorax primarily because it hastens the reabsorption of air. Generally this is used only for a few hours. Although the oxygen also improves gas exchange and prevents hypoxemia, these are not the reasons for its use in this situation. There is no obstruction with a pneumothorax.

19. When performing the physical examination of a child with cystic fibrosis, what would the nurse expect to assess? A. Dullness over the lung fields B. Increased diaphragmatic excursion C. Decreased tactile fremitus D. Hyperresonance over the liver

Answer: C Rationale: Examination of a child with cystic fibrosis typically reveals decreased tactile fremitus over areas of atelectasis, hyperresonance over the lung fields from air trapping, decreased diaphragmatic excursion, and dullness over the liver when enlarged.

13. A child with a pneumothorax has a chest tube attached to a water seal system. When assessing the child, the nurse notices that the chest tube has become disconnected from the drainage system. What would the nurse do first? A. Notify the physician. B. Apply an occlusive dressing. C. Clamp the chest tube. D. Perform a respiratory assessment.

Answer: C Rationale: If a chest tube becomes disconnected from the water seal drainage system, the nurse would first clamp the chest tube to prevent air from entering the child's chest cavity. Then the nurse would perform a respiratory assessment and notify the physician. An occlusive dressing would be applied first if the chest tube became dislodged from the child's chest.

4. Bacterial pneumonia is suspected in a 4-year-old boy with fever, headache, and chest pain. Which assessment finding would most likely indicate the need for this child to be hospitalized? A. Fever B. Oxygen saturation level of 96% C. Tachypnea with retractions D. Pale skin color

Answer: C Rationale: Pneumonia is usually a self-limiting disease. Children with bacterial pneumonia can be successfully managed at home if the work of breathing is not severe and oxygen saturation is within normal limits. Hospitalization would most likely be required for the child with tachypnea, significant retractions, poor oral intake, or lethargy for the administration of supplemental oxygen, intravenous hydration, and antibiotics. Fever, although common in children with pneumonia, would not necessitate hospitalization. An oxygen saturation level of 96% would be within normal limits. Pallor (pale skin color) occurs as a result of peripheral vasoconstriction in an effort to conserve oxygen for vital functions; this finding also would not necessitate hospitalization.

18. A nurse is teaching the parents of a child diagnosed with cystic fibrosis about medication therapy. Which would the nurse instruct the parents to administer orally? A. Recombinant human DNase B. Bronchodilators C. Anti-inflammatory agents D. Pancreatic enzymes

Answer: D Rationale: Pancreatic enzymes are administered orally to promote adequate digestion and absorption of nutrients. Recombinant human DNase, bronchodilators, and anti-inflammatory agents are typically administered by inhalation.

8. The nurse is examining a 5-year-old. Which sign or symptom is a reliable first indication of respiratory illness in children? A. Slow, irregular breathing B. A bluish tinge to the lips C. Increasing lethargy D. Rapid, shallow breathing

Answer: D Rationale: Tachypnea, or increased respiratory rate, is often the first sign of respiratory illness in infants and children. Slow, irregular breathing and increasing listlessness are signs that the child's condition is worsening. Cyanosis (a bluish tinge to the lips) or the degree of cyanosis present is not always an accurate indication of the severity of respiratory involvement.

11. The nurse is providing care to several children who have been brought to the clinic by the parents reporting cold-like symptoms. The nurse would most likely suspect sinusitis in which child? A. A 2-year-old with thin watery nasal discharge B. A 3-year-old with sneezing and coughing C. A 5-year-old with nasal congestion and sore throat D. A 7-year-old with halitosis and thick, yellow nasal discharge

Answer: D Rationale: The frontal sinuses, those most commonly associated with sinus infection, develop by age 6 to 8 years. Therefore, the 7-year-old would most likely experience sinusitis. In addition, this child also exhibits halitosis and a thick, yellow nasal discharge, other findings associated with sinusitis. Thin watery discharge in a 2-year-old is more likely to indicate allergic rhinitis. A 3-year-old with coughing and sneezing or a 5-year-old with nasal congestion and sore throat suggests the common cold.

What would the nurse include when teaching an adolescent about tinea pedis? A) "Keep your feet moist and open to the air as much as possible." B) "Dry the area between your toes really well." C) "Wear nylon or synthetic socks every day." D) "Go barefoot when you are in the locker room at school."

B) "Dry the area between your toes really well." Feedback: Keeping the feet clean and dry is key for the child with tinea pedis. This includes rinsing the feet with water or a water/vinegar mixture and drying them well, especially between the toes. The adolescent should wear cotton socks and shoes that allow the feet to breathe. Going barefoot at home is allowed, but the adolescent should wear flip-flops around swimming pools and locker rooms.

The nurse is providing parental teaching about home care for an 8-year-old boy with widespread sunburn on his back and shoulders. Which response indicates a need for further teaching? A) "Cool compresses may help cool the burn." B) "He should manually peel off any flaking skin." C) "Nonsteroidal anti-inflammatory drugs like ibuprofen are helpful." D) "He should avoid hot showers or baths for a couple of days."

B) "He should manually peel off any flaking skin." Feedback: If skin flaking occurs, the child should be discouraged from manually "peeling" the flaked skin as it can cause further injury. Using cool compresses, taking nonsteroidal anti-inflammatory drugs, and avoiding hot showers or baths are appropriate measures.

The nurse is teaching the mother of a toddler about burn prevention. Which response by the mother indicates a need for further teaching? A) "We will leave fireworks displays to the professionals." B) "I will set our water heater at 130 degrees." C) "All sleepwear should be flame retardant." D) "The handles of pots on the stove should face inward."

B) "I will set our water heater at 130 degrees." Feedback: If the temperature of the water heater is set at 130 degrees, a child can be burned significantly in only 30 seconds. The recommended maximal home hot water heater temperature is 120 degrees. Leaving fireworks to the professionals, using flame-retardant sleepwear, and turning the handles of pots on the stove inward are correct.

A teenage girl with psoriasis tells the nurse that she is so embarrassed by the plaque on her skin that she doesn't want to go to school. What is the best response by the nurse? A) "Have you been applying your medication and emollients to your skin as directed by your physician?" B) "It must be really difficult for you. Tell me how you are taking care of your skin on a daily basis." C) "Sunlight really helps the plaque areas heal. Maybe going to a tanning bed routinely will help." D) "You can't miss school because of your skin. Can you wear clothes that will cover the areas?"

B) "It must be really difficult for you. Tell me how you are taking care of your skin on a daily basis." Feedback: "It must be really difficult for you. Tell me how you are taking care of your skin on a daily basis" shows empathy and allows the nurse to determine how the girl is taking care of the psoriasis and if any suggestions to the treatment plan can be helpful. Questioning the client if she is doing what the physician has prescribed may make her defensive and does not show empathy. Suggesting tanning can cause too much exposure to unwanted UV rays. Telling the girl that she can't miss school and to cover the areas does not elicit open discussion and does not promote self-esteem.

20. The parents of a 10-year-old girl with a refractive error ask the nurse about the possibility of laser surgery to correct the vision. Which statement by the nurse would be most appropriate? A) "As she gets older, her vision will begin to correct itself." B) "Laser surgery typically is not done until she's 18 years old." C) "She looks so cute in her glasses; why put her through surgery?" D) "She can use contact lenses soon, so surgery isn't necessary."

B) "Laser surgery typically is not done until she's 18 years old." Because of the continuing refractive development in the child's vision through adolescence, laser surgery for vision correction is not recommended by the American Academy of Ophthalmology until 18 years of age. The refractive error will continue to change as the child's vision continues to develop, making the refraction unstable. Thus, corrective lens prescription may change but the refraction error will not correct itself. Glasses still carry a stigma and the child may be teased or bullied. The statement about the child looking cute in her glasses ignores the parents' question and concerns and questions the parents' desire for information. The use of contact lenses does not negate the possibility of surgery. However, laser surgery would have to wait until the child is 18 years of age.

27. A child is diagnosed with bacterial conjunctivitis and is prescribed topical antibiotic therapy. The child's mother asks when he can return to school. Which response by the nurse would be most appropriate? A) "You need to wait until you finish the entire prescription of antibiotic." B) "Once the drainage is gone, he can go back to school." C) "You can send him to school this afternoon after his first dose of antibiotic." D) "He needs to be symptom-free for at least 72 hours."

B) "Once the drainage is gone, he can go back to school." For the child with bacterial conjunctivitis, the child may safely return to school or day care when the mucopurulent drainage is no longer present, usually after 24 to 48 hours of treatment with the topical antibiotic. There is no need to wait until the prescription is finished. The antibiotic is being given topically, not systemically. One dose of antibiotic is not sufficient to eradicate the infection. Typically 24 to 48 hours of treatment is needed to stop the drainage, which, when no longer present, indicates that the child can return to school.

14. After teaching a group of new parents about their newborns' eyes and vision, which statement by the group indicates effective teaching? A) "Our newborn can see at distances of about 1 to 2 feet." B) "We won't know the baby's eye color until he's at least 6 months old." C) "A baby can easily distinguish colors, but they must be bright colors." D) "A newborn can focus with both eyes at the same time shortly after birth."

B) "We won't know the baby's eye color until he's at least 6 months old." The eye color of an infant is determined by 6 to 12 months of age. A newborn sees best at distances of about 8 to 10 inches. The optic nerve is not completely myelinated, so color discrimination is incomplete. The rectus muscles are uncoordinated at birth and mature over time, so binocular vision may be achieved by 4 months of age

The nurse is caring for an infant with candidal diaper rash. Which topical agent would the nurse expect the physician to order? A) Corticosteroids B) Antifungals C) Antibiotics D) Retinoids

B) Antifungals Feedback: Candidal diaper rash would require a fungicide. The nurse would expect to administer topical antifungals as ordered. Corticosteroids are not typically recommended for young infants and are used for atopic dermatitis and certain types of contact dermatitis. Antibiotics would be ineffective against fungal infections. Retinoids are indicated for moderate to severe acne.

A nurse is preparing a presentation for a local parent group about burn prevention and care in children. What would the nurse be least likely to include in the presentation when describing how to care for a superficial burn? A) Using cool water over the burned area until the pain lessens B) Applying ice directly to the burned skin area C) Covering the burn with a clean, nonadhesive bandage D) Giving the child acetaminophen for pain relief

B) Applying ice directly to the burned skin area With a superficial burn, ice should not be applied to the skin. Using cool water over the burn area; covering with a clean, nonadhesive bandage; and using acetaminophen for pain relief are appropriate to include in the presentation.

A nurse is inspecting the skin of a child with atopic dermatitis. What would the nurse expect to observe? A) Erythematous papulovesicular rash B) Dry, red, scaly rash with lichenification C) Pustular vesicles with honey-colored exudates D) Hypopigmented oval scaly lesions

B) Dry, red, scaly rash with lichenification Feedback: Atopic dermatitis or eczema is characterized by a dry, red, scaly rash with lichenification and hypertrophy. An erythematous papulovesicular rash is associated with contact dermatitis. Pustules and vesicles with honey-colored exudates suggest nonbullous impetigo. Hypopigmented oval scaly lesions are associated with tinea versicolor.

An instructor is developing a plan for a class of nursing students on various skin disorders. When describing urticaria, what would the instructor include? A) It is a type IV hypersensitivity reaction. B) Histamine release leads to vasodilation. C) Wheals appear first followed by erythema. D) The nonpruritic rash blanches with pressure.

B) Histamine release leads to vasodilation. Feedback: Urticaria is a type I hypersensitivity reaction caused by an immunologically mediated antigen-antibody response of histamine release from the mast cells. Vasodilation and increased vascular permeability result, leading to erythema and then wheals. The rash is pruritic and blanches with pressure.

24. A mother brings her child to the health care clinic because she thinks that the child has conjunctivitis. Which assessment finding would lead the nurse to suspect bacterial conjunctivitis? Select all answers that apply. A) Itching of the eyes B) Inflamed conjunctiva C) Stringy discharge D) Photophobia E) Mild pain F) Tearing

B) Inflamed conjunctiva E) Mild pain terial conjunctivitis is manifested by inflamed conjunctiva, a purulent or mucoid discharge, mild pain, and occasional eyelid edema. Itching and a stringy discharge suggest allergic conjunctivitis. Photophobia and tearing suggest viral conjunctivitis

10. The nurse is examining a 3-year-old boy with acute otitis media who has a mild earache and a temperature of 38.5º C (101.3) . Which of the following actions will be taken? A) Obtain a culture of the middle ear fluid. B) Instruct the parents to watch for worsening symptoms. C) Administer antibiotics. D) Administer antivirals.

B) Instruct the parents to watch for worsening symptoms. In this case, the child will be continually observed. If the symptoms persist or become worse, antibiotics will be prescribed. This clinical practice guideline was developed by the American Academy of Pediatrics and the American Academy of Family Physicians in order to avoid overusing antibiotics or obtaining a middle ear fluid culture with every occurrence of acute otitis media. Administering antiviral agents would not be appropriate for this child. (Refer to Table 39.3 for treatment recommendations for acute otitis media.)

The nurse is caring for a 15-year-old boy who has sustained burn injuries. The nurse observes the burn developing a purplish color with discharge and a foul odor. The nurse suspects which infection? A) Burn wound cellulitis B) Invasive burn cellulitis C) Burn impetigo D) Staphylococcal scalded skin syndrome

B) Invasive burn cellulitis Feedback: Invasive burn cellulitis results in the burn developing a dark brown, black, or purplish color with a discharge and foul odor. In burn wound cellulitis, the area around the burn becomes increasingly red, swollen, and painful early in the course of burn management. Burn impetigo is characterized by multifocal, small, superficial abscesses. Staphylococcal scalded skin syndrome is not a burn infection; however, it is managed similarly to burns.

18. A nurse is reviewing the medical record of a child with hearing loss and notes that the child's hearing loss is in the range 40 to 60 decibels (dB). The nurse interprets this as indicating which of the following? A) Mild loss B) Moderate loss C) Severe loss D) Profound loss

B) Moderate loss A hearing loss of 40 to 60 decibels (dB) indicates a moderate loss; 20 to 40 dB indicates a mild loss; 60 to 80 dB indicates a severe loss; and greater than 80 dB indicates a profound loss.

15. Assessment of a child leads the nurse to suspect viral conjunctivitis based on which of the following? A) Mild pain B) Photophobia C) Itching D) Watery discharge

B) Photophobia Viral conjunctivitis is characterized by lymphadenopathy, photophobia, and tearing. Mild pain is associated with bacterial conjunctivitis. Itching and watery discharge are associated with allergic conjunctivitis.

The mother of a 15-year-old girl has contacted the clinic to report that her daughter has burned the back of her hand with a curling iron. The child's mother reports the burn is mild but states her daughter is complaining of pain. After consulting with the physician, what instructions can the nurse anticipate will be recommended? Select all that apply. A) Apply a thin film of protective cocoa butter. B) Run cool water over the injured area. C) Apply ice for 15 to 20 minutes each hour until the pain subsides. D) Take acetaminophen using the manufacturer's guidelines. E) Apply a thin layer of petroleum jelly to the burned area.

B) Run cool water over the injured area. D) Take acetaminophen using the manufacturer's guidelines. Feedback: Mild burns may be cared for at home. Cool water may be run over the injured tissue. Acetaminophen or ibuprofen may be administered for pain. Ointments and creams including butter, margarine, cocoa butter, and petroleum jelly should not be applied.

2. The nurse is caring for a 3-month-old with nasolacrimal duct obstruction. Which intervention would be most appropriate for the nurse to implement? A) Being careful to prevent spread of infection B) Teaching the parents how to gently massage the duct C) Applying hot, moist compresses to the affected eye D) Referring the child to an ophthalmologist

B) Teaching the parents how to gently massage the duct Massaging the nasolacrimal duct can cause it to open and drain. Teaching the parents how to do this would be part of the nurse's plan of care. Nasolacrimal duct obstruction is not infectious. Applying hot, moist compresses to the eye is an intervention for conjunctivitis. Nasolacrimal duct obstruction is often self-resolving, so there would be no need for a specialist's care.

A nurse is preparing a class for parents of infants about managing diaper dermatitis. What advice would the nurse include in the presentation? Select all that apply. A) Applying topical nystatin to the diaper area B) Using a blow dryer on warm to dry the diaper area C) Refraining from using rubber pants over diapers D) Using scented diaper wipes to clean the area E) Washing the diaper area with an antibacterial soap

B) Using a blow dryer on warm to dry the diaper area C) Refraining from using rubber pants over diapers Feedback: For diaper dermatitis, topical products such as ointments or creams containing vitamins A, D, and E; zinc oxide; or petrolatum help to provide a barrier. Nystatin is an antifungal agent used for diaper candidiasis. Using a blow dryer on warm to dry the area, avoiding the use of rubber pants, and using unscented diaper wipes or ones free of preservatives are appropriate. The area should be washed with a soft cloth, without harsh soaps.

8. The nurse is caring for a newborn and knows that his vision, unlike his hearing, is not fully developed. Which aspect of the child's vision would the nurse expect to be similar to his father's vision? A) Adequate color detection B) Visual acuity of 20/100 C) Nearsightedness D) Monocular vision

B) Visual acuity of 20/100 If the child's father has lost visual acuity, he and his new son could possibly have the same 20/100 vision. Poor color detection, nearsightedness, and monocular vision are characteristic of newborns and are the result of their lack of development.

A nurse is assessing the skin of a child with cellulitis. What would the nurse expect to find? A) Red, raised hair follicles B) Warmth at skin disruption site C) Papules progressing to vesicles D) Honey-colored exudate

B) Warmth at skin disruption site Feedback: Cellulitis is manifested by erythema, pain, edema, and warmth at the site of skin disruption. Red raised hair follicles would indicate folliculitis. Papules progressing to vesicles and a honey-colored exudate would suggest nonbullous impetigo.

The nurse is conducting a physical examination of a 9-month-old baby with a flat, discolored area on the skin. The nurse documents this as a: A) papule. B) macule. C) vesicle. D) scale.

B) macule. Feedback: A macule is a flat, discolored area on the skin. A papule is a small, raised bump on the skin. A vesicle is a fluid-filled bump on the skin. Scaling is flaking of the skin.

The nurse is teaching good sleep habits for toddlers to the mother of a 3-year-old boy. Which response indicates the mother understands sleep requirements for her son? A)"I'll put him to bed at 7 p.m., except Friday and Saturday." B)"He needs 12 hours of sleep per day including his nap." C)"I need to put the side down on the crib so he can get out." D)"His father can give him a horseback ride into his bed."

B)"He needs 12 hours of sleep per day including his nap." The mother understands her child needs 12 hours of sleep and one nap per day. Routines, such as the same bedtime every night, promote good sleep. However, a horseback ride to bed may cause problems because it may not provide a calming transition from play to sleep. A bath and reading a book would be better. If the child can climb out of a crib, he needs to be in a youth bed or regular bed to avoid injury.

A teenage patient tells the nurse that she is being abused by her boyfriend but she doesn't want her parents to know because they won't let her see him any longer. What is the best response by the nurse? A)"It's my responsibility to tell your parents if you are in danger." B)"I understand your fear, but I am obligated to be sure your parents know you are in danger. Would you like for us to talk to them together?" C)"I won't tell them this time, but I must inform you that legally I must inform your parents if abuse is occurring. Next time it happens I will have to tell them." D)"You need to tell them because the abuse isn't going to get any better. It will only escalate no matter what your boyfriend says."

B)"I understand your fear, but I am obligated to be sure your parents know you are in danger. Would you like for us to talk to them together?" The most empathetic and informative response is recognizing the teen's fear. This response also establishes trust by letting the patient know what the nurse's responsibility is while also offering support by talking to the parents with the teen. Responding that the nurse won't inform the parents this time is incorrect because the nurse is legally bound to notify the parents if the child is in danger, as in the case of abuse.

The parents of a 2-year-old girl are frustrated by the frequent confrontations they have with their child. Which is the best anticipatory guidance the nurse can offer to prevent confrontations? A)"Respond in a calm but firm manner." B)"You need to adhere to various routines." C)"Put her in time-out when she misbehaves." D)"It's important to toddler-proof your home."

B)"You need to adhere to various routines." Making expectations known through everyday routines helps to avoid confrontations. This helps the child know what to expect and how to behave. It is the best guidance to give these parents. Calm response and time-out are effective ways to discipline, but do not help to prevent confrontations. Toddler-proofing the house doesn't eliminate all the opportunities for confrontation.

The nurse is discussing vaccination for Haemophilus influenzae type B (Hib) with the mother of a 6-month-old child. Which comment provides the most compelling reason to get the vaccination? A)"These bacteria live in every human." B)"Young children are especially susceptible to these bacteria." C)"You have a choice of two excellent vaccines." D)"Your child needs this final dose for protection."

B)"Young children are especially susceptible to these bacteria." The most compelling reason for vaccination is that the highest rate of illness from influenza is in children. The fact that Hib is an opportunistic bacterium that lives in humans and only causes disease when resistance is lowered may be difficult for the parent to understand. A choice of two vaccines conveys no benefits to the mother. Need for the final dose is vague.

For which children would the nurse conduct an immediate comprehensive health history? A)A child who is brought to the emergency room with labored breathing B)A child who is a new client in a pediatric office C)A child who is a routine client and presents with signs of a sinus infection D)A child whose condition is improving

B)A child who is a new client in a pediatric office The purpose of the examination will determine how comprehensive the history must be. A comprehensive history would be performed for a new child in a pediatric office or a child who is admitted to the hospital. Also, if the physician or nurse practitioner rarely sees the child or if the child is critically ill, a complete and detailed history is in order, no matter what the setting. The child who has received routine health care and presents with a mild illness may need only a problem-focused history. In critical situations, some of the history taking must be delayed until after the child's condition is stabilized.

The nurse is conducting a health history for a 9-year-old child with stomach pains. What is a recommended guideline when approaching the child for information? A)Wear a white examination coat when conducting the interview. B)Allow the child to control the pace and order of the health history. C)Use quick deliberate gestures to get your point across. D)Do not make physical contact with the child during the interview.

B)Allow the child to control the pace and order of the health history. The nurse should elicit the child's cooperation by allowing him or her control over the pace and order of the health history, or anything else that the child can control while still allowing the nurse to obtain the information needed. A white examination coat or all-white uniform may be frightening to children, who may associate the uniform with painful experiences or find it too unfamiliar. The nurse should use slow deliberate gestures rather than very quick or grand ones, which may be frightening to shy children. The nurse should make physical contact with the child in a nonthreatening way at first by briefly cuddling newborns before returning them to caregivers, laying a hand on the head or arm of toddlers and preschoolers, and warmly shaking the hand of older children and teens to convey a gentle demeanor.

When instructing the parents of a toddler about appropriate nutrition, what would the nurse recommend? A)About 12 to 16 ounces of fruit juice per day B)Approximately 16 to 24 ounces of milk per day C)Fat intake of 30% to 40% of total calories D)An average of 10 to 12 grams of fiber per day

B)Approximately 16 to 24 ounces of milk per day Milk intake should be limited to 16 to 24 ounces per day, with fruit juice limited to 4 to 6 ounces per day. A toddler's total fat intake should be 20% to 30% of total calories. The daily recommended fiber intake is 19 grams.

The mother of a 5-year-old child with eczema is getting a check-up for her child before school starts. What will the nurse do during the visit? A)Change the bandage on a cut on the child's hand B)Assess the compliance with treatment regimens C)Discuss systemic corticosteroid therapy D)Assess the child's fluid volume

B)Assess the compliance with treatment regimens Maintaining proper therapy for eczema can be exhausting both physically and mentally. Therefore, it is essential that the nurse assess compliance and support the parents' ability to cope if necessary. Changing a bandage is not part of a health maintenance visit. Hydration is important for a child with eczema; however, fluid volume is not the focus at this visit. Systemic corticosteroid therapy is very rarely used and the success of the current therapy needs to be assessed first.

The nurse is assessing the temperature of a diaphoretic toddler who is crying and being uncooperative. What would be the best method to assess temperature in this child? A)Oral thermometer B)Axillary method C)Temporal scanning D)Rectal route

B)Axillary method The axillary method may be used for children who are uncooperative, neurologically impaired, or immunosuppressed or have injuries or surgery to the oral cavity. Since the child is crying and uncooperative, the oral method would not be a good choice. The accuracy of the temporal method may be affected by excessive sweating. The rectal route is invasive, not well accepted by children or parents, and probably unnecessary with the modern alternative methods now available.

The nurse is preparing to take a tympanic temperature reading of a 4-year-old. In order to get an accurate reading, what does the nurse need to do? A)Pull the earlobe back and down B)Direct the infrared sensor at the tympanic membrane C)Pull the earlobe down and forward D)Remove any visible cerumen from inside the ear canal

B)Direct the infrared sensor at the tympanic membrane The accuracy of tympanic temperature reading is dependent upon appropriate technique. The nurse needs to be sure to direct the infrared sensor at the tympanic membrane. Since the child is older than age 3, the earlobe does not need to be pulled back and down. The nurse would not remove earwax from inside the ear canal.

The nurse is teaching the parents of a 2-year-old toddler methods of dealing with their child's 'negativism.' Based on Erickson's theory of development, what would be an appropriate intervention for this child? A)Discourage solitary play; encourage playing with other children. B)Encourage the child to pick out his own clothes. C)Use 'time-outs' whenever the child says 'no' inappropriately. D)Encourage the child to take turns when playing games.

B)Encourage the child to pick out his own clothes. Erikson defines the toddler period as a time of autonomy versus shame and doubt. It is a time of exerting independence. Allowing the child to choose his own clothes helps him to assert his independence. Negativism and always saying "no" is a normal part of healthy development and is occurring as a result of the toddler's attempt to assert his or her independence. It should not be punished with 'time-outs.' The toddler should be encouraged to play alone and with other children. Toddlers cannot take turns in games until age 3.

The nurse is helping parents prepare a healthy meal plan for their toddler. Which guidelines for promoting nutrition should be followed when planning meals? Select all that apply. A)The child younger than 2 years of age should have his or her fat intake restricted. B)Extending breastfeeding into toddlerhood is believed to be beneficial to the child. C)Weaning from the bottle should occur by 6 to 12 months of age. D)Adequate calcium intake and appropriate exercise lay the foundation for proper bone mineralization. E)The toddler requires an average intake of 500 mg calcium per day. F)Toddlers tend to have the highest daily iron intake of any age group.

B)Extending breastfeeding into toddlerhood is believed to be beneficial to the child. D)Adequate calcium intake and appropriate exercise lay the foundation for proper bone mineralization. E)The toddler requires an average intake of 500 mg calcium per day. Extending breastfeeding into toddlerhood is believed to be beneficial to the child as it is known to help prevent obesity. Adequate calcium intake and appropriate exercise lay the foundation for proper bone mineralization. The toddler requires an average intake of 500 mg calcium per day. The child younger than 2 years of age should not have his or her fat intake restricted, but this does not mean that unhealthy foods such as sweets should be eaten liberally. Weaning from the bottle should occur by 12 to 15 months of age. Prolonged bottle-feeding is associated with the development of dental caries. It is important for toddlers to consume adequate amounts of iron since they tend to have the lowest daily iron intake of any age group.

The nurse is preparing a presentation to a local parent group about pediatric health supervision. Which would the nurse emphasize as the focus? A)Injury prevention B)Wellness C)Health maintenance D)Developmental surveillance

B)Wellness The focus of pediatric health supervision is wellness. Injury and disease prevention, health maintenance and promotion, and developmental surveillance are all critical components of wellness.

The nurse is caring for an infant who had hyperbilirubinemia requiring exchange transfusion. Based on this information, this infant is at risk for what type of disorder? A)Vision loss B)Hearing loss C)Hypertension D)Hyperlipidemia

B)Hearing loss There are many conditions that place an infant at risk for hearing loss, including an exchange transfusion with hyperbilirubinemia. A risk factor for vision loss is history of ocular structural abnormalities. Risk factors for systemic hypertension include preterm birth, very low birthweight, renal disease, organ transplant, congenital heart disease, or other illnesses associated with hypertension. A risk factor for hyperlipidemia is family history.

A 2-week-old child responds to a bell during an initial health supervision examination. The child's records do not show that a newborn hearing screening was done. Which is the best action for the nurse to take? A)Do nothing because responding to the bell proves he does not have a hearing deficit. B)Immediately schedule the infant for a newborn hearing screening. C)Ask the mother to observe for signs that the infant is not hearing well. D)Screen again with the bell at the 2-month-old health supervision visit.

B)Immediately schedule the infant for a newborn hearing screening. Guidelines for infant hearing screening recommend universal screening with an auditory brain stem response (ABR) or evoked otoacoustic emissions (EOAE) test by 1 month of age. All the other answers rely on behavioral observation. Studies have shown that behavioral observations are not a reliable method of screening for hearing loss.

The nurse is teaching the student nurse how to perform a physical assessment based on the child's developmental stage. Which statements accurately describes a recommended guideline for setting the tone of the examination for a school-age child? A)Keep up a running dialogue with the caregiver, explaining each step as you do it. B)Include the child in all parts of the examination; speak to the caregiver before and after the examination. C)Speak to the child using mature language and appeal to his or her desire for self-care. D)Address the child by name; speak to the caregiver and do the most invasive parts last.

B)Include the child in all parts of the examination; speak to the caregiver before and after the examination. For a school-age child, the nurse should include the child in all parts of the examination, and speak to the caregiver before and after the examination. For a newborn the nurse should keep up a running dialogue with the caregiver, explaining each step as it is done. The nurse should speak to the early teen using mature language and appeal to his or her desire for self-care. For an infant, the nurse should address the child by name, and speak to the caregiver and do the most invasive parts last.

The nurse is explaining the difference between active and passive immunity to the student nurse. Which statement accurately describes a characteristic of the process of immunity? A)Active immunity is produced when the immunoglobulins of one person are transferred to another. B)Passive immunity can be obtained by injection of exogenous immunoglobulins. C)Active immunity can be transferred from mothers to infants via colostrum or the placenta. D)Passive immunity is acquired when a person's own immune system generates the immune response.

B)Passive immunity can be obtained by injection of exogenous immunoglobulins. Passive immunity can be obtained by injection of exogenous immunoglobulins. Passive immunity is produced when the immunoglobulins of one person are transferred to another. Passive immunity can also be transferred from mothers to infants via colostrum or the placenta. Active immunity is acquired when a person's own immune system generates the immune response.

The nurse is performing a risk assessment of a 5-year-old and determines the child has a risk factor for cystic fibrosis. What type of screening would the nurse perform to confirm or rule out this disease? A)Universal screening B)Selective screening C)Hyperlipidemia screening D)Developmental screening

B)Selective screening Selective screening is done when a risk assessment indicates the child has one or more risk factors for the disorder. In universal screening, an entire population is screened regardless of the child's individual risk. Selectively screening children at high risk for hyperlipidemia can reduce their lifelong risk of coronary artery disease; it does not screen for cystic fibrosis. Developmental screening is performed to detect developmental delays.

The nurse is assessing a 2-year-old boy who has missed some developmental milestones. Which finding will point to the cause of motor skill delays? A)The mother is suffering from depression. B)The child is homeless and has no toys. C)The mother describes an inadequate diet. D)The child is unperturbed by a loud noise.

B)The child is homeless and has no toys. Children develop through play, so a child without any toys may have trouble developing the motor skills appropriate to his age. Maternal depression is a risk factor for poor cognitive development. Inadequate diet will cause growth deficiencies. A child who does not respond to a loud noise probably has hearing loss, which will lead to a language deficit.

The nurse is observing a 24-month-old boy in a day care center. Which finding suggests delayed motor development? A)The child has trouble undressing himself. B)The child is unable to push a toy lawnmower. C)The child is unable to unscrew a jar lid. D)The child falls when he bends over.

B)The child is unable to push a toy lawnmower. Children with normal motor development are able to push toys with wheels at 24 months of age. He won't be ready to undress himself, unscrew a jar lid, or bend over without falling until about 36 months of age.

The nurse strives to provide culturally competent care for children in a health clinic that follows the principles of health supervision. Which nursing action reflects this type of care? A)The nurse treats all children the same regardless of their culture. B)The nurse negotiates a care plan with the child and family. C)The nurse researches the child's culture and provides care based on the findings. D)The nurse provides future-based care for culturally diverse children.

B)The nurse negotiates a care plan with the child and family. Optimal wellness for the child requires the nurse and the family to negotiate a mutually acceptable plan of care. The nurse must consider the culture of children because if the goals of the health care plan are not consistent with the health belief system of the family, the plan has little chance for success. Researching the culture is helpful, but the nurse should not assume all children follow cultural directives and base the care plan solely on the research. Most health promotion and disease prevention strategies in the United States have a future-based orientation; however, significant numbers of children belong to cultures with a present-based orientation. For these children, health promotion activities need shorter-term goals and outcomes to be useful.

After teaching the mother about follow-up immunizations for her daughter, who received the varicella vaccine at age 14 months, the nurse determines that the teaching was successful when the mother states that a follow-up dose should be given at which time? A)When the child is 20 to 36 months of age B)When the child is 4 to 6 years of age C)When the child is 11 to 12 years of age D)When the child is 13 to 15 years of age

B)When the child is 4 to 6 years of age A second dose of varicella vaccine should be given when the child is 4 to 6 years of age. Hepatitis A vaccine should be given to infants at age 12 months, with a repeat dose given in 6 to 12 months. The human papillomavirus (HPV) vaccine should be given to children beginning at age 11 to 12 years, with catch-up doses to begin at 13 to 14 years of age.

The nurse assesses the spirituality of an adolescent. What are normal moral and spiritual milestones in this age group? Select all that apply. A) Adolescents will base their actions on the avoidance of punishment and the attainment of pleasure. B) Adolescents develop their own set of morals and values and question the status quo. C) Adolescents undergo the process of developing their own set of morals at different rates. D) Adolescents are more interested in the spiritualism of their religion than in the actual practices of their religion. E) Adolescents can understand the concepts of right and wrong and are developing a conscience. F) Adolescents are able to understand and incorporate into their behavior the concept of the "golden rule."

B, C, D

The nurse is assessing a child with acute poststreptococcal glomerulonephritis. What would the nurse expect to assess? Select all that apply. A)Irritability B)Abdominal pain C)Hypertension D)Crackles E)Polyphagia

B, C, D Assessment findings associated with acute poststreptococcal glomerulonephritis include fatigue, lethargy, abdominal pain, hypertension, crackles, and anorexia.

The nurse is assessing a 5-year-old child's genitourinary system. Which findings would the nurse document as normal? Select all that apply. A)Labial fusion B)Round abdomen C)Positive bowel sounds D)Dullness over the spleen E)Undescended testicles

B, C, D Normal findings include a round abdomen, positive bowel sounds, dullness over the spleen, and descended testicles. Labial fusion, a distended abdomen, and undescended testicles are abnormal findings.

The nurse is caring for a child that just returned from a coronary arteriogram in which the catheter was placed through the left femoral artery. Which nursing actions demonstrate knowledge of the procedure? Select all that apply. A)The nurse allows the patient up to the bathroom only. B)The nurse assesses the dorsalis pedis pulse in the left foot. C)The nurse assesses the puncture site frequently. D)The nurse tells the parents that the physician will discuss the results of the procedure with them. E)The nurse assesses the patient's vital signs every 8 hours.

B, C, D The nurse must assess the pulse distal to the puncture site to determine that circulation remains adequate to the extremity. Assessing the puncture site ensures early recognition of bleeding from the site. The physician will be able to inform the parents regarding the results of the procedure after completion. The child should be kept on bedrest for a specified period of time, so they cannot be up to the bathroom. Vital signs will need to be taken more frequently than every 8 hours for early detection of complications.

The nurse is caring for a 6-month-old with a cleft lip and palate. The mother of the child demonstrates understanding of the disorder with which statements? Select all that apply. A)"My smoking during pregnancy didn't have anything to do with this disorder. Smoking primarily causes low birth weight." B)"I know my baby takes a lot longer to feed than most children this age." C)"It really worries me that my baby may have some other disorders that haven't been detected yet." D)"I wonder if my baby will develop speech problems when language development begins?" E)"Thankfully there are doctors that specialize in correcting this type of disorder."

B, C, D, E Feeding and speech are especially difficult for the child with cleft lip and palate until the defect is repaired. Cleft lip and palate occurs frequently in association with other anomalies and has been identified in more than 350 syndromes. Plastic surgeons or craniofacial specialists, oral surgeons, dentists or orthodontists, and prosthodontists are some of the physicians that specialize in repair of this disorder. The mother is incorrect in stating that smoking is not associated with cleft lip or palate. Maternal smoking during pregnancy is a major risk factor for the disorder.

After teaching the parents of a child diagnosed with celiac disease about nutrition, the nurse determines that the teaching was effective when the parents identify which foods as appropriate for their child? Select all answers that apply. A)Wheat germ B)Peanut butter C)Carbonated drinks D)Shellfish E)Jelly F)Flavored yogurt

B, C, D, E Foods allowed in a gluten-free diet include peanut butter, carbonated drinks, shellfish, and jelly. Wheat germ and flavored yogurt should be avoided.

The community health nurse has just completed a presentation to a group of parents regarding drowning prevention. Which statements by the parents indicate understanding of the teaching? Select all that apply. A) "I am so glad our 6-year-old child had swim lessons. We really can't afford a fence around our pool." B) "Since we have a 16-year-old I am really concerned about supervision when our child is swimming in the ocean." C) "We always make sure our babysitter keeps her CPR training up to date." D) "It is scary to think that we have a pool and drowning is the second leading cause of accidental death in children." E) "We make sure to keep our bathroom door closed when our 10-month-old is walking around the house since the door handle is too high to reach."

B, C, D, E In children older than 15 years of age, most drownings occur in natural water settings, such as oceans or lakes. Most incidents of drowning are accidental and result from inadequately supervising children of any age. It is important for any caregivers of children to be current on CPR in case of any accident. Children younger than 1 year old most often drown in bathtubs, buckets, or toilets, so keeping the bathroom door closed helps decrease the risk of drowning.

The nurse is performing a gastrointestinal assessment on a 7-year-old boy. The parents are assisting with the history. Which assessment findings are indicative of constipation? Select all that apply. A)"Our child only has 3 to 4 bowel movements per week." B)"Our child complains of pain because his bowel movements are so hard." C)"Our child tells us that his belly hurts a lot of the time." D)"I can tell he holds his bowel movement much of the time because of the way he stands." E)"I find smears of stool in his underwear almost every day."

B, C, D, E Pain, stool withholding behavior (retentive posturing), and encopresis (soiling of fecal contents into the underwear beyond the age of expected toilet training) are all signs of chronic functional constipation. Less than 3 bowel movements is considered constipation.

A nurse is providing instructions to the parents of a 3-month-old with developmental dysplasia of the hip who is being treated with a Pavlik harness. Which statements by the parents demonstrate understanding of the instructions? Select all that apply. A)"We need to adjust the straps so that they are snug but not too tight." B)"We should change her diaper without taking her out of the harness." C)"We need to check the area behind her knees for redness and irritation." D)"We need to send the harness to the dry cleaners to have it cleaned." E)"We need to call the doctor if she is not able to actively kick her legs."

B, C, E Instructions related to use of a Pavlik harness include changing the child's diaper while in the harness; checking the areas behind the knees and diaper area for redness, irritation, or breakdown; and calling the doctor if the child is unable to actively kick her legs. The straps are not to be adjusted without checking with the physician or nurse practitioner first. The harness can be washed with mild detergent by hand and air dried. A hair dryer can be used to dry the harness but only if the air fluffing setting is used.

The nurse is reviewing the medical record of a child with infective endocarditis. What would the nurse expect to find? Select all that apply. A)White blood cell count revealing leukopenia B)Microscopic hematuria with urinalysis C)Electrocardiogram with prolonged PR interval D)Lungs clear on auscultation E)Petechiae on palpebral conjunctiva

B, C, E With infective endocarditis, leukocytosis, microscopic hematuria, prolonged PR interval, adventitious lung sounds, and petechiae on the palpebral conjunctiva are noted.

A nurse is instituting neutropenic precautions for a child. What information would the nurse most likely include? Select all that apply. A)Placing the child in a semiprivate room B)Avoiding rectal exams, suppositories, and enemas C)Placing a mask on the child when outside the room D)Encouraging an intake of raw fruits and vegetables E)Discouraging fresh flowers in the child's room

B, C, E Generally, neutropenic precautions include placing the child in a private room; avoiding rectal suppositories, enemas, and examinations; placing a mask on the child when outside the room; avoiding the intake of raw fruits and vegetables; and not permitting fresh flowers or live plants in the room.

The nurse is assessing an 11-year-old girl with scoliosis. What would the nurse expect to find? Select all answers that apply. A)Complaints of severe back pain B)Asymmetric shoulder elevation C)Even curve at the waistline D)Pronounced one-sided hump on bending over E)Diminished motor function F)Hyperactive reflexes

B, D Assessment findings associated with scoliosis include asymmetric shoulder elevation, uneven curve at the waistline, rib hump on one side, and a pronounced hump on one side when bending over. Typically, only mild back discomfort is found and balance, motor strength, sensation, and reflexes are normal.

A nursing instructor is preparing for a class discussion on spinal muscular atrophy (SMA). When describing type 2 SMA, which information would the instructor include? Select all answers that apply. A)Onset before 6 months of age B)Weakness most severe in shoulders and hips C)Difficulty with swallowing D)Slowly progressing condition E)Genetic disease with autosomal recessive inheritance

B, D, E Any type of spinal muscular atrophy is a genetic motor neuron disease due to autosomal recessive inheritance. Type 2 SMA usually occurs between 6 and 18 months of age, with weakness that is most severe in the shoulders, hips, thighs, and upper back. It is slower in progression than type 1. Survival into adulthood is common if respiratory status is maintained appropriately. Type 1 SMA occurs before birth to 6 months of age and the child usually has difficulty swallowing, sucking, and breathing.

When assessing a child for slipped capital femoral epiphysis, what would the nurse identify as possible risk factors? Select all answers that apply. A)Age younger than 8 years B)African American ethnicity C)History of cystic fibrosis D)Excessive activity E)Obesity

B, E Risk factors associated with slipped capital femoral epiphysis include age between 9 and 16 years, African American race, sedentary lifestyle, and being overweight or obese. A history of cystic fibrosis may contribute to rickets.

Which exercise would the nurse suggest as most helpful to maintain mobility in a child with juvenile idiopathic arthritis? A)Jogging every other day B)Using a treadmill C)Swimming D)Playing basketball

C. Swimming Swimming is a particularly useful exercise to maintain joint mobility without placing pressure on the joints. Jogging, using a treadmill, and playing basketball would place pressure on the joints of the lower extremities.

While presenting a panel discussion to a group of parents about urinary tract infections (UTIs) in children, one of the parents asks the nurse, "Why would my daughter be more at risk than my son?" Which response by the nurse would be most accurate? A)"Girls have a smaller bladder size than boys do." B)"A girl's urethra is closer to the rectal opening." C)"A girl's urethra is longer than a boy's urethra." D)"Her kidneys are less well protected."

B. "A girl's urethra is closer to the rectal opening" In females, the urethra is shorter, which allows bacteria to enter the bladder. It also is closer in physical proximity to the rectum, leading to possible contamination. Bladder size does not differ between boys and girls. The kidneys are less well protected in the abdomen, increasing the risk for injury but not UTIs.

A nurse is leading a discussion with a group of new mothers about newborn nutrition and its importance for growth and development. One of the mothers asks, "Doesn't the baby get iron from me before birth?" Which response by the nurse would be most appropriate? A)"You give the baby some iron, but it is not enough to sustain him after birth." B)"Because the baby grows rapidly during the first months, he uses up what you gave him." C)"The iron you give him before birth is different from what he needs once he is born." D)"If the baby didn't use up what you gave him before birth, he excretes it soon after birth."

B. "Because the baby grows rapidly during the first months, he uses up what you gave him." In the term infant, a period of physiologic anemia occurs between the age of 2 and 6 months. This is due to the fact that the infant demonstrates rapid growth and an increase in blood volume over the first several months of life, and maternally derived iron stores are depleted by age 4 to 6 months of age. Sufficient iron intake is critical for the appropriate development of hemoglobin and RBCs. Therefore, the infant must ingest adequate quantities of iron either from breast milk or from iron-fortified formula in early infancy and other food sources in later infancy.

The nurse is caring for a 2-month-old with a cleft palate. The child will undergo corrective surgery at age 3 months. The mother would like to continue breastfeeding the baby after surgery and wonders if it is possible. How should the nurse respond? A)"There is a good chance that you will be able to breastfeed almost immediately." B)"Breastfeeding is likely to be possible, but check with the surgeon." C)"After the suture line heals, breastfeeding can resume." D)"We will have to wait and see what happens after the surgery."

B. "Breastfeeding is likely to be possible, but check with the surgeon." Postoperatively, some surgeons allow breastfeeding to be resumed almost immediately. However, the nurse needs to advise the mother to check with the surgeon to determine when breastfeeding can resume. Telling the mother that she has to wait until the suture line heals may be inaccurate. Telling her to wait and see does not answer her question.

A child is scheduled to undergo radiation therapy as part of his treatment plan for newly diagnosed cancer. After teaching the child and parents about this treatment, the nurse determines that additional teaching is needed when the parents state: A)"We should not wash off the markings on his skin." B)"He can use petroleum jelly if the skin becomes reddened." C)"He needs to use a sunscreen with an SPF of 30 or more." D)"He should not apply deodorant to the treatment site."

B. "He can use petroleum jelly if the skin becomes reddened." Aqueous creams and moisturizers may be used on the skin, but not petroleum jelly. Markings on the skin should not be removed or washed off. During and after radiation treatment, the skin will be more photosensitive so the child should use a high-SPF sunscreen of 30 or more. Deodorants and perfumed lotions should not be applied to the radiation treatment site.

The nurse is caring for an infant with osteogenesis imperfecta and is providing instruction on how to reduce the risk of injury. Which response from the mother indicates a need for further teaching? A)"I need to avoid pushing or pulling on an arm or leg." B)"I must carefully lift the baby from under the armpits." C)"I should not bend an arm or leg into an awkward position." D)"We must avoid lifting the legs by the ankles to change diapers."

B. "I must carefully lift the baby from under the armpits." The nurse needs to emphasize that the mother must not lift a baby or young child with osteogenesis imperfecta from under the armpits as it may cause harm. Avoiding pushing or pulling, not bending an arm or leg into an awkward position, and avoiding lifting the legs by the ankles are appropriate responses.

The nurse is providing a class for a group of childcare providers. When discussing allergic reactions, which statement by a participant indicates the need for further instruction? A)"Most allergic reactions will happen within a few minutes of eating a problematic food." B)"If a child has previously eaten a food and not had a reaction they are not 'truly' allergic to it. C)"Allergic reactions can happen hours after eating something." D)"In addition to hives some children may also have vomiting and diarrhea when having an allergic reaction to a food."

B. "If a child has previously eaten a food and not had a reaction they are not 'truly' allergic to it." Previous exposure with no incident does not mean an individual cannot develop a hypersensitivity to a food or other substance. An allergy may develop at any time. The remaining statements are correct.

The mother of a 3-week-old infant old brings her daughter in for an evaluation. During the visit, the mother tells the nurse that her baby is spitting up after feedings. Which response by the nurse would be most appropriate? A)"We need to tell the doctor about this." B)"Infants this age commonly spit up." C)"Your daughter might have an allergy." D)"Don't worry; you're just feeding her too much."

B. "Infants this age commonly spit up." In infants younger than 1 month of age, the lower esophageal sphincter is not fully developed, so infants younger than 1 month of age frequently regurgitate after feedings. Many children younger than 1 year of age continue to regurgitate for several months, but this usually disappears with age. The mother's report is not a cause for concern so the physician does not need to be notified. Additional information would be needed to determine if the infant had an allergy. Although the infant's stomach capacity is small, telling the mother not to worry does not address the mother's concern, and telling her that she is feeding the daughter too much implies that she is doing something wrong.

A newborn is diagnosed with metatarsus adductus. The parents ask the nurse how this occurred. Which response by the nurse would be most appropriate? A)"This condition is due to a genetic defect in the bones." B)"It's most likely from how the baby was positioned in utero." C)"They really don't know what causes this condition." D)"There is probably an underlying deformity of the baby's hip."

B. "It's most likely from how the baby was positioned in utero." Metatarsus adductus is a medial deviation of the forefoot that occurs as a result of in utero positioning. Osteogenesis imperfecta is a genetic bone disorder. The underlying cause of congenital clubfoot is not known. Developmental dysplasia of the hip involves a deformity of the newborn's hip.

The nurse is caring for a 4-year-old with a suspected urinary tract infection. What would be most appropriate when obtaining a urine specimen from the child? A)"I will need a urine sample." B)"Let your mom help you tinkle in this cup." C)"Please tinkle in this cup right now." D)"Please void in this cup instead of the toilet."

B. "Let your mom help you tinkle in this cup." The nurse needs to use familiar terms to explain to the child what is needed and to gain cooperation. The most positive approach would be to let the child's mother help rather than demanding that he tinkle right now. Using the terms "urine sample" or "void" is not appropriate for a 4-year-old.

The nurse is caring for an infant girl with a suspected cardiovascular disorder. Which statement by the mother would warrant further investigation? A)"My baby does not make any grunting noises." B)"The baby seems more comfortable over my shoulder." C)"The baby usually drinks all of her bottle." D)"I don't notice any rapid breathing patterns."

B. "The baby seems more comfortable over my shoulder" The nurse should be alert to statements indicating that the baby seems to be more comfortable when she is sitting up or over her mother's shoulder than when she is lying flat. Grunting or rapid breathing would be a cause for concern. Drinking all of the bottle would be considered normal.

The school nurse is working with a 10-year-old girl with recurrent abdominal pain. The girl's teacher has been less than understanding about the frequent absences and trips to the nurse's office. How should the nurse respond? A)"Be patient; she is trying some new medication." B)"The pain she is having is real." C)"The family is working toward improvement." D)"Please do not add to this family's stress."

B. "The pain she is having is real." It is important to educate the teacher that this recurrent abdominal pain is a true pain that the child feels and it is not "in her mind." Telling the teacher not to add to the family's stress or that the family is working toward improvement does not teach. The nurse must have the permission of the family to discuss the girl's medication.

After teaching the parents of a child with a hydrocele about this condition, which statement indicates that the teaching was successful? A)"If this gets worse and we don't treat it, our son could become infertile." B)"This condition should gradually go away on its own." C)"The surgeon is going to operate on him immediately." D)"It's going to be difficult putting ice packs on his scrotum."

B. "This condition should gradually go away on its own." Hydrocele requires watchful waiting because it will usually resolve spontaneously on its own. Hydrocele is not associated with the development of infertility; a varicocele, if left untreated, can lead to infertility. Immediate surgery is warranted for testicular torsion. Ice packs to the scrotum are helpful in relieving pain associated with epididymitis.

The nurse is talking to the parents of a child who has been diagnosed with severe combined immune deficiency (SCID). Which statement by the parents best indicates that they understand their child's condition? A)"He'll need to receive intravenous immunoglobulin routinely." B)"We'll need to prepare him and ourselves for a bone marrow transplant." C)"He'll need to receive several different types of antiviral medications." D)"We'll make sure that he has his EpiPen with him at all times."

B. "We'll need to prepare him and ourselves for a bone marrow transplant." SCID is a potentially fatal disorder requiring emergency intervention at the time of diagnosis. Gene therapy provides some promise for the future treatment of SCID, but until then bone marrow or stem cell transplantation is necessary. IVIG may be used to help decrease the number of infections until bone marrow or stem cell transplantation can be done. Antiviral medications are used to treat HIV infection. An EpiPen is used for anaphylaxis.

The nurse is caring for a newborn diagnosed with an atrial septal defect (ASD). The parents voice concern and state, "I can't believe this is happening. Will our child be okay?" What is the nurse's best response? A)"If the defect isn't treated it can cause problems such as pulmonary hypertension, heart failure, atrial arrhythmias, or stroke." B)"While each case is different, the majority of these defects correct on their own. Let's see what the tests show, then speak with the doctor." C)"Since there are no symptoms being exhibited right now, your child will likely not require surgery until the age of 3 years." D)"Most children have no symptoms of this defect."

B. "While each case is different, the majority of these defects correct on their own. Let's see what the test show, then speak with the doctor." While all responses supply correct information about the disorder, the best response is, "While each case is different, the majority of these defects correct on their own. Let's see what the tests show, then speak with the doctor." This individualizes the response to this child, offers realistic hope, and verifies that the physician will need to be consulted to answer questions regarding prognosis.

The nurse is preparing an 8-year-old girl for a cystoscopy. Which instruction would be most appropriate to give to the child? A)"You need to make sure that you don't go to the bathroom before the test." B)"You might feel some burning when you go to the bathroom afterward." C)"I'm going to have to put a tube into your bladder to empty it." D)"I have to put a thick tight rubber band around your arm to get a blood specimen."

B. "You might feel some burning when you go to the bathroom afterward." Cystoscopy is an endoscopic visualization of the urethra and bladder. The nurse would instruct the child that she might experience some burning when she voids after the procedure. A full bladder is needed for urodynamic studies. Putting a tube into the bladder describes a catheterization. Putting a thick tight rubber band suggests a tourniquet, which is used to obtain blood specimens.

The nurse is determining maintenance fluid requirements for a child who weighs 25 kg. How much fluid would the child need per day? A)1,560 mL B)1,600 mL C)1,650 mL D)1,700 mL

B. 1600 mL Using the following formula of: 100 mL/kg for the first 10 kg 50 mL/kg for the next 10 kg 20 mL/kg for the remaining kg The child would require (100 × 10) + (50 × 10) + (20 × 5) = 1,000 + 500 + 100 = 1,600 mL in 24 hours.

The nurse is conducting a physical examination of a child with suspected developmental dysplasia of the hip. Which finding would help confirm this diagnosis? A)Abduction occurs to 75 degrees and adduction to within 30 degrees (with stable pelvis). B)A distinct "clunk" is heard with Barlow and Ortolani maneuvers. C)A high-pitched "click" is heard with hip flexion or extension. D)The thigh and gluteal folds are symmetric.

B. A distinct "clunk" is heard with Barlow and Ortolani maneuvers. A distinct "clunk" while performing Barlow and Ortolani maneuvers is caused as the femoral head dislocates or reduces back in to the acetabulum. A higher-pitched "click" may occur with flexion or extension of the hip. This is a benign, adventitious sound that should not be confused with a true "clunk" when assessing for developmental dysplasia of the hip. Abduction to 75 degrees, adduction within 30 degrees, and symmetric thigh and gluteal folds are normal findings.

When teaching a group of students about the skeletal development in children, what information would the instructor include? A)The growth plate is made up of the epiphysis. B)A young child's bones commonly bend instead of break with an injury. C)The infant's skeleton has undergone complete ossification by birth. D)Children's bones have a thin periosteum and limited blood supply.

B. A young child's bones commonly bend instead of break with an injury. A young child's bones are more flexible and more porous with a lower mineral count than adults. Thus, bones will often bend rather than break when an injury occurs. The growth plate is composed of the epiphysis and physis. The infant's skeleton is not fully ossified at birth. Children's bones have a thick periosteum and an abundant blood supply.

The nurse is caring for an infant with a temporary ileostomy. As part of the plan of care, the nurse monitors for skin breakdown around the stoma. If redness occurs, what would be most appropriate to promote healing and prevent further skin breakdown? A)Clean the area well with a scented diaper wipe. B)Apply a barrier/healing cream or paste on the skin. C)Use a barrier wafer to attach the appliance. D)Sanitize the area with an alcohol wipe after each diaper change.

B. Apply a barrier/healing cream or paste on the skin. The nurse should use a barrier/healing cream or paste on the skin around the stoma to promote healing and prevent further skin breakdown. Diaper wipes that contain fragrance or alcohol can sting if used on nonintact skin and can worsen skin breakdown. The barrier wafer would be helpful, but does not address the skin breakdown.

The nurse is caring for an infant with bladder exstrophy. As part of the infant's preoperative plan of care, the nurse monitors for abdominal skin excoriation. Which action would be most appropriate for promoting healing and preventing further skin breakdown? A)Cleaning the area well with a scented diaper wipe B)Applying a barrier/healing cream or paste on skin C)Keeping the bladder moist and covered with a sterile bag D)Covering the area with sterile gauze pads after tub baths

B. Applying a barrier/healing cream or paste on skin The nurse should use a barrier/healing cream or paste on surrounding skin to promote healing and prevent further skin breakdown. Diaper wipes that contain fragrance or alcohol can sting if used on nonintact skin and can worsen skin breakdown. It is important to protect the bladder, but this will not address the skin excoriation. Meticulous attention to cleanliness is important, but the nurse should sponge-bathe the infant rather than immerse him in water to prevent pathogens from the water possibly entering the bladder.

The nurse is caring for a 2-month-old with cerebral palsy. The infant is limp and flaccid with uncontrolled, slow, worm-like, writhing, and twisting movements. What word would the nurse use when documenting these observations? A)Spastic B)Athetoid C)Ataxic D)Mixed

B. Athetoid Athetoid cerebral palsy is characterized by abnormal, involuntary movement. It affects all four extremities with possible involvement of the face, neck, and tongue. The movements increase in periods of stress. Dysarthria and drooling may be present as well. Spastic cerebral palsy is characterized by poor control of posture, balance, and movement; exaggeration of deep tendon reflexes; and hypertonicity of affected extremities. Ataxic is characterized by poor coordination, unsteady gait, and wide-based gait.

After teaching a class about the hemodynamic characteristics of congenital heart disease, the instructor determines that the teaching has been successful when the class identifies which defect as an example of a disorder involving increased pulmonary blood flow? A)Tetralogy of Fallot B)Atrial septal defect C)Hypoplastic left heart syndrome D)Transposition of the great vessels

B. Atrial septal defect Atrial septal defect is an example of a disorder involving increased pulmonary blood flow. Tetralogy of Fallot is a defect involving decreased pulmonary blood flow. Transposition of the great vessels and hypoplastic left heart syndrome are examples of mixed disorders.

A child with suspected sickle cell disease is scheduled for a hemoglobin electrophoresis. When reviewing the child's history, what would the nurse identify as potentially interfering with the accuracy of the results? A)Use of iron supplementation B)Blood transfusion 1 month ago C)Lack of fasting for 12 hours D)History of recent infection

B. Blood transfusion 1 month ago Blood transfusion within the previous 12 weeks may alter the results of the hemoglobin electrophoresis. Iron supplements can increase serum ferritin levels. Children should fast for 12 hours before having a specimen obtained for iron levels. A history of infection might interfere with the white blood cell count results, not hemoglobin electrophoresis.

The nurse is preparing a teaching plan for the parents of a child with a urinary tract infection (UTI). What would the nurse encourage the parents to avoid? A)Liberal fluid intake B)Caffeine C)Cranberry juice D)Cotton underwear

B. Caffeine Caffeine is an irritant to the bladder and should be avoided. Liberal fluid intake and cranberry juice should be encouraged. The child should wear cotton underwear to avoid perineal irritation.

The nurse is visually inspecting a urine specimen from a 12-year-old boy. The nurse documents gross hematuria with a specimen of which color? A)Cloudy yellow B)Cola colored C)Pale to almost clear urine D)Light orange to moderately yellow colored

B. Cola colored Gross hematuria causes the urine to appear tea, cola, or even dirty green colored. Cloudy urine is typically a sign of infection. Normal urine ranges from moderately yellow to pale or almost clear. Orange-colored urine can occur because of medication.

The school nurse is performing health assessments on students in middle school. Of what developmental milestone should the nurse be aware? A) Height in girls increases rapidly after menarche and usually ceases immediately after menarche. B) Boys' growth spurts usually begin between the ages of 8 and 14 years and end between the ages of 131/2 and 171/2 years. C) Peak height velocity (PHV) occurs at approximately 12 years of age in girls or about 6 to 12 months after menarche. D) Boys reach PHV and peak weight velocity (PWV) at about 16 years of age.

C

What information would the nurse include in the preoperative plan of care for an infant with myelomeningocele? A)Positioning supine with a pillow under the buttocks B)Covering the sac with saline-soaked nonadhesive gauze C)Wrapping the infant snugly in a blanket D)Applying a diaper to prevent fecal soiling of the sac

B. Covering the sac with saline-soaked nonadhesive gauze. For the infant with a myelomeningocele, saline-soaked nonadhesive gauze or antibiotic-soaked gauze is used to keep the sac moist. The infant is positioned prone, with a folded towel under the abdomen, so that the urine and feces flow away from the sac. A warmer or isolette is used to keep the infant warm. Blankets are avoided because they could place excess pressure on the sac. Diapering may be contraindicated to avoid placing pressure on the sac.

A child is diagnosed with hemolytic-uremic syndrome (HUS). Review of the child's laboratory test results would reveal which finding? A)Decreased blood urea nitrogen (BUN) and creatinine B)Decreased platelets and leukocytosis C)Hypernatremia and hypokalemia D)Respiratory acidosis and proteinuria

B. Decreased platelets and leukocytosis The child with HUS typically exhibits severe thrombocytopenia (decreased platelets) and leukocytosis. BUN and creatinine are elevated. Hyponatremia, hyperkalemia, metabolic acidosis, and proteinuria also may be noted.

The nurse is caring for an 8-year-old girl who has been diagnosed with leukemia and will have a variety of tests, including a lumbar puncture, before beginning chemotherapy. What action would be the priority? A)Applying EMLA to the lumbar puncture site B)Educating the child and family about the testing procedures C)Administering promethazine as ordered for nausea D)Educating the family about chemotherapy and its side effects

B. Educating the child and family about the testing procedures. The priority would be educating the child and family about the testing procedures so they know what to expect and understand why the tests are being performed. Applying EMLA to the lumbar puncture site will be done prior to the procedure. The family will be educated about chemotherapy and its side effects prior to the therapy beginning, and promethazine or other antiemetics will be administered once chemotherapy has begun.

The nurse is developing a plan of care for a child who is receiving cyclophosphamide. What advice would the nurse expect to include? A)Withholding food and fluids from the child during the infusion B)Encouraging frequent voiding during and after the infusion C)Monitoring for signs of anaphylaxis during infusion D)Assessing the child for complaints of bone pain

B. Encouraging frequent voiding during and after the infusion. Cyclophosphamide may cause hemorrhagic cystitis. Therefore, the nurse needs to provide adequate hydration and have the child void frequently during and after the infusion to decrease the risk of hemorrhagic cystitis. Fluids need to be encouraged, not withheld. Monitoring for anaphylaxis would be appropriate when asparaginase or etoposide is given. Bone pain is associated with the administration of filgrastim or sargramostim.

A nurse is preparing a presentation for a parent group about musculoskeletal injuries. When describing a child's risk for this type of injury, the nurse integrates knowledge that bone growth occurs primarily in which area? A)Growth plate B)Epiphysis C)Physis D)Metaphysis

B. Epiphysis Growth of the bones occurs primarily in the epiphyseal region. This area is vulnerable and structurally weak. Traumatic force applied to the epiphysis during injury may result in fracture in that area of the bone. The growth plate refers to the combination of the epiphysis, the end of a long bone, and the physis, a cartilaginous area between the epiphysis and the metaphysis.

A nurse is preparing a presentation for a local parent group about urinary tract infections (UTIs) in children. Which organism would the nurse incorporate into the presentation as the most common cause? A)Klebsiella B)Escherichia coli C)Staphylococcus aureus D)Pseudomonas

B. Escherichia coli E. coli most commonly causes UTI. Other less common causative organisms include Klebsiella, S. aureus, and Pseudomonas.

The nurse is assessing a newborn who was delivered after a prolonged labor due to an abnormal presentation. The newborn sustained a cranial nerve injury. The nurse would most likely expect to assess deficits related to which cranial nerve? A)Optic B)Facial C)Acoustic D)Trigeminal

B. Facial The most common cranial nerve injury occurring during birth trauma involves facial nerve palsy. The optic, acoustic, and trigeminal nerves are not typically injured during birth trauma.

The nurse is assessing a child with suspected thalassemia. What would the nurse expect to assess? A)Dactylitis B)Frontal bossing C)Presence of clubbing D)Presence of spooning

B. Frontal bossing The nurse would expect to find skeletal deformities such as frontal or maxillary bossing. Dactylitis is associated with sickle cell anemia. Clubbing and spooning are associated with chronic decreases in oxygen supply.

The parents of a 6-week-old boy come to the clinic for evaluation because the infant has been vomiting. The parents report that the vomiting has been increasing in frequency and forcefulness over the last week. The mother says, "Sometimes, it seems like it just bursts out of his mouth." A diagnosis of hypertrophic pyloric stenosis is suspected. When performing the physical examination, what would the nurse most likely find? A)Sausage-shaped mass in the upper midabdomen B)Hard, moveable, olive-shaped mass in the right upper quadrant C)Tenderness over the McBurney point in the right lower quadrant D)Abdominal pain in the epigastric or umbilical region

B. Hard, moveable, olive-shaped mass in the right upper quadrant With hypertrophic pyloric stenosis, a hard, moveable, olive-shaped mass would be palpated in the right upper quadrant. A sausage-shaped mass in the upper midabdomen would suggest intussusception. Tenderness over the McBurney point would be associated with appendicitis. Epigastric or umbilical pain would be associated with peptic ulcer disease.

The nurse is caring for an infant with suspected patent ductus arteriosus. Which assessment finding would the nurse identify as helping to confirm this suspicion? A)Thrill at the base of the heart B)Harsh, continuous, machine-like murmur under the left clavicle C)Faint pulses D)Systolic murmur best heard along the left sternal border

B. Harsh, continuous, machine-like murmur under the left clavicle With patent ductus arteriosus, a harsh, continuous, machine-like murmur (usually loudest under the left clavicle) is heard at the first and second intercostal spaces. A thrill at the base, faint pulses, and systolic murmur heard best along the left sternal border point to aortic stenosis.

The parents of a boy diagnosed with Hirschsprung disease are anxious and fearful of the upcoming surgery. The mother states, "I'm worried about having to care for our son's ostomy." Which intervention would be most helpful for the parents? A)Explaining to them about the diagnosis and surgery B)Having a wound, ostomy, and continence nurse meet with them C)Reinforcing that the ostomy will be temporary D)Teaching them about the medications used to slow stool output

B. Having a wound, ostomy, and continence meet with them Although explaining about the diagnosis and surgery, reinforcing that the ostomy will be temporary, and teaching them about medications would be appropriate, the parents are voicing concerns about caring for the ostomy. Therefore, having a wound, ostomy, and continence nurse meet with them would address these concerns and help them deal with the anxieties and care of a newly placed stoma.

When describing the various changes that occur in organ systems during adolescence, what would the nurse include? A) Significant increase in brain size B) Ossification completed later in girls C) Decrease in heart rate D) Decrease in activity of sebaceous glands

C

The nurse is conducting a physical examination of a child with a ventricular septal defect. Which finding would the nurse expect to assess? A)Right ventricular heave B)Holosystolic harsh murmur along the left sternal border C)Fixed split-second heart sound D)Systolic ejection murmur

B. Holosystolic harsh murmur along the left sternal border With ventricular septal defects, there is often a characteristic holosystolic harsh murmur along the left sternal border. Right ventricular heave, fixed split-second heart sound, and systolic ejection murmur are typically found with atrial septal defects.

When teaching a group of new parents about newborn care and development, which immunoglobulin would the nurse explain as being primarily responsible for the passive immunity exhibited by newborns? A)IgA B)IgG C)IgM D)IgE

B. IgG IgG is acquired transplacentally, providing the newborn with passive immunity to antigens to which the mother had developed antibodies. IgA, IgD, IgE, and IgM do not cross the placenta and require an antigenic challenge for production.

The nurse is caring for a 5-year-old boy undergoing radiation treatment for a neuroblastoma. Which nursing diagnosis would be most applicable for this child? A)Activity intolerance related to anemia and weakness from medications B)Impaired skin integrity related to desquamation from cellular destruction C)Impaired oral mucosa related to the presence of oral lesions from malnutrition D)Imbalanced nutrition, less than body requirements related to nausea and vomiting

B. Impaired skin integrity related to desquamation from cellular destruction A nursing diagnosis for impaired skin integrity evidenced by desquamation of the radiation site would only be made for a child undergoing radiation therapy. Activity intolerance due to anemia and weakness, impaired oral mucosa evidenced by oral lesions, and malnutrition and anorexia due to nausea and vomiting are diagnoses that are common to both radiation and chemotherapy.

A nurse is preparing a plan of care for a child with a primary immunodeficiency. Which nursing diagnosis is the priority? A)Imbalanced nutrition, less than body requirements related to poor appetite B)Ineffective protection related to impaired humoral defenses C)Acute pain related to inflammatory processes D)Risk for delayed growth and development related to chronic illness

B. Ineffective protection related to impaired humoral defenses The child with a primary immunodeficiency lacks the necessary immune responses that provide protection from infection. Therefore, the priority nursing diagnosis would be ineffective protection. Imbalanced nutrition and risk for delayed growth and development may be appropriate, but these would not be the priority. Acute pain would be more appropriate for a child with juvenile idiopathic arthritis.

A nurse is providing care to a child with idiopathic thrombocytopenic purpura with a platelet count of 18,000/mm3. Which medication would the nurse most likely expect to be ordered? A)Folic acid B)Intravenous immune globulin C)Dimercaprol D)Deferoxamine

B. Intravenous immune globulin Intravenous immune globulin would be used to treat idiopathic thrombocytopenic purpura. Folic acid is used to treat folic acid deficiency anemia. Dimercaprol is used to remove lead from the soft tissue and bone to allow for excretion by the kidneys. Deferoxamine is used to treat iron toxicity.

A child is receiving carboplatin as part of a chemotherapy protocol. What would be most important for the nurse to include in the child's plan of care? A)Monitoring for visual changes B)Maintaining adequate hydration C)Using prescribed eye drops to prevent conjunctivitis D)Avoiding administration with food or meals

B. Maintaining adequate hydration When fluorouracil is administered, the nurse must ensure adequate hydration. Monitoring for visual changes is appropriate when giving fludarabine. Eye drops are necessary to prevent conjunctivitis when high doses of cytarabine are administered. Oral mercaptopurine should not be given with meals or food.

The nurse is preparing a teaching plan for the parents of a child who has been diagnosed with a congenital heart defect. What would the nurse be least likely to include? A)Daily weight assessment B)Maintenance of strict bed rest C)Prevention of infection D)Signs of complications

B. Maintenance of strict bed rest A child with congenital heart disease should be allowed to engage in activity as tolerated, with rest periods frequently throughout the day to prevent overexertion. Daily weights, infection prevention measures, and signs of complications are all appropriate to include when teaching parents of a child with a congenital heart defect.

A nurse is caring for a 14-year-old girl following myelography. What is the priority nursing action? A)Monitoring for a decrease in spasticity B)Observing for signs of meningeal irritation C)Assessing motor function D)Observing for mental confusion or hallucinations

B. Observing for signs of meningeal irritation Following myelography, the nurse should carefully observe for signs of meningeal irritation because of what is involved in this procedure. Monitoring for a decrease in muscle spasticity, assessing motor function, and observing for mental confusion or hallucinations is appropriate following an intrathecal test dose of baclofen.

The nurse is conducting a physical examination of a child with suspected Crohn disease. Which finding would be the most suspicious of Crohn disease? A)Normal growth patterns B)Perianal skin tags or fissures C)Poor growth patterns D)Abdominal tenderness

B. Perianal skin tags or fissures. Perianal skin tags and/or fissures are highly suspicious of Crohn disease. Poor growth patterns and abdominal tenderness are common to Crohn disease but are also seen with many other conditions. Normal growth patterns would not point to Crohn disease because of problems with absorbing nutrients.

A child is prescribed monthly injections of vitamin B12. When developing the teaching plan for the family, the nurse would focus on which type of anemia? A)Aplastic anemia B)Pernicious anemia C)Folic acid anemia D)Sickle cell anemia

B. Pernicious anemia Monthly injections of vitamin B12 are used to treat pernicious anemia. Aplastic anemia is characterized by a decrease in all blood cells necessitating a bone marrow transplant. Folic acid deficiency anemia is treated with dietary measures and possible folic acid supplementation. Sickle cell anemia is treated supportively with a focus on preventing sickling crisis, infection, and other complications.

In stage IV neuroblastoma, there is metastasis to the bone, bone marrow, other organs, or distant lymph nodes. Additionally, the tumor was located in the abdomen, which is associated with a poor prognosis. Therefore, the most important diagnosis would be grieving. Although infection, skin integrity, and imbalanced nutrition may be relevant, they would not be the most important. What would be most appropriate to include in the plan of care for a child who has undergone surgery for removal of an astrocytoma? A)Elevating the foot of the bed B)Positioning the child on his unaffected side C)Raising the head of the bed at least 45 degrees D)Administering large volumes of intravenous fluids

B. Positioning the child on his unaffected side Postoperatively, the nurse should position the child on his unaffected side, with the head of the bed flat or at the level prescribed by the neurosurgeon. The foot of the bed is not elevated to prevent increasing intracranial pressure and contributing to bleeding. Fluids are administered carefully to avoid excess fluid intake, which would cause or worsen cerebral edema.

The nurse is evaluating the laboratory test results of a 7-year-old child with a suspected hematologic disorder. Which finding would cause the nurse to be concerned? A)WBC: 5.6 X 103/mm3 B)RBC: 2.8 X 106/mm3 C)Hemoglobin: 11.4 mg/dL D)Hematocrit: 35%

B. RBC: 2.8 X 106/mm3 The RBC listed is below the normal range for a child between the ages of 6 and 16 years (4.0 to 5.2 X 106/mm3). The WBC count, hemoglobin, and hematocrit are within acceptable parameters for a child this age

The nurse is describing the phases of treatment to a child who was diagnosed with leukemia and his parents. How would the nurse describe the induction stage? A)Intense therapy to strengthen remission B)Rapid promotion of complete remission C)Elimination of all residual leukemic cells D)Reduction of risk for central nervous system (CNS) disease

B. Rapid promotion of complete remission Induction is done to rapidly produce a complete remission. Consolidation or intensification is the stage when remission is strengthened and leukemic cell burden is reduced. Maintenance attempts to eliminate all residual leukemic cells, and CNS prophylaxis is the stage that attempts to reduce the development of CNS disease.

The nurse is assessing a child with suspected infective endocarditis. Which assessment finding would the nurse interpret as a sign of extracardiac emboli? A)Pruritus B)Roth spots C)Delayed capillary refill D)Erythema marginatum

B. Roth spots Roth spots are splinter hemorrhages with pale centers on the sclerae, palate, buccal mucosa, chest, fingers, or toes, and are signs of extracardiac emboli. Delayed capillary refill time does not point to extracardiac emboli. Wheezing and pruritus are indicative of a hypersensitivity reaction. Erythema marginatum is a classic rash associated with acute rheumatic fever.

When conducting a physical examination of a child with suspected Kawasaki disease, which finding would the nurse expect to assess? A)Hirsutism or striae B)Strawberry tongue C)Malar rash D)Café au lait spots

B. Strawberry tongue Dry, fissured lips and a strawberry tongue are common findings with Kawasaki disease. Acne, hirsutism, and striae are associated with anabolic steroid use. Malar rash is associated with lupus. Café au lait spots are associated with neurofibromatosis

A nurse is caring for a newborn with congenital heart disease (CHD). Which finding would the nurse interpret as indicating distress? A)Reduced respiratory rate during feeding B)Subcostal retraction at the time of feeding C)Perspiration on body after feeding D)Feeding lasting for 15-20 minutes

B. Subcostal retraction at the time of feeding. Subcostal retraction during feeding is indicative of distress associated with feeding in newborn infants with CHD. Feeding can be a stress to newborns with CHD who are seriously compromised. Additional features indicating distress in infants with CHD include increased respiratory rate, perspiration along the hairline during feeding and feeding time longer than 30 minutes.

The nurse is conducting a physical examination of a child with a brachial plexus injury. Which finding would lead the nurse to be highly suspicious of Erb palsy? A)The child is unable to close one of his eyes. B)The involved extremity is adducted, prone, and internally rotated. C)Asymmetry of the face occurs when the child is crying. D)The mouth is drawn to the noninvolved side.

B. The involved extremity is adducted, prone, and internally rotated. Erb palsy is an upper brachial plexus injury and the involved extremity usually presents as adducted, prone, and internally rotated. Inability to close one eye, facial asymmetry, or drawing of the mouth to the noninvolved side are associated with facial nerve palsy as a result of cranial nerve injuries.

The nurse is assessing a 3-year-old boy whose parents brought him to the clinic when they noticed that the right side of his abdomen was swollen. What finding would suggest this child has a neuroblastoma? A)The child has a maculopapular rash on his palms. B)The parents report that their son is vomiting and not eating well. C)The parents report that their son is irritable and not gaining weight. D)Auscultation reveals wheezing with diminished lung sounds.

B. The parents report that their son is vomiting and not eating well. Along with the swollen abdomen on one side, the parents reporting that the child is vomiting and anorexic points to the possibility of a neuroblastoma. Observing a maculopapular rash on the child's palms is a sign of graft-versus-host disease. The parents reporting that the child is irritable and not gaining weight suggests a possible brain tumor as well as malabsorption problems. Auscultation revealing wheezing with diminished lung sounds would suggest other problems, not a neuroblastoma.

The nurse is teaching a group of students about myelinization in a child. Which statement by the students indicates that the teaching was successful? A)Myelinization is completed by 4 years of age. B)The process occurs in a head-to-toe fashion. C)The speed of nerve impulses slows as myelinization occurs. D)Nerve impulses become less specific in focus with myelinization.

B. The process occurs in a head-to-toe fashion Myelinization occurs in a cephalocaudal, proximodistal manner and is completed by 2 years of age. As myelinization proceeds, nerve impulses become faster and more accurate.

The nurse is caring for a child who is taking corticosteroids for systemic lupus erythematosus. The nurse closely monitors the child based on the understanding that corticosteroids exert which major action? A)They increase liver enzymes. B)They can mask signs of infection. C)They cause bone marrow suppression. D)They decrease renal function.

B. They can mask signs of infection. The nurse understands that corticosteroids may mask signs of infection. Cytotoxic drugs cause bone marrow suppression. Nonsteroidal anti-inflammatory drugs can increase liver enzymes and decrease renal function.

The nurse is caring for a 10-year-old with Duchenne muscular dystrophy. As part of the plan of care, the nurse focuses on maintaining his cardiopulmonary function. Which intervention would the nurse implement to best promote maximum chest expansion? A)Deep-breathing exercises B)Upright positioning C)Coughing D)Chest percussion

B. Upright positioning The nurse should emphasize that the child's position should be arranged to promote maximum chest expansion. This is usually in the upright position. Deep-breathing exercises are for strengthening/maintaining respiratory muscles. Coughing helps clear the airways. Chest percussion helps loosen secretions in lungs.

The nurse is providing suggestions to a female adolescent about foods to help meet her nutritional requirements for iron. Which food would the nurse suggest as a good source of iron? A) Broccoli B) Y ogurt C) Peanut butter D) White beans

C

A nurse is conducting a physical examination of an infant and observes the urethral opening on the dorsal side of the penis. The nurse documents this finding as: A)hypospadias. B)epispadias. C)varicocele. D)hydrocele.

B. epispadias Epispadias is a urethral defect in which the opening is on the dorsal surface of the penis. Hypospadias is a urethral defect in which the opening is on the ventral surface of the penis rather than at the end. Varicocele is a venous varicosity along the spermatic cord manifested as a scrotal swelling. Hydrocele is a benign condition in which fluid accumulates in the scrotal sac.

During a health maintenance visit, a 15-year-old girl mentions that she is not happy with being overweight. Which approach is best for the nurse to take? A) "Good observation. Let's talk about diet and exercise." B) "Don't worry; you are within the weight and height guidelines." C) "What specifically have you been noticing?" D) "Tell me about your parents. Are they overweight?"

C

The nurse has seen a 15-year-old girl and a 16-year-old boy during health surveillance visits. Which physical characteristics would be seen in both teenagers? A) Decreased respiratory rates of 15 to 20 breaths per minute B) Eruption of last four molars C) Increased shoulder, chest, and hip widths D) Fully functioning sweat and sebaceous glands

C

The nurse is discussing ways to promote discipline with parents who are becoming increasingly frustrated with their teenager. What would the nurse identify as most important? A) Establish rules and expectations. B) Collaborate to determine consequence. C) Make your responses consistent. D) Explain the rules to the adolescent.

C

The nurse is promoting learning and school attendance to a 13-year-old girl. Which factor will affect the child's attitude most? A) Her parents' values and desires B) The dramatic changes to her body C) Peer group behaviors and attitudes D) Desire for attention from boys

C

The nurse is interviewing the mother of a 6-month-old being seen at a well-child visit. The mother reports she has used an over-the-counter topical ointment intended for adults on her child for a skin rash. What is the most appropriate response by the nurse? A) "This is dangerous so please do not do this again." B) "Why did you do that instead of contacting your doctor?" C) "Children have thin skin and can absorb medications differently than adults." D) "How often do you use this medication?"

C) "Children have thin skin and can absorb medications differently than adults." Feedback: Children have thinner skin than adults. They will absorb topical medications more rapidly than adults. Medications concentrated for adults should not be used on children. It is important to explain this to the parent. It is confrontational to tell her this is dangerous or to tell her to contact the physician. The frequency of use is information that should be obtained but the education is most important in this scenario.

25. The nurse is instructing a 7-year-old child and his parents about using his prescribed corrective lenses. Which of the following would the nurse include in these instructions? A) "Make sure to take your glasses off from time to time to allow your eyes to rest." B) "Remove your glasses with both hands and lay them with the lens upright on the surface." C) "Clean the glasses every day with a mild soap and water or commercial cleaning agent." D) "Use paper towels or tissues to dry and periodically clean the lenses.

C) "Clean the glasses every day with a mild soap and water or commercial cleaning agent." Eyeglasses should be cleaned daily with mild soap and water or a commercial cleaning agent. The glasses should be worn at all times, but when removed, they should be removed with both hands and placed on their side (not directly on the lens on any surface). A soft cloth, not paper towels, tissues, or toilet paper, should be used to clean the lenses.

22. A child with persistent otitis media with effusion is to undergo insertion of pressure-equalizing tubes via a myringotomy. The child is to be discharged later that day. After teaching the parents about caring for their child after discharge, which statement indicates that the teaching was successful? A) "The tubes will stay in place for about a month and then fall out on their own." B) "His chances for ear infections now have dramatically decreased." C) "He should wear earplugs when swimming in a pool or a lake." D) "We should keep the ears protected with cotton balls for the first 24 hours."

C) "He should wear earplugs when swimming in a pool or a lake." When pressure-equalizing tubes are inserted, the surgeon may recommend avoiding water entry into the ears. Therefore, earplugs are suggested when the child is in the bathtub or swimming. When swimming in a lake, earplugs are especially important because lake water is contaminated with bacteria and entry of that water into the middle ear must be avoided. Typically, the tubes remain in place for at least several months and generally fall out on their own. Placement of pressure-equalizing tubes does not prevent middle ear infection. Other than earplugs for bathing and swimming, nothing else is placed in the child's ear.

The nurse is providing care for a 14-year-old girl with severe acne. The girl expresses sadness and distress about her appearance. Which response by the nurse would be most appropriate? A) "Are you using your medicine every day?" B) "Your condition will most likely improve in a year or two." C) "Many people feel this way; I know someone who can help." D) "If you have any scarring you can undergo dermabrasion."

C) "Many people feel this way; I know someone who can help." Feedback: Depression can occur as a result of body image disturbances with severe acne. The nurse should provide emotional support to adolescents undergoing acne therapy and refer teens for counseling if necessary. Telling the girl that her condition is likely to improve in a year or two is not helpful. Asking the girl whether she uses her medicine every day or reminding her that her scars can be addressed with dermabrasion does not address her feelings of sadness and distress.

After teaching a class about the differences in the skin of infants and adults, the nurse determines that additional teaching is necessary when the class states: A) "An infant's skin is thinner than an adult's, so substances placed on the skin are absorbed more readily." B) "The infant's epidermis is loosely connected to the dermis, increasing the risk for breakdown." C) "The infant has a lower risk for damage from ultraviolet radiation because the skin is more pigmented." D) "An infant has less subcutaneous fat, which places the infant at a higher risk for heat loss."

C) "The infant has a lower risk for damage from ultraviolet radiation because the skin is more pigmented." Feedback: Infants have less pigmentation in their skin, placing them at increased risk for skin damage from ultraviolet radiation. The infant's skin is thinner, the epidermis is loosely connected, and there is less subcutaneous fat.

When developing the plan of care for a child with burns requiring fluid replacement therapy, what information would the nurse expect to include? A) Administration of colloid initially followed by a crystalloid B) Determination of fluid replacement based on the type of burn C) Administration of most of the volume during the first 8 hours D) Monitoring of hourly urine output to achieve less than 1 mL/kg/hour

C) Administration of most of the volume during the first 8 hours Feedback: With fluid replacement therapy, most of the volume is administered during the first 8 hours. Crystalloids (such as Ringer lactate) are administered for the first 24 hours, and then colloids are used once capillary permeability is less of a concern. Fluid replacement is determined by the amount of body surface area burned. Hourly urine output is expected to be at least 1 mL/kg/hour.

The nurse is preparing a class for a group of adolescents about reducing the risk of skin cancer. What information would the nurse include? A) Using a sunscreen with para-aminobenzoic acid (PABA) with an SPF of at least 10 B) Applying sunscreen at least 1 hour before going outside in the sun C) Avoiding sun exposure between the hours of 10 a.m. and 2 p.m. D) Using artificial ultraviolet (UV) tanning beds instead of sun exposure

C) Avoiding sun exposure between the hours of 10 a.m. and 2 p.m. Feedback: Avoiding sun exposure between the hours of 10 a.m. and 2 p.m. is one method of reducing the risk for skin cancer. Sunscreens with an SPF of 15 or greater that are fragrance- and PABA-free should be used. Sunscreen should be applied at least 30 minutes before exposure and then reapplied at least every 2 hours while exposed. Artificial UV light, including tanning beds, should be avoided.

9. The nurse is taking a health history for a 9-year-old girl. Which finding would alert the nurse to a possible risk factor specifically associated with visual impairment? A) Being born at 39 weeks' gestation B) Having several hours of homework daily C) Being of African American heritage D) Being active in sports

C) Being of African American heritage African American heritage is a risk factor specifically for visual impairment. Although family history of the disorder, genetic syndrome, and previous medication use are risk factors for visual impairment, they are also risk factors for hearing impairment.

A 4-year-old is brought to the emergency department with a burn. What would alert the nurse to the possibility of child abuse? A) Burn assessment correlates with mother's report of contact with a portable heater. B) Parents state that the injury occurred approximately 15 to 20 minutes ago. C) Clear delineations are noted between burned and nonburned skin areas. D) The burn area appears asymmetric and nonuniform.

C) Clear delineations are noted between burned and nonburned skin areas. Feedback: Suggested signs of a burn resulting from possible child abuse include a uniform appearance of the burn with clear delineations of burned and nonburned areas. Abuse would also be suspected if the report of the injury was inconsistent with burn injury or there was a delay in seeking treatment. An asymmetric nonuniform burn often correlates with a splatter-type burn resulting from the child pulling a source of hot fluid onto himself or herself.

A 3-year-old child has sustained severe burns and is ordered to receive 100% oxygen. What would the nurse use to administer the oxygen? A) Nasal cannula B) Venturi mask C) Nonrebreather mask D) Oxygen hood

C) Nonrebreather mask Feedback: All children with severe burns should receive 100% oxygen via a nonrebreather mask or bag-valve-mask ventilation. A nasal cannula provides only low oxygen concentrations (22% to 44%); a Venturi mask provides only 24% to 50% oxygen concentrations. An oxygen hood is used for infants only.

A nurse is caring for a 5-year-old in Buck traction. When conducting a skin examination for signs of pressure ulcers, the nurse pays particular attention to which area? A) Sacral area B) Hip area C) Occiput D) Upper arm

C) Occiput Feedback: Common sites of pressure ulcers in hospitalized children include the occiput and toes, while children who require wheelchairs for mobility demonstrate pressure ulcers in the sacral or hip areas more frequently. The upper arm is not a common site for pressure ulcers.

11. A nurse is examining a 7-year-old boy with hordeolum. Which of the following would the nurse expect to find? A) Redness B) Scaling C) Pain D) Edema

C) Pain Pain is typical of hordeolum. Blepharitis has symptoms of redness, scaling, and edema but not pain.

28. The parents of a 5-year-old bring their son to the emergency department because of significant eyelid edema. The mother states, "He scratched himself near his eye a couple of days ago while playing outside in the yard." The nurse suspects periorbital cellulitis based on which of the following? A) Evidence of discharge B) Reddened conjunctiva C) Purplish discoloration of eyelid D) Altered visual acuity

C) Purplish discoloration of eyelid Periorbital cellulitis is a bacterial infection of the eyelids and tissue surrounding the eye. The bacteria may gain entry into the skin via an abrasion, laceration, insect bite, foreign body, or impetiginous lesion. It may also result from a nearby bacterial infection such as sinusitis. Findings include marked eyelid edema, purplish or red color of the eyelid, clear conjunctivae, absence of discharge, and normal visual acuity.

29. After teaching a group of students about visual disorders, the instructor determines that the teaching was successful when the students identify which of the following as the most common cause of visual difficulties in children? A) Astigmatism B) Strabismus C) Refractive errors D) Nystagmus

C) Refractive errors The most common cause of visual difficulties in children is refractive errors. Astigmatism, strabismus, and nystagmus are other common visual disorders in children but are less common than refractive errors.

12. After teaching a group of parents about ear infections in children, which statement indicates that the teaching was successful? A) Infants with congenital deformities have an increased risk for ear infections. B) Ear infections typically increase as the child gets older. C) The shorter and wider eustachian tubes of an infant increase the risk. D) Adenoids shrink as the child grows, allowing more bacteria to enter.

C) The shorter and wider eustachian tubes of an infant increase the risk. The infant has relatively short, wide, horizontally placed Eustachian tubes, allowing bacteria and viruses to gain access to the middle ear and resulting in an increased number of infections as compared to adults. Congenital deformities of the ear are associated with other body system anomalies, but not necessarily an increase in ear infections. As the child matures, the Eustachian tubes assume a more slanted position, so older children and adults have fewer infections. A child's adenoids are often enlarged, leading to obstruction of the Eustachian tubes and infection

4. The nurse is caring for a 6-year-old visually impaired boy and is about to begin the physical examination. Which intervention would be most appropriate to promote effective communication with the child? A) Show him the stethoscope. B) Describe the examination room. C) Use his name before touching him. D) Allow him to explore the exam room.

C) Use his name before touching him. When interacting with a visually impaired child, it is a good communication technique to use his name to gain his attention before touching him. Letting him listen to his heart with the stethoscope, describing the examination room, and promoting exploration by touch are sound ways to interact, but are not specific to communicating with the child at the beginning of the assessment.

The nurse is collecting information from the parents of a 3-year-old child about her sleeping patterns. Which question by the nurse will best elicit information from the parents? A)"How are things going at home?" B)"Is your child sleeping well at night?" C)"How many hours does your child sleep at night?" D)"What time does your child go to bed at night?"

C)"How many hours does your child sleep at night?" Asking an open-ended question will provide the most opportunity for data to be collected from the parents. Asking how things are going at home is vague and may or may not give the needed information. Asking if the child is sleeping well is problematic as the term "well" is subjective and may be interpreted differently by different individuals. Asking when the child goes to bed is a broad question that may not provide the needed information about the quantity of sleep being achieved by the child each night.

During the health history, the mother of a 4-month-old child tells the nurse she is concerned that her baby is not doing what he should be at this age. What is the nurse's best response? A)"I'll be able to tell you more after I do his physical." B)"Fill out the questionnaire and then I can let you know." C)"Tell me what concerns you." D)"All mothers worry about their babies. I'm sure he's doing well."

C)"Tell me what concerns you." Asking about the mother's concerns is assessment and is the first thing the nurse should do. The mother has intimate knowledge of the infant and can provide invaluable information that can help structure the nurse's assessment. Relying on the physical assessment ignores the value of the mother's input. A screening questionnaire is no substitute for a developmental assessment. Minimizing the mother's concerns reduces communication between the mother and the nurse.

The parents of a 2-day-old girl are concerned because her feet and hands are slightly blue. How should the nurse respond? A)"Your daughter has acrocyanosis; this is causing her blue hands and feet." B)"Let's watch her carefully to make sure she does not have a circulatory problem." C)"This is normal; her circulatory system will take a few days to adjust." D)"This is a vasomotor response caused by cooling or warming."

C)"This is normal; her circulatory system will take a few days to adjust." The nurse should tell the parents that this is normal and that the baby's circulatory system is adjusting to extrauterine life. Using the technical term "acrocyanosis" would most likely scare the parents. Telling the parents that the child may have a circulatory problem is inaccurate as this is a normal variation. Acrocyanosis and the mottling caused by cooling and warming are two different variations.

Three children in a family, ages 7 months, 4 years, and 9 years have been tested for lead poisoning. The two younger children's tests reflect elevated lead levels and they will be undergoing treatment. The children's mother questions why her younger children were not "spared" as their older sibling was. What response by the nurse is most correct? A)"Some children are better able to metabolize toxins such as lead after exposure." B)"Your older child has a stronger liver and kidneys, which have helped her to better rid her body of the lead." C)"Younger children are often impacted because of their play behaviors place them on the floors and they often put things into their mouths." D)"It is likely your older child may have had elevated levels earlier in life but has gotten over the condition."

C)"Younger children are often impacted because of their play behaviors place them on the floors and they often put things into their mouths." Lead poisoning is a problem that affects children younger than age 6 the most due to the fact that they are crawling on the ground and putting things in their mouths, and their developing neurologic system is more sensitive to the effects of lead. The liver and kidney development is not an influence on the degree of lead found in children's blood specimens. Metabolism is not the greatest influence on the reason why only the younger children have been impacted by lead poisoning.

Assessment reveals that a child weighs 73 pounds and is 4 feet, 1 inch tall. The nurse calculates this child's body mass index as: A)19.1 B)20.7 C)21.4 D)24.5

C)21.4 Body mass index is determined by dividing the child's weight (in pounds) by the child's height (in inches) squared and then multiplying this figure by 703. Thus, 73 lb divided by (49 inches × 49 inches) equals 0.0304 multiplied by 703 equals 21.37 or 21.4.

The nurse is providing care for children in a pediatric medical home. What is a characteristic of care in these types of facilities? A)All insurance except Medicaid is accepted. B)Ambulatory care is not provided C)A centralized database contains all child information. D)Continuity of care is provided from infancy through adulthood.

C)A centralized database contains all child information. In a medical home a centralized database contains all pertinent information. All insurance including Medicaid is accepted in the medical home and ambulatory care is provided. Continuity of care is also provided from infancy to adolescence.

The nurse is performing a vision screening for a 4-year-old child. Which screening chart would be best for determining the child's visual acuity? A)Snellen B)Ishihara C)Allen figures D)Color Vision Testing Made Easy (CVTME)

C)Allen figures The Allen figures chart is reliable for assessing visual acuity for a preschool child. The Snellen chart requires that the child has a good knowledge of the alphabet. This is not an expectation for a 4-year-old child. The Ishihara and CVTME charts are designed to assess color vision discrimination and not visual acuity.

The nurse is conducting a physical examination of a child following a comprehensive health history. What should be the focus of the physical examination? A)The child B)The parents C)Chief complaint D)Developmental age

C)Chief complaint The next step after the health history is the physical examination. It should focus on the chief complaint or any of the systems that engaged the nurse's critical thinking while obtaining the history. The child and parents are involved in the assessment but the focus is on the health problem. The nurse should conduct a physical examination with the child's developmental age in mind.

What activity would the nurse expect to find in an 18-month-old? A)Standing on tiptoes B)Pedaling a tricycle C)Climbing stairs with assistance D)Carrying a large toy while walking

C)Climbing stairs with assistance Toddlers continue to progress with motor skills. An 18-month-old should be able to climb stairs with assistance. A 24-month-old should be able to stand on his or her tiptoes and carry a large toy while walking. A 36-month-old would be able to pedal a tricycle.

The nurse is teaching parents interventions appropriate to the emotional development of their toddlers. What is a recommended intervention for this age group? A)Remove children's security blankets at this stage to help them assert their autonomy. B)Distract toddlers from exploring their own body parts, particularly their genitals. C)Do not blame toddlers for aggressive behavior; instead, point out the results of their behavior. D)Offer toddlers many choices to foster control over their environment.

C)Do not blame toddlers for aggressive behavior; instead, point out the results of their behavior. Toddlers should not be blamed for their aggressive behavior; adults can assist the toddler in building empathy by pointing out when someone is hurt and explaining what happened. Adults should allow toddlers to rely upon a security item to self-soothe as this is a function of autonomy and is viewed as a sign of a nurturing environment, rather than one of neglect. Toddlers may question parents about the difference between male and female body parts and may begin to explore their own genitals. This is normal behavior in this age group. Offering limited choices is one way of allowing toddlers some control over their environment and helping them to establish a sense of mastery.

The nurse is testing the sensory development of a toddler brought to the clinic for a well visit. What might alert the nurse to a potential problem with the child's sensory development? A)The toddler places the nurse's stethoscope in his mouth. B)The toddler's vision tests at 20/50 in both eyes. C)The toddler does not respond to commands whispered in his ear. D)The toddler's taste discrimination is not at adult levels yet.

C)The toddler does not respond to commands whispered in his ear. Hearing should be at the adult level, as infants are ordinarily born with hearing intact. Therefore, the toddler should hear commands whispered in his ear. Toddlers examine new items by feeling them, looking at them, shaking them to hear what sound they make, smelling them, and placing them in their mouths. Toddler vision continues to progress and should be 20/50 to 20/40 in both eyes. Though taste discrimination is not completely developed, toddlers may exhibit preferences for certain flavors of foods.

The nurse is performing a physical assessment of a 3-year-old girl. What finding would be a concern for the nurse? A)The toddler gained 4 pounds in weight since last year. B)The toddler gained 3 inches in height since last year. C)The toddler's anterior fontanel is not fully closed. D)The circumference of the child's head increased 1 inch since last year.

C)The toddler's anterior fontanel is not fully closed. -The anterior fontanel should be closed by the time the child is 18-months old. The average toddler weight gain is 3 to 5 pounds per year. Length/height increases by an average of 3 inches per year. Head circumference increases about 1 inch from when the child is between 1 and 2 years of age, then increases an average of a half-inch per year until age 5.

The nurse is providing instruction to the parents of a newborn boy. The parents have decided not to circumcise the child. What information should be included in the discussion? Select all answers that apply. A)The foreskin should be pulled back for cleaning at least once per day. B)The foreskin should be pulled back gently with each diaper change. C)Clean the penis gently with soap and water. D)If the foreskin is not retractable do not force it. E)When the foreskin is retracted, gently replace it prior to completing diapering.

C, D, E The newborn's foreskin does not normally retract. This may not be possible until later in infancy. If the foreskin does not retract do not force it. If the foreskin is able to be retracted, do so gently. Return the foreskin to place prior to applying the diaper. Soap and water should be used several times per day to clean the penis and perineal area.

A child with suspected dyslipidemia undergoes laboratory testing. The nurse is reviewing the results. Which finding would the nurse interpret as supporting the diagnosis? Select all that apply. A)Total cholesterol level of 150 mg/dL B)Total cholesterol level of 180 mg/dL C)Total cholesterol level of 220 mg/dL D)LDL level of 90 mg/dL E)LDL level of 120 mg/dL F)LDL level of 140 md/dL

C, F A total cholesterol level over 200 mg/dL and LDL level above 130 mg/dL are considered high and would support the diagnosis of dyslipidemia. Total cholesterol levels between 170 to 199 mg/dL and LDL levels between 110 to 129 mg/dL are considered borderline. Total cholesterol levels less than 170 mg/dL and LDL levels less than 110 mg/dL are acceptable in children.

The nurse is caring for a child who is having an anaphylactic reaction with bronchospasm. The nurse would expect to administer what medication for bronchospasm as ordered? A)Epinephrine B)Corticosteroid C)Albuterol D)Diphenhydramine

C. Albuterol The nurse would expect to administer bronchodilation inhalation treatment (albuterol) if bronchospasm is present. Epinephrine, diphenhydramine, and/or corticosteroids are administered to reverse the allergic process.

The nurse is teaching the mother of a 5-year-old boy with a myelomeningocele who has developed a sensitivity to latex. Which response from his mother indicates a need for further teaching? A)"He needs to get a medical alert identification." B)"I will need to discuss this with his caregivers." C)"A product's label indicates whether it is latex-free." D)"He must avoid all contact with latex."

C. "A product's label indicates whether it is latex-free." The Food and Drug Administration (FDA) requires that all medical supplies be labeled if they contain latex, but this is not the case with consumer products. The mother must be familiar with products that contain latex. The Spina Bifida Association of America maintains an updated list of latex-containing products. Getting a medical alert identification, talking with his caregivers, and avoiding all contact with latex are correct.

A child with a suspected cardiovascular disorder is to undergo diagnostic testing and is scheduled for an echocardiogram. When explaining this test to the child, what would the nurse most likely include? A)"This test will check the pattern of how your heart is beating." B)"They'll take a picture of your chest to look at the heart's size." C)"A special wand that picks up sound is used to check your heart." D)"Small patches are attached to your chest to check the heart rhythm."

C. "A special wand that picks up sound is used to check your heart." An echocardiogram is a noninvasive ultrasound procedure using a gel-coated wand that assesses the heart wall thickness, the size of the chambers, valve and septal motion, and the relationship of the great vessels to other cardiac structures. An electrocardiogram reveals the pattern or rhythm of the heart's beating and involves small patches or electrodes attached to the chest. A chest radiograph involves a radiographic film of the chest to determine the size of the heart and its chambers.

An infant with congenital heart disease is to undergo surgery to correct the defect. The mother states, "I guess I'm going to have to stop breastfeeding her." Which response by the nurse would be most appropriate? A)"That's true, but we'll make sure she gets the best intravenous nutrition." B)"Unfortunately, your baby needs more nutrients than what breast milk can provide." C)"Breast milk may help to boost her immune system, so you can continue to use it." D)"She won't be able to suck, so we have to give her fortified formula through a tube."

C. "Breast milk may help to boost her immune system, so you can continue to use it." Breastfeeding a child before and after cardiac surgery may boost the infant's immune system, which can help fight postoperative infection. If breastfeeding is not possible, mothers can pump milk and the breast milk may be given via bottle, dropper, or gavage feeding. In addition, breastfeeding is associated with decreased energy expenditure during the act of feeding.

A 9-year-old child has undergone a cardiac catheterization and is being prepared for discharge. The nurse is instructing the parents and child about postprocedure care. Which statement by the parents indicates that the teaching was successful? A)"This pressure dressing needs to stay on for 5 days from now." B)"He can't eat but he can drink fluids for the next 24 hours." C)"He should avoid taking a bath for about 3 days but he can shower." D)"It's normal if he says he feels like his heart skipped a beat."

C. "He should avoid taking a bath for about 3 days but he can shower." After a cardiac catheterization, the child should avoid tub baths for about 3 days but he can shower or use sponge baths. The pressure dressing should be removed the day after the procedure and a dry sterile dressing or adhesive bandage is applied for the next several days. After the procedure, the child can resume his usual diet. Any reports of fluttering or the heart skipping a beat should be reported.

A 6-year-old boy with cerebral palsy has been admitted to the hospital for some tests. His condition is stable. The boy's mother remains with her son, but she is obviously exhausted and stressed. Which response by the nurse would be most appropriate? A)"Would you like me to bring you a blanket and pillow?" B)"You are doing such a wonderful job with your son." C)"He's in good hands; consider going home to get some sleep." D)"Are you planning to spend the night or to go home?"

C. "He's in good hands; consider going home to get some sleep." Providing daily, intense care can be quite demanding and tiring. When a child with cerebral palsy is admitted to the hospital, this may serve as a time of respite for family and primary caregivers. The nurse should remind the mother that her son is in good hands and urge her to go home. Asking her whether she is planning to stay might make the mother feel obligated to stay. Asking if she wants a blanket or pillow does not encourage the mother to leave the hospital. Telling the mother she is doing a good job is nice, but does not encourage her to take a break.

The nurse is caring for a 2-month-old infant who has been diagnosed with acute heart failure. The nurse is providing teaching about nutrition. Which statement by the mother indicates a need for further teaching? A)"The baby may need as much as 150 calories/kg/day." B)"Small, frequent feedings are best if tolerated." C)"I need to feed him every hour to make sure he eats enough." D)"Gavage feedings may be required for now."

C. "I need to feed him every hour to make sure he eats enough." Although offering small frequent feedings is appropriate if the infant tolerates them, feeding every hour is not necessary. During the acute phase, continuous or intermittent gavage feedings may be needed to help the infant maintain or gain weight. Due to the increased metabolic demands, the infant may require as much as 150 calories/kg/day.

A nurse is working with an adolescent who is slightly overweight and was recently diagnosed with hypertension. They are discussing nutritional management. Which statement by the adolescent demonstrates understanding of the information? A)"I have to make sure that I don't eat a lot of salty foods." B)"I can eat any amount at a meal as long as I don't eat between meals." C)"I should eat plenty of fresh fruits and vegetables." D)"If I skip breakfast, I can eat a much bigger lunch."

C. "I should eat plenty of fresh fruits and vegetables." Nutritional management includes controlling portion sizes, decreasing the intake of sugary beverages and snacks, eating more fresh fruits and vegetables, and eating a healthy breakfast. Salt restriction and potassium or calcium supplements have not been shown to decrease blood pressure in children.

The nurse is caring for a 13-year-old boy in traction prior to surgery for slipped capital femoral epiphysis. He has been in an acute care setting for 2 weeks and will require an additional 10 days in the hospital. He is complaining that he feels isolated and is resisting further treatment. Which response by the nurse would be most appropriate? A)"I know it is boring, but you must remain immobile for 2 more weeks." B)"If there are no complications, you only have 2 more weeks here." C)"Let's come up with things to do like books, movies, games, and friends to visit." D)"If you resist your treatment, your condition will only get worse."

C. "Let's come up with things to do like books, movies, games and friends to visit." After 2 weeks in traction, a teenager can become easily bored and isolated from usual peer interaction. The most helpful intervention would be to engage the help of the child to develop a list of books, games, movies, and other activities that he would enjoy. The nurse should also encourage visitation and phone calls from friends. Telling the adolescent that he needs to remain immobile or telling him that he has only 2 more weeks do not address the adolescent's issue. Telling the adolescent that his condition will worsen if he resists is threatening and inappropriate.

The nurse is caring for an active 14-year-old boy who has recently been diagnosed with scoliosis. He is dismayed that a "jock" like himself could have this condition, and is afraid it will impact his spot on the water polo team. Which response by the nurse would best address the boy's concerns? A)"If you wear your brace properly, you may not need surgery." B)"The good news is that you have very minimal curvature of your spine." C)"Let's talk to another boy with scoliosis, who is winning trophies for his swim team." D)"Let's talk to the doctor about your treatment options."

C. "Let's talk to another boy with scoliosis, who is winning trophies for his swim team." Because this boy is concerned about limiting his participation in water polo and perceives scoliosis as a disease that does not affect "jocks," putting the child in contact with someone with the same problem would be helpful. Telling the adolescent about not needing surgery if he wears his brace or that his curvature is minimal may or may not be true in his case and thus would be false reassurance. Although these suggestions and also the suggestion about talking to the doctor about treatment options could be helpful by engaging his input in the treatment, these do not address his specific concerns about his body image.

The mother of a 4 week old infant is tearful. She reports the physician has told her that her son has a small atrial septal defect. She reports she is worried and asks the nurse more about the condition. Which statement by the parents best indicates an understanding of the nurse's teaching? A)"This greatly places my son at risk for cardiac failure." B)"If this does not resolve by the time my child is 1 year old he will likely need surgery." C)"Most of the time this condition spontaneously resolves." D)"Since the surgery to correct this condition can be risky my son will need to be at least 40 pounds."

C. "Most of the time this condition spontaneously resolves." Atrial septal defects in children most likely resolve without treatment. Those that are not corrected by the age of 18 months will likely require surgical intervention. When planned, surgery is not usually performed until the child is at least 3 years of age. There is no indication other problems are present so the child is not at an increased risk for cardiac failure.

A nurse is interviewing the parents of a child diagnosed with obstructive uropathy. Which statement by the parents would the nurse identify as significant? A)"She's been constipated quite a few times." B)"We've noticed that her bed is wet in the morning." C)"She had surgery to repair a problem with her anus." D)"She had a bacterial skin infection about a week ago."

C. "She had surgery to repair a problem with her anus." Risk factors associated with obstructive uropathy include prune belly syndrome, chromosome abnormalities, anorectal malformations, and ear defects. The statement about surgery to repair an anal problem suggests an anorectal malformation. Constipation is a risk factor for urinary tract infections. Bedwetting suggests enuresis. A bacterial skin infection is associated with acute glomerulonephritis.

A 16-year-old patient has just been diagnosed with HIV. Which statement by the parent indicates understanding of the diagnosis? A)"It is important for our child to get started on drug therapy for a better chance of a cure of the infection." B)"I must be infected with HIV and passed it to our child while in the uterus for the infection to have occurred." C)"We don't want to face the fact that it is likely our child contracted HIV through sexual contact or IV drug use." D)"Infections as a result of being HIV positive are a low risk since the diagnosis came early."

C. "We don't want to face the fact that it is likely our child contracted HIV through sexual contact or IV drug use." In teenagers, HIV is primarily contracted through sexual intercourse with an infected person or sharing of needles with an infected person during IV drug use. There is no cure for HIV, infants primarily contract the virus from their mothers, and infections as a result of having HIV are not dependent on when the diagnosis occurred.

The nurse is caring for a female infant with torticollis and is providing instructions to the parents about how to help their daughter. Which statement by the parents indicates a need for further teaching? A)"We must encourage our daughter to turn her head both ways." B)"Flatness on one side of the head is a common side effect." C)"We must apply firm pressure and stretching every other day." D)"We will do a daily stretching regimen with multiple sessions."

C. "We must apply firm pressure and stretching every other day." The nurse needs to remind the parents that the stretching exercises should be done several times a day. The stretching is applied with gentle, not firm, pressure and should be done every day for multiple sessions. The statements about turning the head both ways, flatness on one side as common, and daily stretching with multiple sessions are correct.

The nurse is caring for a 13-year-old girl with von Willebrand disease. After teaching the adolescent and her parents about this disorder and care, which response by the parents indicates a need for additional teaching? A)"We need to administer Stimate prior to dental work." B)"We should be aware that she may suffer from menorrhagia." C)"We should administer desmopressin as often as needed." D)"We understand that she may have frequent nosebleeds."

C. "We should administer desmopressin as often as needed." The parents need to know that desmopressin spray Stimate is used for controlling bleeding; the other brands are used for homeostasis and enuresis. Additionally, Stimate should only be used 3 days in a row as lessening of the response (tachyphylaxis) occurs with frequent use. Stimate should be used before dental work. Menorrhagia and nosebleeds may occur.

A 5-year-old girl is diagnosed with iron-deficiency anemia and is to receive iron supplements. The child has difficulty swallowing tablets, so a liquid formulation is prescribed. After teaching the parents about administering the iron supplement, which statement indicates the need for additional teaching? A)"She needs to eat foods that are high in fiber so she doesn't get constipated." B)"We'll try to get her to drink lots of fluids throughout the day." C)"We will place the liquid in the front of her gums, just below her teeth." D)"We need to measure the liquid carefully so that we give her the correct amount."

C. "We will place the liquid in the front of her gums, just below her teeth." When giving liquid iron supplements, the liquid should be placed behind the teeth because it can stain the teeth. Iron can lead to constipation, so increased fluid and fiber intake is appropriate. The dosage needs to be measured carefully to prevent overdosing the child, leading to iron toxicity.

A nurse is conducting a physical examination of a 5-year-old with suspected iron-deficiency anemia. How would the nurse evaluate for changes in neurologic functioning? A)"Open your mouth so I can look inside your cheeks and lips." B)"Do you have any bruises on your feet or shins?" C)"Will you show me how you walk across the room?" D)"Let me see the palms of your hands and soles of your feet."

C. "Will you show me how you walk across the room?" Neurologic effects of iron deficiency may be demonstrated when the child's ability to sit, stand, and walk are impaired. Inspecting the mouth, looking for bruises, and checking the hands and feet provide information about signs of petechiae, purpura, or pallor.

The nurse is caring for a 2-year-old girl in a bilateral brace with tibia vara. Her parents are upset by their toddler's limited mobility. Which response by the nurse would be most appropriate? A)"If you don't follow the therapy, your daughter could develop severe bowing of her legs." B)"It's important to use the brace or your daughter may need surgery." C)"You are doing a great job. Let's put our heads together on how to keep her busy." D)"You'll need to accept this since treatment may be required for several years."

C. "You are doing a great job. Let's put our heads together on how to keep her busy." The nurse should support the parents by encouraging and praising their compliance with bracing. It is also important to work with the parents to help develop age-appropriate diversions to promote normal growth and development. Telling the parents that they must be compliant or their daughter could develop severe bowing does not teach, does not offer solutions, and does not address the parents' concerns. Telling the parents that they must simply accept this and that the treatment could take years is likely to upset them and does not teach. It also does not address their concerns.

A child is scheduled for a lower endoscopy. What would the nurse include in the child's plan of care in preparation for this test? A)Explaining about the need to ingest barium B)Establishing an intravenous access for radionuclide administration C)Administering the prescribed bowel cleansing regimen D)Withholding prescribed proton pump inhibitors for 5 days before

C. Administering the prescribed bowel cleansing regimen Prior to a lower endoscopy, the child must undergo bowel cleansing to allow visualization of the lower gastrointestinal tract via a fiberoptic instrument. Barium is ingested for an upper gastrointestinal and/or small bowel series. Radionuclides are used with a hepatobiliary scan. Proton pump inhibitors are withheld for 5 days before a urea breath test.

A nurse identifies a nursing diagnosis of impaired urinary elimination related to urinary tract infection. When developing the plan of care, what would be most important for the nurse to do first? A)Develop a schedule for bladder emptying. B)Encourage fluid intake. C)Assess usual voiding patterns. D)Monitor intake and output.

C. Assess usual voiding patterns The first action would be to assess the child's usual voiding patterns to establish a baseline to develop an appropriate schedule for bladder emptying. Encouraging fluid intake and monitoring intake and output would be appropriate, but these would not be the first action.

After teaching a class about humoral and cellular immunity, the nurse recognizes that the additional teaching is needed when the class states that: A)humoral immunity crosses the placenta. B)cellular immunity involves the T lymphocytes. C)cellular immunity recognizes antigens. D)humoral immunity does not destroy the foreign cell.

C. Cellular immunity recognizes antigens Humoral immunity recognizes antigens and cellular immunity does not. Humoral immunity crosses the placenta in the form of IgG. Cellular immunity involves the action of T lymphocytes, and humoral immunity does not destroy the foreign cell.

The nurse is caring for a 9-year-old boy who is having chemotherapy. The nurse is developing a teaching plan for the child and family about nutrition. What instruction would the nurse be least likely to include? A)Emphasizing the intake of grains, fruits, and vegetables B)Featuring high-fiber foods if opioid analgesics are being taken C)Concentrating on consuming primarily high-calorie shakes and puddings D)Avoiding milk products if diarrhea is a problem

C. Concentrating on consuming primarily high-calorie shakes and puddings. Providing high-calorie shakes and puddings with diet restrictions can help with weight gain, if that is a problem. However, concentrating on high-calorie shakes and puddings is not a good strategy. It is best to provide a balanced diet emphasizing grains, fruits, and vegetables. If pain is being treated with opioid analgesics, featuring high-fiber foods is important to help relieve constipation. Avoiding milk products is a good idea if diarrhea is a problem because lactose can make diarrhea worse.

The school nurse is walking through the lunchroom when one of the children says she started to feel strange after trading lunches with a friend. Which assessment would be most important? A)Asking if she has a rash anywhere B)Checking if she has any nausea C)Determining if her throat itches D)Asking if she has abdominal pain

C. Determining if her throat itches. Asking if the child's throat itches is most important because this aids in determining airway patency, which is always the priority. Asking about a rash, nausea, or abdominal pain can be done after the nurse is certain the child's airway is not jeopardized.

The nurse is caring for a client with hemolytic-uremic syndrome (HUS). The client is demonstrating oliguria. What does the nurse expect to find when reviewing the client's records? A)A pattern of below-normal blood pressure B)Higher fluid output than fluid intake C)Elevated BUN and creatinine levels D)Increased glomerular filtration rate (GFR)

C. Elevated BUN and creatinine levels Oliguria is the result of acute renal failure associated with HUS. The BUN and creatinine level are indications of kidney function and are elevated with acute renal failure. Hypertension is associated with HUS. Output is decreased with renal failure, as is GFR.

The nurse is caring for a 5-year-old girl with a disseminated medulloblastoma. What intervention would be most appropriate for this situation? A)Providing emotional support to the parents and siblings of the child B)Recommending support groups for people whose children have cancer C)Encouraging the family to cry and express feelings away from the child D)Educating the family about the disease, its treatments, and side effects

C. Encouraging the family to cry and express feelings away from the child. The outcome of this highly malignant medulloblastoma is often not positive. Helping the family through anticipatory grieving by encouraging the family to cry and express feelings away from the child would be unique to this child's situation. Educating the family about the disease, its treatments, and side effects; recommending support groups; and providing emotional support to the parents and siblings would be appropriate for any child with cancer.

The nurse is assessing a child with pauciarticular-type juvenile idiopathic arthritis. What would the nurse expect to assess? A)Fever B)Rash C)Eye inflammation D)Splenomegaly

C. Eye inflammation With pauciarticular juvenile idiopathic arthritis, eye inflammation may be noted. Fever, rash, and enlarged spleen would be noted with systemic juvenile idiopathic arthritis.

A child diagnosed with stage IV neuroblastoma has undergone abdominal surgery to remove the tumor. He is now receiving chemotherapy. Which nursing diagnosis would be most important? A)Risk for infection related to chemotherapy B)Impaired skin integrity related to abdominal surgery C)Grieving related to advanced disease and poor prognosis D)Imbalanced nutrition related to adverse effects of chemotherapy

C. Grieving related to advanced disease and poor prognosis In stage IV neuroblastoma, there is metastasis to the bone, bone marrow, other organs, or distant lymph nodes. Additionally, the tumor was located in the abdomen, which is associated with a poor prognosis. Therefore, the most important diagnosis would be grieving. Although infection, skin integrity, and imbalanced nutrition may be relevant, they would not be the most important.

A nurse is preparing a program for a group of parents about injury prevention. What would the nurse include as an important contributing factor for cervical spine injury in a child? A)Exposure to teratogens while in utero B)Immaturity of the central nervous system C)Increased mobility of the spine D)Incomplete myelinization

C. Increased mobility of the spine Compared to the adult, a child's spine is very mobile, especially in the cervical spine region, resulting in a higher risk for cervical spine injury. Exposure to teratogens in utero may lead to altered growth and development of the brain or spinal cord. Immaturity of the central nervous system places the infant at risk for insults that may result in delayed motor skill attainment or cerebral palsy. Incomplete myelinization reflects the lack of motor control.

A newborn is diagnosed with patent ductus arteriosus. The nurse anticipates that the physician will most likely order which medication? A)Alprostadil B)Heparin C)Indomethacin D)Spironolactone

C. Indomethacin Indomethacin is the drug typically ordered to close a patent ductus arteriosus. Alprostadil would be indicated to maintain the ductus arteriosus temporarily in infants with ductal-dependent congenital heart defects. Heparin would be used for prophylaxis and treatment of thromboembolic disorders, especially after surgery. Spironolactone would be used to manage edema due to heart failure and to treat hypertension.

During a follow-up visit, the parents of a 5-month-old infant diagnosed with congenital heart disease tell the nurse, "We're just so tired and emotionally spent. All these tests and examinations are overwhelming. We just want to have a normal life. We're so focused on the baby that it seems like our 3-year-old is lost in the shuffle." Which nursing diagnosis would the nurse identify as most appropriate? A)Risk for delayed growth and development related to necessary treatments B)Deficient knowledge related to the care of a child with congenital heart disease C)Interrupted family processes related to demands of caring for the ill child D)Fear related to infant's cardiac condition and need for ongoing care

C. Interrupted family process related to demands of caring for the ill child. The statements by the parents indicate that there is disruption in the family resulting from the demands of caring for the ill infant and they verbalized concern about their older child. The child may be at risk for delayed growth and development, but this is not indicated by the parents' statements. The parents may lack knowledge about their infant's condition and they may be experiencing fear about the infant's condition, but the statements reflect issues related to the family functioning.

A child with spastic cerebral palsy is to receive botulin toxin. The nurse prepares the child for administration of this drug by which route? A)Oral B)Subcutaneous injection C)Intramuscular injection D)Intravenous infusion

C. Intramuscular injection Botulin toxin is administered by injection into the muscle. It may cause dry mouth. It is not administered orally, by subcutaneous injection, or by intravenous infusion.

A child is receiving methotrexate as part of his chemotherapy protocol. The nurse would anticipate administering which agent to counteract the toxic effects of methotrexate? A)Mesna B)Cyclosporine C)Leucovorin D)Nystatin

C. Leucovorin Leucovorin is given as an antidote to methotrexate to reduce its toxic effects. Mesna is given when cyclophosphamide and ifosfamide are used to prevent hemorrhagic cystitis. Cyclosporine is an immunosuppressant used to treat graft-versus-host disease after hematopoietic stem cell transplant. Nystatin is used to treat mucositis or systemic fungal infection.

The nurse is caring for a child who has undergone stem cell transplantation for severe combined immune deficiency. What finding would the nurse interpret as indicative of graft-versus-host disease? A)Presence of wheezing B)Splenomegaly C)Maculopapular rash D)Chronic or recurrent diarrhea

C. Maculopapular rash The nurse should monitor the stem cell transplant child closely for a maculopapular rash that usually starts on the palms and soles for indication that graft-versus-host disease is developing. Wheezing and recurrent diarrhea are not typical clinical manifestations of graft-versus-host disease. Splenomegaly is associated with hypogammaglobulinemia.

The nurse is caring for a toddler who is in Piaget's sensorimotor stage of cognitive development. Which task would the nurse expect the toddler to be able to perform? A)Completing puzzles with four pieces B)Winding up a mechanical toy C)Playing make-believe with dolls D)Knowing which are his or her toys

D)Knowing which are his or her toys The toddler in Piaget's sensorimotor stage of cognitive development (18 to 24 months) understands requests, is capable of following simple directions, and has a sense of ownership (knowing which toys are his). The other tasks are accomplished by the child in the preoperational stage (2 to 7 years).

The school nurse is presenting a class to a group of students about common overuse disorders. Which disorder would the school nurse include? A)Dislocated radial head B)Transient synovitis of the hip C)Osgood-Schlatter disease D)Scoliosis

C. Osgood-Schlatter disease Overuse syndromes refer to a group of disorders that result from repeated force applied to normal tissue. An example is Osgood-Schlatter disease. Dislocated radial head, transient synovitis of the hip, and scoliosis are not considered overuse syndromes.

When examining the abdomen of a child, which technique would the nurse use last? A)Auscultation B)Percussion C)Palpation D)Inspection

C. Palpation. Palpation should be the last part of the abdominal examination. Inspection, auscultation, and percussion should be done before palpation.

The nurse is applying a urine bag to a 15-month-old boy to collect a urine specimen. Which action would the nurse take first? A)Apply benzoin to the scrotal area. B)Tuck the bag downward inside the diaper. C)Pat the perineal area dry after cleaning. D)Apply the narrow portion of the bag on the perineal space.

C. Pat the perineal area dry after cleaning When applying a urine bag, the nurse would first cleanse the perineal area well and pat it dry. If a culture was to be obtained, the nurse would cleanse the genital area with povidone-iodine or according to institutional protocol. Next the nurse would apply benzoin around the scrotum and allow it to dry. Then the nurse would apply the urine bag, making sure that the penis is fully inside the bag, tucking it downward inside the diaper to discourage leaking.

The nurse is reviewing the laboratory test results of a child diagnosed with disseminated intravascular coagulation (DIC). What would the nurse interpret as indicative of this disorder? A)Shortened prothrombin time B)Increased fibrinogen level C)Positive fibrin split products D)Increased platelets

C. Positive fibrin split products Laboratory test results associated with DIC include positive fibrin split products; prolonged prothrombin time, partial thromboplastin time, bleeding time, and thrombin time; decreased fibrinogen levels, platelets, clotting factors II, V, VIII, and X, and antithrombin III; and increased levels of fibrinolysin, fibrinopeptide A, and positive D-dimers.

A 6-year-old child has undergone a renal transplant and is receiving cyclosporine. The nurse instructs the parents to be especially alert for which complication? A)Weight loss B)Hypotension C)Signs of infection D)Hair loss

C. Signs of infection The parents should be especially alert for signs of infection as cyclosporine is an immunosuppressant drug. Weight gain instead of weight loss, hypertension instead of hypotension, and increased facial hair instead of hair loss are some other potential side effects.

The nurse is developing a teaching plan for a child who is to have his cast removed. What instruction would the nurse most likely include? A)Applying petroleum jelly to the dry skin B)Rubbing the skin vigorously to remove the dead skin C)Soaking the area in warm water every day D)Washing the skin with dilute peroxide and water

C. Soaking the area in warm water every day After a cast is removed, the child and family should be instructed to soak the area in warm water every day to help soften and remove the dry flaky skin. Moisturizing lotion, not petroleum jelly, should be applied to the skin. Vigorous rubbing would traumatize the skin and should be avoided. Warm soapy water, not dilute peroxide and water, should be used to wash the area.

An 18-month-old was brought to the emergency department by her mother, who states, "I think she broke her arm." The child is sent for a radiograph to confirm the fracture. Additional assessment of the child leads the nurse to suspect possible child abuse. Which type of fracture would the radiograph most likely reveal? A)Plastic deformity B)Buckle fracture C)Spiral fracture D)Greenstick fracture

C. Spiral fracture A spiral fracture is very rare in children. A spiral femoral or humeral fracture, particularly in a child younger than 2 years of age, should always be thoroughly investigated to rule out the possibility of child abuse. Plastic, buckle, and greenstick fractures are common in children and do not usually suggest child abuse.

An 8-year-old boy with a fractured forearm is to have a fiberglass cast applied. What information would the nurse include when teaching the child about the cast? A)The cast will take a day or two to dry completely. B)The edges will be covered with a soft material to prevent irritation. C)The child initially may experience a very warm feeling inside the cast. D)The child will need to keep his arm down at his side for 48 hours.

C. The child initially may experience a very warm feeling inside the cast. A fiberglass cast usually takes only a few minutes to dry and will cause a very warm feeling inside the cast. Therefore, the nurse needs to warn the child that this will occur. Fiberglass casts usually have a soft fabric edge so they usually do not cause skin rubbing at the edges and don't require petaling. The child should be instructed to elevate his arm above the level of the heart for the first 48 hours.

The mother of a child with end-stage renal disease asks the nurse why her son is getting an injection of erythropoietin. When responding to the mother, the nurse explains this as the rationale. A)To treat low calcium levels B)To stimulate growth in stature C)To stimulate red blood cell growth D)To correct acidosis

C. To stimulate red blood cell growth Erythropoietin is given to stimulate red blood cell growth. Vitamin D and calcium are used to correct hypocalcemia. Growth hormone is used to stimulate growth in stature. Bicitra or sodium bicarbonate tablets are used to correct acidosis.

Which finding would lead the nurse to suspect that a child is experiencing moderate dehydration? A)Dusky extremities B)Tenting of skin C)Sunken fontanels D)Hypotension

C. sunken fontanels A child with moderate dehydration would exhibit sunken fontanels. Severe dehydration would be characterized by dusky extremities, skin tenting, and hypotension.

A 12-year-old boy reports to the nurse that he is one of the shortest kids in his class. He asks the nurse if he will ever grow. What response by the nurse is most appropriate? A) "At your age you are largely done growing taller." B) "Since you are the shortest now, you will likely always be the shortest in the class." C) "Boys do not have their growth spurt until about age 17." D) "There is no way to know how tall you will grow because you are still well within the window for growth."

D

The adolescent continues to develop self-concept and self-esteem. What is most important to a teen's self-esteem? A) Strong authority figures B) Spirituality C) Morals and values D) Body image

D

The nurse is performing an assessment of the reproductive system of a 17-year-old girl. What would alert the nurse to a developmental delay in this girl? A) Areola and papilla separate from the contour of the breast B) Mature distribution and coarseness of pubic hair C) Developed breast tissue D) Occurrence of first menstrual period

D

The nurse is promoting nutrition to a 13-year-old boy who is overweight. Which comment should the nurse expect to include in the discussion? A) "You need to go on a low-fat diet." B) "Eat what your parents eat." C) "Go out for a sport at school." D) "Keep a food diary."

D

The nurse teaching safety to teens knows that which of these is the leading cause of death among adolescents? A) Drowning B) Poisoning C) Diseases D) Unintentional injuries

D

13. A nurse develops a plan of care for a child that includes patching the eye. This plan of care would be most appropriate for which condition? A) Astigmatism B) Hyperopia C) Myopia D) Amblyopia

D) Amblyopia Eye patching is used for amblyopia or any condition that results in one eye being weaker than the other. Corrective lenses would be appropriate for astigmatism, hyperopia, and myopia.

5. The nurse is instructing the parents of a school-age child with an eye disorder how to care for her eye. Which of the following conditions would the nurse explain as resolving by itself without the use of antibiotics? A) Blepharitis B) Hordeolum C) Corneal abrasion D) Chalazion

D) Chalazion A chalazion is caused by noninfectious meibomian gland occlusion, whereas a hordeolum usually is caused by infection.

A nurse is caring for a 14-year-old girl who received an electrical burn. The nurse would anticipate preparing the girl for which diagnostic tests as ordered? A) Pulse oximetry B) Fiberoptic bronchoscopy C) Xenon ventilation-perfusion scanning D) Electrocardiographic monitoring

D) Electrocardiographic monitoring Feedback: Electrocardiographic monitoring is important for the child who has suffered an electrical burn to identify possible cardiac arrhythmias, which can be noted for up to 72 hours after a burn injury. Fiberoptic bronchoscopy and xenon ventilation-perfusion scanning may be ordered to evaluate an inhalation injury, not an electrical burn. Pulse oximetry is used to evaluate pulmonary function and would not be indicated in the case of an electrical burn.

A 6-year-old boy has been admitted to the hospital with burns. The nurse notes carbonaceous sputum. What action would be the priority? A) Determining the burn depth B) Eliciting a description of the burn C) Estimating burn extent D) Ensuring a patent airway

D) Ensuring a patent airway Feedback: Carbonaceous sputum is a sign of potential airway injury due to smoke inhalation. Therefore, the nurse should ensure a patent airway while obtaining a brief history and simultaneously evaluating the child and providing emergency care. If the burn does not pose an immediate, life-threatening risk, the nurse would obtain an in-depth history and elicit a description of the burn. Determining the burn depth and extent are part of the secondary survey.

19. A nurse is examining a child who has sustained blunt trauma to the eye area. The nurse suspects a simple contusion based on which of the following? A) Pain in the eye B) Impaired visual acuity C) Blurred vision D) Intact extraocular movements

D) Intact extraocular movements A simple contusion of the eye area is manifested by bruising and edema of the lids or surrounding eye area, intact extraocular eye movement, intact visual acuity, absence of diplopia or blurred vision, pain surrounding the eye but not within the eye, and pupils that are equal, are round, and react to light and accommodation.

6. The nurse is caring for a 4-year-old boy with infectious conjunctivitis. Which intervention would be least appropriate to include in the child's plan of care? A) Rinsing the eye with cool water B) Educating the family about the disease C) Encouraging frequent hand washing D) Promoting eye safety

D) Promoting eye safety Promoting eye safety would be appropriate if the child had an eye injury. Rinsing the eye with cool water, educating the family about the disorder, and encouraging frequent hand washing are interventions for infectious conjunctivitis.

A child is diagnosed with atopic dermatitis. Which laboratory test would the nurse expect the child to undergo to provide additional evidence for this condition? A) Erythrocyte sedimentation rate B) Potassium hydroxide prep C) Wound culture D) Serum immunoglobulin E (IgE) level

D) Serum immunoglobulin E (IgE) level Feedback: IgE levels are often used to evaluate for atopic dermatitis. IgE levels are elevated in this condition. Erythrocyte sedimentation rate may be used but this test is nonspecific and only indicates infection or inflammation. Potassium hydroxide prep is used to identify fungal infections. Wound culture would be done to identify a specific organism if an infection occurs with atopic dermatitis.

As part of a clinical conference with a group of nursing students, the instructor is describing the burn classification. The instructor determines that the teaching has been successful when the group identifies what as characteristic of full thickness burns? A) Skin that is reddened, dry, and slightly swollen B) Skin appearing wet with significant pain C) Skin with blistering and swelling D) Skin that is leathery and dry with some numbness

D) Skin that is leathery and dry with some numbness Feedback: Full thickness burns may be very painful, numb, or pain-free in some areas. They appear red, edematous, leathery, dry, or waxy and may display peeling or charred skin. Superficial burns are painful, red, dry, and possibly edematous. Partial thickness and deep partial thickness burns are very painful and edematous and have a wet appearance or blisters.

26. A group of students are reviewing information about the anatomic differences in the eyes and ears of a child in comparison to an adult. The students demonstrate a need for additional study when they identify which of the following? A) Hearing is completely developed at the time of birth. B) Visual acuity develops from birth throughout childhood. C) Binocular vision is usually achieved by 2 months of age. D) The ability to discriminate colors is completed by birth.

D) The ability to discriminate colors is completed by birth. The optic nerve is not completely myelinated at birth, so color discrimination is incomplete. Hearing is intact at birth and visual acuity develops from birth throughout childhood. Binocular vision is achieved by 4 months of age.

The mother of a 15-month-old child is questioning the nurse about the need for the hepatitis B vaccination. Which comment provides the most compelling reason for the vaccine? A)"The most common side effect is injection site soreness." B)"This is a recombinant or genetically engineered vaccine." C)"Immunizations are needed to protect the general population." D)"This protects your child from infection that can cause liver disease."

D)"This protects your child from infection that can cause liver disease." Up to 90% of neonates infected with hepatitis B develop chronic carrier status and will be predisposed to cirrhosis and hepatic cancer. The mother is not questioning side effects, safety, or disease prevention in general. Therefore, it is best to speak to her concerns.

The nurse is performing a health history on a 6-year-old boy who is having trouble adjusting to school. Which question would be most likely to elicit valuable information? A)'Do you like your new school?' B)'Are you happy with your teacher?' C)'Do you enjoy reading a book?' D)'What are your new classmates like?'

D)'What are your new classmates like?' A careful conversation and interview with the child and/or the caregiver will provide important information about the child's health. Depending on the intent of the health assessment, many of the questions will be direct, and many will require the caregiver or child to answer simply "yes" or "no." In other than emergency situations, though, asking open-ended questions such as 'What are your classmates like?' offers an excellent opportunity to learn more about the child's life.

The nurse is assessing the heart rate of a healthy school-age child. The nurse expects that the child's heart rate will be in what ranges? A)80 to 150 bpm B)70 to 120 bpm C)65 to 110 bpm D)60 to 100 bpm

D)60 to 100 bpm The normal heart rate for a school-age child is 60 to 100 bpm, for an infant is 80 to 150 bpm, for a toddler is 70 to 120 bpm, and for a preschooler is 65 to 110 bpm.

The nurse is developing a teaching plan for toddler safety to present at a parenting seminar. Which safety intervention should the nurse address? A)Encourage parents to enroll toddlers in swimming classes to avoid the need for constant supervision around water. B)Advise parents to keep pot handles on stoves turned outward to avoid accidental burns. C)Encourage parents to smoke only in designated rooms in the house or outside the house. D)Advise parents to use a forward-facing car seat with harness straps and a clip, placed in the backseat of the car.

D)Advise parents to use a forward-facing car seat with harness straps and a clip, placed in the backseat of the car. Safety is of prime concern throughout the toddler period. The safest place for the toddler to ride is in the back seat of the car. Parents should use the appropriate size and style of car seat for the child's weight and age as required by the state. At a minimum, all children over 20 pounds and up to 40 pounds should be in a forward-facing car seat with harness straps and a clip. Parents who want to enroll a toddler in a swimming class should be aware that a water safety skills class would be most appropriate. However, even toddlers who have completed a swimming program still need constant supervision in the water. Pot handles on stoves should be turned inward to avoid accidental burn. Nurses should counsel parents to stop smoking (optimal), but if they continue smoking never to smoke inside the home or car with children present.

The nurse is using pulse oximetry to measure oxygen saturation in a 3-year-old girl. The nurse understands that falsely high readings may be associated with which situation or condition? A)A nonsecure connection B)Cold extremities C)Hypovolemia D)Anemia

D)Anemia Falsely high readings may be associated with anemia. Falsely low readings may be associated with cold extremities, hypovolemia, and a nonsecure connection.

A 3-year-old child is scheduled for a hearing screening. The nurse would prepare the child for screening by which method? A)Auditory brain stem response B)Evoked otoacoustic emissions C)Visual reinforcement audiometry D)Conditioned play audiometry

D)Conditioned play audiometry For children between the ages of 2 and 4 years, conditioned play audiometry would be an appropriate method for hearing screening. Auditory brain stem response and evoked otoacoustic emissions are appropriate hearing screening methods for newborns through age 6 months. Visual reinforcement audiometry is appropriate for children ages 6 months to 2 years.

The nurse is teaching the parents of a 2-year-old girl how to deal with common toddler situations. Which is the best advice? A)Discipline the child for regressive behavior. B)Scold the child for public thumb sucking. C)Tell the older sibling to not act like a baby. D)Have the child help clean up a bowel accident.

D)Have the child help clean up a bowel accident. Having the child help clean up a bowel accident is the best advice. Toddlers should never be punished for bowel or bladder "accidents," but gently reminded about toileting. Regressive behavior is best ignored, while appropriate behavior should be praised. Telling the older sibling to not act like a baby is a negative approach. It would be better to have the child be mother's helper. Calmly telling the child that thumb sucking is something that is done at home is better than scolding the child.

The nurse is designing a nursing care plan for a toddler with lymphoma, who is hospitalized for treatment. What is a priority intervention that the nurse should include in this child's nursing plan? A)Limiting visitors to scheduled visiting hours B)Planning physical therapy for the child C)Introducing the toddler to other toddlers in the unit D)Monitoring the toddler for developmental delays

D)Monitoring the toddler for developmental delays When the toddler is hospitalized, growth and development may be altered. The toddler's primary task is establishing autonomy, and the toddler's focus is mobility and language development. The nurse caring for the hospitalized toddler must use knowledge of normal growth and development to be successful in interactions with the toddler, promote continued development, and recognize delays. Parents should be encouraged to stay with the toddler to avoid separation anxiety. Planning activities and socialization of the toddler is important, but the priority intervention is monitoring for, and addressing, developmental delays that may occur in the hospital.

The nurse is teaching the student nurse about abnormal findings when assessing the breasts of children. What may be associated with renal disorders? A)Swollen nipples upon inspection of a newborn's breasts B)Tender nodule palpated under the nipple of a 10-year-old C)Observation of enlarged breast tissue in a male adolescent D)Observation of a supernumerary nipple along the mammary ridge

D)Observation of a supernumerary nipple along the mammary ridge Supernumerary nipples are usually of no concern as they do not change over time, but they may be associated with renal disorders. Newborns of both genders may have swollen nipples from the influence of maternal estrogen, but by several weeks of age the nipples should be flat. A tender nodule palpated just under the nipple confirms pubertal changes and is a normal finding. Adolescent boys may develop gynecomastia (enlargement of the breast tissue) due to hormonal pubertal changes. When the hormone levels stabilize, male adolescents then have flat nipples.

The nurse is providing anticipatory guidance to an obese teenager. Which intervention would be most likely to promote healthy weight in teenagers? A)Make the focus of the program weight centered. B)Begin directly advising children about their weight at age 6. C)Focus physical activity on competitive sports and activities. D)Obtain nutritional histories directly from the school-age child and adolescent.

D)Obtain nutritional histories directly from the school-age child and adolescent. Before providing education to school-age and teenage children, it is important to obtain nutritional histories directly from them because increasingly they are eating meals away from the family table. The focus of healthy weight promotion should be health centered, not weight centered. Linking success to numbers on a scale increases the possibility of developing eating disorders, nutritional deficiencies, and body hatred. The nurse can begin directly advising children on healthy foods starting at age 3. The focus of physical activity should be on noncompetitive, fun activities.

A mother and her 4-week-old infant have arrived for a health maintenance visit. Which activity will the nurse perform? A)Assess the child for an upper respiratory infection B)Take a health history for a minor injury C)Administer a varicella injection D)Plot the child's head circumference on a growth chart

D)Plot the child's head circumference on a growth chart The nurse will plot the head circumference of the child as part of developmental surveillance and screening. Assessing for an infection and taking a health history for an injury are not part of a health maintenance visit. Administering a vaccination for varicella would not occur until 12 months of age.

The nurse is administering a hepatitis B vaccine to a child. What is the classification of this type of vaccine? A)Killed vaccines B)Toxoid vaccines C)Conjugate vaccines D)Recombinant vaccines

D)Recombinant vaccines Recombinant vaccines use genetically engineered organisms. The hepatitis B vaccine is produced by splicing a gene portion of the virus into a gene of a yeast cell. The yeast cell is then able to produce hepatitis B surface antigen to use for vaccine production. Killed vaccines contain whole dead organisms; they are incapable of reproducing but are capable of producing an immune response. Toxoid vaccines contain protein products produced by bacteria called toxins. The toxin is heat-treated to weaken its effect, but it retains its ability to produce an immune response. Conjugate vaccines are the result of chemically linking the bacterial cell wall polysaccharide (sugar-based) portions with proteins.

The nurse is examining a 2-year-old child who was adopted from Guatemala. What would be a priority screening for this child? A)Screening for congenital defects B)Screening for abuse C)Screening for childhood illnesses D)Screening for infectious diseases

D)Screening for infectious diseases Although all the screenings are important, health supervision of the internationally adopted child must include comprehensive screening for infectious disease. In 2008, approximately 19,600 children were adopted from countries outside the United States, many from areas with a high prevalence of infectious diseases (Intercountry Adoption, Office of Children's Issues, U.S. Department of State, 2010a, 2010b). Guatemala, China, and Russia supplied about half of all international adoptees in 2008, followed by Ethiopia, South Korea, and Vietnam. Proper screening is important not only to the child's health but also to the adopting family and the larger community.

The nurse is choosing foods for a toddler's diet that are high in vitamin A. What foods could be added to the menu? Select all that apply. A)Applesauce B)Avocados C)Broccoli D)Sweet potatoes E)Spinach F)Carrots

D)Sweet potatoes E)Spinach F)Carrots Foods that are high in vitamin A include apricots, cantaloupe, carrots, mangos, spinach and dark greens, and sweet potatoes. Applesauce is high in fiber, and avocados and broccoli are high in folate.

The nurse emphasizes that a toddler younger than the age of 18 months should never be spanked primarily for which reason? A)Spanking in a child this age predisposes the child to a pro-violence attitude. B)The child will become resentful and angry, leading to more outbursts. C)Spanking demonstrates a poor model for problem-solving skills. D)There is an increased risk for physical injury in this age group.

D)There is an increased risk for physical injury in this age group. Spanking should never be used with toddlers younger than 18 months of age because there is an increased possibility of physical injury. Although spanking or other forms of corporal punishment lead to a pro-violence attitude, create resentment and anger in the child, and are a poor model for learning effective problem-solving skills, the risk of physical injury in this age group is paramount.

While providing care to a 5-month-old girl whose family has a history of food allergies, the nurse instructs the parents about foods to be avoided in the first year of life. Which response by the parents indicates a need for further teaching? A)"She cannot have any cow's milk." B)"I should continue breastfeeding until at least 6 months." C)"Peanuts in any form should be avoided." D)"Any kind of fruit is acceptable."

D. "Any kind of fruit is acceptable." The nurse should caution the parents that kiwifruit should be avoided. Other foods to avoid include cow's milk, eggs, peanuts, tree nuts, sesame seeds, fish, and shellfish. Breastfeeding also is recommended for at least the first 6 months.

The nurse is caring for a 4-year-old boy who has undergone an appendectomy. The child is unwilling to use the incentive spirometer. Which approach would be most appropriate to elicit the child's cooperation? A)"Can you cough for me please?" B)"You must blow in this or you might get pneumonia." C)"If you don't try, I will have to get the doctor." D)"Can you blow this cotton ball across the tray?"

D. "Can you blow this cotton ball across the tray?" Children are more likely to cooperate with interventions if play is involved. Encourage deep breathing by playing games. Asking the boy to cough is less likely to engage him. Telling the child he might get pneumonia is not age appropriate and is unhelpful. Threatening to call the doctor is unhelpful and inappropriate. Remember, however, that the incentive spirometer works on the principle of the amount of air inhaled, not exhaled. Having the child take a deep breath prior to blowing the cotton ball is a beginning step.

The mother of a 5-year-old child with allergies to a variety of foods including eggs, milk, peanuts and shellfish, asks if her child will "always have these problems". What response by the nurse is most accurate? A)"Sadly, allergies to foods will persist." B)"Most children with allergies will outgrow them." C)"We cannot be sure at this point but most children who are allergic to peanuts will not have this allergy in adulthood." D)"In most cases allergies to peanuts and shellfish persist into adulthood but the others may diminish and disappear."

D. "In most cases allergies to peanuts and shellfish persist into adulthood but the others may diminish and disappear." Foods such as peanuts, milk, soy, shellfish, tree nuts are common allergens. By adulthood many allergies will diminish or disappear. Allergies to shellfish, peanuts and tree nuts often persist into adulthood.

The nurse is caring for a child recently diagnosed with glucose-6-phosphate dehydrogenase (G6PD) deficiency. The nurse is teaching the parents about triggers that may result in oxidative stress. Which response indicates a need for further teaching? A)"I doubt he will ever eat fava beans, but they could trigger hemolysis." B)"He must avoid exposure to naphthalene, an agent found in mothballs." C)"He must never take methylene blue for a urinary tract infection." D)"My son can never take penicillin for an infection."

D. "My son can never take penicillin for an infection." The nurse should emphasize that penicillin is not a known trigger that may result in oxidative stress and hemolysis. Fava beans, naphthalene, and methylene blue can trigger oxidative stress.

The nurse is caring for a 2-year-old boy with hemophilia. His parents are upset by the possibility that he will become infected with hepatitis or HIV from the clotting factor replacement therapy. Which response by the nurse would be most appropriate? A)"Parents commonly fear the worst; however, the factor will help your child lead a normal life." B)"There are risks with any treatment including using blood products, but these are very minor." C)"Although factor replacement is expensive, there's more financial strain from missing work if he has a bleeding episode." D)"Since dry heat treatment of the factor began in 1986, there have been no reports of virus transmission."

D. "Since dry heat treatment of the factor began in 1986, there have been no reports of virus transmission." The nurse needs to emphasize that since 1986, there have been no reports of virus transmission from factor infusion since the inception of heat treatment of the factor. Telling the parents that there is a minor risk does not teach. Telling the parents that factor is expensive or that it is common to worry does not teach, nor does it address their concerns.

A 7-year-old child with a family history of cardiovascular disease is being screened for hyperlipidemia. When reviewing the child's laboratory test results, which total cholesterol level would be of significant concern? A)120 mg/dL B)150 mg/dL C)180 mg/dL D)210 mg/dL

D. 210 mg/dL A total cholesterol level greater than 200 mg/dL is considered high and would be of the greatest concern. Levels of 120 mg/dL and 150 mg/dL are considered within the normal range. A level of 180 mg/dL would be considered borderline and significant. However, a level greater than 200 mg/dL would be of greater concern.

After teaching the parents of a 6-year-old how to administer an enema, the nurse determines that the teaching was successful when they state that they will give how much solution to their child? A)100 to 200 mL B)200 to 300 mL C)250 to 500 mL D)500 to 1,000 mL

D. 500-1000 For a school-age child, typically 500 to 1,000 mL of enema solution is given. For an infant, 250 mL or less is used; for a toddler or preschooler, 250 to 500 mL is used.

The nurse is administering digoxin as ordered and the child vomits the dose. What should the nurse do next? A)Contact the physician. B)Offer a snack and administer another dose. C)Immediately administer another dose. D)Administer next dose as ordered in 12 hours.

D. Administer next dose as ordered in 12 hours Digoxin should be administered at regular intervals, every 12 hours, 1 hour before or 2 hours after feeding. If the child vomits digoxin, the nurse should not give a second dose and should wait until the next scheduled dose. It is not necessary to contact the physician.

A group of students are reviewing information about renal failure in children. The students demonstrate a need for additional teaching when they identify which agent as a potential contributor to renal failure? A)Vancomycin B)Gentamicin C)Co-trimoxazole D)Amoxicillin

D. Amoxicillin Amoxicillin is a penicillin and is not associated with nephrotoxicity leading to renal failure. Vancomycin, gentamicin (an aminoglycoside), and co-trimoxazole (a sulfonamide) are nephrotoxic.

A group of nursing students are reviewing the various drug classes used for cancer chemotherapy. The students demonstrate an understanding of these classes when they identify which agent as an example of a nitrosourea? A)Busulfan B)Thiotepa C)Cisplatin D)Carmustine

D. Carmustine Carmustine is an example of a nitrosourea. Busulfan, thiotepa, and cisplatin are alkylating agents.

A child with cerebral palsy has undergone surgery for placement of a baclofen pump. Which instruction would the nurse include when teaching the parents about caring for their child? A)Waiting 48 hours before allowing the child to take a tub bath B)Not allowing the child to sleep on his side for about 4 weeks C)Calling the physician if the child's temperature is over 100.5°F D)Discouraging the child from stretching or bending forward for 4 weeks

D. Discouraging the child from stretching or bending forward for 4 weeks. After insertion of a baclofen pump, the parents should discourage any twisting at the waist, reaching high overhead, stretching, or bending forward or backward for 4 weeks. The child would avoid tub baths for about 2 weeks and avoid sleeping on his stomach for 4 weeks. The parents should notify the physician or nurse practitioner if the child's temperature is greater than 101.5°F.

After teaching a class of nursing students about muscular dystrophy, the instructor determines that the teaching was successful when the students identify which type of muscular dystrophy as demonstrating an X-linked recessive pattern of inheritance? A)Limb-girdle B)Myotonic C)Distal D)Duchenne

D. Duchenne Duchenne muscular dystrophy follows an X-linked recessive inheritance pattern. Limb-girdle muscular dystrophy is believed to be autosomal or X-linked inherited. Myotonic and distal muscular dystrophy follow an autosomal dominant inheritance pattern.

Which test result would the nurse least likely expect to find in a child diagnosed with Wilms tumor? A)Complete blood count (CBC) within normal limits B)Urinalysis positive for blood C)Mass on kidney D)Elevated homovanillic acid (HVA) with 24-hour urine collection

D. Elevated homovanillic acid (HVA) with 24 hr urine collection Levels of HVA and vanillylmandelic acid (VMA) will not be elevated with Wilms tumor; they are elevated with neuroblastoma. CBC, blood urea nitrogen (BUN), and creatinine usually are within normal limits. Urinalysis may reveal hematuria or leukocytes. Renal or abdominal ultrasound would reveal a mass on the kidney.

The parents of a child diagnosed with celiac disease ask the nurse what types of food they can offer their child. What recommendation would the nurse include in the teaching plan? A)Frozen yogurt B)Rye bread C)Creamed spinach D)Fruit juice

D. Fruit juice. For the child with celiac disease, foods containing gluten such as frozen yogurt, rye bread, and creamed vegetables should be avoided. Fruit juice would be an appropriate suggestion in a gluten-free diet.

An 8-year-old girl is scheduled for a renal ultrasound. What would the nurse include in the plan of care when preparing the child for this test? A)Withholding food and fluids after midnight B)Checking the child for allergies to shellfish C)Ensuring the child has a full bladder D)Informing the child she should feel no discomfort

D. Informing the child she should feel no discomfort The nurse should inform the child that she should feel no discomfort during the test. No fasting is required and no dye is used, so allergies are not a concern. A full bladder is needed for urodynamic studies.

The nurse is caring for a child who has been admitted for a sickle cell crisis. What would the nurse do first to provide adequate pain management? A)Administer a nonsteroidal anti-inflammatory drug (NSAID) as ordered. B)Use guided imagery and therapeutic touch. C)Administer meperidine as ordered. D)Initiate pain assessment with a standardized pain scale.

D. Initiate pain assessment with a standardized pain scale. The nurse should first initiate pain assessment with a standardized pain scale upon admission and provide frequent evaluations of pain. Administering NSAIDs or meperidine and the use of nonpharmacologic pain management techniques are all appropriate. However, the first action is to assess the child's pain to provide a baseline for future comparison.

The nurse has developed a plan of care for a 12-month-old hospitalized with dehydration as a result of rotavirus. Which intervention would the nurse include in the plan of care? A)Encouraging consumption of fruit juice B)Offering Kool-Aid or popsicles as tolerated C)Encouraging milk products to boost caloric intake D)Maintaining the intravenous (IV) fluid rate as ordered

D. Maintaining the intravenous (IV) fluid rate as ordered. The nurse should maintain an IV line and administer the IV fluid as ordered to maintain fluid volume. High-carbohydrate fluids like fruit juice, Kool-Aid, and popsicles should be avoided as they are low in electrolytes, increase simple carbohydrate consumption, and can decrease stool transit time. Milk products should be avoided during the acute phase of illness as they may worsen diarrhea.

The nurse is caring for a 7-year-old girl who is undergoing a stem cell transplant. What information would the nurse include in the child's postoperative plan of care? A)Assessing for petechiae, purpura, bruising, or bleeding B)Limiting blood draws to the minimum volume required C)Administering antiemetics around the clock as ordered D)Monitoring for severe diarrhea and maculopapular rash

D. Monitoring for severe diarrhea and maculopapular rash In the posttransplant phase, monitor closely for symptoms of graft-versus-host disease (GVHD) such as severe diarrhea and maculopapular rash progressing to redness or desquamation of the skin (especially on the palms of the hands or soles of the feet). During chemotherapy in the pretransplant phase, assess for petechiae, purpura, bruising, or bleeding to prevent hemorrhage; administer antiemetics around the clock as ordered to prevent the cycle of nausea, vomiting, and anorexia; and limit blood draws to the minimum volume required to prevent anemia.

A nursing instructor is developing a class presentation about the medications used to treat peptic ulcer disease. Which drug class would the instructor be least likely to include in the presentation? A)Antibiotics B)Proton pump inhibitors C)Histamine antagonists D)Prokinetics

D. Prokinetics Treatment for peptic ulcer disease includes antibiotics if Helicobacter pylori is verified, histamine antagonists, and/or proton pump inhibitors. Prokinetics are used to stimulate the gastrointestinal tract to help empty the stomach faster and promote intestinal motility. They are not used for peptic ulcer disease.

The nurse has developed a plan of care for a 6-year-old with muscular dystrophy. He was recently injured when he fell out of bed at home. Which intervention would the nurse suggest to prevent further injury? A)Recommend the bed's side rails be raised throughout the day and night. B)Suggest a caregiver be present continuously to prevent falls from bed. C)Encourage a loose restraint to be used when he is in bed. D)Recommend raising the bed's side rails when a caregiver is not present.

D. Recommend raising the bed's side rails when a caregiver is not present. The nurse should recommend that side rails on the bed be elevated when a caregiver is not present. The use of restraints should be avoided if at all possible. Suggesting that a caregiver be present at all times places undue stress on the family. Close observation is more appropriate. Recommending side rails be elevated at all times may be upsetting to the child and make him feel like a "baby."

The nurse is caring for a 16-year-old boy with acute myelogenous leukemia who is having chemotherapy and who has incomplete records for varicella zoster immunization. Which is the priority nursing diagnosis? A)Pain related to adverse effects of treatment verbalized by the child B)Nausea related to side effects of chemotherapy verbalized by the child C)Constipation related to the use of opioid analgesics for pain D)Risk for infection related to neutropenia and immunosuppression

D. Risk for infection related to neutropenia and immunosuppression. The priority nursing diagnosis is risk for infection related to neutropenia and immunosuppression. The incomplete records for varicella zoster immunization can cause a problem since exposure to chickenpox could cause sepsis, so the nurse should contact the oncologist for approval to administer the vaccine. Certain vaccines are not administered when the child is immunosuppressed, so timing is crucial. Diagnoses for pain and nausea are valid for this child because he is undergoing chemotherapy, but they are not a priority. Likewise, the need for constipation management would not be necessary unless opioid use begins.

The nurse is developing a teaching plan for the parents of a child with a myelomeningocele who will require clean intermittent catheterization. What information would the nurse include? A)Applying petroleum jelly to lubricate the catheter B)Cleaning the reusable catheter with peroxide after each use C)Storing the reusable cleaned catheter in a brown paper bag D)Soaking the catheter in a vinegar and water solution to sterilize

D. Soaking the catheter in a vinegar and water solution to sterilize When teaching parents how to perform clean intermittent catheterization, the nurse would instruct the parents to apply a water-based lubricant to the catheter, clean the reusable catheter with soap and water after each use, store the reusable clean catheter in a zip-top bag or other clean storage container, and soak the catheter in a 1:1 vinegar and water solution for about 30 minutes weekly, rinsing well before the next use or placing the catheter in boiling water for 10 minutes.

The nurse is providing postsurgical care for an infant who has undergone a hypospadias repair. Which action by the nurse would be most important to help keep the area clean while maintaining proper position of the drainage tubing? A)Keeping the drainage tube taped in an upright position B)Administering antibiotics as ordered C)Administering analgesics as prescribed D)Using a double-diapering technique

D. Using a double-diapering technique Double diapering is a method used to protect a child's urethra and stent or catheter after surgery and additionally helps to keep the area clean and free from infection. Keeping the drainage tube taped in an upright position, administering antibiotics, and administering analgesics are also important, but double diapering keeps the area clean and helps prevent infection.

The nurse is caring for a newborn whose mother is HIV positive. The nurse would expect to administer a 6-week course of which medication? A)Lopinavir B)Ritonavir C)Nevirapine D)Zidovudine

D. Zidovudine Children born to HIV-positive mothers should receive a 6-week course of zidovudine therapy. Lopinavir, ritonavir, and nevirapine are medications used for treatment of HIV-1 infections as part of a three-drug regimen.

A child is diagnosed with juvenile idiopathic arthritis and is receiving several different medications listed in the medication administration record. Which agent would the nurse identify as being used to prevent disease progression? A)Aspirin B)Prednisone C)Ibuprofen D)Methotrexate

D. methotrexate Disease-modifying antirheumatic drugs (DMARDs), such as methotrexate, are necessary to prevent disease progression. Other agents, such as aspirin and ibuprofen, are helpful with pain relief. Prednisone helps for relief of inflammation.

A group of students are reviewing information about fluid balance and losses in children in comparison to adults. The students demonstrate a need for additional review when they state that: A)children have a proportionately greater amount of body water than do adults. B)fever plays a greater role in insensible fluid losses in infants and children. C)a higher metabolic rate plays a major role in increased insensible fluid losses. D)the infant's immature kidneys have a tendency to over concentrate urine.

D. the infant's immature kidneys have a tendency to over concentrate urine. The young infant's renal immaturity does not allow the kidneys to concentrate urine as well as in older children and adults, placing them at risk for dehydration or over hydration. Children do have a proportionately greater amount of body water than adults, and fever is important in promoting insensible fluid losses in infants and children because children become febrile more readily and their fevers are higher than those in adults. Children also experience a higher metabolic rate, which accounts for increased insensible fluid losses and increased need for water for excretory function.

A nurse is talking with the parents of a child who has had a febrile seizure. The nurse would integrate an understanding of what information into the discussion? A) The child's risk for cognitive problems is greatly increased. B) Structural damage occurs with febrile seizure. C) The child's risk for epilepsy is now increased. D) Febrile seizures are benign in nature.

Febrile seizures are benign in nature. Parents need reassurance that febrile seizures, although frightening, are benign in nature. Children who experience one or more febrile seizures are at no greater risk of developing epilepsy than the general population. No evidence exists that febrile seizures cause structural damage or cognitive declines.

As a result of seizure activity, a computed tomography (CT) scan was performed and showed that an 18-month-old child has intracranial arteriovenous malformation. When developing the child's plan of care, what would the nurse expect to implement actions to prevent? A) Drug interactions B) Developmental disabilities C) Hemorrhagic stroke D) Respiratory paralysis

Hemorrhagic stroke Intracranial hemorrhage or hemorrhagic stroke is a risk for children with intracranial arteriovenous malformation. Drug interactions are a risk for children who are treated with combinations of anticonvulsants for epilepsy. Children with hydrocephalus are at an increased risk for developmental disabilities. Respiratory paralysis is a risk of botulism that typically affects infants younger than 6 months of age.

The nurse is caring for a child hospitalized with Reye syndrome who is in the acute stage of the illness. The nurse would assess the child most carefully for what finding? A) Indications of increased intracranial pressure B) An increase in the blood glucose level C) A decrease in the liver enzymes D) A presence of protein in the urine

Indications of increased intracranial pressure Reye syndrome is characterized by brain swelling, liver failure, and death in hours if treatment is not initiated. Therefore, increased intracranial pressure could occur. Liver enzyme levels typically increase. Blood glucose levels and protein in the urine are not characteristic of this illness.

The nurse assesses a child's level of consciousness, noting that the child falls asleep unless he is stimulated. The nurse documents this finding as: A) Confusion B) Obtunded C) Stupor D) Coma

Obtunded Obtunded is a state in which the child has limited responses to the environment and falls asleep unless stimulation is provided. Confusion involves disorientation; the child may be alert but responds inappropriately to questions. Stupor exists when the child responds only to vigorous stimulation. Coma is a state in which the child cannot be aroused even with painful stimuli.

During class, a student states, "I didn't think children could have strokes. I thought this only occurred in older adults." When responding to the student, what would be most important for the instructor to integrate into the response? A) Strokes in children often have an identifiable cause. B) The signs and symptoms in children are different from an adult. C) Research has identified specific treatments for children. D) Ischemic strokes are more common than hemorrhagic strokes.

Ischemic strokes are more common than hemorrhagic strokes. In children, ischemic strokes are more common than hemorrhagic strokes. However, the cause of the stroke in many children remains unidentified. Signs and symptoms are similar to those in adults and will vary based on age; underlying cause, if known; and location of the stroke. Historically, children have been excluded from adult stroke studies and thus, many treatments used have had to be adapted from adult studies.

The physician has ordered rectal diazepam for a 2-year-old boy with status epilepticus. Which instruction is essential for the nurse to teach the parents? A) Monitor their child's level of sedation. B) Watch for fever indicating infection. C) Gradually reduce the dosage as seizures stop. D) Monitor for an allergic reaction to the medication.

Monitor their child's level of sedation. Diazepam is useful for home management of prolonged seizures and requires that the parents be educated on its proper administration. Monitoring the child's level of sedation is key when giving diazepam because it slows the central nervous system. Parents need to monitor the overall health of the child, including temperature when needed, but that has nothing to do with the diazepam. When the use of an anticonvulsant is stopped, gradual reduction of the dosage is necessary to prevent seizures or status epilepticus. This is not done without a physician's order. Monitoring for allergic reactions is necessary when any medications have been prescribed, but is not specific to diazepam.

A group of nursing students are reviewing information related to seizures that occur in infants and children. The students demonstrate a need for additional review when they identify which type as common in neonates? A) Tonic B) Focal clonic C) Multifocal clonic D) Myoclonic

Myoclonic Five major types of seizures have been recognized in the neonatal period: subtle, tonic, focal clonic, multifocal clonic, and myoclonic. Of these, myoclonic seizures rarely occur during the neonatal period. Subtle seizures affect preterm and full-term neonates. Tonic seizures primarily occur in preterm neonates. Focal clonic and multifocal clonic are more common in full-term neonates.

A nurse is preparing a school-aged child for a lumbar puncture. The nurse would expect to position the child in which manner? A) On her side with the head flexed forward and knees flexed to the abdomen B) Sitting upright with the head flexed forward to the chest C) Supine with arms and legs pronated and extended D) Prone with the arms flexed under the chest

On her side with the head flexed forward and knees flexed to the abdomen When a lumbar puncture is performed on a child, the child is placed on his or her side with the head flexed forward and knees flexed to the abdomen. An infant would be positioned sitting upright with the head flexed forward. A supine position with the arms and legs pronated and extended suggests decerebrate posturing. A prone position is not used for a lumbar puncture.

The nurse has developed a teaching plan for the family of a 2-year-old boy who holds his breath when he gets frustrated. What will be most important to include in this plan? A) Provide cuddle time whenever the child begins to act out. B) Explain the child's behavior to the parents. C) Encourage the parents to interact more with the child. D) Stay close to prevent injury when he gets frustrated.

Stay close to prevent injury when he gets frustrated. Encourage the parents to maintain a safe environment when an episode is occurring, but to avoid giving extra attention to the child after the event since this could encourage repetition of the behavior. It is important for the parents to understand what is happening, but rewarding the child with cuddle time when he is misbehaving provides incorrect reinforcement of behaviors. Encouraging the parents to interact more with the child may be helpful, but the priority is safety for the child.

A nurse is preparing a presentation for a local health fair about meningitis and has developed a display that lists the following causes: Streptococcus group B Haemophilus influenzae type B Streptococcus pneumoniae Neisseria meningitidis What would the nurse highlight as the most common cause of meningitis in newborns? A) Streptococcus group B B) Haemophilus influenzae type B C) Streptococcus pneumoniae D) Neisseria meningitides

Streptococcus group B Meningitis due to Streptococcus group B along with Escherichia coli is most common in newborns and infants. H. influenzae type B is a common cause in infants between the ages of 6 and 9 months. S. pneumoniae and N. meningitides are common causes in children older than 3 months and in adults.

The nurse inspects the eyes of a child and observes that the sclera is showing over the top of the iris. The nurse documents this finding as: A) Decorticate posturing B) Nystagmus C) Doll's eye D) Sunsetting

Sunsetting Sunsetting is when the sclera of the eyes is showing over the top of the iris. Decorticate posturing includes adduction of the arms, flexion at the elbows with the arms held over the chest, and flexion of the wrists with both hands fisted and the lower extremities adducted and extended. Nystagmus is manifested by involuntary rapid rhythmic eye movements. Doll's eye is a maneuver that tests for symmetric eye movement to the opposite side when the head is turned in the other direction.

The nurse is caring for an 8-year-old boy who has chronic epilepsy. What would be most important to address when teaching the child and parents about living with this condition? A) Multiple corrective surgeries to slowly remove diseased parts of his brain B) Physical, occupational, and speech therapy to maximize his potential C) Support for maintaining self-esteem because of his altered lifestyle D) Hyperventilation therapy to counteract the periods of decreased oxygenation

Support for maintaining self-esteem because of his altered lifestyle The effects of living with a seizure disorder can be devastating, and it is essential for the child to receive support to maintain self-esteem. While corrective surgery is possible, it would only be performed once. Physical, occupational, speech, and hyperventilation therapy are not indicated for treatment of epilepsy.

A 10-year-old boy is seen in the emergency department after falling down a flight of stairs and hitting his head. The child will be monitored overnight for complications. Which occurrence in the coming hours will warrant further assessment? A) The child reports a backache. B) The child is increasingly irritable with his mother and caregivers. C) The child refuses offers of snacks. D) The child reports his stomach is upset.

The child is increasingly irritable with his mother and caregivers. After a head injury the patient should be closely observed for neurological changes. Behavioral changes such as lethargy and irritability should be evaluated for the potential development of complications.

During a well-child visit, the nurse assesses an infant's ability to suck on a pacifier. The nurse is assessing which cranial nerve? A) Olfactory B) Trigeminal C) Facial D) Accessory

Trigeminal To test the trigeminal nerve, the nurse would note the strength of the infant's suck on a pacifier, thumb, or bottle. The olfactory nerve is not assessed in infants and young children. The facial nerve is assessed by noting the symmetry of facial expressions. For the infant, this would be assessed during spontaneous crying or smiling. The accessory nerve is assessed when the infant is in the sitting position and symmetry of the head position is noted.

A nurse is performing a primary survey on a child who has sustained partial thickness burns over his upper body areas. What action should the nurse take first? A) Inspect the child's skin color. B) Assess for a patent airway. C) Observe for symmetric breathing. D) Palpate the child's pulse.

When performing a primary survey, the nurse first assesses the child's airway for patency and then intervenes accordingly to ensure that the airway is patent. Next the nurse would evaluate the child's skin color, respiratory effort, and symmetry of breathing and breath sounds. Then the nurse would determine the pulse strength, perfusion status, and heart rate.

During a health history, the nurse explores the sleeping habits of a 3-year-old boy by interviewing his parents. Which statement from the parents reflects a recommended guideline for promoting healthy sleep in this age group? A)'Our son sleeps through the night, and we insist that he takes two naps a day.' B)'We keep a strict bedtime ritual for our son, which includes a bath and bedtime story.' C)'Our son still sleeps in a crib because we feel it is the safest place for him at night.' D)'Our son occasionally experiences night walking so we allow him to stay up later when this happens.'

son, which includes a bath and bedtime story.' Consistent bedtime rituals help the toddler prepare for sleep; the parent should be advised to choose a bedtime and stick to it as much as possible. The nightly routine might include a bath followed by reading a story. A typical toddler should sleep through the night and take one daytime nap. Most children discontinue daytime napping at around 3 years of age. When the crib becomes unsafe (that is, when the toddler becomes physically capable of climbing over the rails), then he or she must make the transition to a bed. Attention during night waking should be minimized so that the toddler receives no reward for being awake at night.


Related study sets

AH 513: Northern Renaissance Art final

View Set

Law Test 2 - The Charter & Political Leaders

View Set

Chapter 10 Plate Tectonics Section 2 Continental Drift

View Set

APUSH AMSCO Chapter 18 Multiple Choice, APUSH sg chp 18

View Set

antibiotics concepts II exam III

View Set